Onco-Nephrology Curriculum - American Society of Nephrology
Transcription
Onco-Nephrology Curriculum - American Society of Nephrology
Chapter 1: Onco-Nephrology: Growth of the Kidney–Cancer Connection Mark A. Perazella, MD,* and Mitchell H. Rosner, MD† *Section of Nephrology, Department of Medicine, Yale University School of Medicine, New Haven, Connecticut; and †Division of Nephrology, Department of Medicine, University of Virginia Health System, Charlottesville, Virginia The question has been asked of many of us interested in the kidney–cancer connection; Why onco-nephrology? Nephrologists have traditionally treated cancer patients with various forms of kidney disease. However, although typical AKI and electrolyte/acid–base disturbances can be handled by the practicing clinical nephrologist, increasingly it has become clear that many of the issues are more complex and highly specialized. For example, many nephrologists were not trained in the era of bone marrow/hematopoietic stem cell transplant, which has a number of unusual and complicated forms of kidney injury. In addition, the number of anticancer drugs with various types of nephrotoxicity has increased dramatically, and their entry into clinical practice continues at a fast pace. These and other issues have led to a burgeoning interest in a more specific focus of nephrologists on the patient with cancer. Recognizing this changing landscape, nephrologists at various large cancer centers began a discussion about the need to address this rapidly growing area of nephrology. In 2009/2010, Abdulla Salahudeen recruited leaders from these cancer centers and asked the American Society of Nephrology (ASN) to sponsor a “forum” where this emerging area of nephrology could be discussed. In 2012, the ASN had its first official meeting of what was later named the “Onco-Nephrology Forum.” Dr. Salahudeen chaired the group, pushed forward the kidney–cancer agenda with the forum members, and put the fledgling area of onco-nephrology on the nephrology map. The description of the Onco-Nephrology Forum on the ASN website is noted below: “Why onco-nephrology? While all nephrologists address nephrology problems in cancer patients, many of these problems are increasingly complex. To provide the best nephrology care for cancer patients, we must understand rapidly changing protocols and therapies. American Society of Nephrology Emerging kidney toxicities associated with drugs targeting VEGF and TKIs and other signaling pathways, tumor lysis syndrome, cytotoxic chemotherapy-induced kidney toxicities, kidney problems in myeloma, tumor or treatment-related microangiopathies and glomerulonephritis, stem cell transplant–associated acute and chronic kidney injuries, obstructive uropathies, severe fluid and electrolytes abnormalities, and dosing and timing of chemotherapy in CKD and ESRD patients: these and other complex problems, and their increasing frequency and severity, provide a unique and unprecedented opportunity for nephrologists to improve treatment for cancer patients worldwide. Onco-nephrologists help cancer care teams prevent kidney problems or resolve them as they arise and improve patient outcomes. Research in cancer nephrology is already improving kidney care in cancer patients. A more focused approach to cancer nephrology may also help address challenges like renal cell carcinoma in ESRD. The American Society of Nephrology believes onconephrology represents an emerging frontier in the fight against kidney disease.” The ASN Onco-Nephrology Forum has worked hard to spread the word and “metastasize” into multiple journal, meetings, and conferences. One very helpful contribution to this endeavor included the appearance of publications on the onconephrology topic in several high-level journals visible to many nephrologists. Entire issues dedicated to Correspondence: Mark A. Perazella, Section of Nephrology, Department of Medicine, Yale University School of Medicine, BB 114, 330 Cedar Street, New Haven, Connecticut. Copyright © 2016 by the American Society of Nephrology Onco-Nephrology Curriculum 1 Table 1. Onco-Nephrology Curriculum committee members Figure 1. Timeline of the birth and growth of onco-nephrology. Formation of the ASN Onco-Nephrology Forum, numerous conference publications, and dedicated journal publications characterize and highlight the process. ASN, American Society of Nephrology; ONF, Onco-Nephrology Forum; NKF, National Kidney Foundation; JCO, Journal of Clinical Oncology; ON, OncoNephrology; ACKD, Advances in Chronic Kidney Disease; CJASN, Clinical Journal of the American Society of Nephrology; KI, Kidney International; JASN, Journal of the American Society of Nephrology; Sem Nephrol, Seminar in Nephrology. onco-nephrology appeared in Seminars in Nephrology and Advances in Chronic Kidney Disease, while a series of articles on this subject was published in the Clinical Journal of the American Society of Nephrology Moving Points in Nephrology feature. In 2011, the ASN had its first Kidney Week Early Program dedicated to onco-nephrology. This Early Program continues on an every other year schedule. Many of the Kidney Week Clinical Nephrology Conferences included sessions covering various onco-nephrology topics. The National Kidney Foundation Annual Spring Meeting similarly dedicated a session to onconephrology (Figure 1). In addition, editorials describing the importance of onco-nephrology, some suggesting the need for a “new subspecialty” appeared in the Journal of the American Society of Nephrology, Kidney International, and the Journal of Clinical Oncology, authored by members of the ASN Onco-Nephrology Forum. With these important accomplishments, the OncoNephrology Forum with Mark Perazella as the new Onco- 2 American Society of Nephrology Mark A. Perazella (ONF Chair, Lead Editor) Mitchell H. Rosner (Lead Editor) Kevin W. Finkel (Section Editor) Ilya Glezerman (Section Editor) Susie L. Hu (Section Editor) Kenar D. Jhaveri (Section Editor) Amit Lahoti (Section Editor) Anushree C. Shirali (Section Editor) Ala Abudayyeh Joseph R. Angelo Joseph V. Bonventre Anthony Chang Eric P. Cohen Farhad R. Danesh Mona D. Doshi Amaka Edeani Carlos Flombaum Sangeeta R. Hingorani Benjamin Humphreys Divya Monga Abdulla K. Salahudeen Nephrology Forum Chair continued to forge ahead and felt the time was ripe for the creation of an Onco-Nephrology Curriculum. After creation of an outline of topics and discussion by the advisory group (Table 1), the core curriculum was submitted to the ASN Education Committee for review. The curriculum was subsequently approved and the Onco-Nephrology Forum group, under the direction of the curriculum committee co-chairs (Perazella and Rosner), put together the ultimate plan for creation of the curriculum document. The lead editors, section editors, and chapter authors (Tables 1 and 2) were identified and the writing began. The chapters are truly an example of outstanding contributions by experts in the subfield of onco-nephrology. All of the authors are to be congratulated on their fine work and keeping to the originally planned timeline for completion. The product of this work will appear on the ASN’s website and will be available to the ASN membership, the nephrology training programs, and all other interested health care providers. We are confident that this curriculum will strengthen the teaching of onco-nephrology and further expand all practitioners’ knowledge of the subject. We hope the readers enjoy the document. Onco-Nephrology Curriculum Table 2. Onco-Nephrology Curriculum chapters and authors 1) Onco-Nephrology: Growth of the Kidney-Cancer Connection 2) Why do we need an Onco-Nephrology Curriculum? 3) AKI associated with Malignancies 4) Tumor Lysis Syndrome 5) Electrolyte and Acid-Base Disorders and Cancer 6) Glomerular Disease and Cancer 7) Hematologic Diseases and Kidney Disease 8) Clinical tests for Monoclonal Proteins 9) Hematopoietic Stem Cell Transplant-related Kidney Disease 10) Radiation-associated Kidney Injury 11) Chemotherapy and Kidney injury 12) Pharmacokinetics of Chemotherapeutic Agents in Kidney Disease 13) CKD as a Complication of Cancer 14) Hereditary Renal Cancer Syndromes 15) Work-up and Management of Small Renal Masses 16) Cancer in Solid Organ Transplantation 17) Cancer Screening in ESRD 18) Ethics of RRT, Initiation and Withdrawal, in Cancer Patients 19) Palliative Care in Patients with Kidney Disease and Cancer American Society of Nephrology Mark Perazella, Mitchell Rosner Mark Perazella, Mitchell Rosner Amit Lahoti, Benjamin Humphreys Amaka Edeani, Anushree Shirali Anushree Shirali Divya Monga, Kenar Jhaveri Ala Abudayyeh, Kevin Finkel Nelson Leung Sangeeta Hingorani, Joseph Angelo Amaka Edeani, Eric Cohen Ilya Glezerman, Edgar Jaimes Sheron Latcha Maurizio Gallieni, Camillo Porta, and Laura Cosmai Katherine Nathanson Susie Hu, Anthony Chang Mona Doshi Jean Holley Michael Germain Alvin Moss Onco-Nephrology Curriculum 3 AUTHOR QUERIES AUTHOR PLEASE ANSWER ALL QUERIES There are no queries in this article. Chapter 2: Why Do We Need an Onco-Nephrology Curriculum? Mark A. Perazella, MD,* and Mitchell H. Rosner, MD† *Section of Nephrology, Department of Medicine, Yale University School of Medicine, New Haven, Connecticut; and † Division of Nephrology, Department of Medicine, University of Virginia Health System, Charlottesville, Virginia As health care providers, we are acutely aware of the National Vital Statistics Report describing the significant toll cancer, as the second leading cause of death, has on our patients (1). Importantly, cancer incidence rates are highest in the elderly (2). At the same time, the US Renal Data System (USRDS) notes that AKI rates are increasing in the elderly, with rates 10-fold higher than the nonelderly population (3). Importantly, both AKI and CKD are highly prevalent in cancer patients, in particular renal cell cancer, liver cancer, multiple myeloma, leukemias, and lymphomas (4,5). The Belgian Renal Insufficiency and Anticancer Medication (BIRMA) study noted the frequent occurrence of kidney disease in five major cancers (Figure 1) (6). Most concerning is the increased mortality noted in patients with AKI/ CKD compared with those without kidney disease. For instance, the development of AKI can be associated with cessation of effective chemotherapeutic regimens, or the presence of preexisting CKD may limit the use of otherwise active regimens that may be curative. This combination of cancer, kidney disease, and mortality has led to the recognition that nephrology and oncology are intricately linked and require our full attention as a subspecialty (Figure 2). Hence, “onco-nephrology” was born in a few large centers but has steadily grown to include many medical centers, hospitals, and clinics. What exactly is onco-nephrology? It is a rapidly growing area of nephrology where kidney disease in cancer patients has become an important source of consultations, with the trend occurring over the last 10–15 years. Oncology patients now make up a significant number of the patients that nephrologists see for kidney-related problems in the outpatient clinic, on the inpatient floors, and in the medical intensive care unit (ICU). There is an increase in the number of patients with kidney disease, in part related to high incidence rates for many malignancies, as well as improvement in the cancer death rates due to more American Society of Nephrology effective chemotherapeutic agents, including biologics, and stem cell therapies. However, this has led to an increase in the number of cancer survivors that often develop acute and/or CKD due to their malignancy and/or its associated treatment. The best example of the bidirectionality of cancer and kidney disease is seen between renal cancer and CKD (Figure 3 ). Cancer can directly injure the kidneys through tumor infiltration or production of nephrotoxic (paraneoplastic) substances. Any one of the growing numbers of therapeutic agents that extend patient lives can cause various types of acute or CKD, along with serious electrolyte and acid–base abnormalities. In addition, patients may develop multiorgan illness requiring ICU-level care and RRT. Certain malignancies are more likely to cause this severe form of multiorgan dysfunction and may be associated with higher mortality rates. When this type of critical illness occurs in the setting of advanced malignancy, it raises questions about the appropriateness of aggressive care in “futile situations” and the role of palliation. Thus, care for oncology patients has become more specialized and complicated, requiring collaboration between nephrologists, oncologists, intensivists, and palliative care specialists. The remarkable advances in cancer management present both new opportunities and complex challenges for the oncology and nephrology communities. It is essential for nephrologists to be informed and actively involved in certain facets of cancer care. A better understanding of the rapidly evolving field of cancer biology and its therapy is required for nephrologists to become valuable members of the Correspondence: Mark A. Perazella, Section of Nephrology, Department of Medicine, Yale University School of Medicine, BB 114, 330 Cedar Street, New Haven, Connecticut. Copyright © 2016 by the American Society of Nephrology Onco-Nephrology Curriculum 1 Figure 1. Kidney injury associated with five different cancers in the BIRMA study. The percentage of patients with kidney injury as defined by SCR, GFR ,90, or GFR ,60 is noted both for the individual cancers and all cancers lumped together. BIRMA, Belgian Renal Insufficiency and Anticancer Medication study; SCR, serum creatinine; aMDRD, abbreviated MDRD. Adapted with permission from reference 6. cancer care team and to provide the best nephrology care possible. The goal of this American Society of Nephrology (ASN) sponsored Onco-Nephrology core curriculum is to provide the ASN membership including veteran nephrologists, newly minted nephro-clinicians, and fellowship trainees with the building blocks on which further information can be added as technology advances. This educational venue will be available outside the ASN membership as well. Nephrologists must be well prepared to care for patients with cancer and its associated renal complications. The renal manifestations of cancer have many unique features, and these conditions often require specialized approaches to manage fluid, electrolyte, and acid–base disturbances, as well as acute and chronic kidney injury. Furthermore, the ever-evolving field of cancer therapy demands a comprehensive team approach with the nephrologist as one of the critically important care providers. As such, it is essential for nephrologists to develop expertise in the practice of onco-nephrology. We hope this curriculum provides the initial framework to achieve this goal. TAKE HOME POINTS c Kidney disease is a frequent and increasing complication of cancer. c There is a bidirectional relationship between cancer and kidney disease. Figure 2. The relationship between cancer and AKI and CKD. Cancer, AKI, and CKD are linked by various exposures and pathways. 2 Onco-Nephrology Curriculum American Society of Nephrology Figure 3. The bidirectionality between renal cancer and CKD. Common exposures that can cause both renal cell cancer and CKD are noted in the middle bidirectional arrow. c Onco-nephrology is a growing area of nephrology that requires clinicians to have a better understanding of the renal complications of cancer including electrolyte/acid–base disturbances, AKI, and CKD. c The Onco-Nephrology Curriculum is an educational tool created by ASN Onco-Nephrology Forum members and other expert nephrologists. REFERENCES 1. Hoybert DL, Xu J. Deaths: Preliminary data for 2011. Natl Vital Stat Rep 61: 2012 2. National Cancer Institute. Age-adjusted SEER incidence rates, 2007– 2011 (Table 2.7). SEER cancer statistics review (CSR) 1975–2011. Surveillance, epidemiology, and end results program. Available at: http://seer.cancer.gov/csr/1975_2011/browse_csr.php?sectionSEL52& pageSEL5sect_02_table.07.html. Accessed March 1, 2015 American Society of Nephrology 3. USRDS. Percent of Medicare patients aged 661 (a) with at least one AKI hospitalization, and (b) with an AKI hospitalization that had dialysis by year, 2003–2012 (Figure 5.1). Chapter 5: Acute kidney injury. Available at: http://www.usrds.org/2014/view/v1_05.aspx. Accessed March 1, 2015 4. Christiansen CF, Johansen MB, Langeberg WJ, Fryzek JP, Sørensen HT. Incidence of acute kidney injury in cancer patients: A Danish populationbased cohort study. Eur J Intern Med 22: 399–406, 2011 5. Schmid M, Abd-El-Barr AE, Gandaglia G, Sood A, Olugbade K Jr, Ruhotina N, Sammon JD, Varda B, Chang SL, Kibel AS, Chun FK, Menon M, Fisch M, Trinh QD. Predictors of 30-day acute kidney injury following radical and partial nephrectomy for renal cell carcinoma. Urol Cancer 32: 1285–1291, 2014 6. Janus N, Launay-Vacher V, Byloos E, Machiels JP, Duck L, Kerger J, Wynendaele W, Canon JL, Lybaert W, Nortier J, Deray G, Wildiers H. Cancer and renal insufficiency results of the BIRMA study. Br J Cancer 103: 1815–1821, 2010 Onco-Nephrology Curriculum 3 REVIEW QUESTIONS 1. Which of the following malignancies has the highest 1-year risk for AKI? a. b. c. d. e. Multiple myeloma Lymphoma Renal cell cancer Liver cancer Leukemia Answer: c is correct. Although all of these cancers are associated with increased AKI risk, renal cell cancer was found to have the highest 1-year risk in a cohort study examining the incidence of AKI in cancer patients (4). 4 Onco-Nephrology Curriculum 2. In a patient with a recent diagnosis of cancer, which of the following complications are increased in the setting of the cancer diagnosis? a. b. c. d. AKI CKD Mortality All of the above Answer: d is correct. Cancer is associated with an increased incidence of AKI, CKD, and overall mortality. These complications are the result of the tumor itself (infiltration or tumor products), drug nephrotoxicity, comorbid diseases, or all of the above. American Society of Nephrology Chapter 3: AKI Associated With Malignancies Amit Lahoti, MD,* and Benjamin D. Humphreys, MD, PhD† *Division of Internal Medicine, Section of Nephrology, The University of Texas MD Anderson Cancer Center, Houston, Texas; and †Division of Nephrology, Washington University School of Medicine, St. Louis, Missouri INTRODUCTION Advances in treatment, risk stratification, and supportive care have improved survival of patients with cancer over the last two decades (1). AKI may result from the cancer itself (e.g., infiltration or obstruction), the treatment of cancer (e.g., chemotherapy toxicity), or associated complications (e.g., sepsis). Cancer, by itself, is not a contraindication for starting RRT, even in the setting of multiorgan failure (2–4). However, decision-making is complex and requires a multidisciplinary approach between the oncologist, intensivist, and nephrologist. The development of AKI may lead to longer length of hospital stay, decreased functional status and quality of life, and exclusion from further cancer therapy. AKI and RRTmay lead to unpredictable levels of chemotherapeutic agents and anti-infective drugs. AKI may also increase inflammatory cytokines in the lung, leading to increased vascular permeability (5) and the need for mechanical ventilation (6). Therefore, early detection and prevention of AKI is crucial in patients with cancer. DEFINITION More than 35 different definitions for AKI have been used in the literature, which has made crosscomparisons between studies difficult. This led to the development of the RIFLE classification, which defined three stages of AKI (risk, injury, and failure) and two stages of renal failure requiring dialysis (loss and ESRD) (7). Stages for AKI are determined by the percent rise in serum creatinine relative to baseline, decreased urine output, or the need for dialysis. It is unclear whether the criteria are well balanced in respect to urine output and serum creatinine, as most studies have not utilized the urine output component. The RIFLE classification has been validated in numerous patient populations and has highlighted the significant effect of American Society of Nephrology mild degrees of renal injury on mortality. Significant renal injury may occur without elevation in serum creatinine, and an elevation of 0.3 mg/dL has been associated with increased mortality in hospitalized patients. The Acute Kidney Injury Network (AKIN) proposed modifications to the RIFLE criteria with three stages of AKI corresponding to the risk, injury, and failure categories (8). Patients with an absolute rise in serum creatinine of 0.3 mg/dL are included into the least severe category (stage 1). The loss and ESRD categories were eliminated, and all patients requiring dialysis were classified into the most severe category (stage 3). Last, a time constraint of 48 hours to reach stage 1 was also included in the AKIN definition. Whether the AKIN modifications to the RIFLE criteria have led to improvements in classification has yet to be determined (9). Recently, The Kidney Disease Improving Global Outcomes (KDIGO) work group combined elements of the RIFLE and AKIN classifications to define AKI as 1) an increase in serum creatinine (SCr) $0.3 mg/dL within 48 hours, 2) an increase in SCr to $1.5 times baseline within the prior 7 days, or 3) a urine volume of ,0.5 mL/kg/h for 6 hours. Severity of AKI is staged similar to the AKIN criteria. Several studies have correlated AKI as defined by these criteria with increased mortality, length of stay, and hospital costs in patients with cancer (10–13). EPIDEMIOLOGY AND PROGNOSIS AKI is common in hospitalized patients with cancer and is associated with increased length of stay and hospital costs. In a Danish population-based study of 1.2 million cancer patients, the incidence of AKI defined by the RIFLE criteria was highest in patients Correspondence: Amit Lahoti, UT MD Anderson, Cancer Center, Unit 1468, PO Box 301402, Houston, Texas 77230. Copyright © 2016 by the American Society of Nephrology Onco-Nephrology Curriculum 1 with renal cell cancer (44%), multiple myeloma (33%), liver cancer (32%), and leukemia (28%) (14). Compared with patients without cancer, critically ill patients with cancer have a higher incidence of AKI requiring RRT. Depending on the definition of AKI and the underlying case mix, it has been reported that 13%–42% of critically ill patients with cancer develop AKI and 8%–60% require RRT (15). The incidence is highest for those patients with hematologic malignancies, multiple myeloma, and septic shock. The 28-day mortality of patients with cancer who require RRT is 66%–88% (16). In one study of critically ill patients with cancer, the odds ratio for 30-day mortality was increased twofold in patients with AKI. However, approximately one-half of the patients with AKI survived to day 30 after admission (17). In one study of AKI in critically ill patients, there was complete recovery of renal function in 82% and partial recovery in 12%, and chronic dialysis was needed in only 6% of patients (18). Overall severity of illness, age, and functional status may have more of an impact on prognosis than underlying malignancy, and the presence of cancer may not be an absolute exclusion criterion for withholding RRT. However, the prognosis of critically ill recipients of stem cell transplants who develop AKI remains poor, with mortality exceeding 80%. A team-based approach between the oncologist, critical care physician, and nephrologist is necessary to identify patients who are most suitable for initiation of RRT. kidney injury molecule 1 (KIM-1), neutrophil gelatinaseassociated lipocalin (NGAL), N-acetyl-b-D-glucosaminidase (NAG), interleukin 18 (IL-18), and matrix metalloproteinase 9 (MMP-9). The accuracy and reliability of these markers varies across individual studies. An assay for serum and urinary NGAL levels has become recently available but is not routinely used in the clinical setting at this time. EPIDEMIOLOGY OF AKI IN CANCER PATIENTS The overall incidence of AKI among cancer patients was recently defined in a large Danish study. Among 1.2 million people followed between 1999 and 2006, there were 37,267 incident cancer patients with a baseline creatinine measurement. The 1-year risk of AKI in this population (defined as a .50% rise in serum creatinine) was 17.5%, with a 27% risk over 5 years (14). Patients with distant metastases were at the highest risk of AKI. More severe AKI, defined as a doubling of serum creatinine (injury in the RIFLE criteria) (20), had an 8.8% and 14.6% risk at 1 and 5 years, respectively. Even more severe AKI, corresponding to failure in RIFLE criteria and reflecting a tripling of serum creatinine or absolute rise .4 mg/dL, was seen in 4.5% and 7.6% of patients at 1 and 5 years, respectively. Among cancer patients with any stage of AKI (9,613 total), 5.1% required dialysis within 1 year of AKI onset. Older patients were most heavily represented in this analysis. ASSESSMENT OF KIDNEY FUNCTION Cancers with highest AKI risk The ideal marker of kidney function would be a substance that is freely filtered, neither secreted nor reabsorbed, and is solely eliminated by the kidney. Although inulin and radiolabeled EDTA and iothalomate demonstrate many of these characteristics, their complexity and cost of measurement have precluded use in daily practice. Serum creatinine has been traditionally used as a marker of kidney function, but when used in isolation, it is not an adequate measure. Serum creatinine values are altered by many other factors including muscle mass, diet, sex, and tubular secretion. Patients with cancer may present with spuriously low serum creatinine levels secondary to cachexia. However, estimating equations for GFR, which factor other variables such as age, sex, and race along with serum creatinine, provide a reasonable estimate of renal function in most patients. The most commonly used estimating equations are the Cockcroft-Gault, the Modification of Diet in Renal Disease (MDRD), and the Chronic Kidney Disease Epidemiology Collaboration (CKD-EPI) formulas. Among patients with cancer who have serum creatinine values within the normal range, 20% of patients have unsuspected CKD when the GFR is estimated by Cockcroft-Gault formula (19). It is well understood that elevation in serum creatinine is a relatively late marker of renal injury, as a significant amount of kidney function may be lost before a rise in serum creatinine is apparent. Several urinary biomarkers of AKI that have greater sensitivity for acute renal injury have been proposed, including Certain cancers carry a much higher risk of AKI than others. In the Danish study above, kidney cancer, multiple myeloma, and liver cancer had the highest 1-year risk of AKI at 44.0%, 33.0%, and 31.8%, respectively. After diagnosis of renal cell carcinoma, many patients still undergo radical nephrectomy, and this procedure itself is associated with a 33.7% risk of AKI and predicts the future development of CKD at 1 year (21). Patients with acute lymphoma or leukemia undergoing induction chemotherapy are also at an especially high risk of AKI. In a series of 537 patients with either acute myelogenous leukemia or high-risk myelodysplastic syndrome undergoing induction, 36% developed AKI. Even among patients with mild AKI (defined as RIFLE risk), 8-week mortality was 13.6% (95% confidence interval, 7.8%–23%) compared with patients with no AKI whose 8-week mortality was 3.8% (95% confidence interval, 2.2%–6.4%). Patients requiring RRT experienced mortality of 61.7% (95% confidence interval, 50%–74%) over the same time frame (12). AKI is common in hospitalized cancer patients and also correlates with increased length of stay, cost, and mortality. Candrilli and colleagues analyzed the 2004 Nationwide Inpatient Sample for patients with hematologic malignancies. They identified 350,601 patients without AKI, 27,654 patients with mild or moderate AKI (not requiring dialysis), and 5,148 patients with severe AKI (requiring dialysis). The average length of stay and costs among these groups were 7.4, 12.2, and 17.6 days, and 2 Onco-Nephrology Curriculum American Society of Nephrology Table 1. Cancer-specific risk factors for AKI Table 2. Common causes of AKI in patients with cancer Age .65 years Congestive heart failure (i.e., exposure to anthracyclines, trastuzumab) CKD Hypovolemia (i.e., chemotherapy-related nausea and vomiting, acute graft-versus-host disease) Distant metastases Multiple myeloma Liver cancer Nephrectomy for renal cell carcinoma Induction chemotherapy for acute lymphoma or leukemia Prerenal azotemia Volume depletion Nausea, vomiting, diarrhea Decreased oral intake owing to mucositis (5-fluorouracil, methotrexate, taxanes) Polyuria caused by hyperglycemia (steroids) or diabetes insipidus (pituitary tumor) “Third spacing” (hypoalbuminemia, liver or peritoneal metastases, interleukin-2) Insensible loss of fluid from skin lesions (mycosis fungoides) Hemodynamic-mediated Sepsis Renal arteriolar vasoconstriction (nonsteroidal antiinflammatory drugs [NSAIDs], calcineurin inhibitors, hypercalcemia) Congestive heart failure Hepatorenal syndrome/hepatic sinusoidal obstruction syndrome Budd-Chiari syndrome Intrahepatic inferior vena cava compression or thrombosis caused by hepatomegaly or a tumor Intravenous iodinated contrast agent Abdominal compartment syndrome Intrinsic renal disease Acute tubular necrosis Chemotherapy (cisplatin, ifosfamide) Anti-infectives (amphotericin B, foscarnet, cidofovir, aminoglycosides, vancomycin) Bisphosphonates Sepsis Prolonged prerenal azotemia Allergic interstitial nephritis (penicillins, cephalosporins, fluoroquinolones, NSAIDs) Crystal nephropathy (methotrexate, acyclovir, ciprofloxacin, sulfonamides, rifampin) Osmotic nephrosis (IV immunoglobulin, mannitol, starch) Thrombotic microangiopathy (post-hematopoietic stem cell transplant, gemcitabine, prior radiation therapy) Myeloma-related kidney disease Postrenal obstruction Bladder outlet obstruction (malignancy of cervix, prostate, bladder, or uterus) Retroperitoneal disease (metastasis, lymphadenopathy, fibrosis) Hemorrhagic cystitis (cyclophosphamide, BK virus) Ureteral strictures (prior radiation therapy, BK virus) $13,947, $25,638, and $44,619, respectively (22). Cancer-specific risk factors for AKI are summarized in Table 1. ETIOLOGY OF AKI The causes of AKI in patients with cancer are numerous (Table 2). The sites along the nephron at which some of these syndromes act are depicted in Figure 1. The specific diagnoses will be discussed in detail elsewhere in the core curriculum, but some notable causes are highlighted in this chapter. Sepsis Sepsis is the most common cause of AKI in patients with cancer. In population-based studies, approximately 15% of critically ill patients with sepsis have underlying cancer (23). Acute tubular necrosis secondary to sepsis remains the leading cause of AKI in critically ill patients with cancer. Patients with hematologic malignancies are especially prone to the development of bacterial infections and sepsis secondary to prolonged neutropenia. Nearly half of patients admitted to the intensive care unit (ICU) with hematologic malignancies have underlying sepsis compared with 12%–25% of patients with solid tumors (24). Studies have demonstrated improved survival of cancer patients with sepsis over the last decade, except in patients that require RRT, where hospital mortality approaches 80% (25,26). Sepsis causes AKI by systemic vasodilation, leading to decreased effective circulating volume, cytokine activation, endothelial damage, and microthrombi formation. The use of vasoconstricting pressor agents further exacerbates an effective prerenal state. Anti-infectives The high incidence of sepsis in critically ill cancer patients necessitates the use of nephrotoxic antibacterial and antifungal agents. Aminoglycosides may cause nephrotoxicity after 5–7 days of therapy, and patients present with nonoliguric AKI, hypokalemia, hypomagnesemia, and hypocalcemia. The risk of renal toxicity may be minimized with once daily dosing. Several alternative drugs to aminoglycosides that do not cause AKI have become available in the treatment of neutropenic fever. Amphotericin B deoxycholate may cause tubular American Society of Nephrology toxicity and vasoconstriction, leading to nonoliguric AKI, hypokalemia, hypomagnesemia, and distal renal tubular acidosis. Newer liposomal and lipid formulations are less nephrotoxic with comparable efficacy. Other novel antifungal agents, caspofungin and voriconizole, are also less nephrotoxic and are often used as first-line therapy. Several studies have reported on the nephrotoxicity of vancomycin, although the biological mechanism remains undefined. Reported risk factors for AKI are higher trough levels (.15 mg/dL) and Onco-Nephrology Curriculum 3 Figure 1. Sites of injury in AKI syndromes. TMA, thrombotic microangiopathy; ATN, acute tubular necrosis. higher daily doses (.4 g/day) (27,28). Patients present with nonoliguric AKI and bland urine sediment, and most patients recover renal function after discontinuation of the drug. Chemotherapy Cisplatin is a DNA alkylating agent used to treat a variety of tumors including sarcomas, small cell lung cancer, ovarian cancer, and germ cell tumors. It is directly tubular toxic and leads to salt wasting, hyponatremia, hypomagnesemia, and AKI. A low chloride environment enhances toxicity, and concurrent saline administration to achieve urine output .3 L/day is the mainstay of prevention. Approximately onethird of patients will experience AKI within days after treatment, and episodes worsen with repeated dosing. Tubular injury may be permanent with doses .100 mg/m2. Amifostine, a free radical scavenger, has been shown to ameliorate cisplatin nephrotoxicity. Newer platinum agents such as carboplatin and oxaliplatin appear to cause less tubular injury. Ifosfamide is an alkylating agent commonly used in treating sarcomas and metastatic germ cell turmors, which may cause AKI in up to 30% of patients. Proximal tubular injury may also lead to glucosuria, hypokalemia, hypophophatemia, and proximal renal tubular acidosis. Severe cases may present with Fanconi’s syndrome. Cumulative doses .100 g/m2 are associated with moderate to severe tubular injury. Risk factors for AKI include prior cisplatin therapy, tumor infiltration of the kidney, and underlying CKD. Mesna protects against bladder toxicity 4 Onco-Nephrology Curriculum from metabolites excreted in the urine, which helps prevent hemorrhagic cystitis. Methotrexate is an antifolate and antimetabolite commonly used in the treatment of leukemia, lymphoma, and sarcoma. High-dose methotrexate (.1 g/m2) may cause AKI by forming intratubular crystals leading to obstruction and direct tubular cell toxicity. Patients generally present with nonoliguric AKI with a subsequent rapid rise in serum creatinine. Intravenous hydration and urinary alkalinization prevent the precipitation of methotrexate crystals. In the setting of AKI, methotrexate may accumulate and cause neutropenia, hepatitis, mucositis, and neurologic impairment. Folinic acid may be given concurrently to replete folic acid stores and minimize toxicities. Dialysis can acutely clear methotrexate from the blood, but levels quickly rebound after discontinuation of treatment. Carboxypeptidase G2 can rapidly convert methotrexate to an inactive metabolite and recently became commercially available. This therapy also suffers from rebound in plasma levels, but to a lesser degree than high-flux dialysis. Targeted therapy Targeted therapy against vascular endothelial growth factor (VEGF) has advanced the treatment of certain tumors including colorectal and renal cell carcinoma. Monoclonal antibody to VEGF (bevacizumab) and tyrosine kinase inhibitors of the VEGF pathway (sunitinib, sorafenib, pazopanib, axitinib, and regorafenib) have been associated with the development of hypertension and American Society of Nephrology proteinuria (29). Rare cases of thrombotic microangiopathy (TMA) have also been reported (30). Symptoms generally resolve with discontinuation of the drug. Multiple myeloma Multiple myeloma involves the clonal proliferation of malignant plasma cells and is the second most common hematologic malignancy after non-Hodgkin lymphoma. Approximately one-half of patients with multiple myeloma present with AKI, and 10% require dialysis on initial presentation (31). The common mechanisms of injury include cast nephropathy, light chain deposition disease, light chain amyloidosis, hypercalcemia, and acute tubular necrosis (ATN) from sepsis. Suppression of normal hematopoiesis predisposes patients to infections and sepsis, which often requires ICU admission. Initial management consists of saline hydration, correction of hypercalcemia, alkalinization of urine, and avoidance of nonsteroidal anti-inflammatory drugs and iodinated contrast. Renal recovery occurs in up to one-half of patients, except in patients who require dialysis, where recovery rates are ,25%. In a randomized controlled trial, the use of plasma exchange did not significantly decrease the composite end point of death, dialysis dependence, or GFR ,30 mL/min (32). With concurrent chemotherapy, the use of high cut-off filters with extended daily dialysis may help to decrease circulating monoclonal light chains. Multicenter randomized controlled trials studying the utility of high cut-off hemofilters are currently ongoing. Hematopoietic cell transplant The number of hematopoietic cell transplants (HCTs) performed has dramatically increased over the last three decades. Refinement in techniques has permitted transplants in older patients with more comorbidities. All patients, regardless of the type of transplant, are susceptible to infection after transplant until engraftment is complete. During this period, patients are at most risk of developing AKI from ischemic and toxic ATN in the setting of sepsis. Patients who receive allogeneic transplants require calcineurin inhibitors to prevent graft-versus-host disease (GVHD), which further increases the risk of AKI. The need for RRT after HCT increases mortality more than 70% (33,34). Engraftment syndrome may occur within days after autologous HCT and is a common reason for ICU admission. It is associated with cytokine release in association with rapid neutrophil recovery after HCT. Patients develop fever, noncardiogenic pulmonary edema, erythrodermatous skin rash, and peripheral edema. Often these patients develop nonoliguric AKI with relatively bland urine sediment. The mainstay of treatment is corticosteroids and diuretics, and most patients will recover renal function without the need for RRT. Hepatic sinusoidal obstruction syndrome (HSOS), formerly termed veno-occlusive disease, is associated with AKI within the first month after allogeneic HCT. Damage to the American Society of Nephrology hepatic sinusoidal endothelium from the pretransplant conditioning regimen leads to sloughing of the endothelium, collagen deposition, fibrosis, and liver failure. In severe cases, patients may subsequently develop AKI from hepatorenal syndrome. Presentation includes right upper quadrant abdominal pain, ascites, edema, and elevated bilirubin. Treatment includes salt restriction, diuretics, and RRT if needed. Severe HSOS, defined as severe liver injury unresponsive to supportive care, often requires ICU admission and is historically associated with near 100% mortality. Defibrotide, an oligonucleotide that has antithrombotic and profibrinolytic properties with minimal anticoagulant effects, has shown promise in patients with severe HSOS. Several clinical trials using defibrotide for treatment of severe HSOS have demonstrated improvement in complete response rates and overall survival, and the drug is currently commercially available in Europe (35,36). A new drug application (NDA) for defibrotide was submitted to the Food and Drug Administration in 2014 and has been granted Fast Track Designation. TMA occurs in approximately 2%–21% of patients after allogeneic stem cell transplant (37). In one study, 3% of all cancer patients admitted with AKI to the ICU had underlying TMA (4). Patients often present with progressive AKI, anemia out of proportion to underlying renal function, and hypertension. Risk factors for transplant-associated TMA (TA-TMA) are acute GVHD, recipient/donor mismatch, total body irradiation .1,200 cGy, and adenovirus infection (37). TA-TMA is not associated with ADAMTS-13 deficiency and is poorly responsive to plasmapheresis. Calcineurin inhibitors are also associated with TMA and should be withheld or decreased in dose if possible. Contrast-induced nephropathy Intravascular administration of iodinated contrast is associated with contrast-induced nephropathy (CIN). Risk factors include underlying CKD, diabetes mellitus, volume depletion, and coadministration of other nephrotoxins. Intra-arterial injection is considered to be more nephrotoxic compared with intravenous administration. In addition, high osmolar (.1400 mOsm/kg) and low osmolar (600–800 mOsm/kg) contrast agents are associated with a higher incidence of AKI in comparison to iso-osmolar (300 mOsm/kg) contrast. Preventive measures should be taken in patients with GFR ,60 mL/min including limiting contrast volume, using iso-osmolar contrast, prehydration with normal saline, and discontinuation of concurrent nephrotoxic agents. Several meta-analyses have examined the use of N-acetylcysteine in the prevention of CIN but results remain inconclusive, as is the use of bicarbonate (38). There is insufficient evidence to recommend hemodialysis or hemofiltration for the prevention or treatment of CIN. Abdominal compartment syndrome Abdominal compartment syndrome (ACS) is most commonly defined as an intra-abdominal pressure (IAP) .10 and clearly Onco-Nephrology Curriculum 5 .20 mmHg with evidence of organ dysfunction that improves with abdominal decompression. Patients may present with tachypnea with high ventilatory pressures, liver dysfunction, intestinal ischemia, and oliguric AKI. In patients with cancer, common causes include malignant ascites, urinary leak from a recent urologic procedure, and colonic dilatation. The IAP, which is measured by transducing a foley catheter filled with saline with a pressure monitoring system, is normally 0–10 mmHg. Values between 12 and 20 mmHg are classified as intra-abdominal hypertension and are not generally associated with organ dysfunction. Depending on the etiology, treatment may involve diuretics, paracentesis, colonic decompression with nasogastric suction, and decompression laparotomy. Generally, urine output and renal function markedly improve with therapy. CONCLUSION AKI is a common complication of cancer or its treatment. Advances in supportive care including RRT have improved outcomes in critically ill patients with cancer, with the exception of patients with allogeneic stem cell transplants. A joint decision-making process between the oncologist, intensivist, and nephrologist is vital to determine which patients are best suited for RRT. Identification of risk factors for AKI, as well as the development of biomarkers of kidney injury, may lead to earlier intervention. TAKE HOME POINTS c The selection of patients best suited for RRT requires a team-based approach between the oncologist, intensivist, and nephrologist. c Manifestations of kidney disease from chemotherapy and targeted therapy include AKI, proteinuria, electrolytes derangements, and TMA. c Nearly one-half of patients with multiple myeloma have evidence of AKI on initial presentation, and 10% require dialysis. c Engraftment syndrome, HSOS, and TMA are unique causes of AKI in patients after stem cell transplant. The mortality of patients that require dialysis after stem cell transplant remains high. REFERENCES 1. Brenner H. Long-term survival rates of cancer patients achieved by the end of the 20th century: A period analysis. Lancet 360: 1131–1135, 2002 2. Benoit DD, Hoste EA, Depuydt PO, Offner FC, Lameire NH, Vandewoude KH, Dhondt AW, Noens LA, Decruyenaere JM. Outcome in critically ill medical patients treated with renal replacement therapy for acute renal failure: comparison between patients with and those without haematological malignancies. Nephrol Dial Transplant 20: 552–558, 2005 3. Berghmans T, Meert AP, Markiewicz E, Sculier JP. Continuous venovenous haemofiltration in cancer patients with renal failure: A singlecentre experience. Support Care Cancer 12: 306–311, 2004 6 Onco-Nephrology Curriculum 4. Darmon M, Thiery G, Ciroldi M, Porcher R, Schlemmer B, Azoulay E. Should dialysis be offered to cancer patients with acute kidney injury? Intensive Care Med 33: 765–772, 2007 5. Kramer AA, Postler G, Salhab KF, Mendez C, Carey LC, Rabb H. Renal ischemia/reperfusion leads to macrophage-mediated increase in pulmonary vascular permeability. Kidney Int 55: 2362–2367, 1999 6. Vieira JM, Jr., Castro I, Curvello-Neto A, Demarzo S, Caruso P, Pastore L, Jr., Imanishe MH, Abdulkader RC, Deheinzelin D. Effect of acute kidney injury on weaning from mechanical ventilation in critically ill patients. Crit Care Med 35: 184–191, 2007 7. Bellomo R, Ronco C, Kellum JA, Mehta RL, Palevsky P. Acute renal failure: Definition, outcome measures, animal models, fluid therapy and information technology needs: the Second International Consensus Conference of the Acute Dialysis Quality Initiative (ADQI) Group. Crit Care 8: R204–R212, 2004 8. Mehta RL, Kellum JA, Shah SV, Molitoris BA, Ronco C, Warnock DG, Levin A. Acute Kidney Injury Network: Report of an initiative to improve outcomes in acute kidney injury. Crit Care 11: R31, 2007 9. Kellum JA. Defining and classifying AKI: One set of criteria. Nephrol Dial Transplant 23: 1471–1472, 2008 10. Lahoti A, Nates JL, Wakefield CD, Price KJ, Salahudeen AK. Costs and outcomes of acute kidney injury in critically ill patients with cancer. J Support Oncol 9: 149–155, 2011 11. Salahudeen AK, Doshi SM, Pawar T, Nowshad G, Lahoti A, Shah P. Incidence rate, clinical correlates, and outcomes of AKI in patients admitted to a comprehensive cancer center. Clin J Am Soc Nephrol 8: 347–354, 2013 12. Lahoti A, Kantarjian H, Salahudeen AK, Ravandi F, Cortes JE, Faderl S, O’Brien S, Wierda W, Mattiuzzi GN. Predictors and outcome of acute kidney injury in patients with acute myelogenous leukemia or high-risk myelodysplastic syndrome. Cancer 116: 4063–4068, 2010 13. Kim CS, Oak CY, Kim HY, Kang YU, Choi JS, Bae EH, Ma SK, Kweon SS, Kim SW. Incidence, predictive factors, and clinical outcomes of acute kidney injury after gastric surgery for gastric cancer. PLoS One 8: e82289, 2013 14. Christiansen CF, Johansen MB, Langeberg WJ, Fryzek JP, Sorensen HT. Incidence of acute kidney injury in cancer patients: A Danish population-based cohort study. Eur J Intern Med 22: 399–406, 2011 15. Darmon M, Ciroldi M, Thiery G, Schlemmer B, Azoulay E. Clinical review: Specific aspects of acute renal failure in cancer patients. Crit Care 10: 211, 2006 16. Benoit DD, Hoste EA. Acute kidney injury in critically ill patients with cancer. Crit Care Clin 26: 151–179, 2009 17. Darmon M, Thiery G, Ciroldi M, de Miranda S, Galicier L, Raffoux E, Le Gall JR, Schlemmer B, Azoulay E. Intensive care in patients with newly diagnosed malignancies and a need for cancer chemotherapy. Crit Care Med 33: 2488–2493, 2005 18. Soares M, Salluh JI, Carvalho MS, Darmon M, Rocco JR, Spector N. Prognosis of critically ill patients with cancer and acute renal dysfunction. J Clin Oncol 24: 4003–4010, 2006 19. Dogan E, Izmirli M, Ceylan K, Erkoc R, Sayarlioglu H, Begenik H, Alici S. Incidence of renal insufficiency in cancer patients. Adv Ther 22: 357– 362, 2005 20. Eheman C, Henley SJ, Ballard-Barbash R, Jacobs EJ, Schymura MJ, Noone AM, Pan L, Anderson RN, Fulton JE, Kohler BA, Jemal A, Ward E, Plescia M, Ries LA, Edwards BK. Annual Report to the Nation on the status of cancer, 1975-2008, featuring cancers associated with excess weight and lack of sufficient physical activity. Cancer 118: 2338–2366, 2012 21. Cho A, Lee JE, Kwon GY, Huh W, Lee HM, Kim YG, Kim DJ, Oh HY, Choi HY. Post-operative acute kidney injury in patients with renal cell carcinoma is a potent risk factor for new-onset chronic kidney disease after radical nephrectomy. Nephrol Dial Transplant 26: 3496–3501, 2011 22. Candrilli S, Bell T, Irish W, Morris E, Goldman S, Cairo MS. A comparison of inpatient length of stay and costs among patients with hematologic malignancies (excluding hodgkin disease) associated with and without acute renal failure. Clin Lymphoma Myeloma 8: 44–51, 2008 American Society of Nephrology 23. Lameire N, Van Biesen W, Vanholder R. Acute renal problems in the critically ill cancer patient. Curr Opin Crit Care 14: 635–646, 2008 24. Taccone FS, Artigas AA, Sprung CL, Moreno R, Sakr Y, Vincent JL. Characteristics and outcomes of cancer patients in European ICUs. Crit Care (London, England) 13: R15, 2009 25. Larche J, Azoulay E, Fieux F, Mesnard L, Moreau D, Thiery G, Darmon M, Le Gall JR, Schlemmer B. Improved survival of critically ill cancer patients with septic shock. Intensive Care Med 29: 1688–1695, 2003 26. Pene F, Percheron S, Lemiale V, Viallon V, Claessens YE, Marque S, Charpentier J, Angus DC, Cariou A, Chiche JD, Mira JP. Temporal changes in management and outcome of septic shock in patients with malignancies in the intensive care unit. Critical Care Med 36: 690–696, 2008 27. Hidayat LK, Hsu DI, Quist R, Shriner KA, Wong-Beringer A. High-dose vancomycin therapy for methicillin-resistant Staphylococcus aureus infections: Efficacy and toxicity. Arch Intern Med 166: 2138–2144, 2006 28. Lodise TP, Lomaestro B, Graves J, Drusano GL. Larger vancomycin doses (at least four grams per day) are associated with an increased incidence of nephrotoxicity. Antimicrob Agents Chemother 52: 1330– 1336, 2008 29. Izzedine H, Rixe O, Billemont B, Baumelou A, Deray G. Angiogenesis inhibitor therapies: Focus on kidney toxicity and hypertension. Am J Kidney Dis 50: 203–218, 2007 30. Eremina V, Jefferson JA, Kowalewska J, Hochster H, Haas M, Weisstuch J, Richardson C, Kopp JB, Kabir MG, Backx PH, Gerber HP, Ferrara N, Barisoni L, Alpers CE, Quaggin SE. VEGF inhibition and renal thrombotic microangiopathy. N Engl J Med 358: 1129–1136, 2008 31. Kyle RA, Gertz MA, Witzig TE, Lust JA, Lacy MQ, Dispenzieri A, Fonseca R, Rajkumar SV, Offord JR, Larson DR, Plevak ME, Therneau TM, Greipp PR. Review of 1027 patients with newly diagnosed multiple myeloma. Mayo Clin Proc 78: 21–33, 2003 American Society of Nephrology 32. Clark WF, Stewart AK, Rock GA, Sternbach M, Sutton DM, Barrett BJ, Heidenheim AP, Garg AX, Churchill DN. Plasma exchange when myeloma presents as acute renal failure: A randomized, controlled trial. Ann Intern Med 143: 777–784, 2005 33. Lopes JA, Jorge S. Acute kidney injury following HCT: Incidence, risk factors and outcome. Bone Marrow Transplantation 46: 1399–1408, 2011 34. Humphreys BD. Onco-nephrology: Kidney disease in the cancer patient: Introduction. Semin Nephrol 30: 531–533, 2010 35. Richardson PG, Murakami C, Jin Z, Warren D, Momtaz P, Hoppensteadt D, Elias AD, Antin JH, Soiffer R, Spitzer T, Avigan D, Bearman SI, Martin PL, Kurtzberg J, Vredenburgh J, Chen AR, Arai S, Vogelsang G, McDonald GB, Guinan EC. Multi-institutional use of defibrotide in 88 patients after stem cell transplantation with severe veno-occlusive disease and multisystem organ failure: response without significant toxicity in a high-risk population and factors predictive of outcome. Blood 100: 4337–4343, 2002 36. Richardson PG, Soiffer RJ, Antin JH, Uno H, Jin Z, Kurtzberg J, Martin PL, Steinbach G, Murray KF, Vogelsang GB, Chen AR, Krishnan A, Kernan NA, Avigan DE, Spitzer TR, Shulman HM, Di Salvo DN, Revta C, Warren D, Momtaz P, Bradwin G, Wei LJ, Iacobelli M, McDonald GB, Guinan EC. Defibrotide for the treatment of severe hepatic venoocclusive disease and multiorgan failure after stem cell transplantation: a multicenter, randomized, dose-finding trial. Biol Blood Marrow Transplant 16: 1005–1017, 2010 37. Changsirikulchai S, Myerson D, Guthrie KA, McDonald GB, Alpers CE, Hingorani SR. Renal thrombotic microangiopathy after hematopoietic cell transplant: Role of GVHD in pathogenesis. Clin J Am Soc Nephrol 4: 345–353, 2009 38. Vaitkus PT, Brar C. N-acetylcysteine in the prevention of contrastinduced nephropathy: Publication bias perpetuated by meta-analyses. Am Heart J 153: 275–280, 2007 Onco-Nephrology Curriculum 7 REVIEW QUESTIONS 1. Criteria for AKI as defined by the KDIGO classification include the following except: a. A rise in SCr $0.3 mg/dL within 48 hours b. An increase in SCr to $1.5 times baseline within the prior 7 days c. A urine volume of ,0.5 mL/kg/h for 6 hours d. An increase in SCr to $1.5 times the upper limit of the “normal” range as listed in the laboratory reference values Answer: d is correct. The KDIGO classification defines AKI as 1) an increase in SCr $0.3 mg/dL within 48 hours; 2) an increase in SCr to $1.5 times baseline within the prior 7 days, or 3) a urine volume of ,0.5 mL/kg/h for 6 hours. The upper limit of normal from a reference range should not be used in diagnosing AKI if the patient’s baseline SCr level is known. 2. Common manifestations of myeloma-related kidney disease include all of the following except: a. b. c. d. 8 Cast nephropathy Light chain deposition disease Thrombotic microangiopathy (TMA) Light chain amyloidosis Onco-Nephrology Curriculum Answer: c is correct. The three most common manifestations of myeloma-related kidney disease include cast nephropathy, light chain deposition disease, and light chain amyloidosis. Other less common manifestations include heavy chain deposition disease, membranoproliferative glomerulonephritis from cryoglobulinemia, and fibrillary glomerulonephritis. TMA is not a common presentation. 3. Which of the following therapies has shown efficacy in the treatment of HSOS after stem cell transplant? a. b. c. d. Heparin Defibrotide Tissue plasminogen activator (tPA) Plasmapheresis Answer: b is correct. Heparin has been used for prophylaxis of HSOS with mixed results. Both heparin and tPA have unacceptable bleeding risks when used for treatment of HSOS. Defibrotide, an oligonucleotide that has antithrombotic and profibrinolytic properties with minimal anticoagulant effects, has shown promise in the treatment of patients with severe HSOS. Plasmapheresis has no role in the treatment of HSOS. American Society of Nephrology Chapter 4: Tumor Lysis Syndrome Amaka Edeani, MD,* and Anushree Shirali, MD† *Kidney Diseases Branch, National Institute of Diabetes and Digestive and Kidney Diseases, National Institutes of Health, Bethesda, Maryland; and †Section of Nephrology, Yale University School of Medicine, New Haven, Connecticut INTRODUCTION Tumor lysis syndrome (TLS) is a constellation of metabolic abnormalities resulting from either spontaneous or chemotherapy-induced tumor cell death. Tumor cytotoxicity releases intracellular contents, including nucleic acids, proteins, and electrolytes into the systemic circulation and may lead to development of hyperuricemia, hyperphosphatemia, hypocalcemia, and hyperkalemia. Clinically, this results in multiorgan effects such as AKI, cardiac arrhythmias, and seizures (1,2). TLS is the most common oncologic emergency (3), and without prompt recognition and early therapeutic intervention, morbidity and mortality is high. DEFINITION Hande and Garrow (4) first initiated a definition of the clinical and pathologic characteristics of patients at risk for developing TLS. Based on a retrospective analysis of 102 patients with non-Hodgkin lymphoma (NHL), they classified TLS as laboratory TLS (LTLS) or clinical TLS (CTLS). Cairo and Bishop (1) modified these criteria to formulate a commonly used classification system for TLS. This system (Table 1) defines LTLS when two or more of the following abnormalities are met within 3 days before or 7 days after the initiation of chemotherapy: 1) 25% decrease from baseline in serum calcium, and/or 2) 25% increase from baseline in the serum values of uric acid, potassium, or phosphorous. The Cairo and Bishop definition assumes adequate volume expansion and prophylaxis with a hypouricemic agent. LTLS is defined as CTLS (Table 1) when LTLS is accompanied by one or more clinical manifestations such as cardiac arrhythmia, death, seizure, or AKI with an elevated serum creatinine .1.5 times upper limit of normal. Additionally, this definition of CTLS assumes that the clinical manifestations are not caused directly by the therapeutic agent. Last, a third American Society of Nephrology class specifies patients with normal laboratory and clinical parameters as having no LTLS or CTLS. Cairo and Bishop also proposed a grading system combining the definitions of no TLS, LTLS, and CTLS, with the maximal clinical manifestations in each affected organ defining the grade of TLS (1). Although this grading system attempts to provide uniform definitions to TLS severity, it is not widely used in clinical practice. The Cairo-Bishop classification is not immune to critique despite its common use. Specifically, patients with TLS may not always have two or more abnormalities present at once, but one metabolic derangement may precede another (2). Furthermore, a 25% change from baseline may not always be significant if it does not result in a value outside the normal range (2). From a renal standpoint, Wilson and Berns (5) have noted that defining AKI on the basis of a creatinine value .1.5 times the upper limit of normal does not clearly distinguish CKD from AKI. Thus, they propose using established definitions of AKI in CTLS such as an absolute 0.3 mg/dL increase or relative 50% increase in creatinine over baseline. Finally, they point out that the Cairo-Bishop classifications cannot be applied to spontaneous TLS, which is common with high-risk malignancies, as chemotherapy is a required criterion for LTLS and CTLS. EPIDEMIOLOGY AND RISK FACTORS TLS is most commonly described in NHL, particularly Burkitt-type lymphoma (BTL), as well as other hematologic malignancies, such as acute lymphocytic and lymphoblastic leukemia (ALL) and acute myeloid leukemia (AML) (6–8), and less commonly in chronic Correspondence: Anushree Shirali, Section of Nephrology, Yale University School of Medicine, PO Box 208029, New Haven, Connecticut 06520-8029. Copyright © 2016 by the American Society of Nephrology Onco-Nephrology Curriculum 1 Table 1. Cairo-Bishop definition of laboratory tumor lysis syndrome and clinical tumor lysis syndrome Laboratory Tumor Lysis Syndrome Metabolite or electrolyte Uric acid Potassium Phosphorus Calcium Criterion for diagnosis $8 mg/dL or 25% increase from baseline $6 mEq/L or 25% increase from baseline $6.5 mg/dL (children), $4.5 mg/dL (adults), or 25% increase from baseline $25% decrease from baseline Clinical Tumor Lysis Syndrome LTLS and one or more of the following: 1) creatinine 3 $1.5 ULN (age .12 years of age or age adjusted); 2) cardiac arrhythmia or sudden death; 3) seizure LTLS, laboratory tumor lysis syndrome; ULN, upper limit of normal. leukemias (9–11) and multiple myeloma (12,13). More rarely, TLS has also been described with solid malignancies (14,15) with particular features, including large tumor burden, metastatic disease, specifically in the liver, short doubling time, increased chemosensitivity, and elevated uric acid and lactate dehydrogenase (LDH) (15). Among solid tumors, small-cell carcinoma of the lung, germ cell tumors, neuroblastoma, and breast carcinoma have all been linked to development of TLS (8). TLS is usually associated with cytotoxic chemotherapy but reports have also linked it to the use of imatinib (11), bortezomib (12), corticosteroids (16,17), rituximab (18), methotrexate (19), and thalidomide (13,20). There are also case reports of TLS following total body irradiation (21) and chemoembolization (22). Last, TLS may also be spontaneous, i.e., not requiring initiation of cytotoxic therapy. This has been most frequently described in BTL (23–25). The incidence of TLS varies based on the underlying malignancy and the definition of TLS. Most incidence data are from older, retrospective studies that precede the CairoBishop classification, so there is considerable heterogeneity in the data. In a review of 102 patients with high-grade NHL and using the Hande-Garrow classification, LTLS was seen in 42% of patients, with CTLS occurring only in 6% (4). In BTL, however, 56% and 11% of patients met criteria for LTLS and CTLS, respectively. Mato et al. (26) studied 194 patients receiving induction therapy for AML and found a TLS incidence of 9.8%. In a mixed adult and pediatric study of 788 European patients with acute leukemia or NHL (27), the overall incidence of LTLS and CTLS was 18.9% and 5%, respectively. When classified by tumor type, LTLS and CTLS incidence rates of 14.7% and 3.4% were seen in AML patients, respectively; 21.4% and 5.2% in ALL patients, respectively; and 19.6% and 6.1% in patients with NHL, respectively (27). Wössman et al. (28) reviewed the incidence and complications of 1,791 children with NHL and reported an overall incidence of 4.4%, of which 26% had B-cell ALL (B-ALL). Risk stratification Risk factors (2,29) for TLS include cancer and patient-specific factors. Increased tumor burden is the most cancer-specific risk factor and is demonstrated by elevated LDH (28), white blood cell count .50,000/mm3, massive liver metastasis (14), bone marrow involvement (2), cancer stage, proliferation rate 2 Onco-Nephrology Curriculum of cancer cells, and cell sensitivity to cytotoxic therapy. Patient-related factors include age, volume depletion, preexisting CKD, hyperuricemia, and hyponatremia. Recognition of these high-risk factors is an important step in the management of TLS. In 2008, an expert panel (7) developed a TLS risk classification system, based on published evidence and expert opinion, in which malignancies as were described as low (,1% chance), intermediate (1%–5% chance), or high risk (.5% chance) for developing TLS. Classification into these risk groups incorporates type of histology, extent of disease, renal involvement or dysfunction, and type of induction therapy (Table 2). Other factors that have been shown to be predictive of TLS include male sex and presence of splenomegaly (26,28,30). Certain cytogenetic shifts may also portend greater risk for TLS. Specifically, MYCN gene mutation in neuroblastoma (31), t (8;14)(q24;q32) in L3 type of acute lymphoblastic leukemia (32), and inv(16)(p13;q22) in acute myelocytic leukemia (33) are all linked to more aggressive disease and greater risk for TLS. PATHOPHYSIOLOGY AND CLINICAL MANIFESTATIONS TLS is a direct consequence of cell lysis and release of intracellular products. When clearance of these products, by excretion (renal or hepatic excretion or phagocytosis by the reticuloendothelial system) (23), is impaired and their serum burden increases, the clinical sequelae of TLS may occur. Of these cellular products, nucleic acids (converted to uric acid), potassium, and phosphorus are particularly important in the pathophysiology of TLS. Hyperuricemia The nucleic acids adenine and guanine are metabolized to xanthine, which is further metabolized by xanthine oxidase to the water-insoluble uric acid (5) (Figure 1). Because humans lack a functional gene for urate oxidase (uricase), which further metabolizes uric acid to the freely soluble and excretable allantoin, patients with high-risk malignancy are susceptible to rapid increases in serum uric acid. Uric acid is freely filtered at the glomerulus, and handling in the renal proximal tubule American Society of Nephrology Table 2. Risk classification of TLS according to type of malignancy, extent of disease, and presence or absence of renal dysfunction Type of malignancy Solid tumor Myeloma Chronic leukemia Lymphoma: Burkitt type Lymphoma: non-Burkitt type Anaplastic large cell Lymphoblastic lymphoma Hodgkin, small lymphocytic, follicular, marginal zone B cell, MALT, nonblastoid mantle cell, cutaneous T cell Adult T-cell lymphoma, diffuse large B cell, peripheral T cell, transformed, or blastoid mantle cell Leukemia:-Burkitt type Leukemia: non-Burkitt type; acute myeloid leukemia (AML); acute lymphoblastic Lleukemia (ALL) Renal dysfunction Absent Present Risk Low Low CML: low CLL w/alkylating agents: low CLL w/targeted or biological agents: intermediate Early stage and LDH ,2 3 ULN: intermediate Early stage and LDH .2 3 ULN: high Advanced stage: high Child with stage III/IV disease: intermediate All others: low Early stage and LDH ,2 3 ULN: intermediate Early stage and LDH .2 3 ULN: high Advanced stage: high Low Adult with normal LDH: low Child with stage I/II disease: low Adult with LDH . ULN and nonbulky disease: intermediate Adult with LDH . ULN and bulky disease: high Child with stage III/IV disease and LDH ,2 3 ULN: intermediate Child with stage III/IV disease and LDH .2 3 ULN: high High AML with WBC ,25 3 109/L and LDH ,2 3 ULN: low AML with WBC ,25 3 109/L and LDH .2 3 ULN: intermediate AML with WBC 5 25–100 3 109/L: intermediate ALL with WBC ,100 3 109/L and LDH ,2 3 ULN: intermediate ALL with WBC ,100 3 109/L and LDH .2 3 ULN: high ALL with WBC .100 3 109/L: high Risk If low risk disease, no change If intermediate risk disease and normal UA, phosphorus, and potassium, no change If UA, phosphorus, or potassium . ULN, intermediate risk disease becomes high risk Low risk disease become intermediate risk Intermediate risk disease becomes high risk CML, chronic myeloid leukemia; CLL, chronic lymphocytic leukemia; MALT, mucosa-associated lymphoid tissue; LDH, lactate dehydrogenase; AML, acute myeloid leukemia; WBC, white blood cell count; ALL, acute lymphocytic and lymphoblastic leukemia; UA, urinalysis; ULN, upper limit of normal. is a combination of reabsorption and secretion via the luminal urate/anion exchanger urate transporter 1 (URAT-1) and the basolateral organic anion transporter (OAT) (34). URAT-1 is an apical membrane transporter and exchanges anions for urate absorption from the tubular lumen. It is critical in regulating urate levels and is targeted by uricosuric and antiuricosuric agents (34). When the capacity to transport luminal uric acid is overwhelmed, there is potential for uric acid to crystallize within the tubular lumen. An acidic urine pH favors this process. Uric acid crystals can cause direct tubular injury by obstruction, but other pathways for injury include induction of chemokine-mediated inflammation from monocyte chemoattractant protein-1 (MCP-1) (35) and macrophage migration inhibition factor (MIF) (36). There are also American Society of Nephrology crystal-independent mechanisms which target hemodynamics. These include increased peritubular capillary pressures, increased vasoconstriction, and decreased blood flow (5,37–39). Uric acid may also prevent recovery from AKI in TLS, as it has been shown to inhibit proximal tubule cell proliferation (38). These diverse mechanisms are united in their propensity to cause AKI. Clinically, hyperuricemia is unlikely to cause symptoms because urinary crystallization of uric acid does not result in the renal colic, which is typical of uric acid nephrolithiasis. Hyperkalemia Massive tumor cell lysis releases potassium into the extracellular environment, leading to severe hyperkalemia when uptake capacity by muscle and liver is exceeded, especially in Onco-Nephrology Curriculum 3 Figure 1. Schematic of purine metabolism. Allopurinol acts an inhibitor of xanthine oxidase via its active metabolite, oxypurinol. Dashed arrow and box indicate arm of metabolism not constitutively present in humans; this conversion of uric acid to water-soluble allantoin is stimulated clinically by the administration of rasburicase (recombinant urate oxidase). Black arrows denote enzyme stimulation; red lines denote inhibition. the setting of CKD or AKI. Muscle weakness may be the initial symptom, but cardiac arrhythmia, manifested initially by peaked Twaves, widened QRS complexes, and sine waves, is the feared complication. population (42). Although the data were not broken down into cause of AKI, the incidence of TLS was similar at 17%, suggesting that AKI and TLS were also linked in this population. AKI due to TLS may be asymptomatic or include symptoms of uremia, including nausea, vomiting, and lethargy. Hyperphosphatemia and hypocalcemia Because phosphate is an intracellular electrolyte, cell lysis releases significant amounts of it. However, malignant hematologic cells may contain four times more intracellular phosphate in comparison to normal mature lymphoid cells (3), making hyperphosphatemia a particular issue with tumor cell lysis. Because phosphorus excretion is tied to kidney function, hyperphosphatemia occurs when the kidney’s excretory capacity is overwhelmed. Thus, preexisting CKD or AKI enhances risk for hyperphosphatemia with TLS. Spontaneous tumor lysis, however, is less commonly associated with hyperphosphatemia and may be due to rapid uptake of extracellular phosphate by residual highly metabolically active tumor cells (5). Hyperphosphatemia may cause nausea, vomiting, diarrhea, or lethargy, but it exerts its predominant toxicity by binding to calcium cations. This results in secondary hypocalcemia and its downstream neuromuscular and cardiovascular effects such as cramps, hypotension, tetany, and arrhythmias. Additionally, calcium–phosphate precipitates may deposit in tissues, as seen in nephrocalcinosis, including the renal interstitium. AKI AKI in TLS may be either due to the aforementioned effects of acute urate nephropathy or hyperphosphatemic nephrocalcinosis affecting the renal tubulointerstitium or a combination of the two. Some studies have suggested that a urine uric acid to creatinine ratio of .1 may be specific to uric acid nephropathy (40), but another study has noted high uric acid to creatinine ratios in AKI from other etiologies (41). The association between AKI and TLS has been demonstrated across various populations and tumor subtypes (5). Annemans et al. (27) found that in patients with leukemia and NHL who had TLS, 45% had AKI. A smaller pediatric cohort of B-cell NHL or ALL noted renal insufficiency in 20% percent of the study 4 Onco-Nephrology Curriculum MANAGEMENT Prophylaxis and monitoring Prevention of TLS begins with recognition of risk factors and close laboratory and clinical monitoring. Patients at highest risk of developing TLS (Table 2) require intensified monitoring with more frequent electrolyte checks. Patients with high-risk disease may be prone to lactic acidosis from massive tumor cell necrosis. Because acidosis inhibits uric acid excretion (43), prompt recognition and correct of acidosis may prevent or ameliorate uric acid nephropathy. Additionally, nonsteroidal anti-inflammatory drugs, iodinated radiocontrast dye, and other potentially nephrotoxic therapeutic agents should be avoided to abrogate the risk of AKI from TLS. Volume expansion Delivery of crystalloid intravenous fluids (IVFs) is recommended for all patients and is essential for those with higher TLS risk. Volume expansion supports adequate intravascular volume and renal blood flow, which maintain glomerular filtration. This is the cornerstone of uric acid, potassium, and phosphate excretion and may delay and prevent the need for renal replacement measures (2,6,44). High-dose IVFs up to 3 L have been recommended (2), for a target urine output of $2 mL/kg/h. Diuretics may be necessary if patients develop volume overload, but routine use is not recommended to avoid volume depletion. Urinary alkalinization Alkalinization makes physiologic sense, as increasing urine pH from 5 to 7 can increase the solubility of uric acid .10-fold (28). However, urinary alkalinization decreases calcium– phosphate solubility (2), thereby exacerbating its precipitation American Society of Nephrology and deposition. Furthermore, if urinary alkalinization results in rising serum pH, free calcium may bind albumin more avidly and further exacerbate hypocalcemia (45). Thus, urinary alkalinization is not recommended in the management of TLS (2,6,45). Allopurinol Allopurinol is converted in vivo to oxypurinol and as a xanthine analog acts as a competitive inhibitor of xanthine oxidase and blocks the conversion of purines to uric acid (6,46) (Figure 1). This prevents hyperuricemia but does not treat preexisting hyperuricemia (6). Furthermore, because oxypurinol also inhibits the conversion of xanthine to uric acid, serum and urine xanthine levels may rise and precipitate xanthine crystal deposition in the renal tubules and xanthine-induced obstructive nephropathy (47). Administration of allopurinol is recommended for prophylaxis in patients with low and intermediate risk of developing TLS (2,6). Smalley et al. (48) studied 1,172 patients to evaluate the efficacy and safety of intravenous allopurinol in patients with hyperuricemia. They noted reduced uric acid levels in 57% of adults and 88% of children. When used as prophylactic therapy, allopurinol prevented an increase in uric acid levels in 93% of adults and 92% of children. Because oxypurinol excretion is by the kidney, dose adjustments are necessary for patients with CKD and AKI. Allopurinol has been associated with a hypersensitivity syndrome with rash, acute hepatitis, and eosinophilia (45,49). Allopurinol reduces the clearance of purine-based chemotherapeutic agents such as 6-mercaptopurine and azathioprine (6). It may also interact with azathioprine and cyclophosphamide in potentiating severe bone marrow suppression (6,45). Febuxostat Febuxostat is a novel xanthine oxidase inhibitor lacking the hypersensitivity profile of allopurinol. Because it is metabolized to inactive metabolites by the liver, adjustment for reduced GFR is not necessary. It has been proposed as a viable alternative to allopurinol in TLS prophylaxis for patients with allopurinol hypersensitivity or renal dysfunction (45). A recently completed phase III study of febuxostat versus allopurinol in TLS prevention found significantly lower serum uric acid in the febuxostat but found no significant difference in serum creatinine change compared with allopurinol (50). Febuxostat use has been limited by its significant cost compared with generically available allopurinol (45). to FDA approval, 1,069 adult and pediatric patients received rasburicase on a compassionate use basis (53). Decreased serum uric acid levels were observed in 99% of children and 100% of adults. Hemodialysis was performed in only 2.8% of patients. In a study of 131 patients with newly diagnosed leukemia or lymphoma, Pui et al. (54) reported a decrease in plasma uric acid concentrations from 9.7 to 1 mg/dL (P 5 0.0001) in 65 patients who presented with hyperuricemia and a decrease from 4.3 to 0.5 mg/dL (P 5 0.0001) in the remaining patients. There was negligible toxicity, and no patients required dialysis. Cortes et al. (55) compared response rates in dosing rasburicase alone versus rasburicase followed by allopurinol versus allopurinol alone. They reported a plasma uric acid response rate of 87% in the rasburicase group, 78% in the rasburicase followed by allopurinol group, and 66% in the allopurinol group, with a significantly greater response for rasburicase compared with allopurinol in the overall study population (P 5 0.001), in patients at high risk for TLS (89% versus 68%; P 5 0.012), and in those with baseline hyperuricemia (90% versus 53%; P 5 0.015). Of note, there are no prospective studies to date that have examined the impact of rasburicase on relevant clinical end points such as morbidity from AKI. Nonetheless, rasburicase should be used for prophylaxis in patients with high risk of developing TLS (7). The FDA-approved dosing guidelines recommend 0.2 mg/kg in 50 mL normal saline as a 30-minute intravenous infusion once daily for up to 5 days (51). Length of treatment is related to control of plasma uric acid levels, but use of rasburicase for .5 days is rarely needed (6,51). In comparison with generically available allopurinol, rasburicase is significantly more expensive (up to $3,600 per 7.5-mg vial) (45), and in most published studies, one-time dosing was sufficient to suppress hyperuricemia. Rasburicase does not require dosing adjustment for GFR and is not known to have any known clinically relevant drug– drug interactions (51,56). Adverse reactions are rare but may include rash, increased liver enzyme levels, headaches, fever, vomiting, and nausea (56). Rasburicase is active ex vivo, so blood samples for serum uric acid levels must be stored on ice to avoid erroneously low results (45). Patients with glucose 6-phosphate dehydrogenase (G6PD) deficiency can develop significant methemoglobinemia and hemolysis due to oxidative stress triggered by hydrogen peroxide (57,58). Accordingly, patients should have G6PD status tested prior to starting rasburicase. RRT Rasburicase Rasburicase (Elitek) is an Aspergillus-derived recombinant urate oxidase approved by the US Food and Drug Administration (FDA) in 2002 for the initial management of hyperuricemia in pediatric patients with leukemia, lymphoma, and solid tumor malignancies receiving anticancer therapy (51). It was subsequently approved for use in adults in 2009 (51). Rasburicase catalyzes the conversion of uric acid to allantoin, carbon dioxide, and hydrogen peroxide (Figure 1). Allantoin is 5- to 10-fold more soluble than uric acid (52) and is readily excreted. Prior American Society of Nephrology The need for renal replacement has significantly reduced since the advent of rasburicase, but about 1.5% of children and 5% of adults require dialysis during induction therapies (53). Indications for RRT are similar to those for AKI from other causes, but due to the rapid onset of the clinical manifestations of TLS, the threshold for initiating dialytic therapies is lower than in other situations. Although intermittent hemodialysis (IHD) may be sufficient for most patients, continuous RRT (CRRT) at high dialysate or replacement fluid flow rates (.3-4 L/h) may be necessary in those patients with severe TLS who Onco-Nephrology Curriculum 5 experience rebound in serum potassium and phosphorous levels with IHD (45,59,60). PROGNOSIS There are many confounding factors that impact clinical outcomes in patients with malignancies, particularly in those who have TLS, but AKI appears to be a significant predictor of short- and long-term mortality from TLS. A study comparing hematologic cancer patients without AKI to patients with AKI (61) showed significantly lower hospital mortality (7% and 21%, respectively) and 6-month mortality (51% and 66%, respectively) in patients without AKI. TLS is most common during initial presentation of disease because relapsed malignancies are significantly more chemoresistant (5). There are fewer case reports of TLS in recurrent disease (62). CONCLUSIONS TLS is a common oncologic emergency that requires immediate diagnosis and prompt treatment to avoid morbidity and mortality. Understanding the diagnostic criteria for TLS, knowing the tumor types at high risk for TLS, and instituting prophylactic and treatment measures are essential for the nephrologist who treats patients with malignant diseases. TAKE HOME POINTS c TLS is the most common oncologic emergency. c The risk of TLS depends on tumor type but is also influenced by other factors. c There is a high burden of AKI in patients with TLS. c Prophylaxis with volume expansion is the mainstay of preventing TLS in any patient-risk category. c Patients at high risk for TLS should receive rasburicase for initial treat- ment of hyperuricemia. ACKNOWLEDGMENTS Dr. Edeani’s work is supported by the intramural Research Program of the National Institute of Diabetes and Digestive and Kidney Diseases, National Institutes of Health. REFERENCES 1. Cairo MS, Bishop M. Tumor lysis syndrome: New therapeutic strategies and classification. Br J Haematol 127: 3–11, 2004 2. Howard SC, Jones DP, Pui C. The tumor lysis syndrome. N Engl J Med 364: 1844–1854, 2011 3. Flombaum CD. Metabolic emergencies in the cancer patient. Semin Oncol 27: 322–334, 2000 4. Hande KR, Garrow GC. Acute tumor lysis syndrome in patients with high-grade non-Hodgkin’s lymphoma. Am J Med 94: 133–139, 1993 6 Onco-Nephrology Curriculum 5. Perry Wilson F, Berns JS. Onco-nephrology: Tumor lysis syndrome. Clin J Am Soc Nephrol 7: 1730–1739, 2012 6. Coiffier B, Altman A, Pui C, Younes A, Cairo MS. Guidelines for the Management of Pediatric and Adult Tumor Lysis Syndrome: An evidence-based review. J Clin Oncol 26: 2767–2778, 2008 7. Cairo MS, Coiffier B, Reiter A, Younes A, Baruchel A, Bosly A, Goldman SC, Leverger G, Ohyashiki K, Panagiotidis P, Pession A, Pui CH, Ribera JM, Rosti G, Rule S, Tsukimoto I, Zinzani PL. Recommendations for the evaluation of risk and prophylaxis of tumour lysis syndrome (TLS) in adults and children with malignant diseases: An expert TLS panel consensus. Br J Haematol 149: 578–586, 2010 8. Baeksgaard L, Sørenson JB. Acute tumor lysis syndrome in solid tumors: A case report and review of the literature. Cancer Chemother Pharmacol 51: 187–192, 2003 9. Jensen M, Winkler U, Manzke O, Diehl V, Engert A. Rapid tumor lysis in a patient with B-cell chronic lymphocytic leukemia and lymphocytosis treated with an anti-CD20 monoclonal antibody (IDEC-C2B8, rituximab). Ann Hematol 77: 89–91, 1998 10. Cech P, Block JB, Cone LA, Stone A. Tumor lysis syndrome after tamoxifen flare. N Engl J Med 315: 263–264, 1986 11. Al-Kali A, Farooq S, Tfayli A. Tumor lysis syndrome after starting treatment with Gleevec in a patient with chronic myelogenous leukemia. J Clin Pharm Ther 34: 607–610, 2009 12. Sezer O, Vesole DH, Singhal S, Richardson P, Stadtmauer E, Jakob C, Boral AL, Esseltine DL, Mehta J. Bortezomib-induced tumor lysis syndrome in multiple myeloma. Clin Lymphoma Myeloma 7: 233–235, 2006 13. Fuente N, Mane JM, Barcelo R, Muñoz A, Perez-Hoyos T, LopezVivanco G. Tumor lysis syndrome in a multiple myeloma patient treated with thalidomide. Ann Oncol 15: 537, 2004 14. Mirrakhimov AE, Ali AM, Khan MK, Barbaryan A. Tumor lysis syndrome in solid tumors: An up to date review of the literature. Rare Tumors 6: 68–76, 2014 15. Gemici C. Tumour lysis syndrome in solid tumors. Clin Oncol 18: 773– 780, 2006 16. Lerza R, Botta M, Barsotti B, Schenone E, Mencoboni M, Bogliolo G, Pannacciulli I, Arboscello E. Dexamethasone-induced acute tumor lysis syndrome in a T-cell malignant lymphoma. Leuk Lymphoma 43: 1129– 1132, 2002 17. Tiley C, Grimwade D, Findlay M, Treleaven J, Height S, Catalano J, Powles R. Tumour lysis following hydrocortisone prior to a blood product transfusion in T-cell acute lymphoblastic leukemia. Leuk Lymphoma 8: 143–146, 1992 18. Jabr F. Acute tumor lysis syndrome induced by rituximab in diffuse large B-cell lymphoma. Int J Hematol 82: 312–314, 2005 19. Simmons ED, Somberg KA. Acute tumor lysis syndrome after intrathecal methotrexate administration. Cancer 67: 2062–2065, 1991 20. Lee CC, Wu YH, Chung SH, et al. Acute tumor lysis syndrome after thalidomide therapy in advanced hepatocellular carcinoma. Oncologist 11: 87–88, 2006 21. Linck D, Basara N, Tran V, Chen WJ. Peracute onset of severe tumor lysis syndrome immediately after 4 Gy fractionated TBI as part of reduced intensity preparative regimen in a patient with T-ALL with high tumor burden. Bone Marrow Transplant. 31: 935–937, 2003 22. Hsieh PM, Hung KC, Chen YS. Tumor lysis syndrome after transarterial chemoembolization of hepatocellular carcinoma: Case reports and literature review. World J Gastroenterol 15: 4726–4728, 2009 23. Cohen LF, Balow JE, Magrath IT, Poplack DG, Zeigler JL. Acute tumor lysis syndrome: A review of 37 patients with Burkitt’s Lymphoma. Am J Med 68: 486–491, 1980 24. Iversen U, Iversen OH, Bluming AZ, Zeigler JL, Kyalwasi S. Cell kinetics of African cases of Burkitt lymphoma. A preliminary report. Eur J Cancer 8: 305–308, 1972 25. Jasek AM, Day HJ. Acute spontaneous tumor lysis syndrome. Am J Hematol 47: 129–131, 1994 26. Mato AR, Riccio BE, Qin L, Heitjan DF, Carroll M, Loren A, Porter DL, Perl A, Stadtmauer E, Tsai D, Gewirtz A, Luger SM. A predictive model American Society of Nephrology 27. 28. 29. 30. 31. 32. 33. 34. 35. 36. 37. 38. 39. 40. 41. 42. for the detection of tumor lysis syndrome during AML induction therapy. Leuk Lymphoma 47: 877–883, 2006 Annemans L, Moeremans K, Lamotte M, Garcia Conde J, van den Berg H, Myint H, Pieters R, Uyttebroeck A. Incidence, medical resource utilization and costs of hyperuricemia and tumour lysis syndrome in patients with acute leukemia and non-Hogkin’s lymphoma in four European countries. Leuk Lymphoma 44: 77–83, 2003 Wössman W, Schrappe M, Meyer U, Zimmermann M, Reiter A. Incidence of tumor lysis syndrome in children with advanced stage Burkitt’s lymphoma/leukemia before and after introduction of prophylactic use of urate oxidase. Ann Hematol 82: 160–165, 2003 Tosi P, Barosi G, Lazzaro C, Lis V, Marchetti M, Morra E, Pession A, Rosti G, Santoro A, Zinzani PL, Tura S. Consensus conference on the management of tumor lysis syndrome. Haematologica 93: 1877– 1885, 2008 Montesinos P, Lorenzo I, Martín G, Sanz J, Pérez-Sirvent ML, Martínez D, Ortí G, Algarra L, Martínez J, Moscardó F, de la Rubia J, Jarque I, Sanz G, Sanz MA. Tumor lysis syndrome in patients with acute myeloid leukemia: Identification of risk factors and development of a predictive model. Haematologica 93: 67–74, 2008 Kushner BH, LaQuaglia MP, Modak S, Cheung NK. Tumor lysis syndrome, neuroblastoma, and correlation between lactate dehydrogenase levels and MYCN-amplification. Med Pediatr Oncol 41: 80–82, 2003 Fenaux P, Lai JL, Miaux O, Zandecki M, Jouet JP, Bauters F. Burkitt cell acute leukemia (L3 ALL) in adults: A report of 18 cases. Br J Haematol 71: 371–376, 1989 Seftel MD, Bruyere H, Copland M, Hogge DE, Horsman DE, Nantel SH, Shepherd JD, Lavoie JC, Le A, Sutherland HJ, Toze CL, Nevill TJ. Fulminant tumour lysis syndrome in acute myelogenous leukemia with inv(16)(p13;q22). Eur J Haematol 69: 193–199, 2002 Enomoto A, Kimura H, Chairoungdua A, Shigeta Y, Jutabha P, Cha SH, Takeda M, Sekine T, Igarashi T, Matsuo H, Kikuchi Y, Oda T, Ichida K, Hosoya T, Shimokata K, Niwa T, Kanai Y, Endou H. Molecular identification of a renal urate-anion exchanger that regulates blood urate levels. Nature 417: 447–452, 2002 Umekawa T, Chegini N, Khan SR. Increased expression of monocyte chemoattractant protein-1 (MCP-1) by renal epithelial cells in culture on exposure to calcium oxalate, phosphate and uric acid crystals. Nephrol Dial Transplant 18: 664–669, 2003 Kim YG, Huang XR, Suga S, Mazzali M, Tang D, Metz C, Bucala R, Kivlighn S, Johnson RJ, Lan HY. Involvement of macrophage migration inhibitory factor (MIF) in experimental uric acid nephropathy. Mol Med 6: 837–848, 2000 Kang DH, Park SK, Lee IK, Johnson RJ. Uric acid-induced C-reactive protein expression: Implication on cell proliferation and nitric acid production of human vascular cells. J Am Soc Nephrol 16: 3553–3562, 2005 Han HJ, Lim MJ, Lee YJ, Lee JH, Yang IS, Taub M. Uric acid inhibits renal proximal tubule cell proliferation via at least two signaling pathways involving PKC, MAPK, cPLA2 and NF-kappaB. Am J Physiol Renal Physiol 292: F373–F381, 2007 Feig DI, Kang DH, Johnson RJ. Uric acid and cardiovascular risk. N Engl J Med 359: 1811–1821, 2008 Davidson MB, Thakkar S, Hix JK, Bhandarkar ND, Wong A, Schreiber MJ. Pathophysiology, clinical consequences, and treatment of tumor lysis syndrome. Am J Med 116: 546–554, 2004 Tungsanga K, Boonwichit D, Lekhakula A, Setprija V. Urine uric acid and urine creatinine ratio in acute renal failure. Arch Intern Med 144: 934– 937, 1984 Cairo MS, Gerrard M, Sposto R, Auperin A, Pinkerton CR, Michon J, Weston C, Perkins SL, Raphael M, McCarthy K, Patte C; FAB LMB96 International Study Committee. Results of a randomized international study of high-risk central nervous system B non-Hodgkin lymphoma and B acute lymphoblastic leukemia in children and adolescents. Blood 109: 2736–2743, 2007 American Society of Nephrology 43. Lieber C, Jones D, Losowsky M, Davidson CS. Interrelation of uric acid and ethanol metabolism in man. J Clin Invest 41: 1863–1870, 1962 44. Jones DP, Mahmoud H, Chesney W. Tumor lysis syndrome: Pathogenesis and management. Pediatr Nephrol 9: 206–212, 1995 45. Perry Wilson F, Berns JS. Tumor lysis syndrome: New challenges and recent advances. Adv Chronic Kidney Dis 21: 18–26, 2014 46. Krakoff IH, Meyer RL. Prevention of hyperuricemia in leukemia and lymphoma: Use of allopurinol, a xanthine oxidase inhibitor. JAMA 193: 1–6, 1965 47. LaRosa C, McMullen L, Bakdash S, Ellis D, Krishnamurti L, Wu HY, Moritz ML. Acute renal failure from xanthine nephropathy during management of acute leukemia. Pediatr Nephrol 22: 132–135, 2007 48. Smalley RV, Guaspari A, Haase-Statz S, Anderson SA, Cederberg D, Hohneker JA. Allopurinol: Intravenous use for prevention and treatment of hyperuricemia. J Clin Oncol 18: 1758–1763, 2000 49. Arellano F, Sacristan JA. Allopurinol hypersensitivity syndrome: A review. Ann Pharmacother 27: 337–343, 1993 50. Spina M, Nagy Z, Ribera JM, Spina M, Nagy Z, Ribera FM, Federico M, Aurer I, Jordan K, Borsaru G, Pristupa AS, Bosi A, Grosicki S, Glushko NL, Ristic D, Jakucs J, Montesinos P, Mayer J, Rego EM, Baldini S, Scartoni S, Capriati A, Maggi CA, Simonelli C. A randomized doubleblind phase III pivotal study of febuxostat versus allopurinol in the prevention of tumor lysis syndrome: Florence study. J Clin Oncol 32: 5s, 2014 51. Elitek (Rasburicase) Package Label. Available at: http://www.accessdata. fda.gov/drugsatfda_docs/label/2009/103946s5083lbl.pdf. Accessed April 30, 2015 52. Brogard JM, Coumaros D, Franckhauser J, Stahl A, Stahl J. Enzymatic uricolysis: A study of the effect of a fungal urate-oxydase. Rev Eur Etudes Clin Biol 17: 890–895, 1972 53. Jeha S, Kantarjian H, Irwin D, Shen V, Shenoy S, Blaney S, Camitta B, Pui CH. Efficacy and safety of rasburicase (ElitekTM), in the management of a malignancy-associated hyperuricemia in pediatric and adult patients: Final results of a multicenter compassionate use trial. Leukemia 19: 34–38, 2005 54. Pui C, Mahmoud HH, Wiley JM, Woods GM, Leverger G, Camitta B, Hastings C, Blaney SM, Relling MV, Reaman GH. Recombinant urate oxidase for the prophylaxis or treatment of hyperuricemia in patients with leukemia or lymphoma. J Clin Oncol 19: 697–704, 2001 55. Cortes J, Moore JO, Maziarz RT, Wetzler M, Craig M, Matous J, Luger S, Dey BR, Schiller GJ, Pham D, Abboud CN, Krishnamurthy M, Brown A Jr, Laadem A, Seiter K. Control of plasma uric acid in adults at risk for tumor lysis syndrome: Efficacy and safety of rasburicase alone and rasburicase followed by allopurinol compared with allopurinol alone—Results of a multicenter phase III study. J Clin Oncol 28: 4207–4213, 2010 56. Sood AR, Burry LD, Cheng DKF. Clarifying the role of rasburicase in tumor lysis syndrome. Pharmacotherapy 27: 111–121, 2007 57. Sonbol MS, Yadav H, Vaidya R, Rana V, Witzig TE. Methemoglobinemia and hemolysis in a patient with G6PD deficiency treated with rasburicase. Am J Hematol 88: 152–154, 2013 58. Bontant T, Le Garrac S, Avran D, Dauger S. Methaemoglobinaemia in a G6PD-deficient child treated with rasburicase. BMJ Case Rep 2014 59. Agha-Razii M, Amyot SL, Pichette V, Cardinal J, Ouimet D, Leblanc M. Continuous veno-venous hemodiafiltration for the treatment of spontaneous tumor lysis syndrome complicated by acute renal failure and severe hyperuricemia. Clin Nephrol 54: 59–63, 2000 60. Sakarcan A, Quigley R. Hyperphosphatemia in tumor lysis syndrome: The role of hemodialysis and continuous veno-venous hemofiltration. Pediatr Nephrol 3: 351–353, 1994 61. Darmon M, Guichard I, Vincent F, Schlemmer B, Azoulay E. Prognostic significance of acute renal injury in acute tumor lysis syndrome. Leuk Lymphoma 51: 221–227, 2010 62. Hummel M, Buchheidt D, Reiter S, Bergmann J, Adam K, Hehlmann R. Recurrent chemotherapy-induced tumor lysis lysis syndrome (TLS) with renal failure in a patient with chronic lymphocytic leukemia: Successful treatment and prevention of TLS with low-dose rasburicase. Eur J Haematol 75: 518–521, 2005 Onco-Nephrology Curriculum 7 REVIEW QUESTIONS 1. Which of the following cancers are considered high risk for tumor lysis syndrome? a. b. c. d. e. Lung cancer Lung cancer and patient has AKI Burkitt-type lymphoma, advanced stage Adult T-cell lymphoma and normal LDH ALL with WBC ,100 3 109/L and LDH ,2 3 ULN Answer: c is correct. As shown in Table 2, the risk of TLS depends on type of malignancy, stage or extent of disease, and presence/absence of renal disease. Burkitt-type lymphoma that is in an advanced stage confers a high risk of TLS. Solid tumors such as lung cancer are considered low risk, and the presence of renal failure raises that to intermediate risk. Thus, answers a and b are incorrect. Adult T-cell lymphoma is considered low risk if LDH is normal and acute lymphoblastic leukemia with WBC ,100 3 109/L is considered intermediate risk if LDH ,2 3 ULN. 2. Which of the following electrolyte abnormalities define laboratory TLS? a. Hypokalemia b. Hypercalcemia c. Hypophosphatemia 8 Onco-Nephrology Curriculum d. Hypernatremia e. Hypocalcemia Answer: e is correct. As show in Table 2, laboratory TLS is defined by two or more abnormalities in serum electrolytes. These include a 25% increase from baseline in phosphorous, potassium, or uric acid or a 25% decrease from baseline in calcium. Thus, answers a, b, and c are incorrect. Serum sodium concentration is not directly affected in TLS; therefore, answer d is incorrect. 3. Rasburicase is part of the treatment regimen for tumor lysis syndrome because a. It increases urinary alkalinization b. It improves the ability of proximal tubular cells to recover from AKI c. It stimulates the URAT1 transporter to increase uptake of uric acid from the tubular lumen d. It catalyzes the conversion of uric acid into allantoin e. It prevents xanthine crystal deposition in tubular lumens Answer: d is correct. Rasburicase, as shown in Figure 3, is recombinant urate oxidase that enzymatically transforms uric acid into allantoin. It has no known effect on urine pH, renal tubular cells, URAT1 transporters, or xanthine crystals. Thus, answers a–c and e are incorrect. American Society of Nephrology Chapter 5: Electrolyte and Acid–Base Disorders in Malignancy Anushree C. Shirali, MD Section of Nephrology, Yale University School of Medicine, New Haven, Connecticut INTRODUCTION Renal complications in cancer patients include AKI, hypertension, or electrolyte and acid–base disorders. Of the latter, there are various types that share the ability to increase morbidity and mortality, delay treatment, and decrease quality of life. Understanding the etiology of electrolyte and acid–base abnormalities in cancer patients is critical to prompt recognition and appropriate treatment so that these complications may be avoided. This section of the Onco-Nephrology Curriculum will review the pathophysiology, clinical presentation, and management of electrolyte and acid–base abnormalities in patients with malignancies. Specifically, disturbances in the following will be reviewed: 1) electrolytes: disorders of sodium, potassium, calcium, magnesium, and phosphorous; and 2) acid–base: metabolic acidosis. BACKGROUND, EPIDEMIOLOGY, AND SPECIFIC DISORDERS Electrolyte and acid–base disturbances are common in cancer patients, either due to the malignancy or treatment of the malignancy. For example, a patient may develop metabolic acidosis from lactate produced by disseminated lymphoma or from chemotherapy-induced diarrhea. Published statistics are not robust for each type of electrolyte or acid–base disorder, but there are data on those associated with greater morbidity or mortality, such as hyponatremia. In one such analysis of cancer-related admissions, 47% of patients with mostly solid tumors had hyponatremia, and of these, 11% had moderate (sodium [Na] 120–129 mEq/L) to severe (Na , 120 mEq/L) hyponatremia (1). This is disproportionately higher than the hyponatremia prevalence rates of 15%–30% reported for general medicine admissions (2). In cancer patients with nonsolid tumors, rates of hyponatremia are less. For example, in acute American Society of Nephrology leukemia, the prevalence of hyponatremia is only 10%, whereas the prevalence of hypokalemia ranges between 43% and 64% (3). This suggests that differences in pathophysiologic mechanisms may drive unique electrolyte disorders in different malignancies. In the next sections, the clinical features, pathophysiology, and treatment of the most common electrolyte and acid–base disorders in cancer patients will be considered. Hyponatremia Cancer is a common etiology for hyponatremia in the hospitalized patient, accounting for 14% of cases in a prospective observational cohort (4). Similar to reports on hyponatremia in the general population, lower serum sodium concentration is associated with increased hospital length of stay and 90-day mortality (1). In patients with small-cell lung cancer (SCLC), in those who had hyponatremia prior to chemotherapy initiation, failure to achieve normonatremia within the first two cycles of chemotherapy was a predictive marker for decreased survival (5). Hyponatremia associated with cancer may have several potential etiologies (Table 1). Regardless of the etiology, patients may be asymptomatic with mild to moderate disease but may experience headache, fatigue, and mental status changes with moderate to severe hyponatremia. Examination findings such as frank or orthostatic hypotension in volume depletion or edema in the third-spacing states of cirrhosis may point to potential causes. In conjunction with examination data, urine studies are indispensable, with urine sodium ,20 mEq/L reflecting the sodium avidity of volume depletion and urine sodium .40 mEq/L suggesting the syndrome of inappropriate antidiuretic Correspondence: Anushree Shirali, Section of Nephrology, Yale University School of Medicine, PO Box 208029, New Haven, Connecticut 06520-8029. Copyright © 2016 by the American Society of Nephrology Onco-Nephrology Curriculum 1 Table 1. Common mechanisms for hyponatremia in the cancer patient Etiology of hyponatremia Pseudohyponatremia Reduced water excretion Decreased circulating volume Decreased effective circulating volume SIADH Nonosmotic stimuli for ADH Salt wasting Clinical examples specific to the cancer patient Paraproteinemias Underlying CKD or AKI Nausea, vomiting, nasogastric suctioning, and diarrhea; hematemesis or hematochezia (gastrointestinal malignancies or steroid-induced ulcer disease) Underlying or new onset of CHF, cirrhosis, ascites, severe hypoalbuminemia, veno-occlusive disease Tumor release of ADH: SCLC and head and neck cancer Chemotherapy: cyclophosphamide, cisplatin/carboplatin, vincristine, vinblastine Other drugs: SSRIs, NSAIDs Pain, nausea, vomiting Cisplatin ADH, antidiuretic hormone; CHF, congestive heart failure; SIADH, syndrome of inappropriate ADH secretion; NSAIDs, nonsteroidal anti-inflammatory drugs; SCLC, small cell lung cancer; SSRI, selective serotonin reuptake inhibitors. hormone secretion (SIADH) in a euvolemic patient or rarely salt wasting due to cisplatin therapy. Although patients with cancer have hyponatremia due to many etiologies (Table 1), SIADH is the most common etiology that is directly attributable to cancer. This is because cancer patients have nonvolume and nonosmotic stimuli for antidiuretic hormone (ADH) release, such as nausea/vomiting and pain or medications such as cyclosphosphamide. An additional factor is the paraneoplastic release of ADH from tumor subtypes, notably SCLC and head and neck cancer (6). Finally, several chemotherapy drugs have been linked to hyponatremia via potentiation of ADH release or action, including vinblastine, vincristine, and cyclophosphamide (7). Cisplatin is another antineoplastic agent linked to hyponatremia through a mechanism that involves salt wasting at the loop of Henle (8). Ten percent of patients on cisplatin therapy at a single center developed hyponatremia with a high urine sodium concentration but with profound volume depletion. Thus, these patients required volume expansion with saline to correct their hyponatremia, in contrast to patients with SIADH who are volume replete. This underscores the need for correct etiologic diagnosis of hyponatremia to provide appropriate therapy. Therapy must be tailored for the patient, the underlying diagnosis, and the severity of hyponatremia. Severe hyponatremia with serum Na concentration ,110 mEq/L and neurologic symptoms may need 3% hypertonic saline for acute management. Fluid restriction is the mainstay of treatment for SIADH, with salt tablets and loop diuretics as adjunctive therapy. However, these measures can hinder quality of life in the cancer patient, and thus, aquaretics that inhibit the vasopressin type 2 receptor (V2-R) to inhibit water reabsorption in the collecting duct are suggested for management of hyponatremia secondary to SIADH in cancer patients if other therapies are not feasible or effective. However, the data on their clinical utility in cancer patients are sparse. In a small, single center safety and efficacy study, tolvaptan, a V2-R antagonist, was superior to placebo in the correction of hyponatremia but did not decrease hospital length of stay (LOS) 2 Onco-Nephrology Curriculum or improve cognitive testing (9). In addition, chronic tolvaptan use may be limited by expense and cumulative dose-dependent hepatotoxicity (10). Hypokalemia Similar to hyponatremia, hypokalemia is commonly encountered in cancer patients, resulting from cancer-distinct and cancer-specific causes (Table 2) and, more commonly, from a combination of the two. Proper diagnosis starts with excluding pseudohypokalemia from postphlebotomy transcellular shifts, which is seen in patients with profound leukocytosis whose blood samples are not refrigerated or immediately analyzed. Once true hypokalemia is confirmed, measurement of urine potassium and the trans-tubular potassium gradient can be helpful in analyzing renal potassium wasting (11). In cancer-distinct causes, chemotherapy leads to hypokalemia either indirectly via side effects of decreased appetite/ intake, vomiting, and diarrhea or directly via renal tubular effects. For example, ifosfamide causes renal potassium wasting, either as an isolated proximal tubulopathy or Fanconi syndrome (FS), which may persist after treatment. Fifteen percent of pediatric cancer patients who received ifosfamide therapy exhibited persistent hypokalemia months to years after the end of treatment (12). Cancer-specific causes of hypokalemia include tumors that secrete ectopic adrenocorticotropin hormone (ACTH) such as SCLC, thymus or bronchial carcinoid, thyroid medullary carcinoma, or neuroendocrine tumors (13). Although uncommon, these tumors stimulate renal potassium wasting via excessive cortisol release that activates the mineralocorticoid pathway. Accordingly, other features of hypercortisolemia are also present including pigmented skin, diabetes, and hypertension (13). Another cancer-specific etiology for hypokalemia is evident in acute myeloid leukemia (AML), M4 and M5 subtypes, which has been long associated with hypokalemia (14,15). These malignancies increase serum lysozyme and lysozymuria, leading to the hypothesis that lysozymemediated tubular injury leaks potassium (and other electrolytes) American Society of Nephrology Table 2. Cancer-distinct and cancer-specific causes of hypokalemia in the patient with malignancy Etiology of hyponatremia Pseuodohypokalemia Redistribution into cells Poor intake Extrarenal losses Renal losses Cancer distinct Cancer specific Phlebotomy error with tight tourniquet Use of GM-CSF, vitamin B12 Anorexia, nausea, mucositis Vomiting, diarrhea from chemotherapy or radiation enteritis Hypomagnesemia, Fanconi syndrome with chemotherapy Clonal leukocytosis with leukemias Blast crisis with leukemias Tumor-induced dysphagia VIPoma, villous adenoma (rare) Lysozymuria, Fanconi syndrome from light chain injury (myeloma), ectopic ACTH production ACTH, adrenocorticotropin hormone; GM-CSF, granulocyte macrophage colony stimulating factor; VIP, vasoactive intestinal peptide. into urine. Another putative mechanism may be renin-like activity by AML blast cells stimulating the mineralocorticoid pathway (16). The potassium losses in these cases may be profound and require aggressive replacement. The choices for replacement are the same as those utilized for hypokalemia in the noncancer patient (11), but it should be noted that given the difficulty cancer patients may have with oral intake due to nausea, mucositis, etc., intravenous dosing is often necessary. Hypokalemia treatment is also ineffective if hypomagnesemia remains uncorrected, due to unchecked potassium losses via the renal outer medullary K1 channel (ROMK) channel in distal nephron tubular cells (17). panitumumab, display tumoricidal activity against a variety of cancers, but they also prevent the insertion of a magnesium channel, transient receptor potential M6 (TRPM6), into the apical membrane of distal tubular cells (Figure 1) (18). As a result, magnesium cannot be reabsorbed from the tubular lumen and serum magnesium levels fall, affecting 10%–36% of patients in early clinical trials of cetuximab (7). A fractional excretion of magnesium .15% in a hypomagnesemic patient indicates renal wasting. Treatment involves replacing magnesium, and intravenous dosing is usually needed because diarrhea is a dose-limiting adverse effect of oral magnesium. Fortunately, renal magnesium wasting subsides over time following discontinuation of the EGFR antagonist. However, this is not the case with the platin drugs, where renal magnesium wasting can be permanent. Hypomagnesemia in cancer patients Hypomagnesemia in the cancer patient may be due to decreased intake or from renal magnesium wasting. Renal losses of magnesium are principally due to chemotherapy-mediated injury to the distal nephron, the site of active magnesium reabsorption in the nephron. This has been noted with cisplatin, but a rising number of cases are being attributed to drugs that target the epidermal growth factor receptor (EGFR) pathway. Monoclonal antibodies against EGFR, such as cetuximab and Hypercalcemia Twenty percent to 30% of cancer patients experience hypercalcemia during the course of their malignancy (19), and this is predictive of poor prognosis (20). Hypercalcemia of malignancy uses one of two mechanisms: 1) osteolytic release of local calcium from bone directly involved by cancer cells or 2) stimulation of osteoclast activity by release of the tumor-derived endocrine factors. Although these mechanisms are distinct, Figure 1. Absorption of magnesium from the tubular lumen is via an EGFR-dependent apical channel, TRPM6. This pathway is inhibited by use of anti-EGFR monoclonal antibodies such cetuximab. EGF, epidermal growth factor; EGFR, epidermal growth factor receptor; TRPM6, transient receptor potential M6. Red line denotes inhibition of interaction. American Society of Nephrology Onco-Nephrology Curriculum 3 the resultant hypercalcemia in either case may be mild and asymptomatic, moderate and accompanied by nausea/vomiting, constipation, bone pain, and fatigue, or severe and manifested by confusion and coma (21). It is important to correct serum calcium concentration for hypoalbuminemia so that hypercalcemia levels can be properly graded. Among solid tumors, primary bone cancers and metastatic breast or prostate cancer stimulate osteolysis, which correlates with the overall tumor burden. Although metastases do not occur in nonsolid tumors, osteolysis may be stimulated by a variety of immune and nonimmune pathways in multiple myeloma. Both result in release of sequestered calcium from bone with the common pathway centered on the interaction between receptor activator of nuclear factor-kB (RANK), which is present on osteoclasts and their precursors, and RANK ligand (RANKL), which is present on osteoblast and bone marrow stromal cell surfaces (22). The putative mechanism involves RANKL binding to its cognate receptor RANK through the influence of parathyroid hormone (PTH) and PTH-related peptide (PTHRP), which subsequently increases osteoclastic activity and release of local calcium (21). Tumor-derived endocrine factors are responsible for the humoral hypercalcemia of malignancy, including PTHRP and 1,25-dihydroxyvitamin D [1,25(OH)2D]. More rarely, there is PTH release from primary parathyroid carcinoma (23) or ovarian cancer (24). PTHRP is most commonly secreted by squamous cell carcinoma of the lung or head and neck, but renal cell, ovarian, breast, and esophageal cancers have all been associated with hypercalcemia from PTHRP release (21). 1,25(OH)2D, however, is more likely to be secreted by lymphoma cells or tumor-associated macrophages that possess inherent 1-a-hydroxylase activity that is not subject to regulation by PTH (25–27). Treatment of hypercalcemia of malignancy is focused on increasing urinary calcium excretion and suppression of the calcium source. The first objective is achieved by volume expansion with saline to drive urinary calcium excretion. Furosemide, once routinely touted as an adjunct to saline, has no proven benefit and should only be reserved for cases of volume overload (28). The second objective may be fulfilled by suppressing osteoclast activity through use of bisphosphonates such as zoledronate or pamidronate. The former causes acute renal tubular injury, and the latter has been linked to a collapsing variant of FSGS, and high intravenous (IV) dosing should be used with caution, particularly with preexisting CKD. An emerging option that directly targets a pathway in hypercalcemia of malignancy is the use of RANKL inhibitors such as denosumab. These agents have shown to be superior to bisphosphonates in the treatment of skeletal related events in cancers with bony metastases (29,30), and early evidence suggests they are useful in the treatment of hypercalcemia of malignancy, particularly in bisphosphonate-resistant cases (31). Hypophosphatemia in cancer patients Cancer patients usually experience hypophosphatemia as a consequence of chemotherapy. This may be due to generalized 4 Onco-Nephrology Curriculum malnutrition from anorexia or malnutrition causing poor intake, or it may be the result of renal phosphate wasting from druginduced proximal tubulopathy and FS. As mentioned previously, FS is common with ifosfamide, but has also been associated with cisplatin and imatinib use (32,33). A fractional excretion of phosphate that is .5% in the setting of hypophosphatemia is diagnostic of renal phosphate wasting. Treatment of hypophosphatemia centers on phosphate replacement, which may approach several grams per day in cases of renal phosphate wasting. A rare cause of hypophosphatemia is tumor-induced osteomalacia, whose mechanism is dependent on the phosphatonin, fibroblast growth factor 23 (FGF-23). The role of the FGF-23 pathway has been detailed previously (6,34). Briefly, FGF-23 is a phosphaturic agent whose expression is tightly regulated by phosphate, 1,25(OH)2D, and other factors. In tumor-induced osteomalacia, constitutive release of FGF-23 without usual regulatory checkpoints leads to persistent FGF-23 activation, resulting in severe phosphaturia, hypophosphatemia, and osteomalacia. Several malignancies are associated with this syndrome including hemangiopericytomas, giant cell tumors, and osteoblastomas (34). Definitive treatment is surgical resection, as the phosphate wasting may be so profound that medical management may not be sufficient. Thus, functional imaging such as F-18 fluorodeoxyglucose positron emission tomography is suggested for diagnosis, which has high sensitivity for these tumors but may not be specific (34). Metabolic acidosis in cancer patients Anion gap (AG) acidosis and non–anion gap (NAG) acidosis is prevalent in cancer patients. Among the various AG acidosis disorders, lactic acidosis (LA) is the most cancer specific. Cancer patients may have type A LA due to tissue hypoxia from sepsis or cardiac failure, but they may also have type B LA with no evidence for tissue ischemia. Type B LA is well described in leukemias and lymphomas (35), but other reported cases include multiple myeloma, gastric cancer, and breast cancer (36–38). The pathophysiology of malignancy-associated LA is unclear, but speculative mechanisms include anaerobic glycolysis by tumor cells, stimulation of lactate production by tumor-derived cytokines, and thiamine deficiency (36). Treatment involves control of tumor burden. Bicarbonate infusion may be necessary for critical drops in serum pH, but paradoxically may stimulate more lactate production. Dialysis is often requested for lactic acidosis, but clearance with either intermittent or continuous dialysis modalities is insufficient to overcome ongoing production. Non-AG acidosis in cancer patients is most likely related to infection or therapy-related diarrhea, but renal tubular acidosis (RTA) should be considered. Tubular injury from chemotherapy can cause RTA either in isolation or as part of the FS. Light chain–associated tubular injury in multiple myeloma is another cause of FS and can present with RTA. Bicarbonate supplementation is sometimes necessary in patients with RTAs, and its success depends on the degree of renal bicarbonate wasting. American Society of Nephrology Other disorders Cancer patients may have electrolyte and acid–base abnormalities beyond those reviewed in this chapter. In particular, hyperkalemia, hyperphosphatemia, and hypocalcemia are diagnostic criteria for tumor lysis syndrome, which is detailed in Chapter 4 of the ASN Onco-Nephrology Curriculum. In acid–base disorders, metabolic alkalosis may accompany the rare renin-producing tumor but is more common with persistent vomiting or diuretic use. Respiratory alkalosis may occur with pontine tumor or infection-associated stimulation of central respiratory centers. 10. 11. 12. 13. 14. CONCLUSION 15. A myriad of cancer and chemotherapy-related electrolyte and acid-base disorders can affect cancer patients. Although most patients develop mild disease, some patients may experience significant morbidity. Diagnosing electrolyte and acid–base abnormalities and initiating treatment quickly is essential for the nephrologist seeking to improve the outcomes of cancer patients. 16. 17. 18. TAKE HOME POINTS c Electrolyte and acid–base abnormalities occur frequently in the cancer pa- tient and contribute to poor quality of life. 19. c Disturbances in electrolyte and acid–base homeostasis may occur due to the cancer itself or due to adverse effects of therapy. 20. c Treatment of electrolyte and acid–base disorders in cancer should be etiology specific and patient centered. 21. REFERENCES Q:1 1. Doshi SM, Shah P, Lei X, Lahoti A, Salahudeen AK. Hyponatremia in hospitalized cancer patients and its impact on clinical outcomes. Am J Kidney Dis 59: 222–228, 2012 2. Verbalis JG, Goldsmith SR, Greenberg A, Korzelius C, Schrier RW, Sterns RH, Thompson CJ. Diagnosis, evaluation, and treatment of hyponatremia: expert panel recommendations. Am J Med 126[Suppl 1]: S1–S42, 2013 3. Filippatos TD, Milionis HJ, Elisaf MS. Alterations in electrolyte equilibrium in patients with acute leukemia. Eur J Haematol 75: 449–460, 2005 4. Gill G, Huda B, Boyd A, Skagen K, Wile D, Watson I, van Heyningen C. Characteristics and mortality of severe hyponatraemia–a hospitalbased study. Clin Endocrinol 65: 246–249, 2006 5. Hansen O, Sorensen P, Hansen KH. The occurrence of hyponatremia in SCLC and the influence on prognosis: a retrospective study of 453 patients treated in a single institution in a 10-year period. Lung cancer 68: 111–114, 2010 6. Rosner MH, Dalkin AC. Electrolyte disorders associated with cancer. Adv Chronic Kidney Dis 21: 7–17, 2014 7. Shirali AC, Perazella MA. Tubulointerstitial injury associated with chemotherapeutic agents. Adv Chronic Kidney Dis 21: 56–63, 2014 8. Hutchison FN, Perez EA, Gandara DR, Lawrence HJ, Kaysen GA. Renal salt wasting in patients treated with cisplatin. Ann Intern Med 108: 21–25, 1988 9. Salahudeen AK, Ali N, George M, Lahoti A, Palla S. Tolvaptan in hospitalized cancer patients with hyponatremia: a double-blind, randomized, American Society of Nephrology 22. 23. 24. 25. 26. 27. 28. placebo-controlled clinical trial on efficacy and safety. Cancer 120: 744–751, 2014 Administration USFaD. FDA Drug Safety Communication: FDA limits duration and usage of Samsca (tolvaptan) due to possible liver injury leading to organ transplant or death 2013. Available from: http://www. fda.gov/Drugs/DrugSafety/ucm350062.htm Unwin RJ, Luft FC, Shirley DG. Pathophysiology and management of hypokalemia: a clinical perspective. Nat Rev Nephrol 7: 75–84, 2011 Skinner R, Cotterill SJ, Stevens MC. Risk factors for nephrotoxicity after ifosfamide treatment in children: a UKCCSG Late Effects Group study. United Kingdom Children’s Cancer Study Group. Br J Cancer 82: 1636–1645, 2000 Alexandraki KI, Grossman AB. The ectopic ACTH syndrome. Rev Endocr Metab Disord 11: 117–126, 2010 Perazella MA, Eisen RN, Frederick WG, Brown E. Renal failure and severe hypokalemia associated with acute myelomonocytic leukemia. Am J Kidney Dis 22: 462–467, 1993 Muggia FM, Heinemann HO, Farhangi M, Osserman EF. Lysozymuria and renal tubular dysfunction in monocytic and myelomonocytic leukemia. Am J Med 47: 351–366, 1969 Wulf GG, Jahns-Streubel G, Strutz F, Basenau D, Hufner M, Buske C, Wormann B, Hiddemann W. Paraneoplastic hypokalemia in acute myeloid leukemia: a case of renin activity in AML blast cells. Ann Hematol 73: 139–141, 1996 Huang CL, Kuo E. Mechanism of hypokalemia in magnesium deficiency. JASN 18: 2649–2652, 2007 Groenestege WM, Thebault S, van der Wijst J, van den Berg D, Janssen R, Tejpar S, van den Heuvel LP, van Cutsem E, Hoenderop JG, Knoers NV, Bindels RJ. Impaired basolateral sorting of pro-EGF causes isolated recessive renal hypomagnesemia. J Clin Invest 117: 2260–2270, 2007 Stewart AF. Clinical practice. Hypercalcemia associated with cancer. N Engl J Med 352: 373–379, 2005 Ralston SH, Gallacher SJ, Patel U, Campbell J, Boyle IT. Cancerassociated hypercalcemia: morbidity and mortality. Clinical experience in 126 treated patients. Ann Intern Med 112: 499–504, 1990 Rosner MH, Dalkin AC. Onco-nephrology: the pathophysiology and treatment of malignancy-associated hypercalcemia. CJASN 7: 1722–1729, 2012 Papadopoulou EC, Batzios SP, Dimitriadou M, Perifanis V, Garipidou V. Multiple myeloma and bone disease: pathogenesis and current therapeutic approaches. Hippokratia 14: 76–81, 2010 Collins MT, Skarulis MC, Bilezikian JP, Silverberg SJ, Spiegel AM, Marx SJ. Treatment of hypercalcemia secondary to parathyroid carcinoma with a novel calcimimetic agent. J Clin Endocrinol Metab 83: 1083–1088, 1998 Nussbaum SR, Gaz RD, Arnold A. Hypercalcemia and ectopic secretion of parathyroid hormone by an ovarian carcinoma with rearrangement of the gene for parathyroid hormone. N Engl J Med 323: 1324–1328, 1990 Maletkovic J, Isorena JP, Palma Diaz MF, Korenman SG, Yeh MW. Multifactorial hypercalcemia and literature review on primary hyperparathyroidism associated with lymphoma. Case Rep Endocrinol 2014: 893134, 2014 Evans KN, Taylor H, Zehnder D, Kilby MD, Bulmer JN, Shah F, Adams JS, Hewison M. Increased expression of 25-hydroxyvitamin D-1alphahydroxylase in dysgerminomas: a novel form of humoral hypercalcemia of malignancy. Am J Pathol 165: 807–813, 2004 Hewison M, Kantorovich V, Liker HR, Van Herle AJ, Cohan P, Zehnder D, Adams JS. Vitamin D-mediated hypercalcemia in lymphoma: evidence for hormone production by tumor-adjacent macrophages. J Bone Miner Res 18: 579–582, 2003 LeGrand SB, Leskuski D, Zama I. Narrative review: furosemide for hypercalcemia: an unproven yet common practice. Ann Intern Med 149: 259–263, 2008 Onco-Nephrology Curriculum 5 29. Fizazi K, Carducci M, Smith M, Damiao R, Brown J, Karsh L, Milecki P, Shore N, Rader M, Wang H, Tadros S, Dansey R, Goessl C. Denosumab versus zoledronic acid for treatment of bone metastases in men with castration-resistant prostate cancer: a randomised, double-blind study. Lancet 377: 1813–822, 2011 30. Stopeck AT, Lipton A, Body JJ, Steger GG, Tonkin K, de Boer RH, Lichinitser M, Fujiwara Y, Yardley DA, Viniegra M, Fan M, Jiang Q, Dansey R, Jun S, Braun A. Denosumab compared with zoledronic acid for the treatment of bone metastases in patients with advanced breast cancer: a randomized, double-blind study. J Clin Oncol 28: 5132–5739, 2010 31. Hu MI, Glezerman I, Leboulleux S, Insogna K, Gucalp R, Misiorowski W, Yu B, Ying W, Jain RK. Denosumab for patients with persistent or relapsed hypercalcemia of malignancy despite recent bisphosphonate treatment. J Natl Cancer Inst 105: 1417–1420, 2013 32. Hall AM, Bass P, Unwin RJ. Drug-induced renal Fanconi syndrome. QJM 107: 261–269, 2014 6 Onco-Nephrology Curriculum 33. Francois H, Coppo P, Hayman JP, Fouqueray B, Mougenot B, Ronco P. Partial fanconi syndrome induced by imatinib therapy: a novel cause of urinary phosphate loss. Am J Kidney Dis 51: 298–301, 2008 34. Chong WH, Molinolo AA, Chen CC, Collins MT. Tumor-induced osteomalacia. Endocr Relat Cancer 18: R53–R77, 2011 35. Friedenberg AS, Brandoff DE, Schiffman FJ. Type B lactic acidosis as a severe metabolic complication in lymphoma and leukemia: a case series from a single institution and literature review. Medicine 86: 225–232, 2007 36. Sia P, Plumb TJ, Fillaus JA. Type B lactic acidosis associated with multiple myeloma. Am J Kidney Dis 62: 633–637, 2013 37. de Groot R, Sprenger RA, Imholz AL, Gerding MN. Type B lactic acidosis as a severe metabolic complication in lymphoma and leukemia: a case series from a single institution and literature review.nger RA, Imholz AL, Gerding MN. Type B lactic acidosis in solid malignancies. Neth J Med 69: 120–123, 2011 38. Hashemi-Sadraei N, Machicado JD, Gupta R, Huapaya JA. Lactic acidosis in gastric cancer. J Gastrointest Cancer 45[Suppl 1]: 192–194, 2014 American Society of Nephrology REVIEW QUESTIONS 1. Which of the following tumors is most likely to be associated with SIADH? a. b. c. d. e. Non–small-cell lung cancer Acute myeloid leukemia, M4 type Small-cell lung cancer Breast cancer Bronchial carcinoid Answer: c is correct. Hyponatremia in cancer patients from persistent ADH stimulation or potentiation may be cancer distinct, e.g., from pain, nausea, or chemotherapy. Certain cancers, however, are known to release ectopic ADH, including small cell lung cancer and head and neck cancer. Non– small cell lung cancer and breast cancer are not associated with SIADH from ectopic ADH release. AML-M4 is associated with lysozyme-mediated renal potassium wasting, whereas bronchial carcinoid secretes ectopic adrenocorticotropin hormone. 2. Which of the following laboratory abnormalities are seen in Fanconi syndrome? a. b. c. d. e. Hyperkalemia Hypomagnesemia Hypophosphatemia Metabolic alkalosis Hyperglycemia Answer: c is correct. Fanconi syndrome is a constellation of metabolic abnormalities, which result following proximal tubule injury. As the proximal tubule is the primary site for American Society of Nephrology reabsorption of bicarbonate, ammonia, amino acids, glucose, and most electrolytes including sodium, potassium, and phosphorus, this syndrome typically results in potassium, phosphorus, and bicarbonate wasting. Patients may exhibit hypokalemia, hypophosphatemia, metabolic acidosis due to renal tubular acidosis, and glucosuria. Because active magnesium reabsorption takes place at the distal tubule, magnesium levels are not affected. 3. Which of the following is true of the lactic acidosis of malignancy? a. Levels of lactic acid decrease with bicarbonate infusion b. Continuous RRT is recommended for control of lactic acid levels c. Measurement of the urinary anion gap can aid in diagnosis d. Control of tumor burden is not necessary in improving lactic acid levels e. It may be the result of anaerobic glycolysis and lactate production by tumor cells Answer: e is correct. Type B lactic acidosis is the production of lactic acid in the absence of ischemia. It is linked with certain malignancies, particularly lymphoma, and hypotheses for its pathophysiology include tumor-induced anaerobic glycolysis and lactate production. Lactic acidosis of malignancy correlates well with tumor burden, and levels improve with control of that burden. The urinary anion gap is not useful in the diagnosis of this disorder. Although acute treatment of acidosis may require bicarbonate, levels of lactic acid may rise with sustained bicarbonate infusion. Last, hemodialysis is an inefficient modality for lactic acid clearance, as production is higher than clearance rates, even in continuous RRT. Onco-Nephrology Curriculum 7 AUTHOR QUERIES AUTHOR PLEASE ANSWER ALL QUERIES Q1: Please verify corrections, especially to references. The end of the references were changed to match style, so please make sure the edits are correct. Chapter 6: Glomerular Diseases and Cancer Divya Monga* and Kenar D. Jhaveri† *Nephrology Division, University of Mississippi Medical Center, Jackson, Mississippi; and †Nephrology Division, Northwell Health, Hofstra Northwell School of Medicine, Great Neck, New York INTRODUCTION Glomerular diseases are associated with many solid and hematologic malignancies. Additionally, many chemotherapeutic agents and post–stem cell transplant– associated glomerular lesions have been described. These glomerular lesions are most likely due to abnormal products produced by tumor cells, although the exact pathogenesis is unclear. The treatment of these cancer-associated glomerulopathies is primarily targeted at treating the underlying malignancy. This chapter will review glomerular diseases associated with cancer, chemotherapy, and hematopoietic stem cell transplantation (HSCT). SOLID TUMOR–ASSOCIATED GLOMERULAR DISEASES Membranous nephropathy Membranous nephropathy (MN) is the most common glomerular pathology (Figure 1, A and B) described in patients with solid tumors (1,2). In a review of 240 patients with biopsy proven MN, Lefaucheur et al. (3) reported a prevalence of malignancy of 10%. Only about half of these patients had symptoms related to cancer at the time of their kidney biopsy. Also, most of these patients were diagnosed with malignancy within a year of MN diagnoses. Review of case series shows a reported prevalence of as low as 1% to as high as 22% (2). Classically, the solid tumors most commonly associated with MN are lung, bronchus, and gastric cancers, followed by renal cell, prostate, and thymoma (2). Other cancers reported with MN are colorectal, pancreatic, esophageal, and hepatic carcinomas. Differentiating primary MN from secondary MN associated with malignancy can be difficult. Our suspicion for a secondary glomerular disease American Society of Nephrology should be high in a patient with known cancer who has presence of proteinuria or nephrotic syndrome. Also development of proteinuria within a year of diagnosis of cancer should raise the suspicion of secondary form of glomerulopathy. Studies have reported risk factors like age .65 years and history of smoking for .20 pack-years for paraneoplastic MN (3). Review of relevant studies (3–7) has suggested certain parameters, which can help differentiate primary from secondary MN, the latter being associated with cancer. These features are summarized in Table 1 (8). In addition to these findings, one should have a high index of suspicion for malignancy when a patient with MN is evaluated. It is reasonable to perform routine age- and sex-appropriate screening for malignancy, once other known causes of secondary MN have been excluded. In patients with high risk of lung cancer, low-dose chest computed tomography should be considered. The risk of cancer persists for $5 years from the time of kidney biopsy (9). This is most likely due to slowgrowing malignancy, use of cytotoxic therapy for MN, or increased surveillance. Therefore, close medical follow-up is needed even if the cancer is not detected on initial screening at the time of MN diagnosis. The possible mechanisms by which solid tumors may be associated with MN include the following (10): 1) In situ immune complex formation: Antibodies are formed against a tumor antigen, which is localized in the sub epithelial location or to a podocyte antigen that is identical or similar to the tumor antigen. Correspondence: Divya Monga, Division of Nephrology, University of Mississippi Medical Center, 2500 N. State St., Jackson, Mississippi 39216. Copyright © 2016 by the American Society of Nephrology Onco-Nephrology Curriculum 1 Figure 1. Membranous nephropathy. (A) Light microscopy showing immune complex deposits. Note the thickened basement membrane, which stains black while deposits within it stain pink, giving a variegated appearance to the capillary wall. Silver [periodic acid silver methamine (PASM)] stain; 603, original magnification. (B) Electron microscopy showing immune complex deposits in a subepithelial location, between effaced podocyte foot processes (top) and the basement membrane (bottom). 2) Tumor antigens may form circulating immune complexes that are subsequently trapped in glomerular capillaries. 3) External factors such as infections with oncogenic viruses or altered immune function that can cause both the malignancy and MN. Other glomerular diseases Minimal change disease (MCD) has been reported in association with solid tumors like lung, colorectal, renal cell cancers, and thymoma. Rarely, pancreatic, bladder, breast, and ovarian cancers have also been associated (2). Focal segmental glomerular sclerosis (FSGS) has been observed with renal cell carcinoma, thymoma, and rarely with lung, breast, and esophageal cancers (2). A membranoproliferative glomerular nephritis (MPGN) pattern of injury has been described with lung, kidney, and stomach cancer (2). Mustonen et al. (11) reported the first known association between IgA nephropathy and solid tumors of the respiratory tract, buccal mucosa, and nasopharynx. Renal cell carcinoma is the most frequently reported solid malignancy associated with IgA nephropathy (12). Treatment of underlying cancer improved the IgA nephropathy (11). Rarely, both solid and hematologic malignancies have been associated with adult Henoch-Schönlein purpura (HSP) (13,14). Endocapillary glomerulonephritis is the most commonly seen lesion on kidney biopsy in adults with HSP (15). Older age and male sex were identified risk factors for cancerassociated HSP (14). Crescentic glomerulonephritis (CGN) has been associated with renal cell, gastric, and lung cancers (2). Thrombotic microangiopathy (TMA) has been associated with mucin-producing gastric, lung, and breast cancers (16). In these patients, ADAMTS13 activity is not impaired, and they respond poorly to plasmapharesis (17). The exact mechanism of these solid tumor malignancy associations with glomerular disease is poorly understood. There have been animal studies (18) done to help us understand the pathomechanisms involved. This animal study suggested that T-cell response might be critical in the development of paraneoplastic glomerular disease. Th1 (T-helper type 1)-predominant responses have been associated with proliferative and crescentic forms of GN and Th2 (T-helper type 2) type responses with a membranous pattern of injury (19). Cancer-associated MCD might be related to vascular endothelial growth factor (VEGF) production (20). Overexpression of VEGF leads to a collapsing variant of FSGS, and underexpression is associated with a TMA pattern of injury (21,22). Thymoma-associated glomerular disease MCD is the most common glomerular disease associated with thymoma (23). The prevalence of thymoma associated glomerulopathy is ;2% (23). Other glomerular lesions Table 1. Differences between primary and tumor-associated secondary MN Compared feature History Serologic markers Histopathologic clues on kidney biopsy Primary MN Younger age, no history of smoking Presence of circulating anti-PLA2R autoantibodies in serum Predominance of glomerular IgG4 deposition Enhanced glomerular PLA2R staining Presence of less than eight inflammatory cells per glomeruli Solid tumor–associated MN Age .65 years, smoking .20 pack-years Absence of anti-PLA2R autoantibodies Predominance of glomerular IgG1/IgG2 deposition Normal glomerular PLA2R staining Presence of greater than eight inflammatory cells per glomeruli IgG, immunoglobulin G; MN, membranous nephropathy; PLA2 R phospholipase A2. Reprinted with permission from reference 73. 2 Onco-Nephrology Curriculum American Society of Nephrology Table 2. Glomerular diseases associated with solid tumors and hematologic malignancies (23) Malignancy Lung cancer a Colon cancer Stomach cancer Pancreas cancer Bladder cancer Renal cell cancer Prostate cancer Breast cancer Esophageal cancer Gastrointestinal stromal tumor Gastric cancer Spleen sarcoma Head and neck cancer Wilms’ tumor Teratoma Ovarian cancer Cervical cancer Endometrial cancer Tongue cancer Mesothelioma Melanoma Skin cancers (basal and squamous cell) Pheochromocytoma Thymoma Hodgkin disease Non-Hodgkin’s disease CLL AML CML MGUS T-cell leukemia HEMATOLOGIC MALIGNANCIES–ASSOCIATED GLOMERULAR DISEASES Glomerular diseases reported MN, MCD, MPGN, IgAN, FSGS, CGN, TMA MN, MCD, CGN MN MN, MCD, IgAN MCD AAA, CGN, IgAN, MCD, FSGS, MPGN MN, CGN MN, FSGS, MPGN, TMA MPGN, FSGS AAA MPGN, CGN, TMA AAA MN, IgAN MN, MPGN MN MN, MCD MN MN IgAN MCD MN, MPGN MN MN MCD, FSGS, CGN, MPGN MCD, MN, MPGN, IgAN, FSGS, CGN, AAA, Anti-GBM MN, MCD, MPGN, IgAN, FSGS MN, MCD, MPGN, FSGS, CGN MN, FSGS MN, MCD, MPGN MPGN FSGS a Includes small-cell, non–small-cell, squamous cell, and bronchogenic cancers. AAA, AA amyloidosis; AML, acute myelogenous leukemia; CGN, Crescentic glomerulonephritis; CLL, chronic lymphocytic leukemia; CML, chronic myelogenous leukemia; FSGS, focal segmental global sclerosis; GBM, glomerular basement membrane; IgAN, IgA nephropathy; MCD, minimal change disease; MGUS, monoclonal gammopathy of unclear significance; MN, membranous nephropathy; MPGN, membranoproliferative glomerular nephritis; TMA, thrombotic microangiopathy. Reprinted with permission from reference 23. described are MN, FSGS, CGN, and lupus-like nephritis (24). MN is associated with thymoma of epithelial origin. MCD is associated with thymoma with lymphocyte predominance. The pathogenesis of thymoma-associated MN seems to be similar to solid tumor–associated MN, and thymomaassociated MN is likely related to T-cell dysfunction (24). Studies (25–27) have suggested a major role of T cells, especially the Th2 subtype, in thymoma-associated nephrotic syndromes. Table 2 summarizes the various solid tumors seen with solid tumors. American Society of Nephrology Minimal change disease MCD is classically associated with Hodgkin lymphoma (HL), more so in the mixed cellularity and nodular sclerosing types. MCD usually presents around the time of diagnosis of the malignancy (28). One case series does report diagnosis of MCD preceding the diagnosis of lymphoma by several months (29). A poor response to the treatment of MCD with corticosteroids should raise suspicion of underlying lymphoma. In the case series by Audard et al., (29), the simultaneous diagnosis of HL and MCD was associated with the remission of proteinuria in response to chemotherapy. Th2-related cytokines such as interleukin (IL)-13 have been reported to cause inflammatory response in Hodgkin disease (30), and rat studies have shown that overexpression of IL-13 induces proteinuria, hypoalbuminemia, and hypercholesterolemia (31). These kidney biopsies showed fusion of foot processes, suggesting MCD like pathology. Membranoproliferative glomerulonephritis Da’as et al. (32) reviewed 42 cases of glomerular disease with chronic lymphocytic leukemia (CLL); of these, 36 had nephrotic syndrome. The most common glomerular lesion was MPGN, followed by MN. Another case series of 13 patients with glomerular disease and either CLL or well-differentiated lymphocytic lymphoma (33) showed that the majority had an MPGN pattern of injury. Most MPGN patients had an associated cryoglobulinemia. An MPGN pattern on kidney biopsy (Figure 2) may also be a clue to a developing of undiagnosed lymphoplasmacytic malignancy (8). Sethi et al. (34) reported an association between MPGN and monoclonal gammopathy of uncertain significance. They showed that patients with monoclonal gammopathy with normal bone marrow biopsies had granular immune deposits on their kidney biopsy, which correlated with their serum and urine monoclonal proteins. This study (34) also demonstrated that monoclonal gammopathy can be seen in the setting of other lymphoplasmacytic diseases, including low-grade B-cell lymphoma, CLL, and multiple myeloma. Although this direct relationship is not proven, the current observation suggests this possibility (34). More of this is discussed in the paraproteinemia chapter of the curriculum. Glomerular diseases associated with myeloproliferative disorders Myeloproliferative disorders include chronic myelogenous leukemia (CML), polycythemia Vera (PCV), and essential thrombocythemia. A recent study (35) of 11 patients with myeloproliferative disorders with proteinuria and renal failure showed mesangial sclerosis with hypercellularity in all patients, segmental sclerosis in eight patients, features of TMA in eight patients, and intracapillary hematopoietic cells in four patients. Glomerular disorders associated with myeloproliferative disorders are usually late complications and tend to Onco-Nephrology Curriculum 3 Figure 2. Membranoproliferative glomerulonephritis. Light microscopy showing basement membrane duplication and increased cells in capillary lumens. Silver (PASM) stain; 603, original magnification. have a poor renal prognosis, with progressive kidney injury occurring in most patients (35). Essential thrombocythemia and PCV have been associated with FSGS and mesangial proliferative glomerular disease. The prevalence of glomerular disease in PCV and essential thrombocythemia is approximately 3%–4% (36). CML is least likely to have an association with glomerular pathology (36). FSGS has also been reported with Hodgkin’s lymphoma (28) with good response to chemotherapy. Other glomerular diseases associated with lymphoproliferative disorders MN has also been reported with CLL, but less commonly compared with MPGN (32). A case of IgA nephropathy has been described with cutaneous T-cell lymphoma (37). Fibrillary glomerulonephritis (FGN) and Immunotactoid glomerulopathy (ITG) are rare groups of disorders characterized by formation of organized glomerular deposits (Figure 3, A and B). These diseases can either occur as primary condition or be secondary to systemic diseases, mainly lymphoproliferative disorders. ITG is more strongly associated with neoplasms, typically paraproteinemias and CLL, compared with FGN (38). ITG on kidney biopsy should warrant an investigation of an underlying hematologic malignancy. Treatment is directed toward underlying malignancy. Glomerular diseases have also been associated with hemophagocytic syndrome. This syndrome is most commonly associated with Epstein-Barr virus; however, it has also been described with T-cell lymphoma (39,40). Thaunat et al. (40) described 11 patients with hemophagocytic syndrome who developed nephrotic syndrome. Renal biopsy showed glomerular lesions consisting of MCD, FSGS, and TMA. In the absence of a causative viral infection, hemophagocytic syndrome is often treated with immunosuppressive therapy with uncertain renal outcomes. HEMATOPOIETIC STEM CELL TRANSPLANT– RELATED GLOMERULAR DISEASES In HSCT patients, the kidney biopsy findings in patients with nephrotic range proteinuria include MN, MCD, and FSGS (41). Although we discuss briefly here, an entire chapter is devoted to HSCT-related kidney disease in the Curriculum. Chronic graft-versus-host disease MN accounts for a majority of cases of HSCT-associated glomerular disease, followed by MCD (41). When MCD occurs in such patients, it is prudent to rule out recurrence of the primary hematologic malignancy. Figure 3. Fibrillary and immunotactoid glomerulonephritis. (A) Electron microscopy view of fibrillary glomerulonephritis. (B) Electron microscopy view of immunotactoid glomerulonephritis. 4 Onco-Nephrology Curriculum American Society of Nephrology A review of literature by Brukamp et al. (41) showed a close temporal relationship between development of nephrotic syndrome shortly after cessation of immunosuppression and the diagnosis of chronic graft-versus-host disease (GVHD). Luo et al. (42) investigated the etiology and pathogenesis of nephrotic syndrome after allogenic HSCT. Nephrotic syndrome after allogenic SCT was associated with the occurrence of chronic GVHD. Autologous HSCT can also develop glomerular diseases (43), although in these patients, GVHD does not occur. It is possible that there is an immune dysregulation that might be causing nephrotic syndrome secondary to induction agents or that these glomerular diseases are de novo. T cell–depleted HSCT recipients are highly unlikely to develop glomerular diseases. However, our knowledge about glomerular diseases in HSCT patients is incomplete, and more research is needed for complete understanding. Thrombotic microangiopathy after HSCT TMA after HSCT is also known as bone marrow transplant nephropathy or, in some specific cases, radiation nephropathy. A diagnosis criteria for HSCT-related TMA included .4% schistocytes in blood, de novo prolonged or progressive thrombocytopenia, sudden persistent increase in lactate dehydrogenase, and a decrease in serum haptoglobin (44). Studies have suggested that acute GVHD grade 2–4, hepatic GVHD, veno-occlusive disease, adenovirus infection, older age, being female, and total body irradiation of .12 Gy are risk factors for the development of TMA (45,46). TMA can also occur in T cell–depleted group of patients where calcineurin inhibitors (CNIs) and GVHD do not exist (47). Treatment of HSCT-related TMA is usually supportive, with control of hypertension and proteinuria. Plasma exchange has not proven to be effective. CHEMOTHERAPY-ASSOCIATED GLOMERULAR DISEASE Thrombotic microangiopathy Mitomycin C, an alkylating agent, used to treat breast, gastric, and pancreatic cancer, can cause TMA-like syndrome. Its nephrotoxicity is dose dependent and usually appears after a cumulative dose of .40–60 mg/m2 given over a period of several months (48). Gemcitabine, commonly used for pancreas, urothelial, and ovarian cancers, has been shown to cause TMA (49). Cessation of these medications is shown to improve TMA. Carfilzomib is a second-generation proteasome inhibitor used for the treatment of relapsed or refractory multiple myeloma. There has been recent case reports (50,51) that reported TMA associated with the use of this agent. One of them (51) had kidney biopsy evidence of TMA (Figure 4). Treatment options include cessation of the drug with uncertain importance of therapeutic plasma exchange. Kidney biopsy–proven renal TMA has been American Society of Nephrology Figure 4. Thrombotic microangiopathy. Light microscopy view showing red cell thrombi in the afferent arteriole and two glomerular capillaries. Some basement membrane duplication, but without increased intracapillary cells, is also visible. Silver (PASM) stain; 603, original magnification. reported by Kwa et al. (52) in patients receiving years of pegylated liposomal doxorubicin for recurrent ovarian cancer. Bisphosphonate-induced glomerular injury Pamidronate is used to treat malignancy associated bone disorders in myeloma. Markowitz et al. (53) showed that pamidronate causes biopsy-proven collapsing FSGS. MCD has also been reported with this agent (54). Interferon-induced glomerular injury Interferons (IFN)-a, -b, and -g have been associated with moderate proteinuria (55). Markowitz et al. (56) reported 11 cases of collapsing FSGS that developed during treatment with IFN. IFNa developed significant proteinuria and renal failure after a short duration of treatment. Patients treated with IFN-b developed proteinuria after a prolonged course of treatment. The authors (56) also reviewed 21 additional cases of IFN-associated glomerular disease. Thirteen of these patients had FSGS, and the rest had MCD. The mechanism of this injury is not fully understood. There is a direct effect of IFN on the podocyte by altering the cellular proliferation and cell metabolism (56). The indirect effects of IFN might be due to adaptive immune mechanism that increase macrophage activation or via 1L-6 or 1L-13 production (56). IFN-a, when used for treatment of CML, has been reported to be associated with TMA (57,58). Calcineurin and mammalian target of rapamycin inhibitors CNIs can cause a rare manifestation of TMA with glomerular changes. The histology is indistinguishable from other causes of TMA (59). The only consensus on treatment is to withdraw the CNIs (60). Mammalian target of rapamycin (mTOR) inhibitors such as sirolimus, tensirolimus, and everolimus can develop Onco-Nephrology Curriculum 5 Table 3. Glomerular toxicity associated with chemotherapeutic agents Type of cells involved Glomerular epithelial cells (podocytes) Minimal change disease Focal segmental glomerular sclerosis Other glomerular diseases Glomerular endothelial cells Thrombotic microangiopathy Chemotherapy agents Interferon-a and -b, pamidronate, tyrosine kinase inhibitors, anthracyclines, mTOR inhibitors Interferon-a and -g, pamidronate, tyrosine kinase inhibitors, clofarabine, anthracyclines, mTOR inhibitors Ipilimumab, mTOR inhibitors Mitomycin-c, gemcitabine, cisplatin, carboplatin, cytarabine, lomustine, tamoxifen, bleomycin, bortezomib, carfilzomib, anthracyclines, hydroxyurea complications including TMA and FSGS in renal transplant patients (61–63). MCD, MN, FSGS, MPGN, and IgA nephropathy have also been associated with sirolimus in the kidney transplant literature (64–66). There is speculation that sirolimus-induced proteinuria is related to collapsing FSGS associated with VEGF overexpression in podocytes. Antiangiogenesis agents Antiangiogenic agents are used primarily for advanced stage solid tumors, including renal cell carcinoma, non–small cell lung carcinoma, colorectal carcinoma, and gastrointestinal stromal tumors. Monoclonal antibodies against VEGF and tyrosine kinase inhibitors (TKIs) (67,68) have been observed to cause hypertension, proteinuria, and renal vascular injury, manifested by proteinuria and TMA (69). VEGF maintains normal functioning of glomerular endothelial cells, podocytes, mesangium, and peritubular capillaries. Hence, inhibition of VEGF can lead to dose-dependent proteinuria, swelling and detachment of glomerular endothelial cells, vacuolization of endothelial cells, disruption of slit diaphragms, and down-regulation of nephrin (70). Examples of anti-VEGF therapy include bevacizumab, and TKIs include sunitinib and sorafenib. In the majority of cases, proteinuria and hypertension resolve or significantly improve with cessation of this therapy (69). VEGF inhibitors are more likely to present as TMA or renal-limited TMA and TKIs as MCD or MCD/FSGS on kidney biopsy (71). New chemotherapeutic agent–associated glomerular disease Several new chemotherapies are now available in clinical practice. Renal toxicity of these novel agents has been increasingly reported in the last decade. Clofaribine is a purine nucleoside analog used to treat relapsed or refractory pediatric acute lymphoblastic leukemia and adult acute myelogenous leukemia. Nephrotoxicity most commonly manifests as elevation in serum creatinine. Kintzel et al. (72) reported AKI following exposure of clofaribine along with nephrotic range proteinuria. Unfortunately a kidney biopsy was not available. Extrapolating from animal studies, Jhaveri et al. (73) postulated that inhibition of ribonucleotide reductase by clofarabine might be the cause of collapsing glomerulopathy and/or kidney injury seen with this agent. Ipilimumab is a monoclonal antibody against human cytotoxic T-lymphocyte antigen 4. It is US Food and Drug 6 Onco-Nephrology Curriculum Administration (FDA) approved for unresectable or metastatic melanoma. Renal biopsy in a patient with ipilimumabassociated AKI with nephrotic range proteinuria revealed lupus nephritis with positive anti–double-stranded DNA antibodies (74). There are also case reports of acute granulomatous interstitial nephritis by this agent (75). Anthracyclines like daunorubicin and doxorubicin have been known to cause nephrotic syndrome with renal lesions consistent with MCD, FSGS not otherwise specified (NOS), or collapsing glomerulopathy (76). Table 3 summarizes the glomerular toxicities associated with chemotherapy agents. Ongoing education and heightened physician awareness regarding these negative associations is central to early recognition and their successful management. CONCLUSION Several cancers are associated with various glomerular diseases. Membranous nephropathy remains the most common glomerular pathology reported in patients with solid tumors. Although MCD disease has been classically associated with HL, MPGN has been recognized in patients with CLL. Several reports and studies in the literature suggest that treating the cancer leads to resolution of the glomerular disease. TAKE HOME POINTS c Many solid and hematologic malignancies are associated with different glomerular diseases. c Several case reports and case series of cancer-associated glomerular diseases have shown that treating the cancer may lead to resolution of the glomerular process. c Although membranous nephropathy has been classically associated with solid malignancies, minimal change disease has been commonly described with hematologic malignancies, especially Hodgkin lymphoma. c Membranoproliferative glomerulonephritis is increasingly being recognized to be associated with chronic hematologic malignancies such as chronic lymphocytic leukemia. c Chemotherapy agents can also lead to glomerular diseases, the most common being TMA associated with targeted therapies. American Society of Nephrology ACKNOWLEDGMENTS Pathology images are courtesy of James Pullman, Albert Einstein Medical Center, NY. 19. 20. REFERENCES 1. Ronco PM. Paraneoplastic glomerulopathies: New insights into an old entity. Kidney Int 56: 355–377, 1999 2. Bacchetta J, Juillard L, Cochat P, Droz JP. Paraneoplastic glomerular diseases and malignancies. Crit Rev Oncol Hematol 70: 39–58, 2009 3. Lefaucheur C, Stengel B, Nochy D, Martel P, Hill GS, Jacquot C, Rossert J, Group G-PS. Membranous nephropathy and cancer: Epidemiologic evidence and determinants of high-risk cancer association. Kidney Int 70: 1510–1517, 2006 4. Beck LH, Jr., Bonegio RG, Lambeau G, Beck DM, Powell DW, Cummins TD, Klein JB, Salant DJ. M-type phospholipase A2 receptor as target antigen in idiopathic membranous nephropathy. New Engl J Med 361: 11–21, 2009 5. Qin W, Beck LH, Jr., Zeng C, Chen Z, Li S, Zuo K, Salant DJ, Liu Z. Antiphospholipase A2 receptor antibody in membranous nephropathy. J Am Soc Nephrol 22: 1137–1143, 2011 6. Hoxha E, Kneissler U, Stege G, Zahner G, Thiele I, Panzer U, Harendza S, Helmchen UM, Stahl RA. Enhanced expression of the M-type phospholipase A2 receptor in glomeruli correlates with serum receptor antibodies in primary membranous nephropathy. Kidney Int 82: 797–804, 2012 7. Ohtani H, Wakui H, Komatsuda A, Okuyama S, Masai R, Maki N, Kigawa A, Sawada K, Imai H. Distribution of glomerular IgG subclass deposits in malignancy-associated membranous nephropathy. Nephrol Dialysis Transplant 19: 574–579, 2004 8. Jhaveri KD, Shah HH, Patel C, Kadiyala A, Stokes MB, Radhakrishnan J. Glomerular diseases associated with cancer, chemotherapy, and hematopoietic stem cell transplantation. Adv Chronic Kidney Dis 21: 48– 55, 2014 9. Bjorneklett R, Vikse BE, Svarstad E, Aasarod K, Bostad L, Langmark F, Iversen BM. Long-term risk of cancer in membranous nephropathy patients. Am J Kidney Dis 50: 396–403, 2007 10. Beck LH Jr. Membranous nephropathy and malignancy. Semin Nephrol 30: 635–644, 2010 11. Mustonen J, Pasternack A, Helin H. IgA mesangial nephropathy in neoplastic diseases. Contribut Nephrol 40: 283–291, 1984 12. Magyarlaki T, Kiss B, Buzogany I, Fazekas A, Sukosd F, Nagy J. Renal cell carcinoma and paraneoplastic IgA nephropathy. Nephron 82: 127– 130, 1999 13. Pertuiset E, Liote F, Launay-Russ E, Kemiche F, Cerf-Payrastre I, Chesneau AM. Adult Henoch-Schonlein purpura associated with malignancy. Semin Arthrit Rheum 29: 360–367, 2000 14. Zurada JM, Ward KM, Grossman ME. Henoch-Schonlein purpura associated with malignancy in adults. J Am Acad Dermatol 55[5 Suppl]: S65–S70, 2006 15. Pillebout E, Thervet E, Hill G, Alberti C, Vanhille P, Nochy D. HenochSchonlein Purpura in adults: Outcome and prognostic factors. J Am Soc Nephrol 13: 1271–1278, 2001 16. Werner TL, Agarwal N, Carney HM, Rodgers GM. Management of cancer-associated thrombotic microangiopathy: What is the right approach? Am J Hematol 82: 295–298, 2007 17. Francis KK, Kalyanam N, Terrell DR, Vesely SK, George JN. Disseminated malignancy misdiagnosed as thrombotic thrombocytopenic purpura: A report of 10 patients and a systematic review of published cases. Oncologist 12: 11–19, 2007 18. Takeda S, Chinda J, Murakami T, Numata A, Iwazu Y, Akimoto T, Hamano Y, Muto S, Takahashi M, Kusano E. Development of features of glomerulopathy in tumor-bearing rats: A potential model for American Society of Nephrology 21. 22. 23. 24. 25. 26. 27. 28. 29. 30. 31. 32. 33. 34. 35. 36. 37. paraneoplastic glomerulopathy. Nephrol Dialysis Transplant 27: 1786– 1792, 2012 Holdsworth SR, Kitching AR, Tipping PG. Th1 and Th2 T helper cell subsets affect patterns of injury and outcomes in glomerulonephritis. Kidney Int 55: 1198–1216, 1999 Taniguchi K, Fujioka H, Torashima Y, Yamaguchi J, Izawa K, Kanematsu T. Rectal cancer with paraneoplastic nephropathy: Association of vascular endothelial growth factor. Digestive Surg 21: 455–457, 2004 Eremina V, Sood M, Haigh J, Nagy A, Lajoie G, Ferrara N, Gerber HP, Kikkawa Y, Miner JH, Quaggin SE. Glomerular-specific alterations of VEGF-A expression lead to distinct congenital and acquired renal diseases. J Clin Invest 111: 707–716, 2003 Eremina V, Jefferson JA, Kowalewska J, Hochster H, Haas M, Weisstuch J, Richardson C, Kopp JB, Kabir MG, Backx PH. VEGF inhibition and renal thrombotic microangiopathy. New Engl J Med 358: 1129–1136, 2008 Jhaveri KD, Shah HH, Calderon K, Campenot ES, Radhakrishnan J. Glomerular diseases seen with cancer and chemotherapy: A narrative review. Kidney Int 84: 34–44, 2013 Karras A, de Montpreville V, Fakhouri F, Grunfeld JP, Lesavre P, Groupe d’Etudes des Nephropathies Associees aux T. Renal and thymic pathology in thymoma-associated nephropathy: Report of 21 cases and review of the literature. Nephrol Dialysis Transplant 20: 1075–1082, 2005 Hirokawa K, Utsuyama M, Kasai M, Konno A, Kurashima C, Moriizumi E. Age-related hyperplasia of the thymus and T-cell system in the Buffalo rat. Immunological and immunohistological studies. Virchows Arch B 59: 38–47, 1990 Le Berre L, Herve C, Buzelin F, Usal C, Soulillou JP, Dantal J. Renal macrophage activation and Th2 polarization precedes the development of nephrotic syndrome in Buffalo/Mna rats. Kidney Int 68: 2079– 2090, 2005 Le Berre L, Bruneau S, Naulet J, Renaudin K, Buzelin F, Usal C, Smit H, Condamine T, Soulillou JP, Dantal J. Induction of T regulatory cells attenuates idiopathic nephrotic syndrome. J Am Soc Nephrol 20: 57– 67, 2009 Mallouk A, Pham PT, Pham PC. Concurrent FSGS and Hodgkin’s lymphoma: Case report and literature review on the link between nephrotic glomerulopathies and hematological malignancies. Clin Exper Nephrol 10: 284–289, 2006 Audard V, Larousserie F, Grimbert P, Abtahi M, Sotto JJ, Delmer A, Boue F, Nochy D, Brousse N, Delarue R. Minimal change nephrotic syndrome and classical Hodgkin’s lymphoma: Report of 21 cases and review of the literature. Kidney Int 69: 2251–2260, 2006 Kuppers R, Schwering I, Brauninger A, Rajewsky K, Hansmann ML. Biology of Hodgkin’s lymphoma. Ann Oncol 13[Suppl 1]: 11–18, 2002 Lai KW, Wei CL, Tan LK, Tan PH, Chiang GS, Lee CG, Jordan SC, Yap HK. Overexpression of interleukin-13 induces minimal-change-like nephropathy in rats. J Am Soc Nephrol 18: 1476–1485, 2007 Da’as N, Polliack A, Cohen Y, Amir G, Darmon D, Kleinman Y, Goldfarb AW, Ben-Yehuda D. Kidney involvement and renal manifestations in nonHodgkin’s lymphoma and lymphocytic leukemia: A retrospective study in 700 patients. Eur J Haematol 67: 158–164, 2001 Moulin B, Ronco PM, Mougenot B, Francois A, Fillastre JP, Mignon F. Glomerulonephritis in chronic lymphocytic leukemia and related B-cell lymphomas. Kidney Int 42: 127–135, 1992 Sethi S, Zand L, Leung N, Smith RJ, Jevremonic D, Herrmann SS, Fervenza FC. Membranoproliferative glomerulonephritis secondary to monoclonal gammopathy. Clin J Am Soc Nephrol 5: 770–782, 2010 Said SM, Leung N, Sethi S, Cornell LD, Fidler ME, Grande JP, Herrmann S, Tefferi A, D’Agati VD, Nasr SH. Myeloproliferative neoplasms cause glomerulopathy. Kidney Int 80: 753–759, 2011 Au WY, Chan KW, Lui SL, Lam CC, Kwong YL. Focal segmental glomerulosclerosis and mesangial sclerosis associated with myeloproliferative disorders. Am J Kidney Dis 34: 889–893, 1999 Bajel A, Yin Lin M, Hill PA, Goodman D, McCormack C, Foley P, Davison J, Honemann D, Kenealy M, Lade S. IgA nephropathy associated with cutaneous T cell lymphoma. Leukemia lymphoma 50: 2083–2085, 2009 Onco-Nephrology Curriculum 7 38. Rosenstock JL, Markowitz GS, Valeri AM, Sacchi G, Appel GB, D’Agati VD. Fibrillary and immunotactoid glomerulonephritis: Distinct entities with different clinical and pathologic features. Kidney Int 63: 1450– 1461, 2003 39. Chang CS, Wang CH, Su IJ, Chen YC, Shen MC. Hematophagic histiocytosis: A clinicopathologic analysis of 23 cases with special reference to the association with peripheral T-cell lymphoma. J. Formosan Med Assoc 93: 421–428, 1994 40. Thaunat O, Delahousse M, Fakhouri F, Martinez F, Stephan JL, Noel LH, Karras A. Nephrotic syndrome associated with hemophagocytic syndrome. Kidney Int 69: 1892–1898, 2006 41. Brukamp K, Doyle AM, Bloom RD, Bunin N, Tomaszewski JE, Cizman B. Nephrotic syndrome after hematopoietic cell transplantation: Do glomerular lesions represent renal graft-versus-host disease? Clin J Am Soc Nephrol 1: 685–694, 2006 42. Luo XD, Liu QF, Zhang Y, Sun J, Wang GB, Fan ZP, Yi ZS, Ling YW, Wei YQ, Liu XL. Nephrotic syndrome after allogeneic hematopoietic stem cell transplantation: Etiology and pathogenesis. Blood Cells Molecules Dis 46: 182–187, 2011 43. Troxell ML, Pilapil M, Miklos DB, Higgins JP, Kambham N. Renal pathology in hematopoietic cell transplantation recipients. Modern Pathol 21: 396–406, 2008 44. Ruutu T, Barosi G, Benjamin RJ, Clark RE, George JN, Gratwohl A, Holler E, Iacobelli M, Kentouche K, Lammle B. Diagnostic criteria for hematopoietic stem cell transplant-associated microangiopathy: Results of a consensus process by an International Working Group. Haematologica 92: 95–100, 2007 45. Chang A, Hingorani S, Kowalewska J, Flowers ME, Aneja T, Smith KD, Meehan SM, Nicosia RF, Alpers CE. Spectrum of renal pathology in hematopoietic cell transplantation: A series of 20 patients and review of the literature. Clin J Am Soc Nephrol 2: 1014–1023, 2007 46. Changsirikulchai S, Myerson D, Guthrie KA, McDonald GB, Alpers CE, Hingorani SR. Renal thrombotic microangiopathy after hematopoietic cell transplant: Role of GVHD in pathogenesis. Clin J Am Soc Nephrol 4: 345–353, 2009 47. Glezerman IG, Jhaveri KD, Watson TH, Edwards AM, Papadopoulos EB, Young JW, Flombaum CD, Jakubowski AA. Chronic kidney disease, thrombotic microangiopathy, and hypertension following T celldepleted hematopoietic stem cell transplantation. Biol Blood Marrow Transplant 16: 976–984, 2010 48. Lesesne JB, Rothschild N, Erickson B, Korec S, Sisk R, Keller J, Arbus M, Woolley PV, Chiazze L, Schein PS. Cancer-associated hemolytic-uremic syndrome: analysis of 85 cases from a national registry. J Clin Oncol 7: 781–789, 1989 49. Glezerman I, Kris MG, Miller V, Seshan S, Flombaum CD. Gemcitabine nephrotoxicity and hemolytic uremic syndrome: Report of 29 cases from a single institution. Clin Nephrol 71: 130–139, 2009 50. Sullivan MR, Danilov AV, Lansigan F, Dunbar NM. Carfilzomib associated thrombotic microangiopathy initially treated with therapeutic plasma exchange. J Clin Apheresis 2014 51. Hobeika L, Self SE, Velez JC. Renal thrombotic microangiopathy and podocytopathy associated with the use of carfilzomib in a patient with multiple myeloma. BMC Nephrol 15: 156, 2014 52. Kwa M, Baumgartner R, Shavit L, Barash I, Michael J, Puzanov I, Kopolovic J, Rosengarten O, Blank S, Curtin JP. Is renal thrombotic angiopathy an emerging problem in the treatment of ovarian cancer recurrences? Oncologist 17: 1534–1540, 2012 53. Markowitz GS, Appel GB, Fine PL, Fenves AZ, Loon NR, Jagannath S, Kuhn JA, Dratch AD, D’Agati VD. Collapsing focal segmental glomerulosclerosis following treatment with high-dose pamidronate. J Am Soc Nephrol 12: 1164–1172, 2001 54. Barri YM, Munshi NC, Sukumalchantra S, Abulezz SR, Bonsib SM, Wallach J, Walker PD. Podocyte injury associated glomerulopathies induced by pamidronate. Kidney Int 65: 634–641, 2004 8 Onco-Nephrology Curriculum 55. Quesada JR, Talpaz M, Rios A, Kurzrock R, Gutterman JU. Clinical toxicity of interferons in cancer patients: a review. J Clin Oncol 4: 234–243, 1986 56. Markowitz GS, Nasr SH, Stokes MB, D’Agati VD. Treatment with IFN-{alpha}, -{beta}, or -{gamma} is associated with collapsing focal segmental glomerulosclerosis. Clin J Am Soc Nephrol 5: 607–615, 2010 57. Badid C, McGregor B, Faivre JM, Guerard A, Juillard L, Fouque D, Laville M. Renal thrombotic microangiopathy induced by interferonalpha. Nephrol Dialysis Transplant 16: 846–848, 2001 58. Ohashi N, Yonemura K, Sugiura T, Isozaki T, Togawa A, Fujigaki Y, Yamamoto T, Hishida A. Withdrawal of interferon-alpha results in prompt resolution of thrombocytopenia and hemolysis but not renal failure in hemolytic uremic syndrome caused by interferon-alpha. Am J Kidney Dis 41: E10, 2003 59. Liptak P, Ivanyi B. Primer: Histopathology of calcineurin-inhibitor toxicity in renal allografts. Nature Clin Pract Nephrol 2: 398–404, 2006 60. Ho VT, Cutler C, Carter S, Martin P, Adams R, Horowitz M, Ferrara J, Soiffer R, Giralt S. Blood and marrow transplant clinical trials network toxicity committee consensus summary: Thrombotic microangiopathy after hematopoietic stem cell transplantation. Biol Blood Marrow Transplant 11: 571–575, 2005 61. Jhaveri KD, Schatz JH, Young JW, Flombaum C. Sirolimus (rapamycin) induced proteinuria in a patient undergoing allogeneic hematopoietic stem cell transplant. Transplantation 86: 180–181, 2008 62. Hochegger K, Wurz E, Nachbaur D, Rosenkranz AR, Clausen J. Rapamycininduced proteinuria following allogeneic hematopoietic stem cell transplantation. Bone Marrow Transplant 44: 63–65, 2009 63. Izzedine H, Boostandoot E, Spano JP, Bardier A, Khayat D. Temsirolimusinduced glomerulopathy. Oncology 76: 170–172, 2009 64. Mainra R, Mulay A, Bell R, Karpinski J, Hoar S, Knoll G, Robertson S, Wang D. Sirolimus use and de novo minimal change nephropathy following renal transplantation. Transplantation 80:1816, 2005 65. Franco AF, Martini D, Abensur H, Noronha IL. Proteinuria in transplant patients associated with sirolimus. Transplant Proc 39: 449–452, 2007 66. Letavernier E, Bruneval P, Mandet C, Duong Van Huyen JP, Peraldi MN, Helal I, Noel LH, Legendre C. High sirolimus levels may induce focal segmental glomerulosclerosis de novo. Clin J Am Soc Nephrol 2: 326– 333, 2007 67. Jhaveri KD, Flombaum CD, Kroog G, Glezerman IG. Nephrotoxicities associated with the use of tyrosine kinase inhibitors: A single-center experience and review of the literature. Nephron Clin Pract 117: c312– c319, 2011 68. Bollee G, Patey N, Cazajous G, Robert C, Goujon JM, Fakhouri F, Bruneval P, Noel LH, Knebelmann B. Thrombotic microangiopathy secondary to VEGF pathway inhibition by sunitinib. Nephrol Dialysis Transplant 24: 682–685, 2009 69. Hayman SR, Leung N, Grande JP, Garovic VD. VEGF inhibition, hypertension, and renal toxicity. Curr Oncol Rep 14: 285–294, 2012 70. Kelly RJ, Billemont B, Rixe O. Renal toxicity of targeted therapies. Target Oncol 4: 121–133, 2009 71. Izzedine H, Escudier B, Lhomme C, Pautier P, Rouvier P, Gueutin V, Baumelou A, Derosa L, Bahleda R, Hollebecque A. Kidney diseases associated with anti-vascular endothelial growth factor (VEGF): An 8-year observational study at a single center. Medicine 93: 333–339, 2014 72. Kintzel PE, Visser JA, Campbell AD. Clofarabine-associated acute kidney injury and proteinuria. Pharmacotherapy 31: 923, 2011 73. Jhaveri KD, Chidella S, Allen SL, Fishbane S. Clofarabine-induced kidney toxicity. J Oncol Pharmacy Pract 20: 305–308, 2013 74. Fadel F, El Karoui K, Knebelmann B. Anti-CTLA4 antibody-induced lupus nephritis. New Engl J Med 361: 211–212, 2009 75. Izzedine H, Gueutin V, Gharbi C, Mateus C, Robert C, Routier E, Thomas M, Baumelou A, Rouvier P. Kidney injuries related to ipilimumab. Invest New Drugs 32: 769–773, 2014 76. Mohamed N, Goldstein J, Schiff J, John R. Collapsing glomerulopathy following anthracycline therapy. Am J Kidney Dis 61: 778–781, 2013 American Society of Nephrology REVIEW QUESTIONS 1. Which of the following statements regarding glomerular diseases seen with cancer is true? a. The most common glomerular pathology seen with solid tumors is minimal change disease b. The most common glomerular pathology seen with hematologic malignancies is membranous nephropathy c. The most common associated glomerular disease with GVHD is membranous nephropathy d. Thymoma has not been associated glomerular diseases Answer: c is correct. The most common glomerular pathology seen with solid tumors is membranous nephropathy (MN). Minimal change disease (MCD) is commonly seen with hematologic malignancies such as Hodgkin lymphoma. MN accounts for the majority of the cases of HSCT-associated glomerular disease. Thymoma has been associated with MCD (lymphocyte predominant) and MN (epithelial origin). 2. A primary care physician refers a 60-year-old white woman for evaluation of nephrotic range proteinuria. She presented with a 1-month history of worsening bilateral lower extremity edema. She has a history of refractory malignant melanoma. She was recently started on ipilimumab after failing standard chemotherapy. Her melanoma has responded to therapy. On physical examination, her BP was normal at 120/80 mmHg, and there was 31 pitting edema of his lower extremities. The rest of the examination was unremarkable. At the time of presentation, serum creatinine was 0.9 mg/dL, serum albumin was 2.8 g/dL, total cholesterol was 290 mg/dL, and low-density lipoprotein (LDL) cholesterol was 197 mg/dL. Liver function tests and complete blood count were normal. A 24-hour urine collection revealed 8.5 g protein. A workup for secondary causes of nephrotic syndrome revealed normal complement levels. Hepatitis B surface antigen, hepatitis C antibody, antinuclear antibody, cryoglobulins, and human immunodeficiency virus (HIV) antibody were negative. Serum free light chains were within the normal ratio. Sonogram revealed normal-sized kidneys. A kidney biopsy reveals a proliferative glomerular disease with immunofluorescence suggestive of a full house pattern and electron microscopy showing mesangial and subendothelial deposits. What is the most likely diagnosis? American Society of Nephrology a. b. c. d. Melanoma-induced proliferative glomerulonephritis Ipilimumab-induced lupus-like nephritis De novo seronegative lupus nephritis Membranoproliferative glomerulonephritis Answer: b is correct. Ipilimumab is a monoclonal antibody against human cytotoxic T-lymphocyte antigen 4, which is FDA approved for unresectable or metastatic melanoma. Nephrotic range proteinuria with a lupus nephritis–like picture on renal biopsy has been reported. 3. A 62-year-old white man with a long-standing history of hypertension and recent history of CLL was referred by his oncologist for evaluation of proteinuria and elevated serum creatinine. He denied any history of diabetes, hepatitis, or blood transfusion. There was no recent infection or travel history. Review of systems was significant for bilateral intermittent lower extremity swelling over the last 4 months. He denied fever, chills, dyspnea, gross hematuria, arthralgia, or rash. His current medication included amlodipine for hypertension management. On physical examination, his BP was elevated at 160/94 mmHg. There was mild edema of his lower extremities. The rest of the examination was unremarkable. At the time of presentation, serum creatinine was 1.5 mg/dL, and serum albumin was 3.5 g/dL. Complete blood count, liver function tests, and a lipid profile were normal. Urinalysis was significant for 10–20 RBC/hpf and 21 proteinuria. A 24-hour urine collection revealed 1.8 g protein. A workup for secondary causes of proteinuria revealed low C3 and C4 levels. Hepatitis B surface antigen, hepatitis C antibody, antinuclear antibody, cryoglobulins, and human immunodeficiency virus (HIV) antibody were negative. Serum and urine immunofixation did not reveal any monoclonal immunoglobulin. Sonogram revealed normal-sized kidneys. A kidney biopsy was subsequently performed. What is the most likely kidney biopsy diagnosis? a. b. c. d. Membranous nephropathy Membranoproliferative glomerulonephritis Focal segmental glomerulosclerosis Acute interstitial nephritis Answer: b is correct. A membranoproliferative glomerulonephritis (MPGN) pattern of injury on renal biopsy has been most commonly associated with CLL, followed by MN. Onco-Nephrology Curriculum 9 Chapter 7: Hematologic Disorders and Kidney Disease Ala Abudayyeh, MD,* and Kevin Finkel, MD, FACP, FASN, FCCM*† *Division of General Internal Medicine, Section of Nephrology, University of Texas MD Anderson Cancer Center, Houston, Texas; and †UTHealth Science Center at Houston Medical School, Department of Medicine, Division of Renal Diseases and Hypertension, Houston, Texas MULIPLE MYELOMA Pathogenesis Multiple myeloma (MM) is a hematologic malignancy involving the pathologic proliferation of terminally differentiated plasma cells. It is the second most common hematologic malignancy behind non-Hodgkin lymphoma, with an annual incidence of 4–7 cases per 100,000 in the United States. Clinical symptoms are due to osteolysis of the bone, suppression of normal hematopoiesis, and the overproduction of monoclonal immunoglobulins that deposit in organ tissues. Clinical symptoms include bone pain and fractures, anemia, infections, hypercalcemia, edema, heart failure, and renal disease. Kidney involvement and pathology More than one-half of patients with MM initially present with varying degrees of AKI. Nearly 20% of patients present with a serum creatinine .2.0 mg/dL, and 10% of patients require dialysis on presentation (1). AKI is associated with higher mortality, but this may be reflective of patients with more advanced disease (2). The major diseases in the spectrum of myelomarelated kidney disease include cast nephropathy, light chain deposition disease (LCDD), and AL-amyloidosis. Renal biopsy demonstrates the presence of monotypic light chains on immunofluorescence exam, as well as characteristic ultrastructural features of deposits on electron microscopy. Less common forms of renal injury include light chain–induced Fanconi syndrome, cryoglobulinemia, proliferative glomerulonephritis, heavy chain deposition disease, and immunotactoid glomerulonephritis (Table 1) (3).3 Cast nephropathy Cast nephropathy has been diagnosed in 41% of patients with MM and renal disease (4). Excess light American Society of Nephrology chains precipitate with Tamm-Horsfall protein (THP) secreted by the thick ascending limb of the loop of Henle and produce casts in the distal tubule. Decreased GFR may increase the concentration of light chains in the distal tubule and enhance the formation of casts. Therefore, hypercalcemia, volume depletion, diuretics, and nonsteroidal antiinflammatory drugs can exacerbate renal injury. In some cases of AKI associated with MM, cast formation is rare on renal biopsy. Instead, renal injury is attributed to the direct toxic effects of urinary free light chains (FLCs) on proximal tubule cells (5,6). After reabsorption, lysosomal degradation of FLCs can activate the NF-kB pathway leading to oxidative stress with an inflammatory response, apoptosis, and fibrosis. This lesion is characterized histologically by loss of brush border and cell vacuolization and necrosis (7). The classic presentation is an elderly patient with unexplained renal failure, anemia, and bone pain or fractures. Proteinuria, when quantitatively measured with a 24-hour urine collection, is usually subnephrotic and primarily composed of monoclonal light chains (Bence-Jones proteins). The qualitative measurement of proteinuria using a urine test strip, which mainly detects albumin, is generally minimally reactive. Most patients with myeloma cast nephropathy are diagnosed without kidney biopsy using serum and urine immunofixation and serum FLC analysis (see Chapter 9). When biopsied, casts are eosin positive, fractured, and waxy in appearance on light microscopy (Figure 1) (8). Multinucleated giant cells may surround casts, and an interstitial Correspondence: Kevin W. Finkel, MD, FACP, FASN, FCCM, Division of Renal Diseases & Hypertension, UTHealth Science Center at Houston-McGovern Medical School, 6431 Fannin St., Houston, Texas 77030. Email: kevin.w.fi[email protected] Copyright © 2016 by the American Society of Nephrology Onco-Nephrology Curriculum 1 Table 1. Kidney manifestations of multiple myeloma Myeloma cast nephropathy AL amyloidosis Light chain deposition disease Heavy chain deposition disease Immunotactoid glomerulopathy Fibrillary glomerulopathy Light chain Fanconi syndrome Plasma cell infiltration Cryoglobulinemia Membranoproliferative glomerulonephritis Membranous glomerulonephritis inflammatory infiltrate composed of lymphocytes and monocytes may also be present. Widespread tubular atrophy and interstitial fibrosis eventually develops. Immunofluorescence staining generally demonstrates light chain restriction within the casts, although patterns may be mixed or nondiagnostic as well. Casts have a lattice-like appearance and may contain needle-shaped crystals on electron microscopy. The glomeruli and vessels appear normal, unless LCDD is concurrently present. Light chain deposition disease LCDD has been diagnosed at autopsy in 19% of patients with MM and renal disease (4). The renal manifestations are most apparent clinically, whereas light chain deposits within the heart, liver, spleen, and peripheral nervous system may remain asymptomatic. The hallmark of the disease is the development of mesangial nodules from mesangial matrix expansion secondary to the up-regulation of platelet derived growth factor-b and transforming growth factor-b. The nodules can occasionally be confused with the Kimmestiel-Wilson lesion of diabetic nephropathy. Clinically, patients present with proteinuria, renal insufficiency, and a nodular sclerosing glomerulopathy. Several retrospective reviews have reported on the clinical characteristics of these patients (9,10). The mean age was 58 years with no significant preference with respect to sex. Marked renal insufficiency was common on presentation, with a median serum creatinine .4 mg/dL, and renal function rapidly declined thereafter. Nephrotic range proteinuria was detected in 26%–40% of patients and correlated with the degree of glomerular involvement. Hypertension and microscopic hematuria were also present in the majority of patients. Light chain deposition stimulates mesangial and matrix expansion leading to nodule formation. On light microscopy, mesangial nodules are more uniform in distribution and size in LCDD compared with diabetic nephropathy (Figure 2) (8). Irregular thickening and double contours of the glomerular basement membrane may also be present. Eosin-positive deposits may be seen diffusely throughout the tubular basement membranes. Immunofluorescence demonstrates a characteristic linear staining of basement membranes with monotypic Figure 1. Pathology of myeloma cast nephropathy. (A) Large atypical casts are seen within distal tubular lumina. The casts appear hypereosinophilic with fractured texture and sharp edges. They are associated with giant cell (arrows) and mononuclear cell reaction, acute proximal tubular cell injury, and interstitial inflammation (hematoxylin and eosin, 3,2003). (B) The casts characteristically are periodic acid–Schiff (PAS) negative (3,2003). (C) Myeloma casts almost always stain for just k or l light chain. This figure shows bright staining of casts for k (immunofluorescence, 3,4003). The casts were negative for l (data not shown). (D) Occasionally, myeloma casts are composed of small rod- or needle-shaped crystals that fill the distal tubular lumina and, as shown, appear highly electron dense on electron microscopy (31,8503). Reprinted from reference 8, with permission of the Elsevier Science and Technology Journals. 2 Onco-Nephrology Curriculum American Society of Nephrology Figure 2. Pathology of light-chain proximal tubulopathy. (A) Large rod- and rhomboid-shaped hypereosinophilic crystals are seen within proximal tubular cells. (hematoxylin and eosin, 3,6003). (B) In this patient with smoldering myeloma who has 20% k-restricted plasma cells in the bone marrow, the proximal tubular crystals stain strongly for k (as shown) with negative l (not shown) by immunofluorescence performed on pronase-digested, paraffin-embedded tissue. The intracellular crystals failed to stain for k or l on standard immunofluorescence on frozen tissue (not shown; 3,4003). (C) Ultrastructurally, the proximal tubular cells are filled with electron-dense light-chain crystals with rod, rhomboid, or rectangular shapes. The proximal tubule brush border appears preserved (31,8503). (D) In this case of light-chain proximal tubulopathy, the proximal tubular cells are loaded with large election-dense phagolysosomes without crystals (electron microscopy, 32,5003). Reprinted from reference 8, with permission of the Elsevier Science and Technology Journals. light chains, which are most commonly k restricted. On electron microscopy, granular-powdery deposits are distributed within the mesangium and midportion of the glomerular, tubular, and vessel wall basement membranes. Therapy of LCDD is directed at the underlying myeloma. Cytotoxic chemotherapy followed by hematopoietic stem cell transplantation (HCT) has met with good success (11,12). AL amyloidosis AL amyloidosis occurs when pathogenic light chains unfold and deposit as insoluble fibrils extracellularly within tissues. It is found in up to 15% of patients with MM on autopsy (13,14). In 40% of patients with AL amyloidosis, the bone marrow will have .10% plasma cells, although only 10% will meet other criteria for MM (15). Amyloid fibrils may deposit within any organ, but most commonly affect the kidneys, heart, liver, and peripheral nervous system. Patients often present with fatigue, weight loss, and nephrotic syndrome. The clinical characteristics of patients with biopsyproven renal amyloidosis were described in a retrospective review of 84 patients at the Mayo Clinic (15). The median age at diagnosis was 61 years, and 62% were male. The median serum creatinine on presentation was 1.1 mg/dL. The majority of patients had nephrotic syndrome (86%) with a median 24-hour American Society of Nephrology protein loss of 7 g/day. RRT was eventually required in 42% of patients, and median survival after starting dialysis was less than 1 year. In general, cardiac involvement occurs in nearly one-third of patients and portends a poor prognosis. AL amyloid presents as an amorphic hyaline substance within the mesangium, glomerular basement membranes, and vessel walls. Mesangial involvement may be diffuse or nodular. Amyloid stains positive for Congo red and reveals a characteristic apple-green birefringence under polarized light. Immunofluorescence staining reveals the underlying monotypic light chain, which has a l: k ratio of 6:1. Electron microscopy demonstrates nonbranching randomly oriented 8- to 10-nm fibrils (Figure 3) (8). Amyloid deposits may appear as subepithelial spikes along the basement membrane similar to membranous nephropathy. Treatment with high-dose melphalan followed by HCT increases hematologic response and overall median survival (16). Improvement in renal function highly correlates with increased survival (17). Treatment of cast nephropathy General measures Volume resuscitation to assure optimum hemodynamic support and adequate urine output (;3 L/day) are of critical Onco-Nephrology Curriculum 3 Figure 3. Pathology of kidney AL amyloidosis. (A) There is extensive global mesangial and segmental glomerular capillary wall deposition of acellular, PAS-negative amyloid deposits. For comparison, the hyaline casts depicted are PAS positive (3,4003). (B) By definition, amyloid deposits should be Congo red positive (i.e., stain red). The figure shows global glomerular and extensive interstitial Congo red–positive amyloid. (3,2003). (C) Congophilic amyloid deposits characteristically show an apple-green birefringence when viewed under polarized light (3,2003). (D) On immunofluorescence, amyloid deposits appear smudgy and only stain for one of the light chains. The figure shows global mesangial and segmental glomerular capillary wall staining for l (3,4003). Staining for k was negative (not shown). (E) A distinctive feature of kidney AL amyloidosis is glomerular amyloid spicules, which result from parallel alignment of amyloid fibrils in the subepithelial space perpendicular to the glomerular basement membrane (electron microscopy, 320,0003). (F) On high magnification, amyloid fibrils appear haphazardly oriented and measure between 7 and 12 nm in diameter (electron microscopy, 326,0003). AL, immunoglobulin light chain; PAS, periodic acid–Schiff. Reprinted from reference 8, with permission of the Elsevier Science and Technology Journals. importance in the initial management. Based on experimental evidence that furosemide promotes intratubular cast formation by increasing sodium delivery to the distal tubule, the use of loop diuretics should be avoided unless there is volume overload. Hypercalcemia should be aggressively treated because it can lead to renal vasoconstriction, volume depletion, and enhanced cast formation. It has been suggested that urinary alkalinization decreases cast formation by reducing the net positive charge of FLCs and the interaction with THP (18). However, there is no clinical data supporting this approach. Given the risk of causing renal calcium precipitation in the setting of hypercalcemia, urinar y 4 Onco-Nephrology Curriculum alkalinization cannot be recommended. Colchicine was shown to reduce cast formation through decreasing THP secretion and binding in rats, but human studies have been disappointing (19,20). Chemotherapy and stem cell transplantation The key to treating myeloma cast nephropathy is rapid reduction in FLC concentrations. An early decrease in FLC levels is associated with the highest rate of renal recovery. In severe AKI due to cast nephropathy, a 60% reduction in FLC levels by day 21 after diagnosis is associated with renal recovery in 80% of cases (21). Previous studies with conventional American Society of Nephrology chemotherapy protocols demonstrated that high-dose dexamethasone rapidly reduced FLCs. Newer, novel agents such as thalidomide and the proteasome inhibitor, bortezomib, also rapidly lower FLC concentrations; this has been referred to as “renoprotective chemotherapy.” Significant improvement in renal dysfunction has been reported for MM patients treated with bortezomib-based regimens (22–24). Reversal of renal dysfunction with bortezomib may be more frequent and rapid than with other agents, based on observational analysis. No dose adjustment for renal function is necessary for bortezomib. Thalidomide and lenalidomide are two related chemotherapeutic agents commonly used in the treatment of MM. Lenalidomide dose must be adjusted for renal dysfunction (25). Thalidomide is not dependent on renal function for clearance and does not require dose adjustment for renal function; however, it may predispose to hyperkalemia in the setting of renal failure (26,27). Regimens with thalidomide or lenalidomide have shown superior effectiveness to traditional therapy with alkylating agents in terms of reversing renal dysfunction in MM; these agents may be nearly as effective as bortezomib regimens (28). Their effects are likely due to rapid lowering of serum FLC levels. Hematopoietic stem cell transplantation is an important and potentially curative therapy in MM; however, patient selection criteria are stringent, and significant renal dysfunction has traditionally excluded patients from transplantation. Recent studies have shown that HCT may be safe and effective in highly selected patients with renal failure (29). Extracorporeal removal of free light chains Light chains are small-molecular-weight proteins. k light chains usually circulate as monomers with a molecular weight of 22.5 kDa, whereas l light chains are typically dimeric with a molecular weight of 45 kDa (30). Because of their size, there has been a keen interest in the use of extracorporeal therapy as a means of FLC removal. A. Therapeutic plasma exchange (TPE): Several small trials initially suggested that TPE was effective in rapidly lowering FLC concentrations and improving renal function. However, these studies were small, single center, and underpowered. The largest randomized controlled trial of TPE did not demonstrate any benefit in patients with cast nephropathy (31). This study assessed the benefit of five to seven TPE sessions in 104 patients (30% were dialysis requiring) with presumed cast nephropathy (not all patients had biopsy confirmation). There was no difference in the two groups with respect to the composite outcome of death, dialysis, or reduced renal function at 6 months. This lack of benefit may be related to the volume of distribution of FLCs. Based on their molecular weights, 85% of light chains are confined to the extravascular space (32). Therefore, a traditional 2-hour TPE session would be ineffective in removing significant amounts of FLCs because of the excessive rebound effect. Most of the previous trials were performed prior to the availability of bortezomib-containing regimens. In a recent study of American Society of Nephrology 14 patients with presumed myeloma kidney treated with bortezomib and TPE, 12 had complete or partial renal response by 6 months; however, there were no control patients (33). Although there is still interest in TPE as a therapy for cast nephropathy, its routine use cannot be recommended based on the current evidence. B. High cutoff hemodialysis (HCO-HD): More recently interest has developed for another method of extracorporeal removal of FLCs: HCO-HD. In this technique, a hemofilter with a large pore size (45 kDa) is used for extended periods of time to remove FLCs. In the largest study of dialysis dependent renal failure secondary to MM, 67 patients were treated with HCO-HD and chemotherapy (34). Only 57% of patients had a renal biopsy, of which 87% had cast nephropathy. Most patients (85%) received combination chemotherapy with dexamethasone and either bortezomib or thalidomide. The median number of HCO-HD sessions was 11, and all patients had extended (.4 hour) treatments. Overall, 63% of the patients became dialysis independent. The factors that predicted renal recovery were the degree of FLC reduction at days 12 and 21 and the time to initiating HCO-HD. Unfortunately, this trial did not have a control group to assess the benefit of HCO-HD compared with renoprotective chemotherapy alone. It is not known whether HCO-HD offers any additional benefit over current chemotherapeutic regimens. Randomized controlled trials proving the benefit of adding HCO-HD to patients with cast nephropathy treated with current chemotherapy will be necessary before its routine use can be recommended. LEUKEMIA AND LYMPHOMA The development of kidney disease is common in patients with lymphoma and leukemia. As with all hospitalized patients, those with lymphoma and leukemia are at risk for developing AKI from hypotension, sepsis, or administration of radiocontrast, antifungal, and antibacterial agents. With the presence of cancer, renal injury can also result from chemotherapy, immunosuppressive drugs, hematopoietic stem cell transplantation, or tumor lysis syndrome. Furthermore, patients are at risk for renal syndromes specific to the presence of lymphoma or leukemia including various forms of paraneoplastic glomerulopathies, electrolyte disorders, urinary tract obstruction, lysozymuria, leukostasis, and infiltration of renal parenchyma (Table 2). Several of these manifestations are discussed elsewhere in the Curriculum. Lymphomatous infiltration Background Renal involvement in lymphoma is often clinically silent so patients can present with slowly progressive CKD attributed to other etiologies. Therefore, a high index of suspicion is needed to make a diagnosis. Patients may present with AKI, but this is rare and is most commonly seen in highly malignant and Onco-Nephrology Curriculum 5 Table 2. Kidney manifestations of lymphoma and leukemia Comorbid factors Sepsis and shock Hypotension Volume depletion Radiocontrast administration Chemotherapeutic agents Anti-infective agents Drug- induced tubulointerstitial nephritis Hematopoietic stem cell transplantation Radiotherapy Disease-related factors Tumor lysis syndrome Thrombotic microangiopathy Malignancy-associated hypercalcemia Paraneoplastic glomerulopathies Urinary tract obstruction Parenchymal infiltration Lysozymuria Leukostasis disseminated disease (35–38). Other presentations include proteinuria in both the nephrotic and nonnephrotic range, as well as a variety of glomerular lesions including pauciimmune crescentic glomerulonephritis (39). Patients may also present with flank pain and hematuria. Although a variety of cancers can metastasize to the kidneys and invade the parenchyma, the most common malignancies to do so are lymphomas and leukemia. The true incidence of renal involvement is unknown because it is usually a silent disease and only occasionally causes renal impairment. Autopsy studies suggest renal involvement occurs in 90% of patients with lymphoma, whereas radiographic evidence is significantly lower. The cause of impaired renal function from lymphomatous infiltration is poorly understood. Based on biopsy series, patients who present with AKI have predominantly bilateral interstitial infiltration of the kidneys with lymphoma cells and uniformly have increased renal size on radiographic imaging (40). These findings suggest that increased interstitial pressure results in reduced intrarenal blood flow with subsequent renal tubular injury. Patients who present with proteinuria, on the other hand, often have intraglomerular infiltration with lymphoma (40). It is not known how proteinuria develops in these cases, but the local release of permeability factors and cytokines has been suggested (41,42). Diagnosis The diagnosis of lymphomatous infiltration is necessarily one of exclusion because more common explanations are often present. Renal ultrasonography or computed tomography (CT) scan may reveal diffusely enlarged kidneys sometimes with multiple focal lesions (Figure 4) (43). However, many times, radiology will be unrevealing. In a study of 668 consecutive patients with lymphoproliferative disease who underwent diagnostic imaging with a CT scan, only 3% with 6 Onco-Nephrology Curriculum Figure 4. Computed tomography scan of abdomen and pelvis without radiocontrast showing bilateral kidney enlargement. Reprinted from reference 43, with permission of the Elsevier Science and Technology Journals. non-Hodgkin lymphoma were found to have kidney abnormalities (44). Both diffuse enlargement and solitary lesions were detected. This discrepancy between radiologic and autopsy/ histopathologic results may due to the fact that renal involvement is often indolent and only detectable in histopathologic examination (Figure 5) (43). Due to increased metabolic activity within lymphomatous deposits, positron emission tomography may be a more sensitive imaging technique (45). Although definitive diagnosis depends on renal biopsy, this procedure often is impossible because of contraindications. In such cases, the following criteria support the diagnosis of kidney disease due to lymphomatous infiltration: 1) renal enlargement without obstruction; 2) absence of other causes of kidney disease; and 3) rapid improvement of kidney function after radiotherapy or systemic chemotherapy. Treatment The treatment of lymphomatous involvement of the kidney is directed at the underlying malignancy. There are numerous case reports of improvement in renal function after initiation of antitumor therapy. In indolent malignant disease that is usually treated by observation alone, kidney involvement is an indication for starting systemic therapy. Leukemic infiltration Background Leukemia cells can infiltrate any organ, and the kidneys are the most frequent extramedullary site of infiltration. Autopsy studies reveal that 60%–90% of patients have renal involvement (46). On biopsy, cells are usually located in the renal interstitium, although occasional glomerular lesions are noted (47). Increased interstitial pressure leads to vascular and tubular compression and subsequent tubular injury. Occasional nodular lesions are found, but this is more common with lymphoma. American Society of Nephrology Figure 5. (A) Kidney biopsy stained with hematoxylin and eosin at 6033 magnification. (B) Tissue stained with B cell–specific marker anti-CD38 at 20033 magnification. Reprinted from reference 43, with permission of the Elsevier Science and Technology Journals. Clinical Features Leukemic infiltration of the kidneys is often an indolent and clinically silent disease. Most often it is incidentally noted after autopsy or by detection of renal enlargement on ultrasound or CTscan. Although uncommon, many cases of AKI attributable to leukemic infiltration have been described (48–50). Patients may also experience hematuria or proteinuria. Occasionally renal enlargement is accompanied by flank pain or fullness. There are also reports of patients with chronic lymphocytic leukemia who develop AKI from leukemic infiltration and are infected with polyomavirus (BK) (51). Urine from patients demonstrates viral inclusions in tubular cells (“decoy” cells) and blood is positive for BK viral DNA. Therefore, in leukemia patients with AKI considered due to leukemic infiltration, evidence for coexisting BK virus infection should be sought. Diagnosis The diagnosis of leukemia infiltration as a cause of AKI requires a high level of vigilance because it is often clinically silent, and leukemic patients usually have multiple alternative explanations for renal injury. A presumptive diagnosis can be made if there is no other obvious cause of AKI, bilateral renal enlargement is demonstrated radiographically, and there is prompt improvement in renal function after chemotherapy. Screening for leukemic infiltration with radiographic imaging is not revealing. In a study of 668 consecutive patients with lymphoproliferative disease who underwent diagnostic imaging with a CT scan, only 5% with leukemia were found to have kidney abnormalities (45). As with lymphoma, this discrepancy between radiologic and autopsy/histopathologic results may be due to the fact that renal involvement is often indolent and only detectable during histopathologic examination. Treatment Treatment is directed by the type of leukemia. Although some patients do not recover, in the majority of cases, renal function does improve as the leukemia responds to systemic treatment. American Society of Nephrology Lysozymuria Lysozyme is a cationic protein produced by macrophages and monocytes and released in response to bacterial infection. It is freely filtered by the glomerulus and reabsorbed by the proximal tubule. In certain leukemias, clonal expansion leads to an excessive production of lysozyme and subsequent proximal tubular injury and AKI (52). Damage to the proximal tubule reduces reabsorption and can result in Fanconi syndrome and nephrotic range proteinuria. The presence of lysozymuria can be confirmed by detection of an increased g globulin level on serum and urine protein electrophoresis with immunofixation negative for monoclonal gammopathy (53). Treatment is directed at the underlying malignancy. Leukostasis Patients with myeloid leukemia and exceedingly high white blood cell counts (usually in excess of 100,000 cells/mm3) can develop organ dysfunction due to intravascular aggregation of leukemic cells. The pulmonary and cerebral circulations are the most severely affected, although there are case reports of patients developing AKI (54,55). Leukemic cells occlude the peritubular and glomerular capillaries, thereby reducing GFR. Patients may be oliguric, but their renal function often improves with therapeutic leukopheresis or chemotherapy. Leukostasis is thought to result from the abnormal morphology of blast cells and the hyperviscocity of the serum. It has been described in both acute and chronic leukemia. Treatment is directed at treatment of the malignancy with appropriate chemotherapeutic regimens; in severe cases, therapeutic leukopheresis can rapidly lower cell counts. CONCLUSION Numerous kidney diseases are associated with hematologic malignancies unique to this population (Tables 1 and 2). The most common cause of kidney injury in MM patients is cast nephropathy (myeloma kidney). Rapid reduction of Onco-Nephrology Curriculum 7 circulating free light chains with newer “renoprotective chemotherapy” can reverse renal failure in the majority of cases. Although infiltration of the kidneys by leukemia or lymphoma is almost universal histologically, clinical renal disease is uncommon. Given the myriad of potential kidney insults in these patients, a high index of suspicion is necessary to diagnose infiltration as the cause of renal dysfunction. 13. 14. TAKE HOME POINTS 15. c Cast nephropathy is the most common cause of renal failure in patients with multiple myeloma. 16. c Rapid lowering of free light chains with “renoprotective” chemotherapy can reverse renal failure from cast nephropathy in the majority of cases. c Based on current evidence, extracorporeal removal of free light chains with either therapeutic plasmapheresis or HCO hemodialysis cannot be 17. recommended. c Leukemic and lymphomatous infiltration of the kidneys is common on autopsy, although it is usually not clinically apparent. c Enlarged kidneys on imaging and resolution of AKI after therapy with 18. systemic chemotherapy or radiation support the diagnosis of kidney infiltration. REFERENCES 1. Kyle RA, Gertz MA, Witzig TE, Lust JA, Lacy MQ, Dispenzieri A, Fonseca R, Rajkumar SV, Offord JR, Larson DR, Plevak ME, Therneau TM, Greipp PR. Review of 1027 patients with newly diagnosed multiple myeloma. Mayo Clinic Proc 78: 21–33, 2003 2. Eleutherakis-Papaiakovou V, Bamias A, Gika D, et al. Renal failure in multiple myeloma: Incidence, correlations, and prognostic significance. Leukemia Lymphoma 48: 337–341, 2007 3. Markowitz GS. Dysproteinemia and the kidney. Adv Anatomic Pathol 11: 49–63, 2004 4. Montseny JJ, Kleinknecht D, Meyrier A, Vanhille P, Simon P, Pruna A, Eladari D. Long-term outcome according to renal histological lesions in 118 patients with monoclonal gammopathies. Nephrol Dialysis Transplant 13:1438–1445, 1998 5. Sanders PW. Mechanisms of light chain injury along the tubular nephron. J Am Soc Nephrol 23: 1777–1781, 2012 6. Batuman V. Proximal tubular injury in myeloma. Contribut Nephrol 153: 87–104, 2007 7. Sanders PW, Herrera GA, Lott RL, Galla JH. Morphologic alterations of the proximal tubules in light chain-related renal disease. Kidney Int 33: 881–889, 1988 8. Leung N, Nasr SH. Myeloma-related kidney disease. Adv Chronic Kidney Dis 21: 36–47, 2014 9. Lin J, Markowitz GS, Valeri AM, Kambham N, Sherman WH, Appel GB, D’Agati V D. Renal monoclonal immunoglobulin deposition disease: The disease spectrum. J Am Soc Nephrol 12:1482–1492, 2001 10. Pozzi C, D’Amico M, Fogazzi GB, Curioni S, Ferrario F, Pasquali S, Quattrocchio G, Rollino C, Segagni S, Locatelli F. Light chain deposition disease with renal involvement: Clinical characteristics and prognostic factors. Am J Kidney Dis 42: 1154–1163, 2003 11. Girnius S, Seldin DC, Quillen K, Dember LM, Segal A, Sanchorawala V. Long-term outcome of patients with monoclonal Ig deposition disease treated with high-dose melphalan and stem cell transplantation. Bone Marrow Transplant 46: 161–162, 2011 12. Hassoun H, Flombaum C, D’Agati VD, Rafferty BT, Cohen A, Klimek VM, Boruchov A, Kewalramani T, Reich L, Nimer SD, Comenzo RL. 8 Onco-Nephrology Curriculum 19. 20. 21. 22. 23. 24. 25. 26. 27. 28. High-dose melphalan and auto-SCT in patients with monoclonal Ig deposition disease. Bone Marrow Transplant 42: 405–412, 2008 Ivanyi B. Frequency of light chain deposition nephropathy relative to renal amyloidosis and Bence Jones cast nephropathy in a necropsy study of patients with myeloma. Arch Pathol Lab Med 114: 986–987, 1990 Oshima K, Kanda Y, Nannya Y, Kaneko M, Hamak T, Suguro M, Yamamoto R, Chizuka A, Matsuyama T, Takezako N, Miwa A, Togawa A, Niino H, Nasu M, Saito K, Morita T. Clinical and pathologic findings in 52 consecutively autopsied cases with multiple myeloma. Am J Hematol 67: 1–5, 2001 Kyle RA, Gertz MA. Primary systemic amyloidosis: Clinical and laboratory features in 474 cases. Semin Hematol 32: 45–59, 1995 Cibeira MT, Sanchorawala V, Seldin DC, Quillen K, Berk JL, Dember LM, Segal A, Ruberg F, Meier-Ewert H, Andrea NT, Sloan JM, Finn KT, Doros G, Blade J, Skinner M. Outcome of AL amyloidosis after highdose melphalan and autologous stem cell transplantation: Long-term results in a series of 421 patients. Blood 118: 4346–4352, 2011 Leung N, Dispenzieri A, Lacy MQ, Kumar SK, Hayman SR, Fervenza FC, Cha SS, Gertz MA. Severity of baseline proteinuria predicts renal response in immunoglobulin light chain-associated amyloidosis after autologous stem cell transplantation. Clin J Am Soc Nephrol 2: 440– 444, 2007 Holland MD, Galla JH, Sanders PW, Luke RG. Effect of urinary pH and diatrizoate on Bence Jones protein nephrotoxicity in the rat. Kidney Int 27: 46–50, 1985 Sanders PW, Booker BB, Bishop JB, Cheung HC. Mechanisms of intranephronal proteinaceous cast formation by low molecular weight proteins. J Clin Invest 85: 570–576, 1990 Cairns HS, Dawnay A, Woolfson RG, Unwin RJ. Evaluation of therapy for cast nephropathy: Failure of colchicine to alter urinary Tamm Horsfall glycoprotein excretion in normal subjects. Exper Nephrol 2:257–258, 1994 Hutchison CA, Cockwell P, Stringer S, Bradwell A, Cook M, Gertz MA, Dispenzieri A, Winters JL, Kumar S, Rajkumar SV, Kyle RA, Leung N. Early reduction of serum-free light chains associates with renal recovery in myeloma kidney. J Am Soc Nephrol 22: 1129–1136, 2011 Moreau P, Richardson PG, Cavo M, Orlowski RZ, San Miguel JF, Palumbo A, Harousseau JL. Proteasome inhibitors in multiple myeloma: 10 years later. Blood 120: 947–959, 2012 Ludwig H, Adam Z, Hajek R, Greil R, Tothova E, Keil F, Autzinger EM, Thaler J, Gisslinger H, Lang A, Egyed M, Womastek I, Zojer N. Light chain-induced acute renal failure can be reversed by bortezomibdoxorubicin-dexamethasone in multiple myeloma: Results of a phase II study. J Clin Oncol 28: 4635–4641, 2010 Dimopoulos MA, Roussou M, Gkotzamanidou M, Nikitas N, Psimenou E, Mparmparoussi D, Matsouka C, Spyropoulou-Vlachou M, Terpos E, Kastritis E. The role of novel agents on the reversibility of renal impairment in newly diagnosed symptomatic patients with multiple myeloma. Leukemia 27: 423–429, 2013 Chen N, Lau H, Kong L, Kumar G, Zeldis JB, Knight R, Laskin OL. Pharmacokinetics of lenalidomide in subjects with various degrees of renal impairment and in subjects on hemodialysis. J Clin Pharmacol 47: 1466–1475, 2007 Eriksson T, Hoglund P, Turesson I, Waage A, Don BR, Vu J, Scheffler M, Kaysen GA. Pharmacokinetics of thalidomide in patients with impaired renal function and while on and off dialysis. J Pharm Pharmacol 55: 1701–1706, 2003 Harris E, Behrens J, Samson D, Rahemtulla A, Russell NH, Byrne JL. Use of thalidomide in patients with myeloma and renal failure may be associated with unexplained hyperkalaemia. Br J Haematol 122: 160– 161, 2003 Roussou M, Kastritis E, Christoulas D, Migkou M, Gavriatopoulou M, Grapsa I, Psimenou E, Gika D, Terpos E, Dimopoulos MA. Reversibility of renal failure in newly diagnosed patients with multiple myeloma and the role of novel agents. Leukemia Res 34: 1395–1397, 2010 American Society of Nephrology 29. Parikh GC, Amjad AI, Saliba RM, Kazmi SM, Khan ZU, Lahoti A, Hosing C, Mendoza F, Qureshi SR, Weber DM, Wang M, Popat U, Alousi AM, Champlin RE, Giralt SA, Qazilbash MH. Autologous hematopoietic stem cell transplantation may reverse renal failure in patients with multiple myeloma. Biol Blood Marrow Transplant 15: 812–816, 2009 30. Pratt G. The evolving use of serum free light chain assays in haematology. Br J Haematol 141: 413–422, 2008 31. Clark WF, Stewart AK, Rock GA, Sternbach M, Sutton DM, Barrett BJ, Heidenheim AP, Garg AX, Churchill DN, Canadian Apheresis G. Plasma exchange when myeloma presents as acute renal failure: a randomized, controlled trial. Ann Intern Med 143: 777–784, 2005 32. Hutchison CA, Cockwell P, Reid S, Chandler K, Mead GP, Harrison J, Hattersley J, Evans ND, Chappell MJ, Cook M, Goehl H, Storr M, Bradwell AR. Efficient removal of immunoglobulin free light chains by hemodialysis for multiple myeloma: In vitro and in vivo studies. J Am Soc Nephrol 18: 886–895, 2007 33. Burnette BL, Leung N, Rajkumar SV. Renal improvement in myeloma with bortezomib plus plasma exchange. New Engl J Med 364: 2365– 2366, 2011 34. Hutchison CA, Heyne N, Airia P, Schindler R, Zickler D, Cook M, Cockwell P, Grima D. Immunoglobulin free light chain levels and recovery from myeloma kidney on treatment with chemotherapy and high cut-off haemodialysis. Nephrol Dialysis Transplant 27: 3823– 3828, 2012 35. Kanfer A, Vandewalle A, Morel-Maroger L, Feintuch MJ, Sraer JD, Roland J. Acute renal insufficiency due to lymphomatous infiltration of the kidneys: Report of six cases. Cancer 38: 2588–2592, 1976 36. Koolen MI, Schipper P, v Liebergen FJ, Kurstjens RM, v Unnik AJ, Bogman MJ. Non-Hodgkin lymphoma with unique localization in the kidneys presenting with acute renal failure. Clin Nephrol 29: 41–46, 1988 37. Malbrain ML, Lambrecht GL, Daelemans R, Lins RL, Hermans P, Zachee P. Acute renal failure due to bilateral lymphomatous infiltrates. Primary extranodal non-Hodgkin’s lymphoma (p-EN-NHL) of the kidneys: Does it really exist? Clin Nephrol 42: 163–169, 1994 38. Miyake JS, Fitterer S, Houghton DC. Diagnosis and characterization of non-Hodgkin’s lymphoma in a patient with acute renal failure. Am J Kidney Dis 16: 262–263, 1990 39. Henriksen KJ, Hong RB, Sobrero MI, Chang A. Rare association of chronic lymphocytic leukemia/small lymphocytic lymphoma, ANCAs, and pauci-immune crescentic glomerulonephritis. Am J Kidney Dis 57: 170–174, 2011 40. Tornroth T, Heiro M, Marcussen N, Franssila K. Lymphomas diagnosed by percutaneous kidney biopsy. Am J Kidney Dis 42: 960–971, 2003 American Society of Nephrology 41. D’Agati V, Sablay LB, Knowles DM, Walter L. Angiotropic large cell lymphoma (intravascular malignant lymphomatosis) of the kidney: Presentation as minimal change disease. Human Pathol 20: 263–268, 1989 42. Agar JW, Gates PC, Vaughan SL, Machet D. Renal biopsy in angiotropic large cell lymphoma. Am J Kidney Dis 24: 92–96, 1994 43. Luciano RL, Brewster UC. Kidney involvement in leukemia and lymphoma. Adv Chronic Kidney Dis 21: 27–35, 2014 44. Bach AG, Behrmann C, Holzhausen HJ, Katzer M, Arnold D, Spielmann RP, Surov A. Prevalence and patterns of renal involvement in imaging of malignant lymphoproliferative diseases. Acta Radiol 53: 343–348, 2012 45. Sheth S, Ali S, Fishman E. Imaging of renal lymphoma: patterns of disease with pathologic correlation. Radiographics 26: 1151–1168, 2006 46. Suh WM, Wainberg ZA, de Vos S, Cohen AH, Kurtz I, Nguyen MK. Acute lymphoblastic leukemia presenting as acute renal failure. Nature Clin Pract Nephrol 3: 106–110, 2007 47. Kowalewska J, Nicosia RF, Smith KD, Kats A, Alpers CE. Patterns of glomerular injury in kidneys infiltrated by lymphoplasmacytic neoplasms. Human Pathol 42: 896–903, 2001 48. Comerma-Coma MI, Sans-Boix A, Tuset-Andujar E, Andreu-Navarro J, Perez-Ruiz A, Naval-Marcos I. Reversible renal failure due to specific infiltration of the kidney in chronic lymphocytic leukaemia. Nephrol Dialysis Transplant 13: 1550–1552, 1998 49. Pagniez DC, Fenaux P, Delvallez L, Dequiedt P, Gosselin B, Tacquet A. Reversible renal failure due to specific infiltration in chronic lymphocytic leukemia. Am J Med 85: 579–580, 1988 50. Phillips JK, Bass PS, Majumdar G, Davies DR, Jones NF, Pearson TC. Renal failure caused by leukaemic infiltration in chronic lymphocytic leukaemia. J Clin Pathol 46: 1131–1133, 1993 51. Boudville N, Latham B, Cordingly F, Warr K. Renal failure in a patient with leukaemic infiltration of the kidney and polyomavirus infection. Nephrol Dialysis Transplant 16: 1059–1061, 2001 52. Patel TV, Rennke HG, Sloan JM, DeAngelo DJ, Charytan DM. A forgotten cause of kidney injury in chronic myelomonocytic leukemia. Am J Kidney Dis 54: 159–164, 2009 53. Levinson SS, Elin RJ, Yam L. Light chain proteinuria and lysozymuria in a patient with acute monocytic leukemia. Clin Chem 48: 1131–1132, 2002 54. McKee LC Jr, Collins RD. Intravascular leukocyte thrombi and aggregates as a cause of morbidity and mortality in leukemia. Medicine 53: 463–478, 1974 55. Dietrich PY, Pedraza E, Casiraghi O, Bayle C, Hayat M, Pico JL. Cardiac arrest due to leucostasis in a case of prolymphocytic leukaemia. Br J Haematol 78: 122–123, 1991 Onco-Nephrology Curriculum 9 REVIEW QUESTIONS 1. An 80-year-old patient with multiple myeloma presents with serum uric acid of 12.0 mg/dL, serum calcium of 11.0 mg/dL, and a phosphate of 8.1 mg/dL. Serum creatinine is 4.0 mg/dL, and free l light chains are 3,700 mg/L. Which ONE of the following therapies is contraindicated in this patient? a. b. c. d. Urinary alkalinization Intravenous saline Rasburicase Allopurinol Answer: a is correct. There is a lack of evidence to support the use of urinary alkalinization to decrease cast formation by reducing the net positive charge of FLCs. In addition, there is an increased risk of renal calcium precipitation with the presence of hypercalcemia. 2. A 57-year-old man with no medical history is diagnosed with chronic lymphocytic leukemia with Richter’s transformation and presents to the emergency room with creatinine of 3.61 mg/dL and white blood cell count of 20,000/ mm3. The patient has undergone a renal ultrasound indicating slightly enlarged kidneys 13 cm bilaterally. On examination he was noted to have hepatosplenomegaly. Urinalysis was negative for proteinuria and hematuria. He is planned for chemotherapy pending a renal consult for his rise in creatinine. Which one of the following likely explains his renal injury? a. Urinary obstruction related to retroperitoneal lymphadenopathy b. Tumor lysis syndrome c. Leukemic infiltration of the kidney d. Membranoproliferative glomerulonephritis with C3 and monoclonal immunoglobulin deposition Answer: c is correct. It is likely leukemic infiltration in the setting of enlarged kidney and hepatosplenomegaly with the lack of other possible etiologies for the patient’s renal dysfunction. 3. A 74-year-old man with chronic myelomonocytic leukemia with a long-standing history of hypertension and hyperlipidemia is seen in the clinic. Peripheral blood monocyte count is .1,000/mm3, and splenomegaly is noted. His creatinine has started to rise in the last 6 months to 2.0 mg/dL from a baseline of 1.3 mg/dL. Urinalysis indicated 31 protein and 31 glucose. His labs included the following: white blood cells, 50,000/mm3; hemoglobin, 9.0 g/dL; platelets, 60,000/mm3; serum potassium, 3.0 mEq/L; calcium, 8.0 mg/dL; phosphorus, 1.8 mg/dL; serum glucose, 80 mg/dL. Renal ultrasound showed normal size kidneys and no hydronephrosis. Which ONE of the following is the likely cause of this patient’s renal dysfunction? 10 Onco-Nephrology Curriculum a. b. c. d. Infiltrative disease secondary to underlying leukemia Membranous glomerulopathy secondary to neoplasm Renal vein thrombosis Lysozyme-induced kidney injury Answer: d is correct. Lysozyme-induced kidney injury has been underdiagnosed and underrecognized. Uncontrolled production of lysozyme secondary to underlying malignancy predisposes the patient to the damage to the proximal tubule reduces reabsorption of electrolytes and can result in Fanconi syndrome and nephrotic range proteinuria. A clue to Fanconi syndrome is glycosuria with normal serum glucose level. On electron microscopy, the kidney biopsies have shown an increase in number and size of lysosomes in the proximal tubules. 4. A 56-year-old woman with a history of diabetes and hypertension (HTN) presented to the hospital with AKI, calcium of 12.0 mg/dL, hemoglobin of 10.0 g/dL, and serum albumin of 3.5 g/dL. Urinalysis revealed a specific gravity 1.015 with trace protein on dipstick examination and occasional granular cast on microscopic examination. Renal ultrasonography revealed normal-sized kidneys without hydronephrosis. Creatinine was noted to be at 5 mg/dL, and blood urea nitrogen was 82 mg/dL with a normal baseline 6 months ago. Because there was a high suspicion for multiple myeloma, the patient had a serum free light chain (FLC) assay to determine monoclonality, and the free serum l light chains were 1,500 mg/L. Which of the following treatments is indicated for AKI due to cast nephropathy? a. Chemotherapy b. Hemodialysis using dialyzers that have a high-molecularweight cutoff c. Plasma exchange therapy d. All of the above Answer: a is correct. Based on the current evidence, performing plasma exchange and high cutoff hemodialysis to treat cast nephropathy cannot be recommended. Significant and rapid reductions in FLC concentrations using “renoprotective chemotherapy” have been shown of benefit in preserving renal function. 5. A 63-year-old man with a medical history of well-controlled diabetes and hypertension presents with a creatinine of 2.0 mg/dL, weight loss, fatigue, edema, and worsening peripheral neuropathy. He was noted to have a serum albumin level of 3.0 g/dL, and spot urine protein to creatinine ratio of 2 g, with no red blood cells in the urinalysis. Liver function studies were normal. He had a negative skeletal survey, and a bone marrow biopsy showed normal cellularity with 1% plasma cells. Serum protein electrophoresis indicated IgA l M-protein. Due to the possibility of amyloidosis, the patient underwent a kidney biopsy which showed a mesangial area expanded by amyloid fibrils. If the patient had AL-type American Society of Nephrology amyloidosis, what is the most appropriate intervention(s) that would improve overall survival? e. Chemotherapy plus extracorporeal removal of the Ig FLC a. Chemotherapy to reduce monoclonal protein overproduction b. Plasma exchange to reduce circulating FLC levels c. High-dose melphalan followed by autologous hematopoietic stem cell transplant d. Hemodialysis using dialyzers with high-molecularweight cutoff Answer: c is correct. Based on retrospective studies, patients with AL amyloidosis whom underwent stem cell transplant have been shown to have improved overall survival and improved quality of life compared with those undergoing chemotherapy alone. There is no role for plasma exchange or hemodialysis with a high cutoff filter in the treatment of amyloidosis. American Society of Nephrology Onco-Nephrology Curriculum 11 Chapter 8: Clinical Tests for Monoclonal Proteins Nelson Leung, MD Division of Nephrology and Hypertension, Hematology, Mayo Clinic, Rochester, Minnesotta INTRODUCTION SERUM PROTEIN ELECTROPHORESIS Monoclonal gammopathy is a hallmark of plasma cell dyscrasias and some B-cell lymphoproliferative disorders (1). They cover a wide spectrum of diseases from the premalignant condition monoclonal gammopathy of undetermined significance (MGUS) to symptomatic multiple myeloma, malignant lymphomas, and chronic lymphocytic leukemia (CLL). The monoclonal (M) proteins can be the entire immunoglobulin, light chain only, or, rarely, heavy chain only. Their ability to cause kidney disease is another characteristic they have in common. In a disease such as multiple myeloma, the risk of AKI correlates with the severity of disease and can be as high as 50% (2,3). In one study, 87% of patients with AKI had the most advanced stage (III) of disease according to the Durie Salmon classification (4). In fact, only 44% of the patients with stage III disease had normal renal function. Other less common causes of AKI in this population include interstitial nephritis and acute tubular necrosis (5–7). A number of glomerular and tubular lesions have also been described in myeloma patients; however, these lesions are actually more common in patients where the diagnostic criteria for multiple myeloma or lymphoma have not been met and are diagnosed with monoclonal gammopathy of renal significance (MGRS) (8). Patients with MGRSrelated kidney disease are more likely to present with proteinuria, hematuria, and mild renal impairment than rapid-onset AKI as in cast nephropathy. In either situation, the identification of a monoclonal protein changes the diagnosis, pathophysiology, and prognosis and directs the clinician toward a hematologic evaluation (9). Monoclonal protein testing should be a part of any workup of AKI, as well as proteinuria with mild reduction of renal function in adults. This article will review the current available tests for monoclonal proteins. Serum protein electrophoresis (SPEP) is the most commonly used laboratory test for the detection of monoclonal proteins. Serum proteins are loaded on to a gel or a capillary tube and are separated by an electrical current based on charge and size. The proteins are then stained for visualization. The proteins migrate into five zones or fractions. These are albumin, a1, a2, b, and g. The b fraction often has two peaks. Albumin is the most abundant protein in the serum and should make up the largest peak in normal serum. When a monoclonal (M) protein is present, a sharp band appears often in the g region. However, M proteins can migrate to the b or even the a fractions. This often occurs when the M protein is comprised of an IgA or free light chain (FLC). In some cases, no band is detected but instead there is a decrease in the g peak (10). The hypogammaglobulinemia is due to the monoclonal gammopathy. Currently, SPEP is the most commonly used test for M proteins globally because of its ease of use and relatively low cost. Fully automated systems are available for both the agarose gel and capillary tube methods, which have increased reproducibility and efficiency. Because SPEP is quantitative, it is used in both diagnostic and response criteria in multiple myeloma (11,12). Despite its utility, its detection limit is not sensitive enough as a single screening test, especially in low-burden diseases like MGRS. The detection limit for an M protein is 0.3–0.5 g/dL in the g region and up to 0.7 g/dL in the a or b region (13). SPEP is positive in 87.6% of multiple myelomas but only 73.8% of immunoglobulin light chain (AL) amyloidosis and 55.6% of light chain deposition disease (LCDD) (14). In addition, the M-band on the SPEP only indicates an M protein but it does not distinguish the isotypes. To find out the type, immunofixation is required. American Society of Nephrology Correspondence: Nelson Leung, Mayo Clinic, Rochester, Minnesotta: Q:1 Email: [email protected] Copyright © 2016 by the American Society of Nephrology Onco-Nephrology Curriculum 1 Table 1. Screening panel Condition SPEP SIFE SFLC UPEP/IFE MGRS AL/MIDD WM Multiple myeloma X X X X X X X X X X X X Immunofixation should be performed if a screening test is positive to help type the monoclonal protein. SPEP, serum protein electrophoresis; SIFE, serum immunofixation; SFLC, serum free light chain; U, urine; MGRS, monoclonal gammopathy of renal significance; AL, AL amyloidosis; MIDD, monoclonal immunoglobulin deposition disease; WM, Waldenström macroglobulinemia. URINE PROTEIN ELECTROPHORESIS The first person to recognize that monoclonal proteins have special properties in the urine was Dr. William MacIntyre (15). Urine of myeloma patients turns opaque when boiled, clears with the addition of acid, and turns opaque again as it cools. The property was attributed to a protein that was named after the physician who misidentified it: Dr. Henry Bence Jones. It was not until later that Bence-Jones protein was identified as immunoglobulin FLC. Urine protein electrophoresis (UPEP) uses the same principle as SPEP and has the same advantages and disadvantages. The urine M-spike is used for diagnosis and response determination in multiple myeloma (16). However, because M proteins are not always present in the urine of patients with monoclonal gammopathy, the sensitivity of UPEP is the lowest of all the tests and should never be used alone for screening. In a study with 2,799 patients of which 4.4% had a plasma cell dyscrasia, SPEP was positive in 94.4% of patients, whereas only 37.7% had a positive UPEP (17). This has led some to suggest that the serum FLC assay should replace the UPEP for screening (see below). However, despite the low sensitivity, UPEP does provide additional information. In patients with renal impairment, the presence of an M protein and low albumin concentration on the UPEP is highly suggestive of cast nephropathy, whereas a high albumin concentration is more likely the results of LCDD or AL amyloidosis (18). In addition, the presence of a monoclonal light chain in the urine significantly increases the risk of renal injury in myeloma patients (19). Finally, urine M-spike is still used in response determination in multiple myeloma (11). Therefore, UPEP remains a useful supplemental test in patients with paraproteinemia. Figure 1. Serum protein electrophoresis (PEL) and immunofixation (IFE). A monoclonal (M) spike was detected in the b region of the protein electrophoresis. The immunofixation identified a band in the IgA and k lanes that corresponded to the band in the b region of the protein electrophoresis. The M protein in this case is a monoclonal IgA k. The blurry smudge in IgG lane indicated that the IgG was polyclonal. monoclonal protein. In multiple myeloma, serum IFE increased the detection rate from 87.6% to 94.4%. Similarly, the sensitivity increases from 65.9% to 73.8% in AL amyloidosis. In a mixed group of patients, the sensitivity increases from 79% to 87%. Not only is the sensitivity improved with IFE, but the type of the monoclonal protein can be identified. IFE is qualitative and not quantitative. Therefore, for response determination in multiple myeloma and AL amyloidosis, it is only used for assessment of complete response. The extra reagents add significantly to the cost, making IFE less affordable. SERUM FLC ASSAY SERUM AND URINE IMMUNOFIXATION Samples are electrophoresed in parallel lanes in immunofixation (IFE). Antibodies against the heavy and light chains of the immunoglobulin are then applied to each lane separately. A reaction forming a sharp band would suggest the presence of monoclonal immunoglobulin component (Figure 1). The M protein composed of the entire monoclonal immunoglobulin would be positive for both a heavy chain and a light chain. The sensitivity of IFE has a detection limit of ;0.1 g/dL of 2 Onco-Nephrology Curriculum In the early 2000s, a new assay was introduced to aid in the detection of monoclonal proteins. Using antibodies against epitopes that are normally hidden in the intact immunoglobulin, the assay detects both k and l FLCs. It is quantitative and automated. The assay is not specific for monoclonal light chains but instead infers monoclonality when an abnormal ratio between the k and l FLCs is detected. The normal ratio is between 0.26 and 1.65. A high ratio suggests a k clone, whereas a ratio ,0.26 suggest a l clone. The addition of the serum FLC assay to SPEP and serum IFE has significantly increased the sensitivity of monoclonal protein American Society of Nephrology testing. Prior to the introduction of the FLC assay, up to 5% of multiple myelomas were thought to be nonsecretory. Using the FLC assay, 19 of 28 nonsecretory myeloma patients were found to have abnormal k to l ratios, and 4 had suppression of one or both FLCs (20). The FLC assay was able to identify abnormalities in 82% of patients classified as nonsecretory by serum and urine PEP and IFN. Most of these were light chain–only myeloma. Recently, the serum FLC ratio was added to the diagnostic criteria of multiple myeloma (12). In AL amyloidosis, the serum FLC assay increases the detection rate from 69% (with serum IFE alone) to 99% (21). The addition of UPEP did not identify any additional patients. Another study found the combination of serum IFE and serum FLC detected 100% of the patients with multiple myeloma, Waldenström macroglobulinemia, and smoldering multiple myeloma (14). The addition of UPEP did not increase the sensitivity for the above diagnoses but did assist in the identification of MGUS, extramedullary plasmacytoma, AL amyloidosis, and LCDD. In addition to diagnostic evaluation, serum FLC is also used in disease monitoring and assessment of response. The degree and speed of serum FLC reduction have been found to be the most important predictors of renal recovery in cast nephropathy (22). In AL amyloidosis, the reduction of serum FLC has been shown to be a better predictor of outcomes than the M-spike. To achieve stringent complete response in multiple myeloma and complete response in AL amyloidosis, the serum FLC ratio has to be normalized (16,23). Although the serum FLC assay increases the detection rate of monoclonal gammopathy, clinicians should be aware of its limitations. First, the assay does not distinguish between polyclonal FLCs and monoclonal FLCs. The higher (or lower) the k to l ratio, the more likely a monoclonal gammopathy exists. However, several conditions are known to cause minor abnormalities. The most common is renal insufficiency. Because FLCs are mostly cleared by the kidney, a reduction in glomerular filtration rate will cause a rise in the FLC levels. This rise, however, is not symmetric because k FLCs are cleared more readily than l. Thus, the asymmetric increase in FLCs results in an increase in the ratio. In patients with severe renal impairment, a renal reference range for the k to l ratio (0.37 to 3.1) has been recommended (24). Patients with autoimmune diseases can also have mildly abnormality k to l ratios. Finally, look for a biclonal gammopathy if there is elevation in both FLCs but the renal function is normal and autoimmune disorders have been ruled out. URINARY FLC The measurement and quantification of FLC can also be performed in the urine. An elevated k to l ratio suggests a k clone, and a depressed ratio suggests a l clone (25). Studies suggest that the urinary FLC can correlate with the serum FLC in an individual patient (26). However, urinary FLC excretion does not always increase even in patients with elevated serum FLC American Society of Nephrology levels (27). Thus, urinary FLC levels do not appear to contribute to diagnostic sensitivity of the current monoclonal testing regimen. SCREENING AND MONITORING The screening test(s) for any disease must have sufficient sensitivity to identify as many patients as possible but also cost effective enough to apply to the general population. Currently, no single test is capable of accomplishing these goals in monoclonal gammopathy. This is especially true for diseases with very low levels of monoclonal protein (28). However, in diseases such as multiple myeloma and Waldenström macroglobulinemia, where the monoclonal protein is usually abundant, the combination of serum IFE and serum FLC has been shown to be nearly 100% sensitive (14). In diseases with lower levels of monoclonal gammopathy, urine IFE can increase the sensitivity but at a higher cost (Table 1). For monitoring, the goals are different. Typically, the type of monoclonal protein is no longer important so IFE is not routinely required. The response is typically based on the reduction of the M protein. Depending on the M protein, this is done with SPEP, UPEP, and/or serum FLC assay. IFE is required when the M-spike is no longer detectable to evaluate for complete response. In multiple myeloma, FLC should be followed even when patient has a M-spike because of the phenomenon light chain escape. This occurs in some heavily treated patients where the clonal evolution produces a clone that makes more FLC than the intact immunoglobulin (29). In these patients, the M-spike would remain low, suggesting persistent response but the involved FLC will rise rapidly. TAKE HOME POINTS c Monoclonal protein testing is an important diagnostic tool for the evaluation of AKI and proteinuria. c The Serum free light chain assay significantly increases the detection rate of monoclonal protein when added to serum immunofixation. c Urine protein electrophoresis can help distinguish between tubular or glomerular injury in patients with multiple myeloma. REFERENCES 1. Kyle RA, Therneau TM, Rajkumar SV, Larson DR, Plevak MF, Offord JR, Dispenzieri A, Katzmann JA, Melton LJ 3rd. Prevalence of monoclonal gammopathy of undetermined significance. N Engl J Med 354: 1362– 1369, 2006 2. Knudsen LM, Hippe E, Hjorth M, Holmberg E, Westin J. Renal function in newly diagnosed multiple myeloma: A demographic study of 1353 patients. The Nordic Myeloma Study Group. Eur J Haematol 53: 207– 212, 1994 3. Ivanyi B. Frequency of light chain deposition nephropathy relative to renal amyloidosis and Bence Jones cast nephropathy in a necropsy study of patients with myeloma. Arch Pathol Lab Med 114: 986–987, 1990 Onco-Nephrology Curriculum 3 4. Blade J, Fernandez-Llama P, Bosch F, Montoliu J, Lens XM, Montoto S, Cases A, Darnell A, Rozman C, Montserrat E. Renal failure in multiple myeloma: Presenting features and predictors of outcome in 94 patients from a single institution. Arch Intern Med 158: 1889–1893, 1998 5. Nasr SH, Valeri AM, Sethi S, Fidler ME, Cornell LD, Gertz MA, Lacy M, Dispenzieri A, Rajkumar SV, Kyle RA, Leung N. Clinicopathologic correlations in multiple myeloma: A case series of 190 patients with kidney biopsies. Am J Kidney Dis 59: 786–794, 2012 6. Pasquali S, Zucchelli P, Casanova S, Cagnoli L, Confalonieri R, Pozzi C, Banfi G, Lupo A, Bertani T. Renal histological lesions and clinical syndromes in multiple myeloma. Renal Immunopathology Group. Clin Nephrol 27: 222–228, 1987 7. Rota S, Mougenot B, Baudouin B, De Meyer-Brasseur M, Lemaitre V, Michel C, Mignon F, Rondeau E, Vanhille P, Verroust P, Ronco P. Multiple myeloma and severe renal failure: A clinicopathologic study of outcome and prognosis in 34 patients. Medicine (Baltimore) 66: 126–137, 1987 8. Leung N, Bridoux F, Hutchison CA, Nasr SH, Cockwell P, Fermand JP, Dispenzieri A, Song KW, Kyle RA. Monoclonal gammopathy of renal significance: When MGUS is no longer undetermined or insignificant. Blood 120: 4292–4295, 2012 9. Bridoux F, Leung N, Hutchison CA, Touchard G, Sethi S, Fermand JP, Picken MM, Herrera GA, Kastritis E, Merlini G, Roussel M, Fervenza FC, Dispenzieri A, Kyle RA, Nasr SH. Diagnosis of monoclonal gammopathy of renal significance. Kidney Int 87: 698–671, 2015 10. O’Connell TX, Horita TJ, Kasravi B. Understanding and interpreting serum protein electrophoresis. Am Fam Physician 71: 105–112, 2005 11. Durie BG, Harousseau JL, Miguel JS, Blade J, Barlogie B, Anderson K, Gertz M, Dimopoulos M, Westin J, Sonneveld P, Ludwig H, Gahrton G, Beksac M, Crowley J, Belch A, Boccadaro M, Cavo M, Turesson I, Joshua D, Vesole D, Kyle R, Alexanian R, Tricot G, Attal M, Merlini G, Powles R, Richardson P, Shimizu K, Tosi P, Morgan G, Rajkumar SV. International uniform response criteria for multiple myeloma. Leukemia 20: 1467–1473, 2006 12. Rajkumar SV, Dimopoulos MA, Palumbo A, Blade J, Merlini G, Mateos MV, Kumar S, Hillengass J, Kastritis E, Richardson P, Landgren O, Paiva B, Dispenzieri A, Weiss B, LeLeu X, Zweegman S, Lonial S, Rosinol L, Zamagni E, Jagannath S, Sezer O, Kristinsson SY, Caers J, Usmani SZ, Lahuerta JJ, Johnsen HE, Beksac M, Cavo M, Goldschmidt H, Terpos E, Kyle RA, Anderson KC, Durie BG, Miguel JF. International Myeloma Working Group updated criteria for the diagnosis of multiple myeloma. The Lancet Oncology 15: e538–e548, 2014 13. Gay-Bellile C, Bengoufa D, Houze P, Le Carrer D, Benlakehal M, Bousquet B, Gourmel B, Le Bricon T. Automated multicapillary electrophoresis for analysis of human serum proteins. Clin Chem 49: 1909– 1915, 2003 14. Katzmann JA, Kyle RA, Benson J, Larson DR, Snyder MR, Lust JA, Rajkumar SV, Dispenzieri A. Screening panels for detection of monoclonal gammopathies. Clin Chem 55: 1517–1522, 2009 15. Steensma DP, Kyle RA. A history of the kidney in plasma cell disorders. Contrib Nephrol 153: 5–24, 2007 16. Durie BGM, Harousseau JL, Miguel JS, Blade J, Barlogie B, Anderson K, Gertz M, Dimopoulos M, Westin J, Sonneveld P, Ludwig H, Gahrton G, Beksac M, Crowley J, Belch A, Boccadaro M, Turesson I, Joshua D, Vesole D, Kyle R, Alexanian R, Tricot G, Attal M, Merlini G, Powles R, Richardson P, Shimizu K, Tosi P, Morgan G, Rajkumar SV, Grp IMW. International uniform response criteria for multiple myeloma (vol. 20, pg 1467, 2006). Leukemia 20: 2220–2220, 2006 4 Onco-Nephrology Curriculum 17. McTaggart MP, Lindsay J, Kearney EM. Replacing urine protein electrophoresis with serum free light chain analysis as a first-line test for detecting plasma cell disorders offers increased diagnostic accuracy and potential health benefit to patients. Am J Clin Pathol 140: 890–897, 2013 18. Leung N, Gertz M, Kyle RA, Fervenza FC, Irazabal MV, Eirin A, Kumar S, Cha SS, Rajkumar SV, Lacy MQ, Zeldenrust SR, Buadi FK, Hayman SR, Nasr SH, Sethi S, Ramirez-Alvarado M, Witzig TE, Herrmann SM, Dispenzieri A. Urinary albumin excretion patterns of patients with cast nephropathy and other monoclonal gammopathy-related kidney diseases. Clin J Am Soc Nephrol 7: 1964–1968, 2012 19. Drayson M, Begum G, Basu S, Makkuni S, Dunn J, Barth N, Child JA. Effects of paraprotein heavy and light chain types and free light chain load on survival in myeloma: An analysis of patients receiving conventional-dose chemotherapy in Medical Research Council UK multiple myeloma trials. Blood 108: 2013–2019, 2006 20. Drayson M, Tang LX, Drew R, Mead GP, Carr-Smith H, Bradwell AR. Serum free light-chain measurements for identifying and monitoring patients with nonsecretory multiple myeloma. Blood 97: 2900–2902, 2001 21. Katzmann JA, Abraham RS, Dispenzieri A, Lust JA, Kyle RA. Diagnostic performance of quantitative kappa and lambda free light chain assays in clinical practice. Clin Chem 51: 878–881, 2005 22. Hutchison CA, Cockwell P, Stringer S, Bradwell A, Cook M, Gertz MA, Dispenzieri A, Winters JL, Kumar S, Rajkumar SV, Kyle RA, Leung N. Early reduction of serum-free light chains associates with renal recovery in myeloma kidney. J Am Soc Nephrol 22: 1129–1136, 2011 23. Palladini G, Dispenzieri A, Gertz MA, Kumar S, Wechalekar A, Hawkins PN, Schonland S, Hegenbart U, Comenzo R, Kastritis E, Dimopoulos MA, Jaccard A, Klersy C, Merlini G. New criteria for response to treatment in immunoglobulin light chain amyloidosis based on free light chain measurement and cardiac biomarkers: impact on survival outcomes. J Clin Oncol 30: 4541–4549, 2012 24. Hutchison CA, Plant T, Drayson M, Cockwell P, Kountouri M, Basnayake K, Harding S, Bradwell AR, Mead G. Serum free light chain measurement aids the diagnosis of myeloma in patients with severe renal failure. BMC Nephrol 9: 11, 2008 25. Nakano T, Nagata A, Takahashi H. Ratio of urinary free immunoglobulin light chain kappa to lambda in the diagnosis of Bence Jones proteinuria. Clin Chem Lab Med 42: 429–434, 2004 26. Abraham RS, Clark RJ, Bryant SC, Lymp JF, Larson T, Kyle RA, Katzmann JA: Correlation of serum immunoglobulin free light chain quantification with urinary Bence Jones protein in light chain myeloma. Clin Chem 48: 655–657, 2002 27. Alyanakian MA, Abbas A, Delarue R, Arnulf B, Aucouturier P. Free immunoglobulin light-chain serum levels in the follow-up of patients with monoclonal gammopathies: Correlation with 24-hr urinary light-chain excretion. Am J Hematol 75: 246–248, 2004 28. Bhutani G, Nasr SH, Said SM, Sethi S, Fervenza FC, Morice WG, Kurtin PJ, Buadi FK, Dingli D, Dispenzieri A, Gertz MA, Lacy MQ, Kapoor P, Kumar S, Kyle RA, Rajkumar SV, Leung N. Hematologic characteristics of proliferative glomerulonephritides with nonorganized monoclonal immunoglobulin deposits. Mayo Clinic Proc 90: 587–596, 2015 29. Kuhnemund A, Liebisch P, Bauchmuller K, zur Hausen A, Veelken H, Wasch R, Engelhardt M. ’Light-chain escape-multiple myeloma’-an escape phenomenon from plateau phase: Report of the largest patient series using LC-monitoring. J Cancer Res Clin Oncol 135: 477– 484, 2009 American Society of Nephrology REVIEW QUESTIONS 1. A 68-year-old man presents with a 1-year history of suddenonset nephrotic range proteinuria, easy bruising with spontaneous “black eyes,” and more recent onset of dyspnea on exertion. Which of the following tests should be performed for screening? a. b. c. d. e. Serum protein electrophoresis Urine protein electrophoresis Serum immunofixation Serum free light chain assay All of the above Answer: e is correct. The presentation is concerning for amyloidosis. Because amyloidosis is usually caused by a low burden plasma cell dyscrasia, all of the tests should be performed to avoid false negativity. 2. Which test is most important in the screening of nonsecretory multiple myeloma? a. b. c. d. e. Urine immunofixation Complete blood count Serum creatinine Serum calcium Serum free light chain assay American Society of Nephrology Answer: e is correct. Prior to the serum free light chain assay, up to 5% of the multiple myeloma cases were thought to be nonsecretory. The serum free light chain assay identifies 80% of these cases as light chain myeloma. 3. A 65-year-old man with hypertension, mild anemia, and CKD stage IV presents with a mildly depressed serum k to l free light chain ratio of 0.12 (normal 5 0.26–1.65). Serum and urine immunofixation are negative. What is the most likely possible explanation for his abnormal k to l free light chain ratio? a. b. c. d. e. CKD Hypertension Chronic myelogenous leukemia Monoclonal gammopathy of undetermined significance None of the above Answer: d is correct. CKD and renal impairment result in a mildly elevated k to l ratio. This is because the reduction of glomerular filtration reduces the clearance of k free light chain more than l. Hypertension does not affect free light chain clearance. Chronic myelogenous leukemia is a myeloid disease and does not produce monoclonal protein. With the history of CKD, the low k to l ratio is likely to be due to the presence of a monoclonal gammopathy. Onco-Nephrology Curriculum 5 AUTHOR QUERIES AUTHOR PLEASE ANSWER ALL QUERIES Q1: Please provide mailing address for correspondence. Chapter 9: Hematopoietic Stem Cell Transplant–Related Kidney Disease Joseph R. Angelo, MD,* and Sangeeta Hingorani, MD, MPH†‡ *Pediatric Nephrology and Hypertension, University of Texas Health Science Center at Houston, University of Texas MD Anderson Cancer Center, Houston, Texas; †Division of Nephrology, Seattle Children’s Hospital, Seattle, Washington; and ‡Department of Pediatrics, Division of Nephrology, University of Washington, Seatlle, Washington INTRODUCTION Acute and chronic kidney diseases are common following hematopoietic cell transplantation (HCT) and can lead to long-term effects. Additionally, the occurrence of kidney disease in the setting of HCT can negatively affect mortality and morbidity. Etiologies of HCT-associated kidney injury are often multifactorial, including conditioning chemotherapy, radiation, nephrotoxic medications, sepsis, sinusoidal obstruction syndrome (SOS), transplantationassociated thrombotic microangiopathy (TA-TMA), and graft-versus-host disease (GVHD). Continued improvement in survival following HCT highlights the importance of monitoring renal function both before and after transplant and continued follow-up of patients with CKD. AKI Pretransplant evaluation of renal function Serum creatinine (SCr) is the most widely used marker of kidney function in patients undergoing HCT (1). Measurement of SCr provides estimation of renal function at the bedside and allows for following trends in renal function. GFR prediction formulas, such as the Modification of Diet in Renal Disease (MDRD) equation for adults and the Schwartz formula in children, are available (2,3). However, several shortcomings are inherent in the properties of SCr as a functional biomarker of AKI. These include the delay between the onset of kidney injury and an increase in SCr, limiting its utility to provide the earliest window for intervention (4). In addition, SCr is affected by factors such as age, muscle mass, and hydration status, issues particularly relevant for HCT patients. Even small changes in SCr in this population can represent significant American Society of Nephrology decline in kidney function. Other methods of GFR estimation include 24-hour urine creatinine clearance, inulin clearance, and use of radioactive isotopes (Tc-DTPA or Cr-EDTA) or iodinated contrast agents (iothalamate or iohexol). Defining AKI Current definitions of AKI are based on increases in SCr and decreased urine output. Two scoring systems, RIFLE (risk, injury, failure, loss, ESRD) and Acute Kidney Injury Network (AKIN), have been developed to standardize stratification of AKI severity (5,6). RIFLE criteria include two additional categories (loss, ESRD) describing two post-AKI clinical outcomes. For children, a modified version of RIFLE criteria, pRIFLE, has been developed (7). Several studies have shown a correlation between these scores and clinical outcomes (8). Recently, a new staging criteria for AKI was created by KDIGO; however, this newer definition has not been prospectively studied in the HCT population. Epidemiology of AKI The incidence of AKI varies, based on the definition of AKI, type of HCT, and chemotherapeutic conditioning regimen. When AKI is defined as a doubling of SCr during the first 100 days after stem cell infusion, the prevalence ranges from 15% to 73% (9). Severity of AKI also varies. In a study of pediatric and adult allogeneic HCTrecipients, up to a third of all patients doubled their SCr in the first 100 days, and 5% required acute dialysis (10). Severity of AKI is associated with increased risk of morbidity and mortality (11–13). Correspondence: Sangeeta Hingorani, Division of Nephrology, Seattle Children’s Hospital, 4800 Sand Point Way NE, Seattle, Washington 98105. Copyright © 2016 by the American Society of Nephrology Onco-Nephrology Curriculum 1 For those receiving high-dose conditioning regimens and allogeneic HCT, the incidence of AKI is as high as 69%. It often occurs before day 28, and risk factors include lung toxicity, hepatic toxicity, SOS, amphotericin exposure, and sepsis (14,15). For patients receiving reduced-intensity chemotherapy (RIC) and allogeneic HCT, AKI occurs less frequently, later after transplant, and less often results in the need for dialysis. A retrospective cohort study found that 47% of RIC patients developed AKI compared with 73% in the high-dose treatment group, developing at a median of 26–60 days after transplant in the RIC group. Fewer RIC patients required dialysis, and mortality was significantly lower (11,16,17). Compared with allogeneic HCT, AKI incidence is lower in autologous HCT, occurring in approximately 21% of these patients (18). Causes of AKI Common risk factors and causes of AKI after HCT include volume depletion, sepsis, nephrotoxic medication exposure, SOS, and GVHD (Table 1). Owing to a propensity for increased gastrointestinal (GI) fluid losses and poor oral intake, HCT patients are highly susceptible to volume depletion. Close tracking of fluid intake, urine output, fluid losses via the GI tract and insensible losses, and daily weight measurement are required. Additional measures that can discriminate prerenal AKI from other types include BUN/Cr ratio, fractional excretion of sodium (FENa), and fractional excretion of urea (FEurea) (Table 2). Sepsis can result in decreased effective circulating volume and hypotension and is a major risk factor for AKI. Sepsisinduced inflammation leads to increased capillary permeability and intravascular fluid leak, resulting in total body volume overload while depleting effective circulating volume and end organ perfusion (19). GVHD is unique to HCT and likely causes tissue and endothelial damage via T cell– and cytokine-mediated injury (20). The GI mucosa is a common site of GVHD, contributing to inadequate fluid intake and increased GI losses. Table 1. Risk factors for AKI in HCT Intravascular volume depletion Vomiting and diarrhea associated with acute gut GVHD Systemic vasodilatation Sepsis Renal vasoconstriction Sinusoidal obstruction syndrome Calcineurin inhibitors Endothelial injury Acute GVHD Calcineurin inhibitors Total body irradiation Thrombotic microangiopathy Tubular injury Medications: amphotericin, vancomycin Conditioning chemotherapy 2 Onco-Nephrology Curriculum Table 2. Urinary indices for AKI classification AKI classification BUN/Cr ratio Prerenal AKI Intrinsic AKI Obstructive AKI .20 Variable Variable FENa* FEurea† ,1% .3% Variable ,35% .35% Variable *FENa 5 (urinary Na 3 SCr)/(serum Na 3 urinary Cr). † FEurea 5 (urinary urea 3 SCr)/(serum urea 3 urinary Cr). SOS and hepatorenal syndrome (HRS) have been identified as an independent risk factor for AKI. HRS results in decreased resistance in the systemic and splanchnic vasculature, leading to renal hypoperfusion and compensatory increase in renal salt and water reabsorption. It presents as oligo-anuric prerenal AKI with edema and low urinary sodium. Septic shock and other causes of AKI must be ruled out. Defibroitide exhibits antithrombotic and fibrinolytic properties and has been studied for use in SOS (21). Common medications related to AKI include vancomycin, aminoglycosides, and amphotericin. Calcineurin inhibitors (CNIs) can lead to renal arteriolar vasoconstriction and have been associated with development of TA-TMA. Management of AKI The management of AKI is mainly supportive and specific to the underlying cause. For situations of renal hypoperfusion, prompt administration of intravenous fluids is required to restore effective circulating volume. However, a critical point is that fluid overload (FO) can itself be an independent predictor of mortality in critically ill patients (22,23). Stem cell transplant recipients are a population that may be particularly sensitive to FO, with one study of critically ill children suggesting that .10% FO in HCT patients correlates with decreased survival (%FO 5 [Fluid In – Fluid Out]/Intensive Care Unit Admission Weight in kilograms) (24,25). Judicious use and dose adjustment of antimicrobials should be used to decrease risk of AKI from nephrotoxin exposure. For those not responsive to medical interventions, dialysis is used as supportive therapy for management of AKI-related fluid and metabolic derangements. CKD AFTER HCT CKD stage 3 is defined as a GFR of ,60 mL/min per 1.73 m2 for $3 months. This is often the definition of CKD used in studies of HCT patients. The five stages of CKD (Table 3) range from mild to ESRD. The prevalence of CKD after HCT, using this definition, is between 20% and 30% (26). Cohorts including children and adults have shown a CKD prevalence of 19% at 1 year and 7% at least 2 years after HCT, with a mean estimated GFR of 46 mL/min per 1.73 m2 (27,28). Other chronic kidney disorders following HCT include albuminuria, hypertension, and renal tubular dysfunction. Regardless of underlying cause, CKD can progress to ESRD and increases mortality risk after American Society of Nephrology Table 3. Stages of CKD (67) Stage 1 2 3a 3b 4 5 2 Description GFR (mL/min per 1.73 m ) Kidney damage with normal GFR Mild Moderate $90 60–89 30–44 45–59 15–29 ,15 Severe ESRD HCT. For those who progress to ESRD, this mortality risk can be as high as 90% (29). Monitoring changes in renal function and management of any existing chronic renal disease are important to the long-term survival and quality of life for postHCT patients. monitoring of levels rather than complete cessation of CNI may be more appropriate (36,37). In patients not responsive to these interventions, pharmacologic therapies include rituximab, defibrotide, and eculizumab (33,38,39). Eculizumab, a monoclonal immunoglobulin that binds complement factor 5, has been used for treatment of TA-TMA. A retrospective analysis of 12 patients with post-HCT TMA treated with eculizumab reported hematologic response of 50% and overall survival of 33% (40). In some cases, there may be dysregulation of the complement system; elevated levels of C5b-9, the membrane attack complex, have been identified; and eculizumab has been used in these patients with mixed results (41). IDIOPATHIC OR GVHD-RELATED CKD Clinical entities associated with CKD after HCT Preexisting CKD before HCT Preexisting CKD is not a contraindication to HCT. A study of 141 adult patients with leukemia and pretransplant kidney dysfunction showed that, at 1 year, these patients did not have worse survival than those who initially had normal kidney function. In addition, for some cancer diagnoses, CKD is related to the underlying disease process and HCT can slow the progression of CKD. Important points in managing patients with preexisting CKD include accurate assessment of renal function prior to HCT, appropriate changes to medication dosing, and avoidance of nephrotoxins. Transplantation-associated TMA TMA is defined by hemolytic anemia with erythrocyte fragmentation, thrombocytopenia, and renal failure. It is characterized by endothelial damage, leading to thickened glomerular and arteriolar vessels, the presence of fragmented red blood cells, thrombosis, and endothelial cell swelling (30). Two consensus guidelines outline the clinical criteria for the diagnosis of TA-TMA. Both require the presence of schistocytes on peripheral smear and an elevated lactate dehydrogenase. The BMT Clinical Trials Network also includes AKI (doubling of serum creatinine), unexplained CNS dysfunction, and a negative Coombs test. The International Guidelines from the European Group for Blood and Marrow Transplantation include thrombocytopenia, anemia, and decreased haptoglobin (31–33). In the setting of HCT, the incidence of TMA ranges from 2% to 21% (31). The clinical course of TMA can be rapid with severe AKI but commonly follows a more indolent course, resulting in CKD and, possibly, progression to ESRD (34). Risk factors for the development of TMA after HCT include CNI use, total body irradiation, and GVHD (35). The mainstay of TA-TMA management remains reduction in dose or stoppage of CNI and therapeutic plasma exchange. A response rate of 50%–63% has been reported using these two interventions (33). Given that GVHD itself can be a risk factor for the development of TA-TMA, an approach with close American Society of Nephrology Many HCT survivors will not present with a clear etiology for CKD and are labeled as idiopathic CKD. Some data support a label of GVHD-related CKD, with renal disease resulting from T cell– and cytokine-mediated tissue damage related to the chronic inflammatory state of GVHD (42,43). The presence of albuminuria in this patient population may be a marker of the renal involvement of GVHD either directly or indirectly as described above. Clinically, albuminuria is monitored using urine albumin to creatinine ratio (ACR) on a spot urine sample. Microalbuminuria is defined as an ACR of 30–300 mg albumin/g creatinine, whereas macroalbuminuria is defined by ACR $300 mg/g creatinine. Albuminuria is common after HCT and can have long-term effects. In a cohort of 142 HCT patients, 94% developed albuminuria within 100 days after HCT, 50% developed it at 1 year, and 4% had an ACR demonstrating overt proteinuria. Microalbuminuria at day 100 was associated with a four times greater risk of CKD, and macroalbuminuria was associated with a seven times greater risk of nonrelapse mortality (28). In a more recent study, both micro- and macroalbuminuria in the first 100 days after HCT were associated with an increased risk of nonrelapse mortality at 1 year (43). Albuminuria can also provide a readily available indicator of other underlying pathologic processes, such as TA-TMA (37). Recent consensus guidelines recommend screening urinalyses and ACR as part of the day 180 post-HCTevaluation and then yearly screening after HCT. If macroalbuminuria is present, more frequent monitoring every 3–6 months is indicated (44). Given its utility as a marker of underlying pathology and the association between macroalbuminuria and long-term outcomes, renal biopsy is indicated in patients with persistent macroalbuminuria. Glomerular disorders Glomerular lesions related to HCT are typically discussed in association with chronic GVHD (cGVHD). HCT-related glomerular disease results in albuminuria, ranging from mild to nephrotic range proteinuria. Rarely, post-HCT glomerular disease presents as glomerulonephritis (45). In Onco-Nephrology Curriculum 3 contrast to albuminuria, more severe glomerular diseases are less common. Among these, membranous nephropathy (67%) and minimal change disease (33%) are the two most common pathologies (45,46). Both tend to occur fairly late after transplant, 8–14 months, and often within several months of development of GVHD or lowering of immunosuppression for GVHD prophylaxis (27,47). Treatment for HCT-associated nephrotic syndrome is similar to that in other settings, with corticosteroids being most common, as well as resumption of GVHD prophylaxis with CNIs (27,48). HCT patients with macroalbuminuria are likely to benefit from antiproteinuric therapy with angiotensin-converting enzyme inhibitors (ACE-Is) or angiotensin receptor blockers (ARBs) (27,49). Hypertension Elevations in BP are a common complication of HCT. In a retrospective analysis of a cohort of children and adults followed for a median of 16 years after HCT, the prevalence of hypertension was 17% (50). Risk factors associated with the development of hypertension include prior AKI, total body irradiation, autologous transplant, TA-TMA, obesity, and diabetes. Consensus recommendations are for BP measurement at each clinic visit, with a maximum interval of yearly (51). Thresholds for treating hypertension in the HCT population follow those of the general population, as recommended in the Report from the Panel Members of the Eighth Joint National Committee. For those .60 years of age, treatment goals are based on a threshold of $150/90 mmHg. For all other adults ($18 years old), including those with CKD, the threshold for treatment initiation is $140/90 mmHg (52). In children, the Fourth Report on the Diagnosis, Treatment, Evaluation, and Treatment of High Blood Pressure in Children and Adolescents defines hypertension as a systolic or diastolic BP .95th percentile, based on sex, age, and height (53). Effective treatment of hypertension can decrease cardiovascular disease risk and slow the progression of CKD. Initial interventions are lifestyle modifications, including dietary sodium restriction and regular exercise. Due to antiproteinuric and renoprotective properties, ACE-I or ARB therapy should be first choice for pharmacologic treatment of hypertension. For patients in which ACE-I/ARB use is contraindicated, choice of antihypertensive should be individualized. study in pediatric HCT patients reported a 22% prevalance of hemorrhagic cystitis, developing at a median of 35 days after HCT, which was associated with worse survival (55). Risks for developing BK-related complications include unrelated donor, myeloablative conditioning, and GVHD (56). Despite a reported prevalence of 10%–30% for BK viremia, less is known about the relationship between BK viremia and nephropathy in the HCT population. In a study of 124 adult allogeneic HCT recipients, 65% developed viruria and 17% developed viremia after a median follow-up of 454 days. Only 2 of 21 patients had persistent viremia and biopsyproven nephropathy; the remaining cases of viremia were mild and transient. BK viremia was an independent risk factor for an increase in post-HCT creatinine (57). In children after HCT, BK viremia has been reported as more predictive of poor renal outcomes than viruria, supporting the use of plasma BK polymerase chain reaction (PCR) levels when monitoring for the development of nephropathy (58). For those with elevations in SCr suspected to be related to BK nephropathy, definitive diagnosis requires kidney biopsy. Current treatment options for hemorrhagic cystitis include pain control, continuous bladder irrigation, and urologic intervention for clearance of clots causing obstruction. Pharmacologic interventions include cidofovir, leflunomide, and fluoroquinolones (59). CMX100 is an oral formulation of cidofovir and may have less kidney toxicity (60). Another novel therapy being investigated is the use of exogenous BK-specific T cells and manipulation of immunosuppression to maximize the patient’s own immune response (61,62). ESRD AND KIDNEY TRANSPLANT AFTER HCT There are limited data on the risk of ESRD after HCT. The reported prevalence ranges from 0.4% to 4.4% (63). One study calculated a risk of end-stage kidney disease as being 16 times higher than the general population 20 years after HCT (64). For those progressing to ESRD, dialysis and renal transplant remain the treatment options. There are several reports of successful kidney transplantation in both children and adults (65,66). CONCLUSION BK nephropathy BK virus is a double-stranded DNA virus in the polyomavirus family with a seroprevalence of 80% reported in healthy blood donors (54). After infection, BK remains dormant in the urothelial cells without clinical effects in immunocompetent individuals. In the immunosuppressed, BK virus has been associated with nephropathy after both kidney transplant HCT; however, hemorrhagic cystitis is more common in the HCT population. In post-HCT patients, BK virus–associated hemorrhagic cystitis occurs in 10%–25% of patients and can lead to obstructive AKI, long-term urologic dysfunction, need for invasive intervention, and increased mortality. A prospective 4 Onco-Nephrology Curriculum HCT patients are a population clearly at risk for the development of kidney disease, necessitating close monitoring with a multidisciplinary approach involving both oncologists and nephrologists. TAKE HOME POINTS c Acute and chronic kidney problems are common following HCT and are associated with an increased risk of nonrelapse mortality and a decrease in overall survival. American Society of Nephrology c Micro- and macroalbuminuria are associated with an increased risk of mor- tality and CKD at 1 year after HCT. It is unclear if the presence of albuminuria is a marker of systemic or local inflammation or a marker of GVHD. 17. c Hypertension should be managed with ACE-Is or ARBs. c A multidisciplinary approach is needed to insure appropriate manage- ment of renal issues occurring after HCT. REFERENCES 1. Tichelli A, Royo A, Gratwohl A. Late pulmonary, cardiovascular, and renal complications after hematopoietic stem cell transplantation and recommended screening practices. Hematology Am Soc Hematol Educ Program 2008: 125–133, 2008 2. Schwartz GJ, Brion LP, Spitzer A. The use of plasma creatinine concentration for estimating glomerular filtration rate in infants, children, and adolescents. Pediatr Clin North Am 34: 571–590, 1987 3. Levey AS, Coresh J, Greene T, Stevens LA, Zhang YL, Hendriksen S, Kusek JW, Van Lente F; Chronic Kidney Disease Epidemiology Collaboration. Using standardized serum creatinine values in the modification of diet in renal diseases study equation for estimating glomerular filtration rate. Ann Intern Med 145: 247–254, 2006 4. Devarajan P. Emerging biomarkers of acute kidney injury. Contrib Nephrol 156: 203–212, 2007 5. Mehta RL, Kellum JA, Shah SV, Molitoris BA, Ronco C, Warnock DG, Levin A; Acute Kidney Injury Network. Acute Kidney Injury Network: Report of an initiative to improve outcomes in acute kidney injury. Crit Care 11: R31, 2007 6. Ronco C, Levin A, Warnock DG, Mehta R, Kellum JA, Shah S, Molitoris BA; AKIN Working Group. Improving outcomews from acute kidney injury (AKI): Report on an initiative. Int J Arfit Organs 30: 373–376, 2007 7. Arkan-Arikan A, Zappitelli M, Loftis LL, Washburn KK, Jefferson LS, Goldstein SL. Modified RIFLE criteria in critically ill children with acute kidney injury. Kidney Int 71: 1028–1035, 2007 8. Hoste EA, Clermont G, Kersten A, Venkataraman R, Angus DC, De Bacquer D, Kellum JA. RIFLE criteria for acute kidney injury are associated with hospital mortality in critically ill patients: A cohort analysis. Crit Care 10: R73, 2006 9. Hingorani SR, Guthrie K, Batchelder A, Schoch G, Aboulhosn N, Manchion J, McDonald GB. Acute renal failure after myeloablative hematopioetic cell transplant: Incidence and risk factors. Kidney Int 67: 272–277, 2005 10. Gooley TA, Chien JW, Peregam SA, Hingorani S, Sorror ML, Boeckh M, Martin PJ, Sandmaier BM, Marr KA, Appelbaum FR, Storb R, McDonald GB. Reduced mortality after allogeneic hematopoietic-cell transplantation. N Engl J Med 363: 2091–2101, 2010 11. Parikh CR, Yarlagadda SG, Storer B, Sorror M, Storb R, Sandmaier B. Impact of acute kidney injury on long-term mortality after nonmyeloablative hematopoietic cell transplantation. Biol Blood Marrow Transplant 14: 309–315, 2008 12. Bao YS, Xie RJ, Wang M, Feng SZ, Han MZ. An evaluation of the RIFLE criteria for acute kidney injury after myeloablative allogeneic haematopoietic stem cell transplantation. Swiss Med Wkly 141: w13225, 2011 13. Ando M, Mori J, Ohashi K, Akiyama H, Morito T, Tsuchiya K, Nitta K, Sakamaki H. A comparative assessment of the RIFLE, AKIN and conventional criteria for acute kidney injury after hematopoietic SCT. Bone Marrow Transplant 45: 1427–1434, 2010 14. Kersting S, Koomans HA, Hene RJ, Verdonck LF. Acute renal failure after allogeneic myeloablative stem cell transplantation: Retrospective analysis of incidence, risk factors and survival. Bone Marrow Transplant 39: 359–365, 2007 15. Parikh CR, Schrier RW, Storer B, Diaconescu R, Sorror ML, Maris MB, Maloney DG, McSweeney P, Storb R, Sandmaier BM. Comparison of ARF after myeloablative and nonmyeloablative hematopioetic cell transplantation. Am J Kidney Dis 45: 502–509, 2005 16. Liu D, Yan C, Xu L, Wang Y, Han W, Zhang X, Liu K, Huang X. Diarrhea during the conditioning regimen is correlated with the occurrence of American Society of Nephrology 18. 19. 20. 21. 22. 23. 24. 25. 26. 27. 28. 29. 30. 31. 32. 33. 34. 35. severe acute graft-versus-host disease through systemic release of inflammatory cytokines. Biol Blood Marrow Transplant 16: 1567–1575, 2010 Kersting S, Drop SV, Theobald M, Verdonck LF. Acute renal failure after nonmyeloablative stem cell transplantation in adults. Biol Blood Marrow Transplant 14: 125–131, 2008 Lopes JA, Jorge S, Silva S, de Almeida E, Abreu F, Martins C, do Carmo JA, Lacerda JF, Parta MM. Acute renal failure following myeloablative autologous and allogeneic hematopoietic cell transplantation. Bone Marrow Transplant 38: 707, 2006 Wan L, Bagshaw SM, Langenberg C, Saotome T, May C, Bellomo R. Pathophysiology of septic acute kidney injury: What do we really know? Crit Care Med 36[4 Suppl]: S198–S203, 2008 Parikh CR, Coca SG. Acute renal failure in hematopoietic cell transplantation. Kidney Int 69: 430–435, 2006 Keating GM. Defibrotide: A review of its use in severe hepatic venoocclusive disease following haematopoietic stem cell transplantation. Clin Drug Investig 34: 895–904, 2014 Murphy CV, Schramm GE, Doherty JA, Reichley RM, Gajic O, Afessa B, Micek ST, Kollef MH. The importance of fluid management in acute lung injury secondary to septic shock. Chest 136: 102–109, 2009 Boyd JH, Forbes J, Nakada TA, Walley KR, Russell JA. Fluid resuscitation in septic shock: A positive fluid balance and elevated central venous pressure are associated witih increased mortality. Crit Care Med 39: 259–265, 2011 Michael M, Kuehnle I, Goldstein SL. Fluid overload and acute renal failure in pediatric stem cell transplant patients. Pediatr Nephrol 19: 91–95, 2004 Goldstein SL, Currier H, Graf C, Cosio CC, Brewer ED, Sachdva R. Outcome in children receiving continuous venovenous hemofiltration. Pediatrics 107: 1309–1312, 2001 Choi M, Sun CL, Kurian S, Carter A, Francisco L, Forman SJ, Bhatia S. Incidence and predictors of delayed chronic kidney disease in longterm survivors of hematopoietic cell transplantation. Cancer 113: 1580– 1587, 2008 Abboud I, Peraldi MN, Hingorani S. Chronic kidney diseases in long-term survivors after allogeneic hematopoietic stem cell transplantation: Monitoring and management guidelines. Semin Hematol 49: 73–82, 2012 Hingorani SR, Seidel K, Lindner A, Aneja T, Schoch G, McDonald G. Albuminuria in hematopoietic cell transplantation patients: Prevalence, clinical associations, and impact on survival. Biol Blood Marrow Transplant 14: 1365–1372, 2008 Cohen EP, Piering WF, Kabler-Babbit C, Moulder JE. End-stage renal disease (ESRD) after bone marrow transplantation: poor survival compared to other causes of ESRD. Nephron 79: 408–412, 1998 Batts ED, Lazarus HM. Diagnosis and treatment of transplantationassociated thrombotic microangiopathy: Real progress or are we still waiting? Bone Marrow Transplant 40: 709–719, 2007 Ho VT, Cutler C, Carter S, Martin P, Adams R, Horowitz M, Ferrara J, Soiffer R, Giralt S. Blood and Marrow Transplant Clinical Trials Network Toxicity Committee consensus summary: Thrombotic microangiopathy after hematopoietic stem cell transplantation. Biol Blood Marrow Transplant 11: 571–575, 2005 Ruutu T, Barosi G, Benjamin RJ, Clark RE, George JN, Gratwohl A, Holler E, Iacobelli M, Kentouche K, Lämmle B, Moake JL, Richardson P, Socié G, Zeigler Z, Niederwieser D, Barbui T; European Group for Blood and Marrow Transplantation; European LeukemiaNet. Diagnostic criteria for hematopoietic stem cell transplant-associated microangiopathy: Results of a consensus process by an international working group. Haematologica 92: 100, 2007 Kim SS, Patel M, Yum K, Keyzner A. Hematopoietic stem cell transplantassociated thrombotic microangiopathy: Review of pharmacologic treatment options. Transfusion 55: 452–458, 2015 Van Why SK, Friedman AL, Wei LJ, Hong R. Renal insufficiency after bone marrow transplantation in children. Bone Marrow Transplant 7: 383–388, 1991 Schriber JR, Herzig GP. Transplantation-associated thrombotic thrombocytopenic purpura and hemolytic uremic syndrome. Semin Hematol 34: 126–133, 1997 Onco-Nephrology Curriculum 5 36. Mii A, Shimizu A, Kaneko T, Fujita E, Fukui M, Fujino T, Utsumi K, Yamaguchi H, Tajika K, Tsuchiya S, Iino Y, Katayama Y, Fukuda Y. Renal thrombotic microangiopathy associated with chronic graft-versus-host disease after allogeneic hematopoietic stem cell transplantation. Pathol Int 61: 518–527, 2011 37. Laskin BL, Goebel J, Davies SM, Jodele S. Small vessels, big trouble in the kidneys and beyond: Hematopoietic stem cell transplantation-associated thrombotic microangiopathy. Blood 118: 1452–1462, 2011 38. Au WY, Ma ES, Lee TL, Ha Sy, Fung AT, Lie AK, Kwong YL. Successful treatment of thrombotic microangiopathy after haematopoietic stem cell transplantation with rituximab. Br J Haematol 137: 475–478, 2007 39. Corti P, Uderzo C, Tagliabue A, Della Volpe A, Annaloro C, Tagliaferri E, Balduzzi A. Defibrotide as a promising treatment for thrombotic thrombocytopenic purpura in atient sundergoing bone marrow transplantation. Bone Marrow Transplant 29: 542–543, 2002 40. de Fontbrune FS, Galambrun C, Sirvent A, Huynh A, Faguer S, Nguyen S, Bay JO, Neven B, Moussi J, Simon L, Xhaard A, Resche-Riggon M, O’Meara A, Fremeaux-Bacchi V, Veyradier A, Socié G, Coppo P, de Latour RP. Use of eculizumab in patients with allogeneic stem cell transplant-associated thrombotic microangiopathy: A study from the SFGM-TC. Transplantation 99: 1953–1959, 2015 41. Jodele S, Davies SM, Lane A, Khoury J, Dandoy C, Goebel J, Myers K, Grimley M, Bleesing J, El-Bietar J, Wallace G, Chima RS, Paff Z, Laskin BL. Diagnostic and risk criteria for HSCT-associated thrombotic microangiopathy: A study in children and young adults. Blood 1214: 645–653, 2014 42. Hingorani S, Guthjrie KA, Schoch G, Weiss NS, McDonald GB. Chronic kidney disease in long-term survivors of hematopoietic cell transplant. Bone Marrow Transplant 39: 223–229, 2007 43. Hingorani S, Gooley T, Pao E, Sandmaier B, McDonald G. Urinary cytokines after HCT: Evidence for renal inflammation in the pathogenesis of proteinuria and kidney disease. Bone Marrow Transplant 49: 403–409, 2014 44. Pulsipher MA, Skinner R, McDonald GB, Hingorani S, Armenian SH, Cooke KR, Gracia C, Petryk A, Bhatia S, Bunin N, Nieder ML, Dvorak CC, Sung L, Sanders JE, Kurtzberg J, Baker KS. National Cancer Institute, National Heart, Lung and Blood Instsitute/Pediatric Blood and Marrow Transplantation Consortium First International consensus conference on late effects after pediatric hematopoietic cell transplantation: The need for pediatric-specific long-term followup guidelines. Biol Blood Marrow Transplant 18: 334–347, 2012 45. Chanswangphuwana C, Townamchai N, Intragumtornchai T, Bunworasate U. Glomerular diseases associated with chronic graft-versus-host disease after allogeneic peripheral blood stem cell transplantation: Case reports. Transplant Proc 46: 3616–3619, 2014 46. Hu SL. The role of graft-versus-host disease in haematopoietic cell transplantation-associated glomerular disease. Nephrol Dial Transplant 26: 2025–2031, 2011 47. Brukamp K, Doyle AM, Bloom RD, Bunin N, Tomaszewski JE, Cizman B. Nephrotic syndrome after hematopoietic cell transplantation: Do glomerular lesions represent renal graft-versus-host disease? Clin J Am Soc Nephrol 1: 685–694, 2006 48. Niscola P, Tendas A, Luo XD, Catalano G, Scaramucci L, Cupelli L, Giovannini M, Ferranini M, Bondanini F, Piccioni D, Dentamaro T, Palumbo R, Perrotti AP, Liu QF, de Fabritiis P. The management of membranous glomerulopathy in allogeneic stem cells transplantation: Updated literature. Cardiovasc Hematol Agents Med Chem 11: 67–76, 2013 49. Osugi Y, Yamada H, Hosoi G, Noma H, Ikemiya M, Ishii T, Sako M. Treatment with candesartan combined with angiotensin-converting enzyme inhibitor for immunosuppressive treatment-resistant nephrotic syndrome after allogeneic stem cell transplantation. Int J Hematol 83: 454–458, 2006 50. Hoffmeister PA, Hingorani SR, Storer BE, Baker KS, Sanders JE. Hypertension in long-term survivors of pediatric hematopoietic cell transplantation. Biol Blood Marrow Transplant 16: 515–524, 2010 51. Rizzo JD, Wingard JR, Tichelli A, Lee SJ, Van Lint MT, Burns LJ, Davies SM, Ferrara JL, Socié G. Recommended screening and preventive practices for long-term survivors after hematopoietic cell transplantation: Joint recommendations of the European Group for Blood and Marrow Transplantation, the Center for International blood and Marrow 6 Onco-Nephrology Curriculum 52. 53. 54. 55. 56. 57. 58. 59. 60. 61. 62. 63. 64. 65. 66. 67. Transplant Research, and the American Society of Blood and Marrow Transplantation. Biol Blood Marrow Transplant 12: 138–151, 2006 James PA, Oparil S, Carter BL, Cushman WC, Dennison-Himmelfarb C, Handler J, Lackland DT, LeFevre ML, MacKenzie TD, Ogedegbe O, Smith SC Jr, Svetkey LP, Taler SJ, Townsend RR, Wright JT Jr, Narva AS, Ortiz E. 2014 evidence-based guideline for the management of high blood pressure in adults: report from the panel members appointed to the Eighth Joint National Committee. JAMA 311: 507–520, 2014 National High Blood Pressure Education Program Working Group on High Blood Pressure in Children and Adolescents. The fourth report on the diagnosis, evaluation, and treatment of high blood pressure in children and adolescents. Pediatrics 114[2 Suppl 4th Report]: 555–576, 2004 Egli A, Infanti L, Dumoulin A, Buster A, Samaridis J, Stebler C, Gosert R, Hirsch HH. Prevalence of polyomavirus Bk and JC infection and replication in 400 healthy blood donors. J Infect Dis 199: 837–846, 2009 Cesaro S, Facchin C, Tridello G, Messina C, Calore E, Biasolo MA, Pillon M, Varotto S, Brugiolo A, Mengoli C, Palù G. A prospective study of Bi-virus-associated haemorrhagic cystitis in paediatric patients undergoing allogeneic haematopoietic stem cell transplantation. Bone Marrow Transplant 41: 363–370, 2008 Giraud G, Priftakis P, Bogdanovic G, Remberger M, Dubrulle M, Hau A, Gutmark R, Mattson J, Svahn BM, Ringden O, Winiarski J, Ljungman P, Dalianis T. BK-viruria and haemorrhagic cystitis are more frequent in allogeneic haematopoietic stem cell transplant patients receiving full conditioning and unrelated-HLA-mismatched grafts. Bone Marrow Transplant 41: 737–742, 2008 O’Donnell PH, Swanson K, Josephson MA, Artz AS, Parsad SD, Ramaprasad C, Pursell K, Rich E, Stock W, van Besien K. BK virus infection is associated with hematuria and renal impairment in recipients of allogeneic hematopoetic stem cell transplants. Biol Blood Marrow Transplant 15: 1038–1048, 2009 Haines HL, Laskin BL, Goebel J, Davies SM, Yin HJ, Lawrence J, Mehta PA, Bleesing JJ, Filipovich AH, Marsh RA, Jodele S. Blood, and not urine, BK viral load predicts renal outcome in children with hemorrhagic cystitis following hematopoietic stem cell transplantation. Biol Blood Marrow Transplant 17: 1512–1519, 2011 Ramos E, Drachenberg CB, Wali R, Hirsch HH. The decade of polyomavirus Bi-associated nephropathy: State of affairs. Transplantation 87: 621–630, 2009 Dropoulic LK, Cohen JI. Update on new antivirals under development for the treatment of double-stranded DNA virus infections. Clin Pharmacol Ther 88: 610–619, 2010 Babel N, Volk HD, Reinke P. BK polyomavirus infection and nephropathy: The virus-immune system interplay. Nat Rev Nephrol 7: 399–406, 2011 Mani J, Jin N, Schmitt M. Cellular immunotherapy for patients with reactivation of JC and BK polyomaviruses after transplantation. Cytotherapy pii(S1465–3249(14)00559-3), 2014 Touzot M, Elie C, van Massenhove J, Maillard N, Buzyn A, Fakhouri F. Long-term renal function after allogenic haematopoietic stem cell transplantation in adult patients: A single-centre study. Nephrol Dial Transplant 25: 624–627, 2010 Cohen EP, Drobyski WR, Moulder JE. Significant increase in end-stage renal disease after hematopoietic stem cell transplantation. Bone Marrow Transplant 39: 571–572, 2007 Hingorani S. Chronic kidney disease in long-term survivors of hematopoietic cell transplantation: Epidemiology, pathogenesis, and treatment. J Am Soc Nephrol 17: 1995–2005, 2006 Bunin N, Guzikowski V, Rand ER, Goldfarb S, Baluarte J, Meyers K, Olthoff KM. Solid organ transplants following hematopoietic stem cell transplant in children. Pediatr Transplant 14: 1030–1035, 2010 Levey AS, Coresh J, Balk E, Kausz AT, Levin A, Steffes MW, Hogg RJ, Perrone RD, Lau J, Eknoyan G; National Kidney Foundation. National Kidney Foundation practice guidelines for chronic kidney disease: Evaluation, classificaiton, and stratification. Ann Intern Med 139: 137– 147, 2003 American Society of Nephrology REVIEW QUESTIONS 1. Following hematopoietic stem cell transplant your patient is having a progressive rise in serum creatinine. A recent CBC and blood smear showed anemia, thrombocytopenia, and the presence of schistocytes. Considering the possibility of transplant-associated thrombotic microangiopathy, labs are sent and show an elevated lactate dehydrogenase level, an undetectable haptoglobin level, and the Coombs test is negative. Of the following, the best first step(s) in the treatment of this patient is: a. b. c. d. e. Administration of rituximab Therapeutic plasma exchange (TPE) Red blood cell and platelet transfusion Decreasing dose of calcineurin inhibitor therapy Both b and d Answer: e is correct. Calcineurin inhibitor therapy, used for GVHD prophylaxis, has been associated with TA-TMA in the HCT population. First-line therapy includes lowering of CNI dose or stoppage and TPE, with the majority of patients responding to these two interventions. Other therapies being studied include rituximab, defibrotide, and eculizumab. 2. In septic HCT patients, the pathophysiologic mechanism most likely to lead to total body volume overload and edema is: a. Increased capillary leak related to sepsis-induced inflammatory response b. Heart failure c. Administration of pressors d. Endothelial damage related to high-dose antibiotics American Society of Nephrology e. Decreased venous return related to positive pressure ventilation Answer: a is correct. Sepsis results in a cytokine and complement-stimulated systemic inflammatory response causing increased capillary leak, total body volume overload, and decreased effective circulating volume. Sepsis is a common cause of AKI following HCT. Other common causes include volume depletion, nephrotoxic medication exposure, sinusoidal obstruction syndrome, and GVHD. Volume overload has been associated with increased morbidity and mortality in critically ill patients and HCT specifically. 3. As part of annual screening after HCT, your patient is noted to have a urine albumin/creatinine ratio of 500 mg/g Cr. Your frequency of monitoring for proteinuria should be changed to: a. b. c. d. e. Monthly Weekly Continue with annual monitoring Every 3–6 months You no longer need to check for proteinuria Answer: d is correct. Albuminuria is a common long-term consequence of HCTand has been associated with progression to CKD and non–relapse-associated mortality. Microalbuminuria is defined as an ACR of 30–300 mg albumin/g creatinine, whereas macroalbuminuria is defined by ACR $ 300 mg/g creatinine. Recent consensus guidelines recommend screening ACR as part of the day 180 post-HCT evaluation and then yearly screening after HCT. If proteinuria is present, more frequent monitoring every 3–6 months is indicated. Additionally, for those with persistent macroalbuminuria, renal biopsy is indicated to make definitive diagnosis of the underlying etiology. Onco-Nephrology Curriculum 7 Chapter 10: Radiation Nephropathy Amaka Edeani, MBBS,* and Eric P. Cohen, MD† *Kidney Diseases Branch, National Institute of Diabetes and Digestive and Kidney Diseases, National Institutes of Health, Bethesda, Maryland; and †Nephrology Division, Department of Medicine, University of Maryland School of Medicine, and Baltimore Veterans Affairs Medical Center, Baltimore, Maryland INTRODUCTION The occurrence of renal dysfunction as a consequence of ionizing radiation has been known for more than 100 years (1,2). Initial reports termed this condition “radiation nephritis,” but that is a misnomer, because it is not an inflammatory condition. Renal radiation injury may be avoided by the exclusion of an adequate volume of kidney exposure during radiation therapy, but the kidneys’ central location can make this difficult to impossible when tumors of the abdomen or retroperitoneum are treated, or during total body irradiation (TBI) (3). BACKGROUND/CLINICAL SIGNIFICANCE Radiation nephropathy is renal injury and loss of function caused by ionizing radiation; this will occur after sufficient irradiation of both kidneys (4). Ionizing radiation of sufficient energy disrupts chemical bonds and knocks electrons out of atoms. It generates oxygen radicals that cause prompt DNA injury within milliseconds of irradiation. This is the desired action of therapeutic irradiation to cause death of cancer cells. It is also the cause of injury to irradiated normal tissues such as the kidney. The doses of diagnostic X-ray are orders of magnitude less than those of therapeutic irradiation; whereas there may be a very small effect of carcinogenesis from a diagnostic X-ray, there is no risk of normal tissue injury. The kidneys are the dose-limiting organs for radiation therapy for gastrointestinal cancers, gynecologic cancers, lymphomas, and sarcomas of the upper abdomen and during TBI (5). Damage to normal tissues can be reduced by shielding of nontarget tissues, shaping radiation beams that focus the high-dose radiation on the cancer and attempting to avoid surrounding normal tissues, and fractionated radiation dosing. Fractionation enables DNA repair in normal cells between dosing. American Society of Nephrology Classical radiation nephropathy occurred after external beam radiation for treatment of solid cancers such as seminomas (6); the incidence has declined with the advent of more effective chemotherapy. In recent years, radiation nephropathy has occurred due to TBI used as part of chemo-irradiation conditioning just before hematopoietic stem cell transplantation (HSCT) and also from targeted radionuclide therapy used for instance in the treatment of neuroendocrine malignancies. TBI may be myeloablative or nonmyeloablative, with myeloablative regimens using radiation doses of 10–12 Gy to destroy or suppress the recipient’s bone marrow. These doses are given in a single fraction or in nine fractions over 3 days (4). In addition, TBI for bone marrow transplantation (BMT) is preceded or accompanied by cytotoxic chemotherapy, which potentiates the effects of ionizing radiation (7). CKD after HSCT occurs in 10%–30% of HSCT survivors in pediatric and adult populations (8). CKD following HSCT can have many causes including medication-induced nephrotoxicity and graftversus-host disease (9); the role of radiation exposure at the time of the HSCT is also well established (4). CKD has significant patient impact by predisposing to hypertension, by altering medication pharmacokinetics, and by predisposing to ESRD. Threshold dose Luxton identified 23 Gy as the threshold dose for radiation nephropathy (6), from radiation of both kidneys when given in 20 fractions over 4 weeks. If the total irradiated renal volume is ,30% of both kidneys, CKD will not occur from irradiation alone Correspondence: Amaka Edeani, Kidney Diseases Branch, National Institute of Diabetes and Digestive and Kidney Diseases, National Institutes of Health, 10 Center Dr., Rm. 5-5744, Bethesda, Maryland 20892. Copyright © 2016 by the American Society of Nephrology Onco-Nephrology Curriculum 1 (10), although there may be injury to the small, irradiated volume of kidneys that leads to hypertension. In the case of radiation nephropathy after BMT, a 10-Gy TBI single dose of X-rays can cause radiation nephropathy within a year after irradiation, as may 14 Gy fractionated over 3 days (11). Lower radiation doses may cause kidney injury after many years of follow-up. Thus, in survivors of the HiroshimaNagasaki atomic bombs, estimated single fraction doses of ,200 cGy were associated with CKD after many decades (12). Radiosensitivity Radiation nephropathy occurs as a late phenomenon, usually presenting months to years after the radiation exposure. This latency is associated with slower cell turnover rates in renal tissue compared to early-responding (rapidly proliferating) tissue such as gastrointestinal epithelium or bone marrow (4). Normal renal tissue has low mitotic rates, which correlate with delayed expression of renal injury after radiation (4). Less than half of subjects exposed to threshold or higher radiation doses will develop radiation nephropathy. The determinants of individual radiosensitivity are not well known. EXPERIMENTAL MODELS AND PATHOGENESIS Radiation nephropathy has been reproduced in many animal models, including in mice, rats, dogs, and nonhuman primates. A recent rat model demonstrates injury at similar doses to those relevant in humans, a similar latency period between time of irradiation and manifestations of injury, expression of injury as proteinuria, hypertension, and azotemia, and a similar histopathology (4,13). In this model, suppression of the renin– angiotensin system is beneficial, and angiotensin II infusion exacerbates injury (14). However, in the irradiated rat, there is no evidence of activation of the renin–angiotensin system, which suggests other countervailing mechanisms relevant in pathogenesis. Other notable features in this model include a lack for evidence of chronic oxidative stress (15). Multiple aspects of pathogenesis have been tested for causality in the experimental model of TBI-HSCT in which the major lethal toxicity is renal failure (8). These include oxidative stress, cell proliferation, transforming growth factor-b, glomerular permeability, fibrosis, renin–angiotensin system, and vascular injury. Of these, the roles of the renin–angiotensin system and vascular injury have not been disproven to date; the others are either absent or do not play a causal role (8). Krochak and Baker (16) noted a sequence of events in which the initial reaction to irradiation was an increase in endothelial permeability with an increase of ultrafiltrate extruded from the capillaries in the glomerular tuft, with an escape of increased protein and other high-molecular-weight blood components; these events were transient with normal fluid dynamics restored within a few days, but these permeability changes did appear to contribute to eventual early and late clinical syndromes. 2 Onco-Nephrology Curriculum CLINICAL PRESENTATION The clinical features of radiation nephropathy will vary according to dose and volume of irradiation (17). Presentation can be acute and irreversible or subtle, with a gradual progressive dysfunction over years (5). There is a latent period that is clinically “silent” until a stage is reached when there are clinical manifestations of disease (11). These were well described by Luxton in his observation of 137 men with seminoma who were irradiated with 2250–3250 cGy over 5 weeks (3). The incidence of radiation nephropathy in his cohort was about 20%, with four clinical syndromes as identified in Table 1. One may expect similar presentations after sufficient renal irradiation from any source, external or internal. Acute radiation nephritis The onset of acute radiation nephritis is only acute relative to the other variants. Patients present 6–12 months following radiation exposure with fatigue, varying degrees of edema, shortness of breath with exertion and headaches with azotemia, malignant hypertension, and severe anemia disproportionate to degree of renal failure (16,18). Malignant hypertension may present with headaches, encephalopathy, and retinopathy. There may be associated congestive heart failure with anasarca, pleuropericardial effusions, and pulmonary edema. Anemia is normochromic and normocytic with a nonaplastic marrow (17). This has been described as “bone marrow transplant nephropathy” (4,19) when it occurs after HSCT. Some of these cases may present as hemolytic uremic syndrome or thrombotic thrombocytopenic purpura, i.e., as full-blown thrombotic microangiopathies. Proteinuria is generally not severe, with an average of 2 g/g urine creatinine. Prognosis has been variably related to occurrence of malignant hypertension (6) or severity of fluid retention, with oliguria being a terminal event. Patients surviving this acute phase usually are left with progression to CKD. Chronic radiation nephritis There are two variants of chronic nephritis (6): 1) primary chronic radiation nephritis—initial presentation up to 2 years or longer following irradiation with proteinuria and other evidence of chronic nephritis; and 2) secondary chronic radiation nephritis—seen in patients who survived acute radiation nephritis and continued with signs of chronic renal damage. Signs and symptoms are indistinguishable from renal failure from any other cause, with hypertension, albuminuria, anemia, azotemia, and small atrophic kidneys on imaging. Table 1. Clinical syndromes following renal irradiation (3) Type Latent period Acute radiation nephritis (nephropathy) Chronic radiation nephritis Malignant hypertension Benign hypertension 6–12 months $18 months 12–18 months $18 months American Society of Nephrology Malignant hypertension In Luxton’s series of patients, malignant hypertension developed either during the acute phase or later, 12–18 months following irradiation (6,20), with some presenting many years after exposure. Clinical features included hypertensive encephalopathy, retinopathy, and seizures. Renal size is variable. Benign hypertension Luxton showed that hypertension may occur as a manifestation of renal radiation injury in the absence of renal failure, with variable degrees of proteinuria. However, other studies have shown close correlation between degree of azotemia and prevalence of hypertension (1). This is also valid for the hypertensive CKD that occurs after HSCT (4,21). Hypertension following unilateral renal irradiation Hypertension alone may be the presenting feature if only a single kidney is irradiated. This may occur because the irradiated kidney shrinks and creates renin-dependent hypertension in the manner of a Page kidney (22). Such cases have been successfully treated by unilateral nephrectomy (23–25). Histopathology There are early and late changes following renal irradiation (3,26). Early changes include endothelial microvascular injury with cell swelling, subendothelial expansion, and capillary loop occlusion (Figure 1). There is often mesangiolysis and variable tubular injury. Ultrastructural examination shows amorphous material within the subendothelial space, which appears to extend the lamina rara interna (Figure 2). Late changes include sclerosis of interlobular and arcuate arteries, with residual parenchymal damage with increased mesangial matrix, glomerular scarring, tubular atrophy, and renal mass reduction. Fibrosis may be extensive. Figure 1. Photomicrograph by light microscopy of a renal biopsy specimen in case of BMT nephropathy (4). There is mesangiolysis (*) and extreme widening of the space between endothelium and glomerular basement membrane (arrow). The tubular epithelium is intact, but the tubules are separated by an expanded interstitium. periodic acid-Schiff stain; magnification, 2503. Reproduced with permission from reference 4. trial was based on the hypothesis of mitigation: when a subject has received sufficient irradiation to kidneys, use of an angiotensin-converting enzyme inhibitor after exposure but before expression of injury may be beneficial in mitigating the later injury. Despite treatment, patients with radiation nephropathy may evolve to ESRD and require chronic dialysis therapy or undergo kidney transplant. When this occurs after HSCT, survival on dialysis is poor (29). Such patients may be eligible for kidney transplantation and could avoid the need for immunosuppression if the transplanted kidney was from the same donor as gave them their hematopoietic stem cells (30,31). RISK OF CANCER FROM IMAGING TREATMENT Because it is uncommon, there are no controlled trials to guide the management of radiation nephropathy. Thus, treatment of radiation nephropathy is guided by the same principles of treatment of any hypertensive kidney disease, including blood pressure control and correction of metabolic acidosis. Standard management of anemia, secondary hyperparathyroidism, and electrolyte disturbances may be useful. Experimental data show that angiotensin converting enzyme inhibitors or angiotensin receptor blockers are preferentially beneficial in radiation nephropathy (14). Supportive measures are beneficial including treatment of peripheral and pulmonary edema and treatment of anemia with blood transfusions and/or erythropoietin stimulating agents. Prevention may be better than treatment. There are favorable trends for the benefit of captopril to mitigate the late occurrence of CKD after TBI-based HSCT (28). This clinical American Society of Nephrology High-dose ionizing radiation can cause cancer. The threshold dose at which there is increased excess risk after a single exposure is 34 mSv (32). For reference, an absorbed dose of 1 mGy is equivalent to an effective dose of 1 mSv, the body radiation dose of a chest X-ray is ,0.1 mSv, and that of a body computed tomography (CT) scan is approximately 10 mSv. Recent reports express concern about the possible cancer risks of radiologil imaging in patients with kidney disease. Kinsella et al. (33) reported estimated X-ray doses received by dialysis patients, and Nguyen (34) reported X-ray exposure in subjects undergoing workup for kidney transplantation. However, rather than create real worry, these reports actually underline the lack of risk, and neither one reports cancers caused by radiation. The median cumulative doses were 22 and 29 mSv in each report, respectively. The single fraction equivalent doses are about half of those values, well below the single fraction dose at which there is significant excess cancer risk. Furthermore, because radiation-induced cancer takes Onco-Nephrology Curriculum 3 Figure 2. Electron micrographs of glomerular capillary loop in a case of radiation nephropathy (27). (A) Wrinkled capillary basement membrane (BM) with variable widening of lamina rara interna (arrows). Capillary lumen (CL) contains circulating red blood cells (RBCs) and has intact endothelial cell lining. Endothelial cells (ENs) are occasionally swollen and contain prominent organelles. Original magnification, 6,3003; reduced by 31%. (B) Higher magnification of A reveals variable widening of lamina rara interna and deposits of fluffy material (*) resembling that within the capillary lumen. Newly formed basement membrane material is also evident adjacent to the lining endothelium (long arrows). Original magnification, 13,5003; reduced by 31%. M, mesangial cells; EP, epithelial cells; US, urinary space. Reproduced with permission from reference 27. 5–10 years to develop, for the average dialysis patient, i.e., those starting dialysis at age 65 or more, radiation-induced excess risk of cancer cannot be a major risk because the expected remaining lifetime is 4 years or less for a 65 year old starting dialysis (35). Although cancer after kidney transplant is a genuine concern, one would expect a surfeit of leukemias if pretransplant radiation exposure was the culprit, and leukemia after kidney transplant is rare. An unjustified fear of cancer should not get in the way of essential radiologic imaging. However, prevalent kidney transplant patients received cumulative median doses of 17 mSv in another report, and 12% of that cohort had cumulative exposures of 100 mSv (36). Thus, there may be reason for concern in individual patients, perhaps especially in younger people with longer expected lifetimes and therefore more possibility of late radiationinduced cancers. Finally, the concern about exposure to ionizing radiation has affected practice in surveillance for nephrolithiasis; ultrasound, rather than CT may be preferable for serial imaging. TAKE HOME POINTS c Radiation nephropathy can result from external or internal radiation exposures. c Radiation nephropathy will not occur after diagnostic X-ray exposures. c Accidental or belligerent radiation exposures may result in renal radia- tion injury. c Radiation nephropathy is associated with mesangiolysis and glomerular capillary thrombosis on renal biopsy; it can lead to a full blown thrombotic microangiopathy. c Mitigation of radiation nephropathy may be possible with angiotensin converting enzyme inhibitors. ACKNOWLEDGEMENTS This work was supported (in part) by the intramural Research Program of the National Institutes of Health, the National Institute of Diabetes and Digestive and Kidney Diseases, and Merit Review Awards IO1 BX002256 from the US Department of Veterans Affairs Biomedical Laboratory Research and Development and IO1 CX000569 Clinical Sciences Research and Development (E.P.C., principal investigator, both grants). DISCLOSURES FUTURE CONCERNS The apparent excess of CKD in some Hiroshima-Nagasaki survivors emphasizes that wartime or terrorist radionuclear events could cause significant renal injury in those exposed to acutely survivable irradiation. Space exploration is another potential risk for both cardiovascular and renal disease. 4 Onco-Nephrology Curriculum None. REFERENCES Q:1 1. Baerman G, Linser P. Review of localized and general effects of radiation. Munch Med Wschr 7: 996, 1904 American Society of Nephrology 2. Edsall DL. The attitude of the clinician in regard to exposing patients to the x-ray. JAMA 47: 1425–1429, 1906 3. Cassady JR. Clinical radiation nephropathy. Int J Radiat Oncol Biol Phys 31: 1249–1256, 1995 4. Cohen EP. Radiation nephropathy after bone marrow transplantation. Kidney Int 58: 903–918, 2000 5. Dawson LA, Kavanagh BD, Paulino AC, Das SK, Miften M, Li XA, Pan C, Ten Haken RK, Schultheiss TE. Radiation-associated kidney injury. Int J Radiat Oncol Biol Phys 76: S108–S115, 2010 6. Luxton RW. Radiation nephritis: A long term study of 54 patients. Lancet 2: 1221–1224, 1961 7. Phillips TL, Wharam MD, Margolis LW. Modification of radiation injury to normal tissues by chemotherapeutic agents. Cancer 35: 1678–1684, 1975 8. Cohen EP, Pais P, Moulder JE. Chronic kidney disease after hematopoietic stem cell transplantation. Semin Nephrol 30: 627–634, 2010 9. Abboud I, Peraldi M, Hingorani S. Chronic kidney diseases in long-term survivors after allogeneic hematopoietic stem cell transplantation: Monitoring and management guidelines. Semin Hematol 49: 73–82, 2012 10. Dawson LA, Horgan A, Cohen EP. Kidney and ureter. In: ALERT c Adverse Late Effects of Cancer Treatment, Volume 2: Normal Tissue Specific Sites and Systems, edited by Rubin P, Constine LS, Marks LB, New York, Springer, 2014 11. Luxton RW. Radiation nephritis. Q J Med 22: 215–242, 1953 12. Sera N, Hida A, Imaizumi M, Nakashima E, Akahoshi M. The association between chronic kidney disease and cardiovascular risk factors in atomic bomb survivors. Radiat Res 179: 46–52, 2013 13. Moulder JE, Fish BL. Late toxicity of total body irradiation with bone marrow transplantation in a rat model. Int J Radiat Oncol Biol Phys 16: 1501–1509, 1989 14. Cohen EP, Fish BL, Moulder JE. Mitigation of radiation injuries via suppression of the renin-angiotensin system: Emphasis on radiation nephropathy. Curr Drug Targets 11: 1423–1429, 2010 15. Cohen SR, Cohen EP. Chronic oxidative stress after irradiation: An unproven hypothesis. Med Hypoth 80: 172–175, 2013 16. Krochak RJ, Baker DG. Radiation nephritis: Clinical manifestations and pathophysiologic mechanisms. Urology 27: 389–393, 1986 17. Baldwin JN, Hagstrom JWC. Acute radiation nephritis. Calif Med 97: 359–362, 1962 18. Cohen EP, Lawton CA, Moulder JE, Becker CG, Ash RC. Clinical course of late onset bone marrow transplant nephropathy. Nephron 64: 626– 635, 1993 19. Cohen EP, Robbins MEC. Radiation nephropathy. Semin Nephrol 23: 486–499, 2003 20. Fowler JF. Brief summary of radiobiological principles in fractionated radiotherapy. Semin Radiat Oncol 2: 16–21, 1992 21. Kersting S, Hené RJ, Koomans HA, Verdonck LF. Chronic kidney disease after myelablative allogeneic hematopoietic stem cell transplantation. Biol Blood Marrow Transplant 13: 1169–1175, 2007 American Society of Nephrology 22. Cohen EP. Fibrosis causes progressive kidney failure. Med Hypoth 45: 459–462, 1995 23. Crummy Jr AB, Hellman S, Stansel Jr HC, Hukill PB. Renal hypertension secondary to unilateral radiation damage relieved by nephrectomy. Radiology 84: 108–111, 1965 24. Dhaliwal RS, Adelman RD, Turner E, Russo JC, Ruebner B. Radiation nephritis with hypertension and hyperreninemia: cure by nephrectomy. J Pediat 96: 68–70, 1980 25. Salvi S, Green DM, Brecher ML, Magoos I, Gamboa LN, Fisher JE, Baliah T, Afshani E. Renal artery stenosis and hypertension after abdominal irradiation for Hodgkins disease: Successful treatment with nephrectomy. Urology 21: 611–615, 1983 26. White DC. The histopathologic basis for functional decrement in late radiation injury in diverse organs. Cancer 37: 1126–1143, 1976 27. Keane WF, Crosson JT, Staley NA, Anderson WR, Shapiro FL. Radiationinduced renal disease. Am J Med 60: 127–137, 1976 28. Cohen EP, Bedi M, Irving AA, Jacobs E, Tomic R, Klein J, Lawton CA, Moulder JE. Mitigation of late renal and pulmonary injury after hematopoietic stem cell transplantation. Int J Radiat Oncol Biol Phys 83: 292–296, 2012 29. Cohen EP, Piering WF, Kabler-Babbitt C, Moulder JE. End-stage-renaldisease after bone marrow transplantation: Poor survival compared to other causes of ESRD. Nephron 79: 408–412, 1998 30. Butcher JA, Hariharan S, Adams MB, Johnson CJ, Roza AM, Cohen EP. Renal transplantation for end stage renal disease following bone marrow transplantation. Clin Transplant 13: 330–335, 1999 31. Sayegh MH, Fine NA, Smith JL, Rennke HG, Milford EL, Tilney NL. Immunologic tolerance to renal allografts after bone marrow transplants from the same donor. Ann Intern Med 114: 964–965, 1991 32. Brenner DJ, Doll R, Goodhead DT, Hall EJ, Land CE, Little JB, Lubin JH, Preston DL, Preston RJ, Puskin JS, Ron E, Sachs RK, Samet JM, Setlow RB, Zaider M. Cancer risks attributable to low doses of ionizing radiation: Assessing what we really know. Proc Natl Acad Sci U S A 100: 13761–13766, 2003 33. Kinsella SM, Coyle JP, Long EB, McWilliams SR, Maher MM, Clarkson MR, Eustace JA. Maintenance hemodialysis patients have high cumulative radiation exposure. Kidney Int 78: 789–793, 2010 34. Nguyen KN, Patel AM, Weng FL. Ionizing radiation exposure among kidney transplant recipients due to medical imaging during the pretransplant evaluation. Clin J Am Soc Nephrol 8: 833–839, 2013 35. Collins AJ. USRDS 2013 Annual Data Report, Bethesda, MD, National Institutes of Health, 2013 36. De Mauri A, Brambilla M, Izzo C, Matheoud R, Chiarinotti D, Carriero A, Stratta P, De Leo M. Cumulative radiation dose from medical imaging in kidney transplant patients. Nephrol Dial Transplant 27: 3645–3651, 2012 Onco-Nephrology Curriculum 5 REVIEW QUESTIONS 1. Which of the following statements is false? a. Radiation nephropathy may result from radionuclide therapy b. Acute radiation nephritis may be associated with thrombotic thrombocytopenic purpura c. A single dose of 10 Gy of X-rays can lead to radiation nephropathy d. CKD has been associated solely with myeloablative total body irradiation regimens e. Cytotoxic chemotherapy can potentiate the effects of ionizing radiation. Answer: d is correct. CKD can complicate both myeloablative and nonmyeloablative regimens. The other statements are correct. 2. The pathologic features of radiation nephropathy include the following except: a. b. c. d. e. 6 Subendothelial expansion with amorphous material Mesangiolysis Neutrophilic infiltration of the mesangium Arteriolar sclerosis Varying degrees of tubular atrophy and interstitial fibrosis Onco-Nephrology Curriculum f. Endothelial cell swelling Answer: c is correct. Pathologic features of radiation nephropathy include endothelial cell swelling with subendothelial expansion with amorphous material and mesangiolysis, and late changes include arteriolar sclerosis and varying degrees of tubular atrophy and interstitial fibrosis; there is no evidence of glomerular leucocytic infiltration. 3. Which of the following statements is false regarding hypertension associated with radiation nephropathy: a. Hypertension following unilateral radiation may be treated with nephrectomy of the affected kidney b. Acute radiation nephritis is not associated with malignant hypertension c. Hypertension may occur in the absence of renal failure d. ACE inhibitors and/or angiotensin receptor blockers have been shown to be beneficial in management of hypertension Answer: b is correct. Malignant hypertension in the setting of radiation nephropathy may occur as part of the acute radiation nephritis complex, and it may also occur as an initial presentation, 12–18 months after irradiation. The other statements are correct. American Society of Nephrology Chapter 11. Chemotherapy and Kidney Injury Ilya G. Glezerman, MD,*† and Edgar A. Jaimes, MD*† *Renal Service, Department of Medicine, Memorial Sloan Kettering Cancer Center, New York, New York; and † Department of Medicine, Weill Cornell Medical College, New York, New York INTRODUCTION By January 1, 2011, 4,594,732 people in the United States carried the diagnosis of invasive malignancy. On the other hand, with significant advances in anticancer therapies, the 5-year survival for cancer patients has increased from 48.9% in 1975–1979 to 68.5% in 2006 (1). These statistics show that a significant percentage of the population is likely to be exposed to chemotherapy and suffer various shortterm and, in case of survivors, long-term adverse effects of treatment. Kidneys are vulnerable to the development of drug toxicity due to their role in the metabolism and excretion of toxic agents. The kidneys receive close to 25% of cardiac output, and the renal tubules and proximal segment in particular have significant capacity for uptake of drugs via endocytosis or transporter proteins. The high rate of delivery and uptake results in high intracellular concentration of various substances that then undergo extensive metabolism, leading to formation of potentially toxic metabolites and reactive oxygen species (ROS) (2). Numerous chemotherapy agents have been associated with various renal toxicities including tubulointerstitial damage, glomerular disease, electrolyte abnormalities, hypertension, and proteinuria (Table 1). AGENTS WITH PREDOMONANLTY TUBULAR TOXICITY Platinum compounds Cisplatin (cis-dichlorodiammineplatinum)–platinum coordination complex is an effective chemotherapy against a wide spectrum of tumors such as testicular, head and neck, ovarian, lung, cervical, and bladder cancers. Nephrotoxicity is the dose-limiting toxicity of cisplatin. Cisplatin induces the production of ROS and inhibits several antioxidant enzymes, leading to oxidative stress injury. It increases renal expression of tumor necrosis factor a, leading to increased tubular American Society of Nephrology cell apoptosis and production of ROS (3). Cisplatin is excreted and concentrated in the kidneys entering renal tubular cells via organic cation transporter 2, which is kidney specific (3). Initial renal toxicity manifests as a decrease in renal blood flow and subsequent decline in GFR within 3 hours of cisplatin administration. These changes are probably due to increased vascular resistance secondary to tubulo-glomerular feedback and increased sodium chloride delivery to macula densa. The decline in GFR appears to be dose dependent. In a group of patients who received four cycles of 100 mg/m2, the 51Cr-EDTA–measured GFR declined by 11.7%, whereas in patients who received three cycles of 200 mg/m2, the mean decline was 35.7%. This effect was noted to be lasting, as GFR was still 30% below baseline at 2 years (4). Acute tubular toxicity of cisplatin causes mitochondrial dysfunction, decreased ATPase activity, impaired solute transport, and altered cation balance. As a result, sodium and water reabsorption is decreased, and salt and water excretion is increased, leading to polyuria (3). Cisplatin also causes dose-dependent renal magnesium wasting. Tubulointerstitial injury is a predominant finding on pathologic examination of human kidneys affected by cisplatin toxicity. Both proximal and distal tubules are affected, and in patients with AKI, there is usually acute tubular necrosis. Long-term cisplatin exposure may also cause cyst formation and interstitial fibrosis (Figure 1) (3). Patients with cisplatin toxicity typically present with progressive azotemia in the setting of bland urinalysis and minimal proteinuria. Although renal function improves in most patients, a subgroup of patients developed permanent renal impairment. Hypomagnesemia is common and may be present in Correspondence: Ilya G. Glezerman, Memorial Sloan Kettering Cancer Center, 1275 York Ave., New York, New York 10024. Copyright © 2016 by the American Society of Nephrology Onco-Nephrology Curriculum 1 Table 1. Nephrotoxicity of common chemotherapy drugs Agent Drugs with tubular toxicity Cisplatin Ifosfamide Methotrexate Pemetrexed Ipilimumab Drugs with glomerular toxicity Gemcitabine Common pathologic finding Clinical syndromes ATN Chronic interstitial fibrosis and cyst formation ATN AKI, hypomagnesemia, renal sodium wasting, CKD Fanconi syndrome (partial or complete) AKI ESRD Nonoliguric AKI AKI CKD Nephrogenic diabetes insipidus AKI Crystal nephropathy ATN AIN Tubular atrophy and interstitial fibrosis AIN Thrombotic microangiopathy Mitomycin Thrombotic microangiopathy Bevacizumab Thrombotic microangiopathy VEGFR mTKI Sunitinib Sorafenib Axitinib Pazopanib Drugs causing electrolyte abnormalities EGFR antibody Cetuximab Panitumumab Imatinib Thrombotic microangiopathy MCD/cFSGS AKI MAHA Hypertension Dose dependent: AKI, MAHA Hypertension Proteinuria Hypertension Less common: Nephrotic syndrome AKI, MAHA Proteinuria Hypertension Less common: Nephrotic syndrome AKI, MAHA Inhibition of TRMP6 in distal convoluted tubule Hypomagnesemia Unknown Hypophosphatemia ATN, acute tubular necrosis; AIN, acute interstitial nephritis; MAHA, microangiopathic hemolytic anemia; VEGFR mTKI, vascular endothelial growth factor multitarget tyrosine kinase inhibitors; MCD/cFSGS, minimal change disease and/or collapsing-like focal segmental glomerulosclerosis; EGFR, epithelial growth factor receptor; TRMP, transient receptor potential cation channel, subfamily M, member 6. 42%–100% of patients depending on total cisplatin dose and length of exposure. Hypomagnesemia and renal magnesium wasting may persist for up to 6 years after initial dose (5). Renal salt wasting syndrome has been reported in up to 10% of patients, manifesting as hyponatremia and severe orthostatic hypotension in the setting of high urinary sodium concentration. This syndrome may present 2–4 months after initiation of cisplatin therapy (6). Rare cases of thrombotic microangiopathy have been reported in patients who were also receiving bleomycin with cisplatin (4). Syndrome of inappropriate antidiuretic hormone secretion (SIADH) has been documented in patients receiving vigorous hydration (7) but is less common now as cisplatin-associated nausea is treated with new-generation antiemetics, diminishing the stimulus for antidiuretic hormone secretion. Vigorous hydration has been shown to reduce the incidence of AKI in patients receiving cisplatin. Both mannitol and loop diuretics have also been used to ameliorate toxicity; however, randomized studies have not shown a clear benefit (8). 2 Onco-Nephrology Curriculum Numerous compounds have been studied to prevent cisplatin nephrotoxicity but only amifostine is US Food and Drug Administration (FDA) approved for protection against cumulative nephrotoxicity from cisplatin therapy. Amifostine is protective by increasing the binding of ROS to thiol groups. Side effects, cost, and concerns that it also diminishes antitumor effect have limited its use in clinical practice (3). A recent study in a murine model showed that magnesium supplementation during cisplatin therapy may attenuate renal damage; however, further studies in humans are needed to validate these findings (9). Carboplatin is also a platinum-based agent with a lower potential for nephrotoxicity compared with cisplatin but can be nephrotoxic at myeloablative doses of .800 mg/m2 (10). Oxaliplatin, another platinum compound, has no nephrotoxic potential. Ifosfamide Ifosfamide is an alkylating agent used in the treatment of a variety of childhood and adult malignancies. Its use, however, is American Society of Nephrology Figure 1. Cisplatin-induced acute tubular injury and necrosis (ATI/ATN). Light microscopy of the kidney biopsy specimen reveals dilated tubules with flattened epithelium. There is also apical blebbing of tubular cells and drop out of tubular cells from the basement membrane. associated with a significant risk for nephrotoxicity. Because it is commonly used in children, most of the data pertaining to nephrotoxicity of ifosfamide have been obtained in pediatric patients. It has been reported that GFR goes to ,90 mL/min per 1.73 m2 in 50% of and ,60 mL/min per 1.73 m2 in 11% of patients treated with ifosfamide, with an average reduction of GFR of 35.1 mL/min per 1.73 m2 at a median of 6 months after treatment (range, 1–47 months) (11). In adults, ifosfamide has been shown to reduce mean GFR from 81.5 to 68.5 mL/min per 1.73 m2 1 year after treatment in patients with prior exposure to cisplatin (12). Fanconi syndrome characterized by proximal tubular dysfunction with variable degrees of glucosuria in the setting of normoglycemia, renal phosphate and potassium wasting, proximal tubular acidosis, hypouricemia, and aminoaciduria has been reported in 5% of patients treated with ifosfamide (13). Patients who receive cumulative dose ,60 g/m2 are at lower risk of renal toxicity, whereas patients receiving .100 g/m2 are at highest. Platinum combination therapy, renal irradiation, nephrectomy, and hydronephrosis are additional risk factors (14). Renal disease may progress even after ifosfamide is discontinued and may lead to ESRD (15). Although the precise incidence of severe kidney dysfunction after ifosfamide exposure is unknown, recent review indicates that it appears to be a sporadic complication without clear relationship to cumulative dose (16). Methotrexate Methotrexate (MTX) is an antifolate agent that inhibits dihydrofolate reductase (DHFR), an important step in DNA synthesis. Fifty percent to 70% of the drug is bound to plasma proteins, and 95% is found in the urine 30 hours after administration in subjects with normal renal function (17). MTX is both filtered American Society of Nephrology and secreted by the kidneys. It is a weak organic acid and is poorly soluble in acidic urine (18). Although it is administered over a large therapeutic range, only high-dose methotrexate (HDMTX) therapy of .1 g/m2 has the potential for nephrotoxicity. MTX renal toxicity is presumed to be due to direct precipitation of the drug, as well to direct toxic effects on renal tubules. In a large clinical trial of 3,887 patients treated with HDMTX, renal dysfunction occurred in 1.8% of the subjects and was associated with a 4.4% mortality in this group (19). Affected patients usually develop nonoliguric and in more severe cases oliguric AKI shortly after the administration of HDMTX. Urinalysis is generally bland and without proteinuria. Because MTX is excreted in the urine, renal impairment affects the clearance of the drug. Prolonged exposure to toxic levels of MTX (.10 mmol/L at 24 hours; .1 mmol/L at 48 hours, and .0.1 mmol/L at 72 hours) may lead to life-threatening nonrenal toxicities such as prolonged cytopenias, mucositis, neurotoxicity, and hepatic dysfunction. MTX solubility is 10-fold higher in urine with a pH of 7.5 than in acidic urine, and therefore urinary alkalinization and aggressive hydration (2.5–3.5 L/m 2 per 24 hours starting 12 hours prior to chemotherapy administration) are important steps to establish brisk diuresis and prevent methotrexate precipitation in the tubules. Probenecid, penicillins, salicylates, sulfisoxazole, and nonsteroid anti-inflammatory drugs may increase the risk of nephrotoxicity as they interfere with renal tubular secretion of MTX and delay excretion. Leucovorin rescue is used in patients who develop nephrotoxicity and is aimed at prevention of nonrenal complications. Leucovorin acts as an antidote by bypassing blocked DHFR pathway. In patients who have toxic levels of MTX, the leucovorin rescue dose is given according to established nomograms (17) with doses of 100– 1,000 mg/m2 administered every 6 hours. Leucovorin rescue is an effective sole therapy in patients with MTX toxicity (20). Hemodialysis and hemoperfusion have been used in attempt to remove MTX from circulation. Although both modalities result in lower MTX plasma levels immediately after treatment, there is significant rebound effect with levels reaching 90%– 100% of preprocedure MTX concentrations (19). Glucarpidase (carboxypeptidase-G2), a recombinant bacterial enzyme that rapidly metabolizes MTX to inactive compounds, is able to decrease MTX plasma level .98% within 15 minutes after administration and is effective as a single dose (21,22). Although a number of studies showed rapid rates of MTX removal in patients with HDMTX nephrotoxicity, none had a control group, and true clinical impact of glucarpidase is difficult to assess (22). Time to renal recovery in most studies was similar to that of the leucovorin rescue case series (20,22). In one study, glucarpidase was associated with lower risk of grade 4 nonrenal toxicity if administered ,96 hours after HDMTX, but in the same study, inadequate leucovorin rescue was predictive of nonrenal toxicities. Glucarpidase only affects extracellular levels of MTX, which may explain the delay in renal recovery after MTX removal from circulation (23). The use of glucarpidase is limited by its high cost (.$100,000/patient), Onco-Nephrology Curriculum 3 other patients had AKI but did not undergo a kidney biopsy (30). Most patients improved with prompt discontinuation of ipilimumab and steroid therapy. AGENTS WITH PREDOMINANTLY GLOMERULAR TOXICITY Gemcitabine Pemetrexed is an antifolate agent that inhibits several enzymes involved in DNA synthesis. This drug is not metabolized significantly, and 70%–90% of the drug is excreted unchanged in the urine within the first 24 hours after administration. The half-life of pemetrexed is 3.5 hours in patients with normal renal function but is increased in patients with renal insufficiency resulting in higher exposure to the drug (24). Pemetrexed has not been studied in patients with creatinine clearance (CrCl) , 45 mL/min, but a fatality was reported in a patient with CrCl of 19 mL/min who received this drug (25). Mild and reversible renal toxicity has been reported in patients who received high-dose therapy ($600 mg/m2). Recently, several cases of pemetrexed-induced tubular injury were reported (26–29) including interstitial nephritis and fibrosis, as well as diabetes insipidus. After discontinuation of pemetrexed, the renal function stabilized in these patients, but did not return to pretreatment baseline. Gemcitabine is a pyrimidine analog used in the treatment of a variety of solid tumors. Nephrotoxicity of this agent manifests as thrombotic microangiopathy (TMA). During early clinical experience, TMA was reported at a low rate of 0.015%; however, as the drug became more widely used, the incidence was noted to increase to as high as 2.2%. TMA presents as new-onset renal insufficiency, various degrees of microangiopathic hemolytic anemia (MAHA), and new or worsening hypertension (HTN). In a single institution experience of 29 cases of gemcitabineinduced TMA, de novo renal dysfunction or worsening of preexisting CKD was noted in all patients (32). Kidney biopsies were performed in four cases and showed thrombi in small blood vessels, glomerular mesangiolysis, and widening of subendothelial space with detachment of endothelial cells from the glomerular basement membrane consistent with TMA (Figure 2). In this study, the development of TMA was independent of cumulative dose, which ranged from 4 to 81 g/m2. After discontinuation of gemcitabine 28% of patients had complete recovery of renal function, and 48% had partial recovery or stable renal function. Although patients in this study did not undergo plasmapheresis, some authors advocate this treatment for patients with TMA due to gemcitabine. Literature reviews show no difference in outcomes between patients treated with plasmapheresis and conservative management with drug withdrawal (32,33). Eculizumab, a monoclonal antibody directed against the complement protein C5 approved for treatment of atypical hemolytic uremic syndrome, has been used to treat gemcitabine-induced TMA (34–36). Of the six patients reported, two had complete renal response, two had partial improvement in renal function, and two patients showed no improvement. Given the response rates similar to supportive care alone, the use of eculizumab should be carefully weighed against its high cost. Ipilimumab Mitomycin Ipilimumab is a novel immunotherapy agent that has shown significant promise in the treatment of metastatic melanoma. Ipilimumab is a fully human monoclonal antibody directed against cytotoxic T-lymphocyte antigen-4 (CTLA-4), a key negative regulator of T-cell activation. Because of its immunomodulatory effects, ipilimumab has been associated with a number of immune-mediated side effects involving skin, liver, gastrointestinal tract, and endocrine system (30). Renal involvement appears less common, but two cases of biopsy proven granulomatous acute interstitial nephritis (AIN) and one case of lupus nephritis have been reported (30,31). Three Mitomycin is an antitumor antibiotic isolated from Streptomyces caespitosus used for treatment of gastrointestinal and other solid tumors. It has been associated with life-threatening TMA with renal failure and MAHA. Mitomycin nephrotoxicity is dose dependent, with the risk of TMA being 1.6% with cumulative doses #49 mg/m2 and as high as 30% at doses exceeding 70 mg/m2 (37). Therefore, doses exceeding 40 mg/m2 are not recommended. Figure 2. Gemcitabine-induced thrombotic microangiopathy. Light microscopy (H&E stain) of the kidney biopsy shows intraarteriolar microthrombus. (Courtesy of Dr. Surya V. Seshan.) and therefore its use should be considered only after standard supportive measures are maximized (22). For most patients, supportive care in the form of leucovorin rescue results in recovery of renal function, and additional doses of HDMTX may be given without untoward side effects (20). Pemetrexed 4 Onco-Nephrology Curriculum Antiangiogenic agents In the last several years, a group of agents called antiangiogenic therapies have been utilized in the treatment of a variety of solid American Society of Nephrology Figure 3. Vascular endothelial growth factor angiogenic pathway inhibition. VEGF binds its receptor expressed on the surface of endothelial cells and podocytes triggering intracellular and extracellular processes resulting in vascular proliferation. Several drugs classes have been employed to inhibit the activation of VEGFR and prevent angiogenesis, which is seminal for tumor growth. VEGF, vascular endothelial growth factor; VEGFR, vascular endothelial growth factor receptor; TK, intracellular VEGFR tyrosine kinases; mTKI, multitarget tyrosine kinase inhibitors. tumors. Angiogenesis is seminal for tumor growth and development of metastases, making it an attractive target for therapeutic intervention. Vascular endothelial growth factor (VEGF) is a proangiogenic factor that binds to a family of VEGF receptors (VEGFRs), with tyrosine kinase activity (TKR). The receptor binding triggers intracytoplasmic signaling pathways, leading to proliferation of endothelial cells and pericytes, recruitment of endothelial cell precursors, and growth of capillaries (38). In the kidneys, VEGF is expressed in podocytes, signals glomerular endothelial cells, and regulates survival of podocytes via autocrine mechanisms. VEGF maintains podocyte cytosolic calcium concentration and selective barrier to macromolecules (39). In addition, VEGF influences BP by up-regulating the synthesis of nitric oxide in the vascular endothelium and increasing the production of prostacycline resulting in vasodilatation (40). Several classes of antiangiogenic therapies targeting VEGF pathway are now available (Figure 3). Bevacizumab is a blocking humanized monoclonal antibody directed against VEGF. Another class is represented by a group of drugs known as small molecule multitarget tyrosine kinase inhibitors (mTKIs). These agents inhibit VEGFR and a number of other TKRs and include sunitinib, sorafenib, axitinib, and other drugs. Ramucirumab is a recombinant human monoclonal antibody directed against VEGFR. The renal effects of VEGF inhibition have been studied in murine models. When the VEGF gene is deleted only from podocytes in mice, they become hypertensive and proteinuric. Pathologic findings in the kidneys revealed typical features of American Society of Nephrology TMA with intracapillary thrombi, endotheliosis, and obliterated capillary loops (41). In humans, a similar spectrum of disorders has been associated with VEGF inhibition. Hypertension, proteinuria, and TMA have all been reported after VEGF inhibition. The effects of anti-VEGF antibody therapy on blood pressure were recently reviewed in a meta-analysis of seven randomized clinical trials that included 1,850 patients treated with bevacizumab. In patients who received a low dose (3–7.5 mg/kg/dose) of the drug, the relative risk (RR) of developing HTN was 3.0 (95% CI, 2.2–4.2; P,0.001). In a high-dose group (10–15 mg/kg/dose), the RR was 7.5 (95% CI, 4.2–13.4; P,0.001). Grade III HTN (requiring therapy or more intense therapy) was observed in 8.7% of patients in low-dose and 16.0% in high-dose groups. Proteinuria was also more common in treated patients. In the low-dose group, RR for proteinuria was 1.4 (95% CI, 1.1–1.7; P50.003), and in the high-dose group, RR was 2.2 (95% CI, 1.6–2.9; P,0.001). Grade III (.3.5 g/24 h) proteinuria was noted in 1.8% of patients in the high-dose group vs. only 0.1% of controls (42). TMA is the predominant glomerular lesion associated with anti-VEGF antibody therapy. It has been reported after intravenous (41,43–45) and ntraocular administration (46). However, concurrent mesangial IgA deposits, cryoglobulinemic glomerulonephritis (47), and immune complex–mediated focal proliferative glomerulonephritis (48) have also been reported. In patients with kidney biopsy findings of TMA, the clinical course varied from subnephrotic range proteinuria to more fulminant disease with worsening renal function, hypertension, and microangiopathic anemia (41,43–45). Hypertension is another major side effect of mTKI therapy. In a meta-analysis of 13 clinical trials that included 4,999 patients with renal cell carcinoma (RCC) and other malignancies treated with sunitinib, the incidence of all-grade hypertension was 21.6% and high grade was 6.8%. However, the RR was only statically significant for patients with high-grade hypertension at 22.72 (95% CI, 4.48–115.3). Furthermore, when patients were analyzed by the type of malignancy, only those with RCC had a statistically significant RR of developing both all-grade and high-grade HTN. More pronounced effects of mTKI in RCC may be due to higher VEGF levels in patients with RCC, resulting in a more evident anti-VEGF effect. In addition, the majority of patients with RCC also undergo nephrectomies, resulting in reduction in renal function and decreased excretion of sunitinib, leading to prolonged exposure to the drug (49). In phase 2 trials of axitinib proteinuria, all-grade proteinuria was reported in 18%–36% of subjects and grade $3 was 0%–5% (50). Proteinuria and nephrotic syndrome due to sunitinib or sorafenib have been described in a number of case reports and one case series (51–55). In these publications, proteinuria of up to 20 g/24 h has been reported, usually in association with new or worsening hypertension. These side effects generally resolved after discontinuation of mTKI. Two patients had a kidney biopsy that showed features of TMA in one patient and TMA and podocyte effacement in another. MAHA was not present in either case (53,55). Onco-Nephrology Curriculum 5 Several more fulminant cases of TMA with worsening renal function, severe hypertension and MAHAwith low haptoglobin, high lactate dehydrogenase levels, and schistocytosis have also been reported (43,56–58). However, in a cohort study of 29 patients treated with mTKIs who developed proteinuria and HTN and underwent a biopsy, minimal change disease and/or collapsing-like focal segmental glomerulosclerosis (MCD/cFSGS) was found in 20 cases (45). However, .55.5% of these patients had a history of nephrectomy, and hyperfiltration injury as a cause of MCD/cFSGS could not be completely ruled out. Because the putative mechanism of hypertension in patients treated with anti-VEGF therapies is intricately related to the antitumor action of these drugs, it has been proposed that the development of HTN could be used as biomarker of response (59). Two small retrospective studies showed that the development of hypertension was associated with improved oncologic outcomes in patients with RCC treated with axitinib and sunitinib (60,61). In a retrospective analysis of .500 patients treated with sunitinib for RCC, the overall survival (OS) and progression-free survival (PFS) was more than four-fold higher in the group of patients who developed sunitinib-induced hypertension defined as a maximum systolic blood pressure of $140 mmHg. However, hypertensive patients had more renal adverse events (5% versus 3%, P 5 0.013) (62). OS and PFS were also improved in patients with advanced non–small-cell lung cancer treated with bevacizumab who developed treatmentrelated hypertension. In this study, hypertension was defined as BP .150/100 mmHg or a $20-mmHg rise in diastolic blood pressure (DBP) (63). Nephrologists should be aware of these data as recommendations to discontinue anti-VEGF therapy due to the development of hypertension and proteinuria should be weighed against the possible enhanced antitumor effects in this setting. An expert panel from the National Institute of Cancer has issued guidelines in management of anti-VEGF therapy–induced hypertension. It recommends careful assessment of the patients prior to the initiation of therapy to identify those with cardiovascular risk factors, addressing preexisting hypertension prior to initiation of anti-VEGF therapy, and frequent monitoring of BP particularly during the first cycle. The patients should be treated if they develop BP .140/90 mmHg or DBP $20 mmHg higher than baseline. The panel did not make any specific recommendations about antihypertensive regimen due to lack of data and stated that treatment should be individualized to fit the patient’s comorbid conditions and to minimize drug interaction (64). Other considerations include concurrent development of proteinuria as a complication of anti-VEGF therapy. In this setting, it may be appropriate to use angiotensin converting enzyme inhibitors (ACE-Is) or angiotensin receptor blockers (ARBs) for their antiproteinuric effect. Additionally, use of ACE-Is or ARBs in combination with anti-VEGF therapy may have a synergistic effect on OS in patients with RCC (65). Although long-term effects of anti-VEGF therapy–induced HTN and proteinuria are unknown, it is probably prudent to continue the anticancer therapy if HTN and proteinuria are controlled with medical therapy. 6 Onco-Nephrology Curriculum However, if complications such as nephrotic syndrome, HTN with end-organ damage, renal insufficiency, or evidence of MAHA develop, discontinuation of antiangiogenic therapy should be considered promptly. In addition to HTN, proteinuria, and TMA, both mTKI and anti-VEGF antibody agents have been reported to cause acute AIN (54,66–69). Although some cases have been confirmed by renal biopsy, in others the diagnosis was made on clinical grounds because biopsy was precluded by thrombocytopenia or the presence of a solitary kidney. These patients had eosinophilia, eosinophiluria, and kidney dysfunction, and renal function either improved or stabilized after discontinuation of antiangiogenic therapy. In two cases, mTKI was administered intermittently (4 weeks on and 2 weeks off), and the patients exhibited “saw tooth” fluctuations in eosinophilia and SCr levels, with both parameters improving just before the initiation of the next cycle (54). AGENTS ASSOCIATED WITH ELECTROLYTE ABNORMALITIES Hypomagnesemia as a common complication of cisplatin therapy and Fanconi syndrome due to ifosfamide treatment have been addressed by this review already. However, a number of targeted biological agents have been associated with electrolyte imbalance. Cetuximab Cetuximab is a chimeric monoclonal antibody directed against epithelial growth factor receptor (EGFR). The EGFR is overexpressed in several tumors of epithelial origin, and cetuximab is often used in combination with chemotherapy for their treatment. Although in initial clinical trials hypomagnesemia was not reported (70), numerous published reports have established a link between low serum Mg21 and use of cetuximab. Active Mg21 transport in the kidney occurs predominantly in the distal convoluted tubule (DCT) and where EGFR is also expressed. TRPM 6 (transient receptor potential cation channel, subfamily M, member 6) has been demonstrated to play a role in this process. The epithelial growth factor (EGF) markedly increases the activity of TRMP 6, leading to the hypothesis that EGFR activation is necessary for reabsorption of Mg21 and that blockade of EGFR leads to renal Mg21 wasting (71) by blocking the activity of TRMP6. In one of the earlier reports, 34 patients on cetuximab had their Mg21 level measured at least once. Of these patients, 23% had grade 3 (,0.9–0.7 mg/dL) and 6% had grade 4 (,0.7 mg/dL) hypomagnesemia (72). In another report, the incidence of grade 3/4 hypomagnesemia was 27% (73). The severity of hypomagnesemia appears to correlate with duration of exposure and is difficult to manage. Daily infusions of up to 6–10 g of MgSO4 were required to correct the deficit in one cohort (73). Hypomagnesemia resolved in all cases after 4 weeks of discontinuation of cetuximab. Patients who develop clinically significant hypomagnesemia are also hypocalcemic due to parathyroid hormone resistance, American Society of Nephrology which is often seen in the presence of hypomagnesemia and resolves after Mg21 levels are normalized (72,73). In more recent randomized trials, the incidence of grade 3/4 hypomagnesemia ranged between 1.8%–5.8% in the cetuximab arm and 0%–0.4% in chemotherapy or best supportive care (BSC) arms. However, in these studies, the Mg21 levels were not routinely measured, which likely explain the lower incidence of hypomagnesemia (74). Panitumumab Panitumumab is a fully human antibody directed at EGFR and is used in treatment of metastatic colorectal cancer. In randomized trials, it has also been shown to cause low serum Mg21 levels, with an incidence of grade 3/4 hypomagnesemia ranging between 3%–5% in the panitumumab arm and 0%– ,1% in chemotherapy or BSC arms (75,76). therapies have been associated with kidney disease due to interference with signaling pathways in nonmalignant cells. TAKE HOME POINTS c A high percentage of the US population undergoes chemotherapy treatments and is at risk for renal complications because kidneys are a major route of elimination of these drugs. c Cisplatin and ifosfamide are major tubular toxins leading to both AKI and CKD. c Glomerular toxicity of chemotherapy most commonly manifests as TMA, with gemcitabine and mitomycin as major offenders. c VEGF antagonists such as anti-VEGF antibody and VEGFR TKI are associated with development of hypertension and proteinuria and in severe cases TMA. Most patients are managed with antihypertensive drugs, with discontinuation of therapy only in patients who develop nephrotic syndrome, malignant hypertension, or TMA. Imatinib Imatinib is a small molecule mTKI with specificity for BCRAbl, C-kit, and platelet-derived growth factor receptor (PDGFR) and activity against tumors characterized by dysregulation of function of these enzymes. Use of imatinib has been shown to cause hypophosphatemia. In the initial report, hypophosphatemia developed in 25 (51%) of 49 patients who had at least one measurement of serum phosphorus. Patients with both low and normal serum phosphate levels were found to have high urine fractional excretion of phosphate compared with controls, but only hypophosphatemic patients had elevated parathyroid hormone (PTH) levels (77). In another study, 14 (39%) of 36 patients treated with imatinib developed hypophosphatemia and low PTH levels (78). Additionally, serum phosphate levels were measured routinely in two clinical trials of 403 patients with chronic myeloid leukemia receiving imatinib. Hypophosphatemia was observed in 50% of the patients, but hypophosphatemia as an adverse event was only reported in 3% of the patients (79). The exact mechanism by which imatinib causes hypophosphatemia is unknown, but it has been proposed that it may inhibit bone resorption via inhibition of PDGFR and lead to decreased calcium and phosphate efflux from the bone. Lower calcium egress from bone has been postulated to cause mild secondary hyperparathyroidism, which in turn leads to increased renal phosphate losses (77). CONCLUSIONS Despite advances in diagnosis, treatment, and prevention of chemotherapy-induced kidney injury, significant challenges still remain. In many cases, the only therapeutic intervention available is the discontinuation of the offending agent. Future research should be directed toward development of antidote agents that protect normal cells and allow continuation of chemotherapy without compromising antitumor effects. In addition to traditional cytotoxic agents, new targeted biological American Society of Nephrology REFERENCES 1. National Cancer Institute. Surveillance, Epidemiology and End Results Program. Cancer Statistics. Available at: http://seer.cancer.gov/csr/ 1975_2011/sections.html. Accessed March 9, 2015 2. Perazella MA. Renal vulnerability to drug toxicity. Clin J Am Soc Nephrol 4: 1275–1283, 2009 3. Yao X, Panichpisal K, Kurtzman N, Nugent K. Cisplatin nephrotoxicity: A review. Am J Med Sci 334: 115–124, 2007 4. Meyer KB, Madias NE. Cisplatin nephrotoxicity. Miner Electrolyte Metab 20: 201–213, 1994 5. Lajer H, Daugaard G. Cisplatin and hypomagnesemia. Cancer Treat Rev 25: 47–58 1999 6. Hutchison FN, Perez EA, Gandara DR, Lawrence HJ, Kaysen GA. Renal salt wasting in patients treated with cisplatin. Ann Intern Med 108: 21– 25, 1988 7. Littlewood TJ, Smith AP. Syndrome of inappropriate antidiuretic hormone secretion due to treatment of lung cancer with cisplatin. Thorax 39: 636–637, 1984 8. Launay-Vacher V, Rey JB, Isnard-Bagnis C, Deray G, Daouphars M. Prevention of cisplatin nephrotoxicity: State of the art and recommendations from the European Society of Clinical Pharmacy Special Interest Group on Cancer Care. Cancer Chemother Pharmacol. 61: 903–909, 2008 9. Solanki MH, Chatterjee PK, Gupta M, Xue X, Plagov A, Metz MH, Mintz R, Singhal PC, Metz C. Magnesium protects against cisplatininduced acute kidney injury by regulating platinum accumulation. Am J Physiol Renal Physiol 307: F369–F384, 2014 10. Isnard-Bagnis C, Launay-Vacher V, Karie S, Deray G. Anticancer drugs. In: Clinical Nephrotoxins Renal Injury from Drug and Chemicals, edited by De Broe M, Porter G, Bennett W, Deray G, 3rd Ed., New York, Springer Scientific, 511–535 2008 11. Skinner R, Cotterill SJ, Stevens MC. Risk factors for nephrotoxicity after ifosfamide treatment in children: A UKCCSG Late Effects Group study. United Kingdom Children’s Cancer Study Group. Br J Cancer 82: 1636– 1645, 2000 12. Farry JK, Flombaum CD, Latcha S. Long term renal toxicity of ifosfamide in adult patients: 5 year data. Eur J Cancer 48: 1326–1331, 2012 13. Suarez A, McDowell H, Niaudet P, Comoy E, Flamant F. Long-term follow-up of ifosfamide renal toxicity in children treated for malignant mesenchymal tumors: An International Society of Pediatric Oncology report. J Clin Oncol 9: 2177–2182, 1991 Onco-Nephrology Curriculum 7 14. Jones DP, Spunt SL, Green D, Springate JE. Renal late effects in patients treated for cancer in childhood: A report from the Children’s Oncology Group. Pediatr Blood Cancer 51: 724–731, 2008 15. Berns JS, Haghighat A, Staddon A, Cohen RM, Schmidt R, Fisher S, Rudnick MR, Tomaszewski JE. Severe, irreversible renal failure after ifosfamide treatment. A clinicopathologic report of two patients. Cancer 76:497–500, 1995 16. Akilesh S, Juaire N, Duffield JS, Smith KD. Chronic Ifosfamide toxicity: Kidney pathology and pathophysiology. Am J Kidney Dis 63:843–850, 2014 17. Bleyer WA. The clinical pharmacology of methotrexate: new applications of an old drug. Cancer 41: 36–51, 1978 18. Smith SW, Nelson LS. Case files of the New York City Poison Control Center: Antidotal strategies for the management of methotrexate toxicity. J Med Toxicol 4: 132–140, 2008 19. Widemann BC, Adamson PC. Understanding and managing methotrexate nephrotoxicity. Oncologist 11: 694–703, 2006 20. Flombaum CD, Meyers PA. High-dose leucovorin as sole therapy for methotrexate toxicity. J Clin Oncol 17: 1589–1594, 1999 21. Widemann BC, Balis FM, Kim A, Boron M, Jayaprakash N, Shalabi A, O’Brien M, Eby M, Cole DE, Murphy RF, Fox E, Ivy P, Adamson PC. Glucarpidase, leucovorin, and thymidine for high-dose methotrexateinduced renal dysfunction: Clinical and pharmacologic factors affecting outcome. J Clin Oncol 28: 3979–3986, 2010 22. Cavone JL, Yang D, Wang A. Glucarpidase intervention for delayed methotrexate clearance. Ann Pharmacother 48: 897–907, 2014 23. Meyers PA, Flombaum C. High-dose methotrexate-induced renal dysfunction: Is glucarpidase necessary for rescue? J Clin Oncol 29: e180, 2011 24. Villela LR, Stanford BL, Shah SR. Pemetrexed, a novel antifolate therapeutic alternative for cancer chemotherapy. Pharmacotherapy 26: 641–654, 2006 25. Mita AC, Sweeney CJ, Baker SD, Goetz A, Hammond LA, Patnaik A, Tolcher AW, Villalona-Calero M, Sandler A, Chaudhuri T, Molpus K, Latz JE, Simms L, Chaudhary AK, Johnson RD, Rowinsky EK, Takimoto CH. Phase I and pharmacokinetic study of pemetrexed administered every 3 weeks to advanced cancer patients with normal and impaired renal function. J Clin Oncol 24: 552–562, 2006 26. Glezerman IG, Pietanza MC, Miller V, Seshan SV. Kidney tubular toxicity of maintenance pemetrexed therapy. Am J Kidney Dis 58: 817–820, 2011 27. Michels J, Spano JP, Brocheriou I, Deray G, Khayat D, Izzdine H. Acute tubular necrosis and interstitial nephritits during pemetrexed Ttherapy. Case Rep Oncol 2: 53–56, 2009 28. Stavroulopoulos A, Nakopoulou L, Xydakis AM, Aresti V, Nikolakopoulou A, Klouvas G. Interstitial nephritis and nephrogenic diabetes insipidus in a patient treated with pemetrexed. Ren Fail 32: 1000–1004, 2010 29. Vootukuru V, Liew YP, Nally JV Jr. Pemetrexed-induced acute renal failure, nephrogenic diabetes insipidus, and renal tubular acidosis in a patient with non-small cell lung cancer. Med Oncol 23: 419–422, 2006 30. Izzedine H, Gueutin V, Gharbi C, Mateus C, Robert C, Routier E, Thomas M, Baumelou A, Rouvier P. Kidney injuries related to ipilimumab. Investig New Drugs 32: 769–773, 2014 31. Fadel F, El Karoui K, Knebelmann B. Anti-CTLA4 antibody-induced lupus nephritis. N Engl J Med 361: 211–212, 2009 32. Glezerman I, Kris MG, Miller V, Seshan S, Flombaum CD. Gemcitabine nephrotoxicity and hemolytic uremic syndrome: Report of 29 cases from a single institution. Clin Nephrol 71: 130–139, 2009 33. Gore EM, Jones BS, Marques MB. Is therapeutic plasma exchange indicated for patients with gemcitabine-induced hemolytic uremic syndrome? J Clin Apheresis 24: 209–214, 2009 34. Starck M, Wendtner CM. Use of eculizumab in refractory gemcitabineinduced thrombotic microangiopathy. Br J Haematol 164: 894–896, 2014 35. Al Ustwani O, Lohr J, Dy G, Levea C, Connolly G, Arora P, Iyer R. Eculizumab therapy for gemcitabine induced hemolytic uremic syndrome: Case series and concise review. J Gastrointestinal Oncol 5: E30–E33, 2014 8 Onco-Nephrology Curriculum 36. Tsai HM, Kuo E. Eculizumab therapy leads to rapid resolution of thrombocytopenia in atypical hemolytic uremic syndrome. Adv Hematol 2014: 295323, 2014 37. Valavaara R, Nordman E. Renal complications of mitomycin C therapy with special reference to the total dose. Cancer 55: 47–50, 1985 38. Mena AC, Pulido EG, Guillen-Ponce C. Understanding the molecularbased mechanism of action of the tyrosine kinase inhibitor: Sunitinib. Anticancer Drugs 21(Suppl 1): S3–S11, 2010 39. Breen EC. VEGF in biological control. J Cell Biochem 102: 1358–1367, 2007 40. Yogi A, O’Connor SE, Callera GE, Tostes RC, Touyz RM. Receptor and nonreceptor tyrosine kinases in vascular biology of hypertension. Curr Opin Nephrol Hypertens 19: 169–176, 2010 41. Eremina V, Jefferson JA, Kowalewska J, Hochster H, Haas M, Weisstuch J, Richardson C, Kopp JB, Kabir MG, Backx PH, Gerber HP, Ferrara N, Barisoni L, Alpers CE, Quaggin SE. VEGF inhibition and renal thrombotic microangiopathy. N Engl J Med 358:1129–1136, 2008 42. Zhu X, Wu S, Dahut WL, Parikh CR. Risks of proteinuria and hypertension with bevacizumab, an antibody against vascular endothelial growth factor: Systematic review and meta-analysis. Am J Kidney Dis 49: 186–193, 2007 43. Frangie C, Lefaucheur C, Medioni J, Jacquot C, Hill GS, Nochy D. Renal thrombotic microangiopathy caused by anti-VEGF-antibody treatment for metastatic renal-cell carcinoma. Lancet Oncol 8: 177– 178, 2007 44. Roncone D, Satoskar A, Nadasdy T, Monk JP, Rovin BH. Proteinuria in a patient receiving anti-VEGF therapy for metastatic renal cell carcinoma. Nat Clin Pract Nephrol 3: 287–293, 2007 45. Izzedine H, Escudier B, Lhomme C, Pautier P, Rouvier P, Gueutin V, Baumelou A, Derosa L, Bahleda R, Hollebecgue A, Sahali D, Soria JC. Kidney diseases associated with anti-vascular endothelial growth factor (VEGF): An 8-year observational study at a single center. Medicine 93: 333–339, 2014 46. Pelle G, Shweke N, Van Huyen JP, Tricot L, Hessaine S, FremeauxBacchi V, Hiesse C, Delahousse M. Systemic and kidney toxicity of intraocular administration of vascular endothelial growth factor inhibitors. Am J Kidney Dis 57: 756–759, 2011 47. Johnson DH, Fehrenbacher L, Novotny WF, Herbst RS, Nemunaitis JJ, Jablons DM, Langer CJ, DeVore RF 3rd, Gaudreault J, Damico LA, Holmgren E, Kabbinavar F. Randomized phase II trial comparing bevacizumab plus carboplatin and paclitaxel with carboplatin and paclitaxel alone in previously untreated locally advanced or metastatic non-small-cell lung cancer. J Clin Oncol 22: 2184–2191, 2004 48. George BA, Zhou XJ, Toto R. Nephrotic syndrome after bevacizumab: Case report and literature review. Am J Kidney Dis. 49: e23–e29, 2007 49. Zhu X, Stergiopoulos K, Wu S. Risk of hypertension and renal dysfunction with an angiogenesis inhibitor sunitinib: Systematic review and meta-analysis. Acta Oncol 48: 9–17, 2009 50. Izzedine H, Massard C, Spano JP, Goldwasser F, Khayat D, Soria JC. VEGF signalling inhibition-induced proteinuria: Mechanisms, significance and management. Eur J Cancer 46: 439–448, 2010 51. Patel TV, Morgan JA, Demetri GD, George S, Maki RG, Quigley M, Humphreys BD. A preeclampsia-like syndrome characterized by reversible hypertension and proteinuria induced by the multitargeted kinase inhibitors sunitinib and sorafenib. J Natl Cancer Inst 100: 282– 284, 2008 52. Obhrai JS, Patel TV, Humphreys BD. The case/progressive hypertension and proteinuria on anti-angiogenic therapy. Kidney Int 74: 685– 686, 2008 53. Overkleeft EN, Goldschmeding R, van Reekum F, Voest EE, Verheul HM. Nephrotic syndrome caused by the angiogenesis inhibitor sorafenib. Ann Oncol 21: 184–185, 2010 54. Jhaveri KD, Flombaum CD, Kroog G, Glezerman IG. Nephrotoxicities associated with the use of tyrosine kinase inhibitors: A single-center experience and review of the literature. Nephron Clin Pract 117: c312– c319, 2011 American Society of Nephrology 55. Bollee G, Patey N, Cazajous G, Robert C, Goujon JM, Fakhouri F, Bruneval P, Noel LH, Knebelmann B. Thrombotic microangiopathy secondary to VEGF pathway inhibition by sunitinib. Nephrol Dial Transplant 24: 682–685, 2009 56. Kapiteijn E, Brand A, Kroep J, Gelderblom H. Sunitinib induced hypertension, thrombotic microangiopathy and reversible posterior leukencephalopathy syndrome. Ann Oncol 18: 1745–1747, 2007 57. Choi MK, Hong JY, Jang JH, Lim HY. TTP-HUS associated with sunitinib. Cancer Res Treat 40: 211–213, 2008 58. Levey SA, Bajwa RS, Picken MM, Clark JI, Baron K, Leehey DJ. Thrombotic microangiopathy associated with sunutinib, a VEGF inhibitor, in a patient with factor V Leiden mutation. Nephrol Dialysis Transplant Plus 3: 154–156, 2008 59. van Heeckeren WJ, Ortiz J, Cooney MM, Remick SC. Hypertension, proteinuria, and antagonism of vascular endothelial growth factor signaling: Clinical toxicity, therapeutic target, or novel biomarker? J Clin Oncol 25: 2993–2995, 2007 60. Rixe O, Billemont B, Izzedine H. Hypertension as a predictive factor of Sunitinib activity. Ann Oncol 18: 1117, 2007 61. Rixe O, Dutcher JP, Motzer RJ, Wilding G, Stadler WM, Kim S, Tarazi J, Motzer RJ. Association between diastolic blood pressure (DBP) $ 90 mm Hg and efficacy in patients (pts) with metastatic renal cell carcinoma (MRCC) receiving axitinib (AG-013736; AG). Ann Oncol 19(Suppl 8): viii189, 2008 62. Rini BI, Cohen DP, Lu DR, Chen I, Hariharan S, Gore ME, Figlin RA, Baum MS, Motzer RJ. Hypertension as a biomarker of efficacy in patients with metastatic renal cell carcinoma treated with sunitinib. J Natl Cancer Inst 103: 763–773, 2011 63. Dahlberg SE, Sandler AB, Brahmer JR, Schiller JH, Johnson DH. Clinical course of advanced non-small-cell lung cancer patients experiencing hypertension during treatment with bevacizumab in combination with carboplatin and paclitaxel on ECOG 4599. J Clin Oncol 28: 949–954, 2010 64. Maitland ML, Bakris GL, Black HR, Chen HX, Durand JB, Elliott WJ, Ivy SP, Leier CV, Lindenfeld J, Liu G, Remick SC, Steingart R, Tang WH. Initial assessment, surveillance, and management of blood pressure in patients receiving vascular endothelial growth factor signaling pathway inhibitors. J Natl Cancer Inst 102: 596–604, 2010 65. McKay RRRGE, Lin X, Simantov R, Choueiri TK. Impact of angiotensin system inhibitors on outcomes in patients with metastatic renal cell carcinoma: Results from a pooled clinical trials database. J Clin Oncol 32(4 Suppl): 437, 2014 66. Winn SK, Ellis S, Savage P, Sampson S, Marsh JE. Biopsy-proven acute interstitial nephritis associated with the tyrosine kinase inhibitor sunitinib: a class effect? Nephrol Dial Transplant 24: 673–675, 2009 American Society of Nephrology 67. Izzedine H, Brocheriou I, Rixe O, Deray G. Interstitial nephritis in a patient taking sorafenib. Nephrol Dial Transplant 22 :2411, 2007 68. Khurana A. Allergic interstitial nephritis possibly related to sunitinib use. Am J Geriatr Pharmacother 5: 341–344, 2007 69. Barakat RK, Singh N, Lal R, Verani RR, Finkel KW, Foringer JR. Interstitial nephritis secondary to bevacizumab treatment in metastatic leiomyosarcoma. Ann Pharmacother 41: 707–710, 2007 70. Cunningham D, Humblet Y, Siena S, Khayat D, Bleiberg H, Santoro A, bets D, Mueser M, Harstrick A, Verslype C, Chau I, van Cutsen E. Cetuximab monotherapy and cetuximab plus irinotecan in irinotecan-refractory metastatic colorectal cancer. N Engl J Med 351: 337–345, 2004 71. Izzedine H, Bahleda R, Khayat D, Massard C, Magne N, Spano JP, Soria JC. Electrolyte disorders related to EGFR-targeting drugs. Crit Rev Oncol Hematol 73: 213–219, 2010 72. Schrag D, Chung KY, Flombaum C, Saltz L. Cetuximab therapy and symptomatic hypomagnesemia. J Natl Cancer Inst 97: 1221–1224, 2005 73. Fakih MG, Wilding G, Lombardo J. Cetuximab-induced hypomagnesemia in patients with colorectal cancer. Clin Colorectal Cancer 6: 152– 156, 2006 74. Fakih M. Management of anti-EGFR-targeting monoclonal antibodyinduced hypomagnesemia. Oncology (Williston Park) 22: 74–76, 2008 75. Peeters M, Price TJ, Cervantes A, Sobrero AF, Ducreux M, Hotko Y, Andre T, Chan E, Lordick F, Punt CJ, Strickland AH, Wilson G, Ciuleanu TE, Roman L, Van Cutsen E, Tzekova V, Collins S, Oliner KS, Rong A, Gansert J. Randomized phase III study of panitumumab with fluorouracil, leucovorin, and irinotecan (FOLFIRI) compared with FOLFIRI alone as second-line treatment in patients with metastatic colorectal cancer. J Clin Oncol 28: 4706–4713, 2010 76. Van Cutsem E, Peeters M, Siena S, Humblet Y, Hendlisz A, Neyns B, Canon JL, Van Laethem JL, Maurel J, Richardson G, Wolf M, Amado RG. Open-label phase III trial of panitumumab plus best supportive care compared with best supportive care alone in patients with chemotherapy-refractory metastatic colorectal cancer. J Clin Oncol 25: 1658–1664, 2007 77. Berman E, Nicolaides M, Maki RG, Fleisher M, Chanel S, Scheu K, Wilson BA, Heller G, Sauter NP. Altered bone and mineral metabolism in patients receiving imatinib mesylate. N Engl J Med 354:2006–2013, 2006 78. Osorio S, Noblejas AG, Duran A, Steegmann JL. Imatinib mesylate induces hypophosphatemia in patients with chronic myeloid leukemia in late chronic phase, and this effect is associated with response. Am J Hematol 82:394–395, 2007 79. Owen S, Hatfield A, Letvak L. Imatinib and altered bone and mineral metabolism. N Engl J Med 355: 627, 2006 Onco-Nephrology Curriculum 9 REVIEW QUESTIONS 1. The best treatment options for gemcitabine induced thrombotic microangiopathy are: a. Plasmapheresis b. Administration of eculizumab c. Discontinuation of gemcitabine and best supportive care d. All of the above Answer: c is correct. Whereas both plasmapheresis and eculizumab have been used in the treatment of gemcitabineinduced thrombotic microangiopathy, there is little evidence that outcomes of these treatments are superior to supportive care alone. 2. Which presentation is most consistent with cisplatin nephrotoxicity? a. Elevated serum creatinine, minimal proteinuria, hypomagnesemia b. Hypertension, elevated serum creatinine, low platelet count c. Nephrotic range proteinuria, hypertension, and edema d. Elevated serum creatinine, hypophosphatemia, glucosuria Answer: a is correct. Cisplatin toxicity involves damage to the tubulo-interstitial compartment and manifests as AKI with relatively normal urinalysis. Hypomagnesemia is a common manifestation of cisplatin tubular toxicity. Cisplatin does not 10 Onco-Nephrology Curriculum cause nephrotic syndrome, and thrombotic microangiopathy and Fanconi syndrome are rare. 3. A 55-year-old man with a history of metastatic renal cell carcinoma was begun on treatment with sunitinib (VEGFR TKI). Two months after starting the treatment, he was noted to have a BP of 154/90 mmHg, and his random urinary protein to creatinine ratio was 2.3. The patient was asymptomatic. His renal function remained normal, and there was no evidence of hemolysis on his blood work. The next step is: a. Discontinue sunitinib and offer best supportive care b. Begin antihypertensive therapy aimed at reducing his blood pressure to ,140/90 mmHg and continue to monitor urinary protein to creatinine ratio closely c. Reduce sunitinib dose d. Switch the patient from sunitinib to sorafenib Answer: b is correct. There are no evidence-based recommendations on management of proteinuria and hypertension induced by VEGF inhibitors. In practice, these agents are generally continued unless patients develop nephrotic syndrome, malignant hypertension, or thrombotic microangiopathy. Reduction of the sunitinib dose may be attempted as a next step if hypertension is difficult to control (answer c). Sorafenib is likely to have a similar side effect profile (answer d). American Society of Nephrology Chapter 12: Pharmacokinetics of Chemotherapeutic Agents in Kidney Disease Sheron Latcha, MD, FASN Department of Medicine, Memorial Sloan Kettering Cancer Center, New York, New York INTRODUCTION The liver and kidneys serve as the major pathways for drug metabolism and elimination, with much smaller contributions from the fecal and reticuloendothelial systems. As shown in Figure 1, some unique aspects of renal physiology that make the kidneys particularly susceptible to drug exposure and injury include 1) a high blood flow rate and therefore high drug delivery rate to the kidney (blood flow to the kidney approximates 25% of cardiac output); 2) the medulla’s considerable concentrating ability, which enhances local drug tissue concentration; 3) the presence of organic anion transporters within the tubules, which allow nephrotoxic medications and their toxic metabolites to become concentrated within the tubules; and 4) the presence of renal enzymes, which can form toxic metabolites and reactive oxygen species (CYP450 and flavin-containing monooxygenase) (1,2). Cancer drugs have been demonstrated to cause nephrotoxicity via direct tubular injury, tubular obstruction, injury to the tubulointerstitium, and glomerular damage. Certainly, the prevalence of renal insufficiency in cancer patients, and the kidney’s role in drug metabolism, has implications for the choice and dosing of chemotherapeutic agents. In this section, recommendations for dose modifications for some of the more frequently used chemotherapeutic agents, which require adjustments in patients with various levels of CKD, will be discussed. For a more complete discussion of all chemotherapeutic agents that require renal dosing, the reader is referred elsewhere (3–6). Published guidelines for dose modification of chemotherapy for cancer patients with CKD are largely based on limited pharmacokinetic (PK) and pharmacodynamic (PD) data and often on studies of poor quality (physician-initiated postmarketing studies, small sample sizes). Historically, the Cockcroft and Gault (CG) formula was most often used to estimate GFR, and this equation has been shown to overestimate GFR. Additionally, before the advent of American Society of Nephrology the isotope dilution mass spectroscopy (IDMS) creatinine assay, the variability in creatinine assays likely affected the PK and PD data from past studies, and therefore, the resulting dosing recommendations being used currently (7). What remains largely unaddressed in all PK and PD studies to date is that CKD can significantly alter nonrenal clearance and modify bioavailability of drugs predominantly metabolized by the liver and intestine. It has been shown that CKD suppresses various liver metabolic enzymes (CPT2C9, CYP2C19, and CYP3A4), and these effects are clinically significant (8). GFR is the metric used to guide dose adjustment, and as shown in Table 1 a number of equations are available to calculate GFR. In cancer patients, compared with Tc99mDPTA clearance, the Martin and Wright formulae seem to have the best concordance, followed by the Chronic Kidney Disease Epidemiology Collaboration (CKD-EPI) equation (60.2%, 56.5%, and 56.3%, respectively). However, there were similar levels concordance in dosage selection between the assorted formulae and Tc99mDPTA clearance when selecting carboplatin dose (9), so the variations in concordance may not appreciably change final dose recommendations. CARBOPLATIN Notwithstanding all of these limitations, carboplatin is one of the few chemotherapeutic agents with good prospective PK and PD data in CKD patients (10–12). It is the third most commonly prescribed cytotoxic agent (3). About 70% of the administered dose is eliminated by the kidneys, and the drug has Correspondence: Department of Medicine, Memorial Sloan Kettering Cancer Center, 1275 York Ave., Suite 1204b, New York, New York 10065. Copyright © 2016 by the American Society of Nephrology Onco-Nephrology Curriculum 1 The drug can be administered to hemodialysis (HD) patients to achieve an AUC of 4–6 such that carboplatin dose (mg)=Target AUCx25. The drug is not avidly protein bound shortly after administration, and approximately 70% is cleared by HD if performed within several hours after infusion (15). Importantly, a majority of carboplatin becomes protein bound after 24 hours (12), so carboplatin administration should be coordinated to perform dialysis within 24 hours of dosing (16). CISPLATIN Figure 1. Susceptibility of the kidneys to drug exposure and delivery. only rarely been associated with AKI at high doses (1,600– 2,400 mg/m2) following bone marrow transplant (BMT) (13,14). At the usual doses range from 400 to 600 mg/m2, the drug is much less nephrotoxic. Neuropathy and myelosuppression are its main toxicities. Calvert’s formula is used to calculate the area under the curve (AUC): carboplatin dose (mg) 5 target AUC 3 (GFR 1 25). Although they are members of the same chemical family, cisplatin is considered to be superior therapy for specific tumor types compared with carboplatin. Unfortunately, nephrotoxicity is the dose-limiting side effect of this very effective chemotherapeutic agent (17–19). In the earliest reports of renal toxicity, incidence rates of 28%–36% were reported in patients receiving a single dose of 50 mg/m2 (Bristol Meyer Packaging), but the severity of renal toxicity decreased following institution of vigorous hydration protocols with normal saline (NS). The chloride in NS decreases the formation of toxic reactive platinum compounds (20). Initial declines in a GFR range from 12% to 19% (21–23), but some patients have shown improvement in renal function over time, implying that renal recovery is possible in some cases (23,24). However, there is usually persistent subclinical renal injury following cisplatin exposure (20). A detailed understanding of the mechanisms by which cisplatin induces renal toxicity remains unclear. The kidney Table 1. eGFR formulas in cancer patients 2 Onco-Nephrology Curriculum American Society of Nephrology selectively accumulates cisplatin and its analogues to a greater extent than other organs(18). On microscopic examination, cisplatin produces a tubulointerstitial lesion, with acute tubular necrosis being the predominant lesion, whereas the glomeruli are spared (20,25). Cisplatin-induced tubular damage affects the PK of subsequent cisplatin doses and causes a decrease in the percentage of cisplatin excreted and an increase in the AUC following sequential doses of the drug (26,27). The drug is 90% protein bound within 2 hours after infusion, and 30% is excreted in the urine within 24 hours. Dosing guidelines for patients with CKD are empiric. Kintzel recommends for a creatinine clearance (CrCl) of 46–60 mL/min, give 75% of the usual dose; for a CrCl of 31–45 mL/min, give 50% of the usual dose, and for a CrCl ,30 mL/min, the drug is not recommended (4). Arnoff recommends for CrCl of 10–50 mL/min, give 75% of the usual dose, and for CrCl ,10 mL/min, give 50% of the usual dose. The manufacturer recommends withholding repeat administration of the drug until the serum creatinine concentration is ,1.5 mg/dL. Cisplatin has been successfully administered to HD patients with similar tolerance as in patients with normal renal function. Because the drug is highly and irreversibly protein bound and because free cisplatin is well dialyzed, drug that is dialyzed off cannot be replaced by bound drug. As such, cisplatin must be given on a nondialysis day. For HD patients, it is recommended that the initial dose be reduced by 50%, or 25–50 mg/m2 every 3–6 weeks (16). IFOSFAMIDE Although hemorrhagic cystitis is the predominant toxicity of this alkylating agent, renal toxicity can be dose limiting. A number of histopathologic changes in the kidney have been observed with ifosfamide and include segmental glomerular sclerosis, tubulointerstitial nephritis, tubular atrophy, and interstitial fibrosis (28). Chloroacetaldehyde, a metabolite of ifosfamide, is toxic to renal epithelial cells and may contribute to nephrotoxicity of the parent drug (29,30). Clinical manifestations of renal tubular toxicity include Fanconi syndrome, proximal and distal renal tubular acidosis, hypophosphatemia, hypokalemia, and nephrogenic diabetes insipidus. Up to 87% of ifosfamide and its metabolites are recovered in the urine, and up to 41% of the dose is recovered in the urine as alkylating activity (31). Central nervous system (CNS) toxicity may be linked to the accumulation of the metabolite chloroacetaldehyde, and the risk of CNS toxicity is greater in patients with abnormal renal function (32). Dosing guidelines are empiric and vary widely. Kintzel recommends for CrCl of 46–60 mL/min, give 80% of dose; for CrCl of 31–45 mL/min, give 75% of dose; and for CrCl of ,30 mL/min, give 70% of dose. Aronoff recommends giving 75% of the usual dose for a GFR ,10 mL/min (4,5). The drug has been used without significant myelosuppression or neurotoxicity in anuric and oliguric patients on HD at starting doses of 1.5 g/m2 at 48- to 72-hour intervals, with HD to American Society of Nephrology follow in the range of 3–14 hours after drug administration. Subsequent dose adjustments were based on signs of neurotoxicity or myelosuppression (33). For HD patients with urine output, hydration and Mesna are needed to prevent hemorrhagic cystitis, and a hydration protocol is outlined elsewhere (33,34). CYCLOPHOSPHAMIDE There are no clear guidelines for dose adjustment of cyclophosphamide in the setting of renal insufficiency. Although cyclophosphamide is largely cleared by hepatic metabolism, up to 60% of the total dose is eliminated by the kidney as the parent drug or metabolites (35), and renal insufficiency is associated in changes in the PK profile of the parent drug and its metabolites. Aronoff recommends giving 75% of the usual dose for GFR ,10 mL/min. Because the AUC of cyclophosphamide is increased in HD patients, it is recommended that the dose be reduced by 25% in this group (36,37). The drug should be given after HD because the drug and its metabolites are dialyzable. METHOTREXATE (MTX) Renal excretion of this antifolate agent is 60% and 94% when the drug is administered over 6 and 24 hours, respectively. Nephrotoxicity has been observed at doses exceeding 1 g/m2 (38) and results from intratubular precipitation of MTX and its metabolites in the distal tubules, which causes an obstructive tubulopathy and decreased glomerular filtration (4,39). Pretreatment with cisplatin has been reported to increase MTX toxicity, possibly by decreasing renal clearance (40). AKI prolongs extrarenal toxicity (myelosuppression and gastrointestinal toxicity) and is observed at plasma concentrations 5–20 mmol/L at 24 hours, 0.5–2 mmol/L at 48 hours, and 0.05–0.1 mmol/L at 72 hours following drug administration. When serum drug levels reach toxic levels, intravenous leucovorin is generally used to circumvent MTX’s inhibition of dihydrofolate reductase to “rescue” normal cells from MTX (38). MTX is poorly soluble in acid urine, and decreased flow rates increase its concentration in the renal tubules. Prevention remains the best treatment for MTX toxicity and includes measures to alkalinize and maximally dilute the urine with bicarbonate containing intravenous fluids as needed to achieve a urine pH . 7.0 and a urine output of $150 mL/h. Because the drug is not lipophilic, it accumulates at high concentrations in ascites and pleural fluid, which can prolong drug elimination and toxicity, especially in the setting of AKI. Consequently, it is recommended that fluid collections be drained prior to high-dose MTX. When AKI occurs, it is reversible in 70%–100% of cases with conservative medical management (38), and the drug can be Onco-Nephrology Curriculum 3 readministered following recovery of renal function. In cases where dialysis requiring AKI occurs and is associated with prolonged myelosuppression and/or gastrointestinal (GI) ulceration, carboxypeptidase-G(2) can be obtained on a compassionate, albeit expensive basis, for management of severe MTX intoxication (41–43). Carboxypeptidase-G(2) rapidly hydrolyzes MTX into its inactive metabolites and decreases median MTX concentrations by 98.7%. Kintzel recommends for CrCl of 46–60 mL/min, give 65% of dose; for CrCl of 31–45 mL/min, give 50% of dose, and for CrCl ,30 mL/min, do not administer. Aronoff recommends a dose reduction of 50% with a CrCl of .10–50 mL/min and avoiding the drug for CrCl of ,10 mL/min (4,5). Although the drug is removed by high-flux HD and charcoal hemoperfusion, because it is 50% protein bound, there is postdialysis rebound in MTX concentrations of 90%–100% of the preprocedure levels. Therefore, patients may require daily or continuous renal replacement therapy to avoid rebound toxicity. PEMETREXED Pemetrexed is a derivative of MTX, and up to 90% of the drug is excreted unchanged in the urine. The drug has been associated with acute tubular necrosis and interstitial fibrosis (44,45). The manufacturer recommends avoiding the drug for a CrCl of ,45 mL/min and avoiding nonsteroidal anti-inflammatory medications in the days before and after pemetrexed dosing in patients with a CrCl of 45–79 mL/min (46). When nephrotoxicity occurs, it is probably best to avoid re-exposure. MELPHALAN Melphalan is effective therapy for multiple myeloma (MM) and amyloidosis. Urinary excretion of melphalan ranges from 10% to 34%, and the AUC for melphalan is inversely correlated with the GFR (47). Bone marrow suppression, the limiting side effect of melphalan, increases when the drug is given in CKD patients (48). Kintzel recommends for CrCl of 46–60 mL/min, give 85% of dose; for CrCl of 31–45 mL/min, give 75% of dose; and for CrCl of ,30 mL/min, give 70% of dose. Aronoff recommends giving 75% of the dose for a CrCl of .10–50 mL/min and 50% for those with a CrCl of ,10 mL/min (4,5). Patients on dialysis have been safely and successfully treated with melphalan prior to stem cell transplant at doses between 60 and 140 mg/m2 (49,50). CKD patients (51,52). Rare cases of AKI due to acute and chronic interstitial nephritis (AIN) have been reported with lenalidomide (53–55). For MDS, the manufacturer recommends 5 mg daily for CrCl of ,59 mL/min, and increasing the dosing interval to every 48 hours for CrCl of ,30 mL/min. For patients on HD, a dose of 5 mg should be given three times a week following each hemodialysis. For MM, the manufacturer recommends a 10-mg daily dose for CrCl of .30–59 mL/min, a 15-mg dose every 48 hours for CrCl of ,30 mL/min, and 5 mg daily for HD patients to be given after HD on dialysis days (56). CYTARABINE Cytarabine is an antimetabolite that is extensively converted to uridine-arabinoside (Ara-U), and 10%–30% of the parent drug and 85% of the inactive metabolite are eliminated by the kidneys (57). High-dose cytarabine (HDAC) can produce significant and sometimes irreversible neurotoxicity. Renal dysfunction is an independent risk factor for cerebellar (dysarthria, nystagmus, gait ataxia, dysdiadochokinesia) and noncerebellar (somnolence, seizures) neurotoxicity. The inactive metabolite Ara-U inhibits cytidine deaminase activity, and in the setting of renal dysfunction, further delays cytarabine clearance and increases serum and cerebral spinal fluid levels of the parent drug (58). For patients with CKD receiving HDAC, Kintzel recommends for CrCl of 46–60 mL/min, give 60% of the dose; for CrCl of 31–45 mL/min, give 50% of the dose; and for CrCl of ,30 mL/min, do not administer (4). Smith developed a dosing algorithm for HDAC based on daily serum creatinine measurements while the patients were on therapy. If the patient’s baseline serum creatinine concentration was between 1.5 and 1.9 mg/day, or if baseline serum creatinine level increased by 0.5–1.2 mg/dL during treatment, then the dose of Ara-C was reduced to 1 g/m2 per dose. If the serum creatinine level was .2 or the change was .1.2 mg/dL, then the dose of Ara-C was reduced to 0.1 g/m2/day (standard dose) as a continuous infusion. (59). Cytarabine and Ara-U are both cleared by HD. There are a few case reports of patients on HD who have tolerated treatment with standard doses cytarabine (continuous infusion, 100 mg/m2 per day) when HD was performed on day 1 of cytarabine infusion and then every other day. In one case, HD was performed consecutively on days 1 and 2 as well (60,61). HDAC has been safely used in a patient with lymphoma on hemodialysis. The patient received two doses of HDAC 1 g/m2, 24 hours apart, was dialyzed 6 hours after each dose, and then resumed his usual dialysis schedule (62). LENALIDOMIDE Lenalidomide is a thalidomide analogue used to treat multiple myeloma (MM) and myelodysplastic syndrome (MDS). Eighty-two percent of the drug is excreted unchanged in the urine, and the AUC and risk for drug toxicity are increased in 4 Onco-Nephrology Curriculum CAPECITABINE Capecitabine is a pyrimidine analogue that is preferentially converted to 5-fluorouracil (5-FU) within tumor cells. American Society of Nephrology Although neither capecitabine nor 5-FU is renally cleared, in patients with CKD, there is retention of active metabolites and a resultant increase in systemic toxicity (63). The manufacturer recommends a dose reduction of 75% from the starting dose of 1,250 mg/m2 for patients with a CrCl between 30 and 50 mL/min, and for patients with a CrCl of ,30 mL/min, it is recommended that capecitabine be discontinued (64). With careful monitoring and dose reduction, the drug has been effectively and safely used in patients with advanced CKD (GFR , 30 mL/min) and patients on hemodialysis (65). BLEOMYCIN Bleomycin is not nephrotoxic, but urinary excretion accounts for close to 70% of the intravenous dose of bleomycin (66,67), and patients with renal dysfunction appeared to be at higher risk for bleomycin pulmonary toxicity, the major dose-limiting toxicity of this drug (48,68). Kintzel recommends for CrCl of 46–60 mL/min, give 70% of dose; for CrCl of 31–45 mL/min, give 60% of dose; and for CrCl of ,30 mL/min, avoid the drug. Aronoff recommends giving 75% of the dose for CrCl of .10– 50 mL/min and 50% for those with a CrCl of ,10 mL/min (4,5). Because pulmonary toxicity can be cumulative in patients with CKD, if bleomycin is administered to patients with CKD, repeat pulmonary function tests prior to each drug administration may be prudent. MOLECULARLY TARGETED AGENTS Vascular endothelial growth factor (VEGF) pathway inhibitors (bevacizumab), tyrosine kinase inhibitors (sorafenib, nilotinib, and dasatinib), and epithelial growth factor receptor (EGFR) pathway inhibitors (erlotinib) have been described to cause nephrotoxicity. Observed renal toxicities among these relatively new agents include acute tubular necrosis, proteinuria, hypertension, thrombotic microangiopathy, acute interstitial nephritis, tumor lysis syndrome, and glomerulonephritis (69–71). Bevacizumab As per the manufacturer’s guidelines, no studies were done to investigate the PK of bevacizumab in patients with CKD. For patients with a CrCl between 20 and 39 mL/min, the manufacturer recommends a 50% decrease in the usual starting dose with increases in subsequent doses as tolerated, but no .400 mg. For patients with a CrCl of 40–59 mL/min, doses .600 mg are not recommended (72). There is only one report on the PK of bevacizumab in a dialysis-dependent patient with metastatic renal cancer who received 5 mg/kg every 2 weeks. The drug was not dialyzable, and its pharmacokinetic parameters were similar to the reference values of patients with normal renal function. The drug can be administered any time before or after hemodialysis (73). American Society of Nephrology Tyrosine kinase inhibitors Erlotinib The PK of erlotinib were never studied in patients with renal insufficiency. There are case reports of patients with CrCl between 25 and 41 mL/min who tolerated the usual dose of 150 mg/day without significant toxicity (74). There are no data on its use in hemodialysis patients. Imatinib Although there is no significant renal excretion, the manufacturer recommends that patients with a CrCl of 20–30 mL/min receive 50% of the starting dose, with dose escalation as tolerated but not to exceed 400 mg/day. For those with CrCl of 40–59 mL/min, doses .600 mg are not recommended. Imatinib exposure can increase up to two-fold in patients even with mild renal impairment (CrCl , 60 mL/min) and that there is a significant correlation between decreased renal function and the incidence of serious adverse events (75). PK data on one patient with ESRD on dialysis that received 400 mg/day indicates that imatinib and its metabolite are unchanged in patients with ESRD on hemodialysis (74). Lenvatinib The package insert recommends a dose reduction to 14 mg daily for patients with CrCl is ,30 mL/min. There are no data on the use of lenvatinib in patients on hemodialysis. Sunitinib Sunitinib use was studied in two ESRD patients. The PK parameters of sunitinib and its major metabolite were similar in patients on HD and those with normal renal function. Furthermore, sunitinib is nondialyzable. Doses of 50 mg/day for 4 weeks every 6 weeks were well tolerated (76). Sorafenib Although the manufacturer does not recommend any dose adjustment for patients with any level of renal insufficiency (77), based on dose-limiting toxicity in a phase 1 study, a starting dose of 200 mg twice a day for CrCl of 20–39 mL/min and 200 mg once daily for patients on hemodialysis was recommended. No recommendations could be made for those with a CrCl ,20 mL/min and not on dialysis (78). Vandetanib The manufacturer recommends that the starting dose should be reduced to 200 mg in patients with a CrCl of ,50 mL/min (79). There are no data on use of this drug in hemodialysis patients. BISPHOSPHONATES Bisphosphonates are frequently administered to cancer patients for management of hypercalcemia of malignancy and osteolytic bone lesions. Zoledronic acid has been associated with acute tubular necrosis, especially after repeated dosing. Onco-Nephrology Curriculum 5 Table 2. Recommended dose adjustments for selected chemotherapies for patients with CKD and ESRD on HD Although pamidronate is more notoriously associated with collapsing glomerulopathy, there are case reports of acute tubular necrosis with this agent as well (80). Dehydration, concomitant use of nephrotoxic medication, and overly frequent dosing of the bisphosphonates all increase susceptibility to deterioration of renal function following bisphosphonate exposure. The American Society of Clinical Oncology (ASCO) recommends that his zoledronate be avoided in patients with CrCl of ,30 mL/min and that the initial dose of 4 mg be reduced to 3.5 mg for CrCl of 50–60 mL/min; 3.3 mg for CrCl of 40–49 mL/min; and 3 mg for CrCl of 30–39 mL/ min. For pamidronate, the usual dose of 90 mg over 2–3 hours and ASCO recommends giving 90 mg over 4–6 hours for CrCl of ,59 mL/min. Neither agent should be used more frequently than every 3–4 weeks, the serum creatinine should be checked prior to each administration, and the medication should be held if the creatinine increases .0.5 mg/dL with normal renal function or .1 mg/dL if there is abnormal renal function at baseline (81). Table 2 includes commonly used chemotherapeutic drugs and their dose adjustment in the setting of CKD. CONCLUSIONS In summary, although the kidneys are a major pathway for drug metabolism, unfortunately, the quality of PK and PD data on 6 Onco-Nephrology Curriculum commonly prescribed chemotherapeutic agents for CKD patients requiring chemotherapy is quite poor. This chapter was an effort to summarize the data that are available and to provide the treating physician with some guidance when treating patients with cancer and CKD. TAKE HOME POINTS c The liver and the kidneys serve the major pathways of drug metabolism and elimination with much smaller contributions from the fecal and reticuloendothelial systems. c Published guidelines for dose modification of chemotherapy for cancer patients, with the exception of carboplatin, are largely based on limited pharmacokinetic and pharmacodynamic data. c In several cases, the parent drug and its metabolites are responsible for systemic toxicity, and the presence of renal insufficiency can potentiate toxicity of the parent drug and its metabolites. REFERENCES 1. Perazella MA. Renal vulnerability to drug toxicity. Clin J Am Soc Nephrol 4: 1275–1283, 2009 2. Aleksa K, Matsell D, Krausz K, Gelboin H, Ito S, Koren G. Cytochrome P450 3A and 2B6 in the developing kidney: implications for ifosfamide nephrotoxicity. Pediatr Nephrol 20: 872–885, 2005 American Society of Nephrology 3. Launay-Vacher V, Oudard S, Janus N, Gligorov J, Pourrat X, Rixe O, Morere JF, Beuzeboc P, Deray G; Renal Insufficiency and Cancer Medications (IRMA) Study Group. Prevalence of Renal Insufficiency in cancer patients and implications for anticancer drug management: the renal insufficiency and anticancer medications (IRMA) study. Cancer 110: 1376–1384, 2007 4. Kintzel PE, Dorr RT. Anticancer drug renal toxicity and elimination: dosing guidelines for altered renal function. Cancer Treat Rev 21: 33– 64, 1995 5. Aronoff GR. Drug Prescribing in Renal Failure, Philadelphia, American College of Physicians, 2007 6. Lichtman SM, Wildiers H, Launay-Vacher V, Steer C, Chatelut E, Aapro M. International Society of Geriatric Oncology (SIOG) recommendations for the adjustment of dosing in elderly cancer patients with renal insufficiency. Eur J Cancer 43: 14–34, 2007 7. Matzke GR, Aronoff GR, Atkinson AJ Jr, Bennett WM, Decker BS, Eckardt KU, Golper T, Grabe DW, Kasiske B, Keller F, Kielstein JT, Mehta R, Mueller BA, Pasko DA, Schaefer F, Sica DA, Inker LA, Umans JG, Murray P. Drug dosing consideration in patients with acute and chronic kidney disease: A clinical update from Kidney Disease: Improving Global Outcomes (KDIGO). Kidney Int 80: 1122–1137, 2011 8. Dreisbach AW, Lertora JJ. The effect of chronic renal failure on drug metabolism and transport. Expert Opin Drug Metab Toxicol 4: 1065– 1074, 2008 9. Dooley MJ, Poole SG, Rischin D. Dosing of cytotoxic chemotherapy: impact of renal function estimates on dose. Ann Oncol 24: 2746–2752, 2013 10. Chatelut E, Canal P, Brunner V, Chevreau C, Pujol A, Boneu A, Roché H, Houin G, Bugat R. Prediction of carboplatin clearance from standard morphological and biological patient characteristics. J Natl Cancer Inst 87: 573–580, 1995 11. Calvert AH, Newell DR, Gumbrell LA, O’Reilly S, Burnell M, Boxall FE, Siddik ZH, Judson IR, Gore ME, Wiltshaw E: Carboplatin dosage: Prospective evaluation of a simple formula based on renal function. J Clin Oncol 7: 1748–1756, 1989 12. Egorin MJ, Van Echo DA, Tipping SJ, Olman EA, Whitacre MY, Thompson BW, Aisner J. Pharmacokinetics and dosage reduction of cis-diammine(1,1-cyclobutanedicarboxylato)platinum in patients with impaired renal function. Cancer Res 44: 5432–5438, 1984 13. Shea TC, Storniolo AM, Mason JR, Newton B, Mullen M, Taetle R, Green MR. A dose-escalation study of carboplatin/cyclophosphamide/ etoposide along with autologous bone marrow or peripheral blood stem cell rescue. Semin Oncol 19[Suppl 2]: 139–144, 1992 14. Beyer J, Rick O, Weinknecht S, Kingreen D, Lenz K, Siegert W. Nephrotoxicity after high-dose carboplatin, etoposide and ifosfamide in germ-cell tumors: Incidence and implications for hematologic recovery and clinical outcome. Bone Marrow Transplant 20: 813–819, 1997 15. Guddati AK, Joy PS, Marak CP. Dose adjustment of carboplatin in patients on hemodialysis. Med Oncol 31: 848, 2014 16. Janus N, Thariat J, Boulanger H, Deray G, Launay-Vacher V. Proposal for dosage adjustment and timing of chemotherapy in hemodialyzed patients. Ann Oncol 21: 1395–1403, 2010 17. Sastry J, Kellie SJ. Severe neurotoxicity, ototoxicity and nephrotoxicity following high-dose cisplatin and amifostine. Pediatr Hematol Oncol 22: 441–445, 2005 18. Arany I, Safirstein RL. Cisplatin nephrotoxicity. Semin Nephrol 23: 460– 464, 2003 19. Boulikas T, Vougiouka M:. Cisplatin and platinum drugs at the molecular level. (Review). Oncol Rep 10: 1663–1682, 2003 20. Dentino M, Luft FC, Yum MN, Williams SD, Einhorn LH. Long term effect of cis-diamminedichloride platinum (CDDP) on renal function and structure in man. Cancer 41: 1274–1281, 1978 21. Meijer S, Mulder NH, Sleijfer DT, Donker AJ, Sluiter WJ, de Jong PE, Schraffordt Koops H, van der Hem GK. Influence of combination chemotherapy with cis-diamminedichloroplatinum on renal function: Long-term effects. Oncology 40: 170–173, 1983 American Society of Nephrology 22. Hamilton CR, Bliss JM, Horwich A. The late effects of cis-platinum on renal function. Eur J Cancer Clin Oncol 25: 185–189, 1989 23. Hansen SW, Groth S, Daugaard G, Rossing N, Rørth M. Long-term effects on renal function and blood pressure of treatment with cisplatin, vinblastine, and bleomycin in patients with germ cell cancer. J Clin Oncol 6: 1728–1731, 1988 24. Fjeldborg P, Sørensen J, Helkjaer PE. The long-term effect of cisplatin on renal function. Cancer 58: 2214–2217, 1986 25. Madias NE, Harrington JT. Platinum nephrotoxicity. Am J Med 65: 307– 314, 1978 26. Erlichman C, Soldin SJ, Thiessen JJ, Sturgeon JF, Fine S. Disposition of total and free cisplatin on two consecutive treatment cycles in patients with ovarian cancer. Cancer Chemother Pharmacol 19: 75–79, 1987 27. Reece PA, Stafford I, Russell J, Gill PG. Reduced ability to clear ultrafilterable platinum with repeated courses of cisplatin. J Clin Oncol 4: 1392–1398, 1986 28. Morland BJ, Mann JR, Milford DV, Raafat F, Stevens MC. Ifosfamide nephrotoxicity in children: Histopathological features in two cases. Med Pediatr Oncol 27: 57–61, 1996 29. Zaki EL, Springate JE, Taub M. Comparative toxicity of ifosfamide metabolites and protective effect of mesna and amifostine in cultured renal tubule cells.Toxicol In Vitro 17: 397–402, 2003 30. Furlanut M, Franceschi L. Pharmacology of ifosfamide. Oncology 65 [Suppl 2]: 2–6, 2003 31. Allen LM, Creaven PJ. Pharmacokinetics of ifosfamide. Clin Pharmacol Ther 17: 492–498, 1975 32. Meanwell CA, Blake AE, Kelly KA, Honigsberger L, Blackledge G. Prediction of ifosfamide/mesna associated encephalopathy. Eur J Cancer Clin Oncol 22: 815–819, 1986 33. Latcha S, Maki RG, Schwartz GK, Flombaum CD. Ifosfamide may be safely used in patients with end stage renal disease on hemodialysis. Sarcoma 2009: 575629, 2009 34. Carlson L, Goren MP, Bush DA, Griener JC, Quigley R, Tkaczewski I, Kamen BA, Weitman SD. Toxicity, pharmacokinetics, and in vitro hemodialysis clearance of ifosfamide and metabolites in an anephric pediatric patient with Wilms’ tumor. Cancer Chemother Pharmacol 41: 140–146, 1998 35. de Jonge ME, Huitema AD, Rodenhuis S, Beijnen JH. Clinical pharmacokinetics of cyclophosphamide. Clin Pharmacokinet 44: 1135– 1164, 2005 36. Haubitz M, Bohnenstengel F, Brunkhorst R, Schwab M, Hofmann U, Busse D. Cyclophosphamide pharmacokinetics and dose requirements in patients with renal insufficiency. Kidney Int 61: 1495–1501, 2002 37. Juma FD, Rogers HJ, Trounce JR. Effect of renal insufficiency on the pharmacokinetics of cyclophosphamide and some of its metabolites. Eur J Clin Pharmacol 19: 443–451, 1981 38. Flombaum CD, Meyers PA. High-dose leucovorin as sole therapy for methotrexate toxicity. J Clin Oncol 17: 1589–1594, 1999 39. Howell SB, Carmody J. Changes in glomerular filtration rate associated with high-dose methotrexate therapy in adults. Cancer Treat Rep 61: 1389–1391, 1977 40. Crom WR, Pratt CB, Green AA, Champion JE, Crom DB, Stewart CF, Evans WE. The effect of prior cisplatin therapy on the pharmacokinetics of high-dose methotrexate. J Clin Oncol 2: 655–661, 1984 41. Wall SM, Johansen MJ, Molony DA, DuBose TD Jr, Jaffe N, Madden T. Effective clearance of methotrexate using high-flux hemodialysis membranes. Am J Kidney Dis 28: 846–854, 1996 42. Relling MV, Stapleton FB, Ochs J, Jones DP, Meyer W, Wainer IW, Crom WR, McKay CP, Evans WE. Removal of methotrexate, leucovorin, and their metabolites by combined hemodialysis and hemoperfusion. Cancer 62: 884–888, 1988 43. Widemann BC, Adamson PC. Understanding and managing methotrexate nephrotoxicity. Oncologist 11: 694–703, 2006 44. Chauvet S, Courbebaisse M, Ronco P, Plaisier E. Pemetrexed-induced acute kidney injury leading to chronic kidney disease. Clin Nephrol 82: 402–406, 2014 Onco-Nephrology Curriculum 7 45. Glezerman IG, Pietanza MC, Miller V, Seshan SV. Kidney tubular toxicity of maintenance pemetrexed therapy. Am J Kidney Dis 58: 817–820, 2011 46. Eli Lilly Canada Inc: Product Monograph Altima Pemetrexed Disodium for Injection, 2013. Available at: http://www.lilly.ca/en/pdf/productmonograph/14_alimta-pm-10may2013.pdf. Accessed January 28, 2016 47. Alberts DS, Chang SY, Chen HS, Moon TE, Evans TL, Furner RL, Himmelstein K, Gross JF. Kinetics of intravenous melphalan. Clin Pharmacol Ther 26: 73–80, 1979 48. Cornwell GG 3rd, Pajak TF, McIntyre OR, Kochwa S, Dosik H. Influence of renal failure on myelosuppressive effects of melphalan: Cancer and Leukemia Group B experience. Cancer Treat Rep 66: 475–481, 1982 49. Badros A, Barlogie B, Siegel E, Roberts J, Langmaid C, Zangari M, Desikan R, Shaver MJ, Fassas A, McConnell S, Muwalla F, Barri Y, Anaissie E, Munshi N, Tricot G. Results of autologous stem cell transplant in multiple myeloma patients with renal failure. Br J Haematol 114: 822–829, 2001 50. Hamaki T, Katori H, Kami M, Yamato T, Yamakado H, Itoh T, Kusumi E, Igarashi M, Ueyama J, Kanda Y, Miyakoshi S, Mineishi S, Morinaga S, Mukai M, Hayashi M, Takaue Y, Hara S, Mutou Y. Successful allogeneic blood stem cell transplantation for aplastic anemia in a patient with renal insufficiency requiring dialysis. Bone Marrow Transplant 30: 195– 198, 2002 51. Chen N, Lau H, Kong L, Kumar G, Zeldis JB, Knight R, Laskin OL. Pharmacokinetics of lenalidomide in subjects with various degrees of renal impairment and in subjects on hemodialysis. J Clin Pharmacol 47: 1466–1475, 2007 52. Niesvizky R, Naib T, Christos PJ, Jayabalan D, Furst JR, Jalbrzikowski J, Zafar F, Mark T, Lent R, Pearse RN, Ely S, Leonard JP, Mazumdar M, Chen-Kiang S, Coleman M. Lenalidomide-induced myelosuppression is associated with renal dysfunction: Adverse events evaluation of treatment-naïve patients undergoing front-line lenalidomide and dexamethasone therapy. Br J Haematol 138: 640–643, 2007 53. Shaaban H, Layne T, Guron G. A case of DRESS (drug reaction with eosinophilia and systemic symptoms) with acute interstitial nephritis secondary to lenalidomide. J Oncol Pharm Pract 20: 302–304, 2013 54. Batts ED, Sanchorawala V, Hegerfeldt Y, Lazarus HM. Azotemia associated with use of lenalidomide in plasma cell dyscrasias. Leuk Lymphoma 49: 1108–1115, 2008 55. Lipson EJ, Huff CA, Holanda DG, McDevitt MA, Fine DM. Lenalidomideinduced acute interstitial nephritis. Oncologist 15: 961–964, 2010 56. Available at: http://www.celgenecanada.net/pdfs/Revlimid%20 Product_Monograph_-_English_Version.pdf 57. Powis G. Effect of human renal and hepatic disease on the pharmacokinetics of anticancer drugs. Cancer Treat Rev 9: 85–124, 1982 58. Damon LE, Mass R, Linker CA. The association between high-dose cytarabine neurotoxicity and renal insufficiency. J Clin Oncol 7: 1563– 1568, 1989 59. Smith GA, Damon LE, Rugo HS, Ries CA, Linker CA. High-dose cytarabine dose modification reduces the incidence of neurotoxicity in patients with renal insufficiency. J Clin Oncol 15: 833–839, 1997 60. Pöschl JM, Klaus G, Querfeld U, Ludwig R, Mehls O. Chemotherapy with cytosine arabinoside in a child with Burkitt’s lymphoma on maintenance hemodialysis and hemofiltration. Ann Hematol 67: 37–39, 1993 61. Tsuchiya Y, Ubara Y, Suwabe T, Hoshino J, Sumida K, Hiramatsu R, Hasegawa E, Yamanouchi M, Hayami N, Marui Y, Sawa N, Takemoto F, Takaichi K. Successful treatment of acute promyelocytic leukemia in a patient on hemodialysis. Clin Exp Nephrol 15: 434–437, 2011 62. Radeski D, Cull GM, Cain M, Hackett LP, Ilett KF. Effective clearance of Ara-U the major metabolite of cytosine arabinoside (Ara-C) by hemodialysis in a patient with lymphoma and end-stage renal failure. Cancer Chemother Pharmacol 67: 765–768, 2011 63. Poole C, Gardiner J, Twelves C, Johnston P, Harper P, Cassidy J, Monkhouse J, Banken L, Weidekamm E, Reigner B. Effect of renal 8 Onco-Nephrology Curriculum 64. 65. 66. 67. 68. 69. 70. 71. 72. 73. 74. 75. 76. 77. 78. 79. 80. 81. impairment on the pharmacokinetics and tolerability of capecitabine (Xeloda) in cancer patients. Cancer Chemother Pharmacol 49: 225– 234, 2002 Hoffman-LaRoche Limited: Product Monograph Xeloda. Available at: http://www.rochecanada.com/fmfiles/re7234008/Research/ ClinicalTrialsForms/Products/ConsumerInformation/Monographsand PublicAdvisories/Xeloda/Xeloda_PM_E.pdf. Accessed January 28, 2016 Jhaveri KD, Flombaum C, Shah M, Latcha S. A retrospective observational study on the use of capecitabine in patients with severe renal impairment (GFR ,30 mL/min) and end stage renal disease on hemodialysis. J Oncol Pharm Pract 18: 140–147, 2012 Alberts DS, Chen HS, Mayersohn M, Perrier D, Moon TE, Gross JF. Bleomycin pharmacokinetics in man. II. Intracavitary administration. Cancer Chemother Pharmacol 2: 127–132, 1979 Crooke ST, Comis RL, Einhorn LH, Strong JE, Broughton A, Prestayko AW. Effects of variations in renal function on the clinical pharmacology of bleomycin administered as an iv bolus. Cancer Treat Rep 61: 1631– 1636, 1977 Dalgleish AG, Woods RL, Levi JA. Bleomycin pulmonary toxicity: Its relationship to renal dysfunction. Med Pediatr Oncol 12: 313–317, 1984 Gafter-Gvili A, Ram R, Gafter U, Shpilberg O, Raanani P. Renal failure associated with tyrosine kinase inhibitors: Case report and review of the literature. Leuk Res 34: 123–127, 2010 Kurita N, Mise N, Fujii A, Ikeda S, Sugimoto T. Crescentic glomerulonephritis in a patient with advanced lung cancer during erlotinib therapy. NDT Plus 2: 512–513, 2009 Usui J, Glezerman IG, Salvatore SP, Chandran CB, Flombaum CD, Seshan SV. Clinicopathological spectrum of kidney diseases in cancer patients treated with vascular endothelial growth factor inhibitors: A report of 5 cases and review of literature. Hum Pathol 45: 1918–1927, 2014 Hoffman-LaRoche Limited: Product Monograph Avastin. Available at: http://rochecanada.com/fmfiles/re7234008/Research/Clinical TrialsForms/Products/ConsumerInformation/MonographsandPublic Advisories/Avastin/Avastin_PM_E.pdf. Accessed January 28, 2016 Garnier-Viougeat N, Rixe O, Paintaud G, Ternant D, Degenne D, Mouawad R, Deray G, Izzedine H. Pharmacokinetics of bevacizumab in haemodialysis. Nephrol Dial Transplant 22: 975, 2007 Pappas P, Karavasilis V, Briasoulis E, Pavlidis N, Marselos M. Pharmacokinetics of imatinib mesylate in end stage renal disease. A case study. Cancer Chemother Pharmacol 56: 358–360, 2005 Novartis Pharmaceuticals Corporation: Highlights of Prescribing Information Gleevac, 2015. Available at: https://www.pharma.us. novartis.com/product/pi/pdf/gleevec_tabs.pdf. Accessed January 28, 2016 Izzedine H, Etienne-Grimaldi MC, Renée N, Vignot S, Milano G. Pharmacokinetics of sunitinib in hemodialysis. Ann Oncol 20: 190–192, 2009 Baryer Inc: Product Monograph Nexavar, 2014. Available at: http:// www.bayer.ca/omr/online/nexavar-pm-en-17dec2014-178086.pdf. Accessed January 28, 2016 Miller AA, Murry DJ, Owzar K, Hollis DR, Kennedy EB, Abou-Alfa G, Desai A, Hwang J, Villalona-Calero MA, Dees EC, Lewis LD, Fakih MG, Edelman MJ, Millard F, Frank RC, Hohl RJ, Ratain MJ. Phase I and pharmacokinetic study of sorafenib in patients with hepatic or renal dysfunction: CALGB 60301. J Clin Oncol 27: 1800–1805, 2009 AstraZeneca Pharmaceuticals Corporation: Highlights of Prescribing Information Caprelsa. Available at: http://www1.astrazeneca-us.com/ pi/vandetanib.pdf. Accessed January 28, 2016 Perazella MA, Markowitz GS. Bisphosphonate nephrotoxicity. Kidney Int 74: 1385–1393, 2008 Kyle RA, Yee GC, Somerfield MR, Flynn PJ, Halabi S, Jagannath S, Orlowski RZ, Roodman DG, Twilde P, Anderson K; American Society of Clinical Oncology. American Society of Clinical Oncology 2007 clinical practice guideline update on the role of bisphosphonates in multiple myeloma. J Clin Oncol 25: 2464–2472, 2007 American Society of Nephrology REVIEW QUESTIONS 1. What is the best option for management of a patient on cisplatin who has lost approximately 50% of the GFR following the first three cycles? a. Continue cisplatin at the present dose b. Continue cisplatin at a reduced dose c. Consider alternative therapies Answer: c is correct. Patients can have episodes of AKI following each dose of cis-platinum. With each subsequent episode of AKI, the baseline serum creatinine may not return to its pretreatment value. Up to 30% of patients may have residual renal insufficiency due to cis-platinum nephrotoxicity. In this patient who has lost about 50% of his GFR following his initial treatment, cisplatin is not recommended, and it may be best to consider alternative therapies. 2. What is the dose of zoledronic acid recommended by American Society of Clinical Oncology for the administration to patients with CrCl of 30–39 mL/min? American Society of Nephrology a. b. c. d. 4 mg 3.5 mg 3 mg Do not administer Answer: c is correct. The American Society of Clinical Oncology recommends that zoledronate should be avoided in patients with CrCl of ,30 mL/min and that the initial dose of 4 mg be reduced to 3.5 mg for CrCl of 50–60 mL/min; 3.3 mg for CrCl of 40–49 mL/min; and 3 mg for CrCl of 30–39 mL/min. 3. What is the appropriate timing of cisplatin administration in an ESRD patient receiving hemodialysis? a. b. c. d. 2 hours prior to hemodialysis 6 hours prior to hemodialysis 12 hours prior to hemodialysis On the off-dialysis day Answer: d is correct. Because the drug is highly and irreversibly protein bound and because free cisplatin is well dialyzed, drug that is removed by dialysis cannot be replaced by bound drug. As such, cisplatin must be given on a nondialysis day. Onco-Nephrology Curriculum 9 Chapter 13: CKD as a Complication of Cancer Laura Cosmai, MD,* Camillo Porta, MD,† and Maurizio Gallieni, MD, FASN‡ *Nephrology and Dialysis, Istituti Ospitalieri Cremona, Cremona, Italy; †Medical Oncology, I.R.C.C.S., San Matteo University Hospital Foundation, Pavia, Italy; and ‡Nephrology and Dialysis, San Carlo Borromeo Hospital, University of Milano, Milano, Italy CKD AND CANCER: A “CIRCULAR” RELATIONSHIP CKD is recognized as a disease that may complicate cancer and its therapy. This is in part related to the fact that preexisting CKD is highly prevalent in oncologic patients. Indeed, as observed in the Renal Insufficiency and Anticancer Medications (IRMA)-1 and -2 studies (1,2), more than half of patients with an active malignancy present with an eGFR of ,90 mL/min per 1.73 m2. Furthermore, the prevalence of more severe CKD (i.e., stages 3–5), not requiring dialysis, was 12.0% and 11.8%, respectively (1,2). Similar results have been reported in other series from different countries, thereby confirming that CKD is a relatively common occurrence in cancer patients, irrespective of the type of malignancy. As a whole, causes potentially able to have a negative impact on kidney function are summarized in Table 1. Interestingly, the relationship between the kidney and cancer appears to be bidirectional (3). For example, preexisting CKD may impact the bioavailability and/or safety profile of an anticancer drug, potentially leading to different and sometimes suboptimal treatment choices. On the other hand, it is also possible that the renal effects of a novel anticancer drug may lead to progressive kidney injury or to worsening of preexisting CKD (3). In addition to the observed increase in CKD prevalence in cancer patients, both CKD and ESRD are risk factors for a number of malignancies (4). However, not all solid tumors appear to be equally represented in this population. A retrospective cohort study of 1,190,538 adults assessed the association between eGFR level and the risk of incident cancer (5). During 6,000,420 personyears of follow-up, 76,809 incident cancers were identified in 72,875 subjects. After adjustment for time-updated confounders, lower eGFR was associated with an increased risk of renal cancer, with an adjusted hazard ratio (HR) of 2.28 (95% CI, 1.78– 2.92) for an eGFR ,30 mL/min per 1.73 m2 (5). American Society of Nephrology The authors also observed an increased risk of urothelial cancer at an eGFR ,30 mL/min per 1.73 m2 but no significant associations between eGFR and other cancers. Finally, CKD conferred an increased cancer-specific mortality in patients with kidney and urinary tract cancer (6). In ESRD patients on dialysis, the observed increased risk for renal parenchymal cancer is related to the development of acquired renal cystic disease, which increases with time on dialysis (7). CKD and antineoplastic drugs Acute and chronic kidney injury associated with antineoplastic drug exposure is well described for the classic cytotoxic agents that are used. In addition, there is a large body of literature that describes dosing of these drugs in patients with underlying renal dysfunction and those on dialysis; however, little is known about the appropriate use of the new targeted agents in this population. This creates a complicated issue for oncologists and nephrologists who care for these patients and must provide both safe and effective anticancer therapy. After decades of use of common cytotoxic drugs, clinicians versed in cancer care and its complications are well aware of the main toxicities of these agents. The new, molecularly targeted, anticancer drugs that are entering clinical practice have a wide array of previously unrecognized and ill-defined adverse effects (8). Ultimately, these toxicities must be readily recognized and managed by those providing care for patients exposed to these drugs. This Correspondence: Laura Cosmai, Division of Nephrology and Dialysis, Istituti Ospitalieri di Cremona, Largo Priori, 1, 26100, Cremona, Italy. Email: [email protected] All authors are members of the Joint Italian Association of Medical Oncology (AIOM)/Italian Society of Nephrology (SIN) Working Group on Onco-Nephrology. L.C. and C.P. contributed equally to the preparation of this chapter. Copyright © 2016 by the American Society of Nephrology Onco-Nephrology Curriculum 1 Table 1. Causes of kidney disease in cancer patients Cause Antineoplastic drugs Traditional chemotherapeutic agents Novel targeted therapies Mechanism(s) c Direct nephrotoxicity (e.g., cisplatin) c Hypertension and/or proteinuria (e.g., VEGF[Rs]-targeted agents) c TMA (e.g., VEGF-targeted agents) c Interstitial nephritis and other glomerulonephritis c Autoimmune nephropathies (e.g., anti-CTLA4 and anti-PD1/PDL1 antibodies) c Indirect toxicities (e.g., nausea/vomiting, diarrhea, dysgeusia) leading to dehydration/volume depletion Other drugs used in cancer patients Anti-pain drugs Bisphosphonates Radiation therapy Contrast medium Paraneoplastic renal syndromes Nephrectomy For cancer For other causes c Direct nephrotoxicity (e.g., NSAIDs, bisphosphonates) Obstruction/compression c Mechanical injury Tumor Infiltration c Kidney infiltration Comorbid risk factors c Hypertension c Preexisting CKD c Still ill defined c Direct nephrotoxicity c Autoimmune mechanism? c Loss of nephrons c AKI c Diabetes mellitus c AKI c Previous use of nephrotoxic cancer therapies VEGF(Rs), vascular endothelial growth factor (receptors); TMA, thrombotic microangiopathies; CTLA4, cytotoxic T-lymphocyte antigen 4; PD1, programmed cell death 1; PDL1, programmed cell death ligand 1; NSAIDs, nonsteroidal anti-inflammatory drugs. includes understanding risk factors for targeted drug-induced kidney injury, appropriate drug dosing (if known) for the patient with AKI, CKD, and those on dialysis, the clinical manifestations of drug nephrotoxicity, and the optimal management of nephrotoxic complications (8). Frequently, oncologists ask their nephrology colleagues to assess the degree of kidney function impairment to provide insight intodosageadjustmentofanticancer therapy.Toaccomplish this, a thorough knowledge of the specific metabolism of anticancer agents and of their pharmacokinetic and pharmacodynamic properties is mandatory. The thought process includes deciding “if” the drug should be administered, “when” it is appropriate to dose the drugs, and to “what extent” dosage adjustment should be used in the setting of underlying kidney disease (3). This approach must be carefully done, as unnecessary treatment interruptions and drug dose reductions may be associated with suboptimal cancer therapy and hamper the clinical benefits of cancer therapy. Optimal management of underlying CKD and its complications, which may be significantly ameliorated in many cases, as well as prevention of further kidney damage from other exogenous nephrotoxins (e.g., contrast medium, nonsteroidal anti-inflammatory drugs, bisphosphonates) in cancer patients with preexisting CKD, is also key to minimizing drug-related complications. Unfortunately, patients with CDK and those on dialysis are often undertreated for their neoplastic disease due to the fear of drug-induced adverse effects. 2 Onco-Nephrology Curriculum Although the relationship between kidney function and cytotoxic agents will be covered in other chapters of the curriculum, the pharmacokinetic properties and specific renal toxicities of novel anticancer agents are summarized in Table 2. The pharmacokinetics of these drugs are similar as most are 90%–98% bound to plasma proteins, and their excretion occurs predominantly via the feces (or the reticulo-endothelial system), whereas urinary excretion is quite variable from one drug to the other (8). As stated in the drugs’ Summary of Product Characteristics (SmPC), the pharmacokinetic properties of the majority of these drugs are not influenced by kidney function (3). A population pharmacokinetic model, which includes data from subjects with baseline creatinine clearance ranging from 30 to 150 mL/min, indicated that it is unlikely that renal insufficiency has a clinically relevant effect on the pharmacokinetics of targeted therapies (9). Thus, no dosage adjustment is recommended in patients with a creatinine clearance .30 mL/min. To date, only patients with adequate kidney function (serum creatinine #1.5 times the upper limit of normal) have been included in registrative randomized controlled trials. In patients with a creatinine clearance ,30 mL/min, a patient population that is poorly studied, caution is recommended (10). Interestingly, this conservative recommendation is not based on data, as drug exposure in patients with severe renal impairment was similar to that observed in patients with normal kidney function (10). American Society of Nephrology Table 2. Renal toxicities of anticancer targeted agents (modified from reference 3) Drug Patients with renal function impairment Renal included in pivotal excretion trial VEGF/VEGFR-tageting agents Bevacizumab Aflibercept Sunitinib Pazopanib Axitinib Other multikinase inhibitors Sorafenib Dose reduction required? Most-frequent renal AEs Patients with mild to moderate CKD Patients with severe CKD Patients receiving dialysis No No No No No No No 16% ,4% 23% Hypertension, proteinuria Hypertension, proteinuria Hypertension, proteinuria Hypertension, proteinuria Hypertension, (proteinuria) No No No No No No (no data) No (no data) No (no data) No (no data) No (no data) No No No No No No 19% No No (no data) No Regorafenib No 19% No No (no data) No data Vandetanib Imatinib mTOR inhibitors Everolimus No No 25% 13% Hypertension, proteinuria, hypophosphatemia Hypertension, proteinuria, electrolyte disorders Hypertension, proteinuria, AKI More renoprotective effects No No Yes (few data) No (no data) No No No 2% No No 4.6% No (no data); suspend if AKI No (no data); suspend if AKI No Temsirolimus Proteinuria, AKI, electrolyte disorders As for everolimus, but less frequent EGFR inhibitors Gefitinib Erlotinib Afatinib Cetuximab No No No No ,4% ,9% ,5% No No No No No No (no data) No (no data) No (no data) No (no data) No No No (no data) No Panitumumab No No Electrolyte disorders Electrolyte disorders Electrolyte disorders Hypomagnesemia, other electrolyte disorders Hypomagnesemia, other electrolyte disorders No No (no data) No (no data) No 1% AKI (tubular necrosis?) No No (no data) Dabrafenib No 23% No No (no data) Possible (risk of arrhythmia) No (no data) Trametinib No No No (no data) No (no data) No No (no data) No No No No No (no data) No (no data) No (no data) No (no data) No No (no data) No No (no data) No (no data) B-Raf inhibitors and MEK inhibitors Vemurafenib ERBB2-targeting agents Trastuzumab Hypophosphatemia, (granulomatous nephritis?) ,20% Hypertension, hyponatremia (with dabrafenib) No No Hypertension, AKI (with cisplatin) No issues No issues Hypokalemia Pertuzumab Lapatinib Trastuzumab emtansine Antibodies against CTLA4 Ipilimumab No No No No 2 ,5% No No Autoimmune nephritis, (drug reaction with eosinophilia and systemic symptom syndrome?) Other agents Crizotinib No No No No Reduction of eGFR (tubular necrosis?), renal cysts No issues Catumaxomab No Possible, with caution No No Possible, with No (no data) caution (no data) No (no data) No (no data) AE, adverse event. American Society of Nephrology Onco-Nephrology Curriculum 3 CKD postnephrectomy for kidney cancer Surgical resection remains the gold standard treatment for localized renal cell carcinoma (RCC) and is also commonly performed for synchronous metastatic disease. The type of surgical resection utilized to treat RCC for the last several decades has been radical nephrectomy, although, more recently, increasing emphasis has been placed on the concept of nephron-sparing procedures (11). Although radical nephrectomy and nephron-sparing surgery do not appear to differ in terms of oncologic outcome, the two different strategies differ in terms of incidence of postoperative CKD and of cardiovascular complications (12). Less invasive surgical procedures for RCC are associated with improved outcomes with less postoperative AKI and CKD and less cardiovascular complications (12). This is not trivial considering that 22% of patients with renal tumors had a pre-nephrectomy stage 3 or greater CKD (eGFR , 60 mL/min per 1.73 m2) (13). In patients 70 years of age and older, the percentage approaches 40% (13). Overall, among 662 patients scheduled for partial or radical nephrectomy, the prevalence of stage 3 or greater CKD was 26% (14). Furthermore, patients that had postoperative AKI following radical nephrectomy had a 4.24-fold higher risk of developing new-onset CKD (15). Progression of underlying CKD was also noted in patients undergoing a radical nephrectomy procedure for an RCC (15). Accordingly, both the American Urological Association (16) and the European Association of Urology (17) endorsed partial nephrectomy as the novel standard of care for organ-confined tumors #4 cm (T1a) and suggested that it be considered as a viable option for patients with tumors .4 but #7 cm (T1b). Evidence for a relationship between the extent of kidney tissue removal and the risk of CKD comes from single-center retrospective studies, population-based studies, and a single randomized, controlled, phase 3 study (12). Among the population-based studies available between 1990 and 2011, a meta-analysis of 36 studies, of which only one was prospective, examined 31,729 patients treated with radical nephrectomy and 9,281 patients managed conservatively (18). The results demonstrated that partial nephrectomy was associated with a 19% reduction in the risk of all-cause mortality (HR, 0.81; P ,0.0001), a 29% reduction in cancer-specific mortality (HR, 0.71; P ,0.001), and a 61% reduction in the risk of severe CKD (HR, 0.39; P ,0.0001), supporting the findings observed in a number of smaller studies (12). In contrast to these findings, however, the results of a highly controversial randomized controlled trial, conducted by the European Organization for the Research and Treatment of Cancer (EORTC) (19), complicate the surgical approach to renal cell cancer. In this trial, a more favorable outcome was observed in patients treated with radical nephrectomy compared with those treated conservatively. During a median follow-up of .9 years, death occurred in 25% of patients treated by partial nephrectomy and in 18.3% of those undergoing radical nephrectomy. Cardiovascular diseases were noted to be the most common cause of death. The intention-to-treat analysis showed a 10-year overall survival rate of 81.1% in the radical nephrectomy group compared with 75.5% in the partial nephrectomy group (HR, 1.5; 95% CI, 1.03–2.16). Interestingly, partial nephrectomy was 4 Onco-Nephrology Curriculum associated with a 21% reduction in the absolute risk of developing moderate CKD (eGFR , 60 mL/min per 1.73 m2) over a median follow-up of 6.7 years, whereas the difference in the incidence of severe CKD (eGFR ,30 mL/min per 1.73 m2) between the two groups was 3.7% (19). These results have generated various hypotheses about the effect of medical- versus surgical-associated CKD on patient survival. It is speculated that medical disease–related CKD has worse outcomes than CKD due to surgery (nephron loss), likely related to other comorbidities and the primary renal disease (i.e., diabetes mellitus). Indeed, a recent report suggested that patients with medical risk factors for CKD are at increased risk of progressive renal impairment, irrespective of the use of partial nephrectomy (20). The duration of renal ischemia during partial nephrectomy may also play a key role in the development of postoperative CKD. In a recent collaborative review (21), a strong association was noted between the quality and quantity of renal tissue that is preserved after surgery and long-term kidney function, and also the duration of ischemia proved to be an important modifiable predictor of postoperative kidney function. Prolonged warm ischemia time (WIT) proved to be significantly associated with adverse postoperative kidney function (21). Available data suggest a renal benefit of keeping WIT ,25 minutes (21). Conversely, cold ischemia appears to safely allow longer durations of ischemia (21). Finally, patients with CKD (irrespective of its cause) within the setting of a metastatic cancer usually tolerate cancer-targeted agents poorly, experiencing higher-grade adverse events compared with patients with normal kidney function (3). Tumor-induced CKD End-stage cancer is often associated with malignant ureteral obstruction (MUO), leading to obstructive nephropathy and CKD. Direct ureteral infiltration by tumor or extrinsic ureter compression by bulky tumor masses frequently cause these sequelae (22). In general, cervical, bladder, and prostate cancers are the most common culprits (22). However, urinary obstruction can also occur from retroperitoneal fibrosis due to surgery, chemotherapy, and/or radiotherapy. In the setting of malignant obstruction, percutaneous nephrostomy tube placement or retrograde ureteral double-J stent placement should be urgently performed, recognizing the associated procedural complications (22). Recently, a prognostic model for survival after palliative urinary diversion for malignant urinary obstruction has been developed (23). Two risk factors (at least four events related to malignancy and an Eastern Cooperative Oncology Group [ECOG] index $2) were used to stratify patients into three groups by survival type: favorable (no factors), intermediate (one factor), and unfavorable (two factors). The median survival at 1, 6, and 12 months was 94.4%, 57.3%, and 44.9% in the favorable group, 78.0%, 36.3%, and 15.5% in the intermediate group, and 46.4%, 14.3%, and 7.1% in the unfavorable group, respectively (23). This simple model could help to guide clinical decisions when choosing which patients are reasonable candidates for urinary diversion in a palliative setting. American Society of Nephrology CONCLUSION CKD is highly prevalent in oncologic patients and appears to be a risk factor for the development of cancer. Furthermore, the use of antineoplastic drugs in patients with underlying CKD raises several specific issues: 1) the direct nephrotoxicity of several anticancer agents (especially novel molecularly targeted agents); 2) the need to adjust antineoplastic doses due to concomitant CKD; 3) the lack of prospective drug dosing data in patients with advanced CKD or those on dialysis; and 4) the nihilistic approach to the treatment of this population of patients with CKD, leading to the frequent undertreatment (or even absence of treatment) of these patients, ultimately denying them potentially life-prolonging options (3). Indeed, in our opinion, CKD should not be regarded, in and of itself, as a reason to reduce or hold targeted therapies in the absence of other comorbidities or medical indications (3). TAKE HOME POINTS c The relationship between CKD and cancer should be regarded as bidirectional. c The use of targeted agents in cancer patients with CKD is ill defined. c CKD and ESRD requiring dialysis, per se, should not be regarded as reasons not to administer anticancer treatments. c Nephrectomy for kidney cancer is another common cause of CKD, but the use of nephron-sparing surgical techniques have been developed to limit this issue. c Medical disease–related CKD has worse outcomes than CKD due to surgery. c The tumor may cause CKD by causing urinary obstruction with obstructive nephropathy. REFERENCES 1. Launay-Vacher V, Oudard S, Janus N, Gligorov J, Pourrat X, Rixe O, Morere JF, Beuzeboc P, Deray G; Renal Insufficiency and Cancer Medications (IRMA) Study Group. Prevalence of renal insufficiency in cancer patients and implications for anticancer drug management: The renal insufficiency and anticancer medications (IRMA) study. Cancer 110: 1376–1384, 2007 2. Launay-Vacher V. Epidemiology of chronic kidney disease in cancer patients: Lessons from the IRMA study group. Semin Nephrol 30: 548– 556, 2010 3. Porta C, Cosmai L, Gallieni M, Pedrazzoli P, Malberti F. Renal effects of targeted anticancer therapies. Nat Rev Nephrol 11: 354–370, 2015 4. Stengel B. Chronic kidney disease and cancer: A troubling connection. J Nephrol 23: 253–262, 2010 5. Lowrance WT, Ordoñez J, Udaltsova N, Russo P, Go AS. CKD and the risk of incident cancer. J Am Soc Nephrol 25: 2327–2334, 2014 6. Weng PH, Hung KY, Huang HL, Chen JH, Sung PK, Huang KC. Cancerspecific mortality in chronic kidney disease: Longitudinal follow-up of a large cohort. Clin J Am Soc Nephrol 6: 1121–1128, 2011 7. Stewart JH, Buccianti G, Agodoa L, Gellert R, McCredie MR, Lowenfels AB, Disney AP, Wolfe RA, Boyle P, Maisonneuve P. Cancers of the kidney and urinary tract in patients on dialysis for end-stage renal disease: Analysis of data from the United States, Europe, and Australia and New Zealand. J Am Soc Nephrol 14: 197–207, 2003 American Society of Nephrology 8. Porta C, Paglino C, Imarisio I, Bonomi L. Uncovering Pandora’s vase: The growing problem of new toxicities from novel anticancer agents. The case of sorafenib and sunitinib. Clin Exp Med 7: 127–134, 2007 9. Thomas SM, Grandis JR. Pharmacokinetic and pharmacodynamic properties of EGFR inhibitors under clinical investigation. Cancer Treat Rev 30: 255–268, 2004 10. Josephs D, Hutson TE, Cowey CL, Pickering LM, Larkin JM, Gore ME, Van Hemelrijck M, McDermott DF, Powles T, Chowdhury P, Karapetis C, Harper PG, Choueiri TK, Chowdhury S. Efficacy and toxicity of sunitinib in patients with metastatic renal cell carcinoma with severe renal impairment or on haemodialysis. BJU Int 108: 1279–1283, 2011 11. Manikandan R, Srinivasan V, Rané A. Which is the real gold standard for small-volume renal tumors? Radical nephrectomy versus nephronsparing surgery. J Endourol 18: 39–44, 2004 12. Li L, Lau WL, Rhee CM, Harley K, Kovesdy CP, Sim JJ, Jacobsen S, Chang A, Landman J, Kalantar-Zadeh K. Risk of chronic kidney disease after cancer nephrectomy. Nat Rev Nephrol 10: 135–145, 2014 13. Canter D, Kutikov A, Sirohi M, Street R, Viterbo R, Chen DY, Greenberg RE, Uzzo RG. Prevalence of baseline chronic kidney disease in patients presenting with solid renal tumors. Urology 77: 781–785, 2011 14. Huang WC, Levey AS, Serio AM, Snyder M, Vickers AJ, Raj GV, Scardino PT, Russo P. Chronic kidney disease after nephrectomy in patients with renal cortical tumours: a retrospective cohort study. Lancet Oncol 7: 735–740, 2006 15. Cho A, Lee JE, Kwon GY, Huh W, Lee HM, Kim YG, Kim DJ, Oh HY, Choi HY. Post-operative acute kidney injury in patients with renal cell carcinoma is a potent risk factor for new-onset chronic kidney disease after radical nephrectomy. Nephrol Dial Transplant 26: 3496–3501, 2011 16. Campbell SC, Novick AC, Belldegrun A, Blute ML, Chow GK, Derweesh IH, Faraday MM, Kaouk JH, Leveillee RJ, Matin SF, Russo P, Uzzo RG; Practice Guidelines Committee of the American Urological Association. Guideline for management of the clinical T1 renal mass. J Urol 182: 1271–1279, 2009 17. Ljungberg B, Bensalah K, Canfield S, Dabestani S, Hofmann F, Hora M, Kuczyk MA, Lam T, Marconi L, Merseburger AS, Mulders P, Powles T, Staehler M, Volpe A, Bex A. EAU guidelines on renal cell carcinoma: 2014 update. Eur Urol 67: 913–924, 2015 18. Kim SP, Thompson RH, Boorjian SA, Weight CJ, Han LC, Murad MH, Shippee ND, Erwin PJ, Costello BA, Chow GK, Leibovich BC. Comparative effectiveness for survival and renal function of partial and radical nephrectomy for localized renal tumors: A systematic review and meta-analysis. J Urol 188: 51–57, 2012 19. Van Poppel H, Da Pozzo L, Albrecht W, Matveev V, Bono A, Borkowski A, Colombel M, Klotz L, Skinner E, Keane T, Marreaud S, Collette S, Sylvester R. A prospective, randomised EORTC intergroup phase 3 study comparing the oncologic outcome of elective nephron-sparing surgery and radical nephrectomy for low-stage renal cell carcinoma. Eur Urol 59: 543–552, 2011 20. Satasivam P, Reeves F, Rao K, Ivey Z, Basto M, Yip M, Roth H, Grummet J, Goad J, Moon D, Murphy D, Appu S, Lawrentschuk N, Bolton D, Kearsley J, Costello A, Frydenberg M. Patients with medical risk factors for chronic kidney disease are at increased risk of renal impairment despite the use of nephron-sparing surgery. BJU Int 116: 590–595, 2015 21. Volpe A, Blute ML, Ficarra V, Gill IS, Kutikov A, Porpiglia F, Rogers C, Touijer KA, Van Poppel H, Thompson RH. Renal ischemia and function after partial nephrectomy: A collaborative review of the literature. Eur Urol 68: 61–74, 2015 22. Russo P: Ureteral obstruction and stents: Still a difficult problem for patients and urologists alike. J Urol 174: 2088, 2005 23. Cordeiro MD, Coelho RF, Chade DC, Pessoa RR, Chaib MS, ColomboJúnior JR, Pontes-Júnior J, Guglielmetti GB, Srougi M. A prognostic model for survival after palliative urinary diversion for malignant ureteric obstruction: A prospective study of 208 patients. BJU Int 117: 266–271, 2016 Onco-Nephrology Curriculum 5 REVIEW QUESTIONS 1. Which types of cancer are more commonly observed in CKD patients? a. b. c. d. Gastrointestinal cancers Lung cancers Breast cancers Renal and urothelial cancers Answer: d is correct. A large retrospective cohort study assessed the association between eGFR level and the risk of incident cancer. Lower eGFR was associated with an increased risk of renal cancer with an adjusted HR of 2.28 (95% CI, 1.78–2.92) for an eGFR ,30 mL/min per 1.73 m2. An increased risk of urothelial cancer at an eGFR ,30 mL/min per 1.73 m2 was also evidenced. Finally, CKD conferred an increased cancer-specific mortality in patients with kidney and urinary tract cancer. 2. Should the dose of a targeted anticancer agent be reduced in a patient with mild to moderate CKD? a. b. c. d. Yes, almost always No, almost never Yes, depending on the drug’s pharmacokinetic properties The targeted anticancer agent should not be used in this setting Answer: b is correct. The pharmacokinetics of all targeted anticancer agents are similar. Most are 90%–98% bound to 6 Onco-Nephrology Curriculum plasma proteins, and their excretion occurs predominantly via the feces (or the reticulo-endothelial system), whereas urinary excretion is quite variable among the drugs. A population pharmacokinetic model indicated that it is unlikely that renal insufficiency has a clinically relevant effect on the pharmacokinetics of targeted therapies. Thus, no dosage adjustment is recommended in patients with a creatinine clearance .30 mL/min. 3. Which of the following contributes to the development of CKD in nephrectomized patients? a. b. c. d. Nephrectomy itself (loss of nephrons) Concomitant comorbidities Ischemia time All the above Answer: d is correct. Radical nephrectomy and nephronsparing surgery greatly differ in terms of incidence of postoperative CKD, due to the different amount of nephron loss. Less invasive surgical procedures for renal cell carcinoma are associated with less postoperative AKI and CKD. Despite this, a recent report suggested that patients with medical risk factors for CKD are at increased risk of progressive renal impairment, irrespective of the use of partial nephrectomy, thus highlighting the key role played by comorbidities in the development of post-nephrectomy CKD. The duration of ischemia is another important predictor of postoperative kidney function. Prolonged warm ischemia time is significantly associated with adverse postoperative kidney function. American Society of Nephrology Chapter 14: Hereditary Renal Cancer Syndromes Katherine L. Nathanson, MD Department of Medicine, Division of Translational Medicine and Human Genetics, and Cancer Control Program, Abramson Cancer Center, Perelman School of Medicine at the University of Pennsylvania, Philadelphia, Pennsylvania INTRODUCTION Inherited forms of renal cancer are estimated to account for 2%–5% of all kidney cancer (1). Currently, 10 inherited cancer susceptibility syndromes are definitively associated with an increased risk of renal cancer (Table 1) and are described in more detail below. Patients with these inherited syndromes develop kidney cancer at an earlier age; furthermore, the lesions can be multifocal, bilateral, and heterogeneous. Several, including Birt-Hogg-Dubé syndrome (BHD), familial clear cell renal cancer due to chromosome 3 translocation, hereditary papillary renal cancer, hereditary leiomyomatosis and renal cell cancer (HLRCC), and von Hippel-Lindau disease (vHL) have renal cancer as a primary feature, whereas in another inherited cancer susceptibility syndrome, such as BAP1 mutation–associated disease, Lynch syndrome, phosphatase and tensin homologue (PTEN) hamartoma syndrome, hereditary pheochromocytoma and paraganglioma (due to SDHx mutations), and tuberous sclerosis complex, it is a secondary feature. Recently, mutations in CDKN2B and PBRM1 also have been reported to predispose to clear cell renal cancer in single case series (2,3) and, as such, need further validation. Many of the genes identified through the studies of familial renal cancer have proven to play a critical role in renal cancer development through somatic mutation, with vHL disease being the exemplar of this paradigm. The description of families with inherited cancer susceptibility syndromes associated with an increased risk of renal cancer has and will lead to the discovery of mutated genes important in the pathogenesis of renal cancer. Below, the features of the inherited cancer susceptibility syndromes associated with an increased risk of renal cancer, with a focus on the renal manifestations and pathologic features, are reviewed. VON-HIPPEL LINDAU DISEASE Patients with this highly penetrant autosomal dominant cancer susceptibility syndrome can present with American Society of Nephrology a wide spectrum of hemangioblastomas of the brain, spine, and retina, pancreatic cysts and neuroendocrine tumors, renal cysts and clear cell renal tumors, endolympatic sac tumors, and pheochromocytomas. vHL disease is found across all ethnic groups, with approximately one-quarter of the incidence due to de novo mutations; genetic testing for mutations in VHL detects nearly 100% of individuals with vHL disease (6). Disease usually presents in the late teens to early 20s, although an occasional individual may be diagnosed in their mid-40s. The presentation of renal disease is quite variable even within family members, with some patients never developing renal cancer, others having a few renal cysts, and others with who have bilateral renal cancers and hundreds of lesions within each kidney. In part, this variability is due to a strong genotype–phenotype correlation that is seen with a mutational type predictive of disease (7). Patients with type 1 (truncating) mutations have a decreased incidence of pheochromocytoma compared with those with type 2 (missense) mutations (8). Frameshift and nonsense mutations in VHL are associated with a high penetrance of clear cell renal cancer, with a risk at age 50 of 70% (8). Full and partial gene deletions of VHL confer a lower risk at age 50 of 40%. Families with type 2 mutations have either a low (type 2A) or high risk of clear cell renal cell carcinoma (ccRCC) (type 2B); type 2C families develop pheochromocytoma only. Type 2A disease is associated with the “Black Forest’ founder mutation (Tyr98His) originating from southwestern Germany, commonly found in the “Pennsylvania Dutch” population (9). Despite the variability in phenotype, screening recommendations for vHL patients are standardized, and in adult include annual CNS (brain, spine) and abdominal/pelvic magnetic resonance Correspondence: Katherine L. Nathanson, Perelman School of Medicine at the University of Pennsylvania, 351 BRB 2/3, 421 Curie Blvd., Philadelphia, Pennsylvania 19104. Email: knathans@ exchange.upenn.edu Copyright © 2016 by the American Society of Nephrology Onco-Nephrology Curriculum 1 Table 1. Inherited cancer susceptibility syndromes associated with an increased risk of renal cancer Syndrome Gene Protein Predominant renal Cancer Type BAP1 mutation associated BAP1 cancer susceptibility BRCA associated protein Clear cell Birt-Hogg-Dubé syndrome FLCN folliculin Oncocytic, chromophobe, hybrid Clear cell Other Cancers Non-Neoplastic Findings Melanoma Uveal melanoma Mesothelioma — Epithelioid atypical Spitz tumors Fibrofolliculomas Lung cysts, pneumothorax — — Familial clear cell renal cancer due to chromosome 3 translocation Hereditary Leiomyomatosis and Renal Cell Cancer Translocation chr 3 FH fumarate hydratase Papillary type 2 — Hereditary Papillary Renal Cancer Hereditary Pheochromocytoma and Paraganglioma MET c-MET Papillary type 1 — SDHB SDHC SDHD succinate dehydrogenase subunits B, C, D Lynch Syndrome MLH1 MSH2 Mismatch repair proteins MSH6 PMS2 PTEN Clear cell Paraganglioma (distinct phenotype) Pheochromocytoma Gastrointestinal Stromal Tumor Urothelial cancer Colorectal cancer (upper tract) Endometrial (uterine) cancer Ovarian cancer PTEN Clear cell Breast cancer Thyroid cancer TSC1 TSC2 hamartin Tuberin Angiomyolipoma Epitheliod angiomyolipoma Angiomyolipomas Subependymal giant cell astrocytomas VHL pVHL Clear cell Clear cell papillary CNS - hemangioblastoma (brain, spine, retina) Adrenal - pheochromocytoma Inner ear - endolymphatic sac tumors Pancreas - neuroendocrine tumors PTEN Hamartoma Syndrome (Cowden syndrome) Tuberous Sclerosis Complex Von Hippel Lindau disease (MR) imaging, ophthalmologic evaluation, plasma metanephrines, and consultation with vHL expert. The implementation of screening guidelines has led to vast improvements in survival (10). Classically, vHL disease and mutations in VHL have been associated with clear cell renal cancers. However, clear cell papillary renal cell carcinoma (CCPRCC), a relatively recently described entity with prominent papillary architecture, exclusive clear cell morphology, and a partially cystic appearance, has been reported in patients with vHL disease (11). The exact characteristics of CCPRCC in this context, and relationship to sporadic disease are somewhat controversial, due to loss of 2 Onco-Nephrology Curriculum Cutaneous leiomyomas Uterine leiomyomas — — Mucocutaneous papules, hamartomas, lipomas, macrocephaly Facial angiofibroma Hypomelanotic macule Connective tissue nevus Forehead plaque Ungal and peri-ungal fibromas Pancreatic, renal cysts chromosome 3p in these tumors and differing findings in regards to cytokeratin 7 staining (12). Based on multiple natural history studies done at the National Cancer Institute, the standard of care for timing of resection of renal cancer in patients with vHL disease is when there is a solid component of 3 cm. In the initial series, with a follow-up of .5 years, Walther et al. reported no evidence of metastatic disease progression and no need for renal transplantation or dialysis among 52 patients with tumors ,3 cm at diagnosis. In contrast, distant metastases developed in 11 of 44 patients (25%) with lesions .3 cm in size, including 3 of 27 patients (11%) with lesions between 3 and 6 cm (13). Similar results were obtained in a follow-up study 5 years later (14). American Society of Nephrology HEREDITARY PAPILLARY RENAL CELL CARCINOMA (TYPE 1 PAPILLARY) Hereditary papillary renal cell carcinoma (HPRCC) is an autosomal dominant syndrome characterized by multifocal, bilateral type 1 papillary renal cell carcinomas, with hundreds of tumors observed due to mutations in MET (15). The tumors are indistinguishable pathologically from sporadic type 1 papillary renal cancer. Mutations of the MET gene on 7q31 have been causally associated with HPRCC. Families with inherited mutations in MET leading to multifocal papillary renal cancer (type 1) are quite rare, more so than other described inherited renal cancer syndromes. HEREDITARY LEIOMYOMATOSIS AND RENAL CELL CANCER (TYPE 2 PAPILLARY) Hereditary leiomyomatosis and renal cell cancer (HLRCC), otherwise known as Reed’s syndrome, is an autosomal cancer susceptibility syndrome characterized by the development of cutaneous and uterine leiomyomas and renal cancer, due to mutations in fumarate hydratase (FH) (16). The lifetime risk of renal cancer is currently estimated to be 15% (17). The pattern of renal cancer in HLRCC differs from other inherited renal cancer susceptibility syndromes in that the tumors tend to be solitary and unilateral and have a more aggressive course of disease. Independent of underlying architecture, which is most commonly described as a subtype of type 2 papillary renal cancer, cells in the renal cancers associated with HLRCC have a characteristic pathologic appearance with large nuclei, with inclusion-like orangiophilic or eosinophilic nucleoi surrounded by a clear halo, which can be recognized by knowledgeable pathologists (18). Recently, several studies have focused on using immunohistochemistry with S-(2-succinyl) cysteine (2SC) as a surrogate for FH deficiency, as an adjunct to pathologic features to accurately diagnose HLRCC associated renal cancer. An initial study from Bardella et al. suggested that positive staining with 2SC was sensitive and specific to detect renal cancers associated with FH mutations (19). An independent followup study confirmed that 2SC demonstrated diffuse and strong cytoplasmic staining in the confirmed HLRCC tumors compared with other tumor types (20). Thus, in addition to family and personal history, histology and immunohistochemistry can be used to assist in the diagnosis of HLRCC when the initial presenting manifestation is renal cancer. The mean age of renal cancer diagnosis is 40 years, but metastatic renal cancer can present in the teens. Given the potential early age of renal cancer diagnosis, genetic testing is recommended at age 8–10 for familial FH mutations with annual MRI; however, the risk of renal cancers is relatively low before age 20, so the drawbacks of screening should be considered (17). American Society of Nephrology BIRT-HOGG-DUBÉ DISEASE BHD disease is an autosomal dominant cancer susceptibility syndrome characterized by the development of fibrofolliculomas (dysplastic hair follicules), lung cysts and spontaneous pneumothorax, and renal cancer, and is due to mutations in folliculin (FLCN) (21). The dermatologic features and lung disease are the most common presenting features; BHD is underdiagnosed due to its variable, often mild, presentation. Awide spectrum of renal cancers (papillary RCC, ccRCC, mixed, and oncocytomas) has been observed in patients with BHD, even within the same kidney (22). The renal parenchyma surrounding the renal tumor can often contain multifocal oncocytosis. The most common type of tumor is an unusual hybrid oncocytic tumor (mixed oncocytoma and chromophobe). As a hybrid oncocytic tumor is characteristic of BHD, any patient presenting with one should be evaluated for BHD. Renal cancer is observed in approximately 30% of patients with a highly variable age of diagnosis ranging from 20 to 83 years, with an average age of 46 years (23). Given the low malignant potential of these tumors, it is generally recommended that screening with abdominal MRI take place every 2 years and that the tumors can be observed until they are 3 cm in diameter prior to resection (24). OTHER INHERITED SYNDROMES WITH AN INCREASED RISK OF RENAL CANCER BAP1 mutations and familial renal cancer Mutations in BAP1 (BRCA-associated protein 1) were initially identified through somatic sequencing of renal tumors. One of the tumors in the initial study was found to carry a germ-line BAP1 mutation, and subsequent studies suggested that BAP1 mutations predispose to familial clear cell renal cancer, along with uveal and cutaneous melanoma and mesothelioma (25,26). Chromosome 3 translocations associated with clear cell renal cancer The first genetic changes identified as associated with inherited risk of clear cell renal cancer were balanced translocations involving chromosome 3; since then, multiple families have been reported with multifocal bilateral disease (27). The mechanism behind the increased risk of multifocal clear cell renal cancer is thought to be loss of the rearranged chromosome during mitosis, which requires a quadrivalent (four chromosomes coming together), leading to greater errors during chromosomal segregation. As multiple genes involved in the pathogenesis of clear cell renal cancer are located on chromosome 3p, including VHL, PBRM1, BAP1, and SETD2 (28), it is not surprising that a mechanism of increased loss of one allele leads to an increased risk of clear cell renal cancer. Histologically, the clear cell renal cancers are indistinguishable from those associated with VHL mutations, although the age of onset tends to be later than in vHL disease. Onco-Nephrology Curriculum 3 Lynch syndrome Although Lynch syndrome (also known hereditary nonpolyposis colorectal cancer), due to mutations in the mismatch repair genes (MMR) MLH1, MSH2, MSH6, and PMS2, is most commonly associated with an increased risk of colorectal, endometrial, and ovarian cancers, upper tract urothelial cancers are a well-recognized feature (29). The estimated risk ranges from 2.6% to 11% (30,31). The risk appears to be highest in patients with MSH2 and MLH1 mutations and presents at earlier ages than sporadic disease (32). A recent review for urologists suggests that evaluation for Lynch syndrome should be done when a patient with upper tract urothelial cancer presents before age 60 or the family meets Amsterdam I or II criteria, which includes colorectal, small bowel, ureter, endometrial, and ovarian cancers (29). Screening for Lynch syndrome can be done using immunohistochemistry for the MMR proteins, in which loss of staining suggests the diagnosis. PTEN hamartoma tumor syndrome Mutations in PTEN are associated with a pleomorphic syndrome (PHTS, also known as Cowden disease and Bannayan-RileyRuvalcaba), which has a variety of disease manifestations ranging from cancer susceptibility to intellectual disability. Patients are at increased risk of benign and malignant tumors of the thyroid, breast, and endometrium; it has been suggested that renal cancer is an underappreciated component of the cancer spectrum (33). Recent estimates suggesting that 3%–5% of of patients with PHTS have renal cancer, with a standardized incidence ratio of 31.7 (95% CI, 15.4–58.4); however, these estimates are based on small numbers and may be due to ascertainment basis (34). When centrally reviewed, the pathology of the renal cancers was either papillary or chromophobe (34). Screening for renal cancers is part of the standard surveillance recommendations for patients with PHTS and is recommended biennially starting at age 40. SDHx-associated paraganglioma/pheochromocytoma Mutations in the five proteins (SDHA, SDHB, SDHC, SDHD, and associated cofactor SDHAF2) that comprise the succinate dehydrogenase complex, which participates in both the Krebs cycle, converting fumarate to succinate, and as mitochondrial respiratory chain complex II, have been associated with an increased risk of pheochromocytomas, paragangliomas, gastrointestinal stromal tumors, and renal cancer (35). Renal cancer has been most commonly observed in association with patients carrying SDHB mutations, which has the highest risk of malignant disease, and these renal tumors have been reported to be particularly aggressive (36). SDHB-associated renal cancer displays a characteristic histopathology with solid or focally cystic growth, uniform cytology with eosinophilic flocculent cytoplasm, intracytoplasmic vacuolations and inclusions, and round to oval low-grade nuclei (37). Renal cancer can be the sentinel diagnosis in the family, so pathologists should be aware of and alert to this potential diagnosis. SDHB 4 Onco-Nephrology Curriculum immunohistochemistry (absence of staining) also can be used to assist in making the diagnosis. Tuberous sclerosis complex Tuberous sclerosis complex (TSC) is an autosomal dominant genetic disorder characterized by the formation of hamartomas in multiple organs, including brain, kidney, skin, and lung. The formation of hamartomas leads to neurologic disorders, including epilepsy, mental retardation, and autism, as well as dermatologic manifestations such as facial angiofibromas, renal angiomyolipomas, and pulmonary lymphangiomyomatosis (38). Inactivating mutations in TSC1 encoding hamartin, or TSC2 encoding tuberin are responsible for the phenotype. Patients with TSC2 mutations are more severely affected with greater renal involvement among other features. Fifty percent to 80% of patients with TSC develop renal lesions including angiomyolipomas (AMLs), cysts, and oncocytomas; renal cell cancer is estimated to occur in ,5% (with precise estimates varying across studies) (39). A recent review of TSC-associated renal cancer demonstrated young age at diagnosis, multifocal disease, an indolent clinical course, and three morphologies: renal angiomyoadenomatous tumor, chromophobe renal cancer, and a granular eosinophilicmacrocystic morphology (40). In patients with TSC2 mutations and multiple renal tumors, it has been shown that they are due to a “shower” of second hits with different secondary TSC2 mutations in each tumor (41). Everolimus is US Food and Drug Administration (FDA) approved for treating AMLs in the setting of TSC and also should be considered for TSCassociated renal cancer. CONTROVERSIES IN GENETIC TESTING FOR INHERITED SUSCEPTIBILITY TO RENAL CANCER Two questions recently have arisen in relationship to genetic testing for inherited susceptibility to renal cancer. Marston Linehan and colleagues from the National Cancer Institute have suggested that all patients with renal cancer diagnosed under the age of 45 should have consideration of genetic counseling/germline mutation testing, even in the absence of a personal or family medical history suggestive of an inherited syndrome (4). The rates of renal cancer in patients under age 50 is steadily increasing and has doubled since 1995, going from 3 per 100,000 to 6 per 100,000 (5), presumably due to the increasing number of incidental renal cancers detected on imaging studies. Although a cutoff age of 45 for detecting cases of inherited renal cancer may be quite sensitive, with the background rate increasing so dramatically, it would require testing of many individuals to identify only a few with inherited disease. Population-based studies of mutation testing in early-onset renal cancer are required to answer the question of utility of general genetic counseling and testing in this setting. An earlier age cutoff, such as at 30, may emerge as a more feasible alternative. As with many other genetic diseases, multiplex gene panels using massively parallel American Society of Nephrology sequencing have emerged as an alternative for genetic testing for renal cancer susceptibility syndromes (e.g., RenalNext from Ambry Genetics). Given the usefulness of renal pathology and extra-kidney manifestations to guide genetic testing, which is relatively unusual for other cancer types, the usefulness of panels that include genes in which mutations predispose to vastly different diseases (e.g., vHL and HLRCC) is not immediately apparent. However, many institutions do not always differentiate renal cancer pathologies to the degree needed for prioritization of genetic testing studies, and even at experienced centers, discrimination of renal cancer pathologies can be complex on occasion. Thus, there has been uptake of massively parallel sequencing panels for renal cancer–associated syndromes, but not to the same extent as for other cancer types (e.g., breast and ovarian cancers, pheochromocytoma). TAKE HOME POINTS c Many different types of hereditary renal cancer exist, and in general, each is associated with a histologic subtype. c Renal cancer can either be a major or a minor feature of a cancer suscep- tibility syndrome, but early age of onset, unusual or pathognomic pathology, and multiplicity of tumors all should be red flags, which prompt questions about family history and consideration of inherited disease. c Standard of care surveillance recommendations are available for essen- tially all renal cancer susceptibility syndromes and should be followed. REFERENCES 1. Cho E, Adami HO, Lindblad P. Epidemiology of renal cell cancer. Hematol Oncol Clin North Am 25: 651–665, 2011 2. Jafri M, Wake NC, Ascher DB, Pires DE, Gentle D, Morris MR, Rattenberry E, Simpson MA, Trembath RC, Weber A, Woodward ER, Donaldson A, Blundell TL, Latif F, Maher ER. Germline Mutations in the CDKN2B tumor suppressor gene predispose to renal cell carcinoma. Cancer Discov 5: 723–729, 2015 3. Benusiglio PR, Couvé S, Gilbert-Dussardier B, Deveaux S, Le Jeune H, Da Costa M, Fromont G, Memeteau F, Yacoub M, Coupier I, Leroux D, Méjean A, Escudier B, Giraud S, Gimenez-Roqueplo AP, Blondel C, Frouin E, Teh BT, Ferlicot S, Bressac-de Paillerets B, Richard S, Gad S. A germline mutation in PBRM1 predisposes to renal cell carcinoma. J Med Genet 52: 426–430, 2015 4. Shuch B, Vourganti S, Ricketts CJ, Middleton L, Peterson J, Merino MJ, Metwalli AR, Srinivasan R, Linehan WM. Defining early-onset kidney cancer: implications for germline and somatic mutation testing and clinical management. J Clin Oncol 32: 431–437, 2014 5. Howlader N, Noone AM, Krapcho M, Garshell J, Miller D, Altekruse SF, Kosary CL, Yu M, Ruhl J, Tatalovich Z, Mariotto A, Lewis DR, Chen HS, Feuer EJ, Cronin KA. SEER Cancer Statistics Review, 1975-2012, Bethesda, MD, National Cancer Institute, 2015 6. Crossey PA, Foster K, Richards FM, Phipps ME, Latif F, Tory K, Jones MH, Bentley E, Kumar R, Lerman MI. Molecular genetic investigations of the mechanism of tumourigenesis in von Hippel-Lindau disease: Analysis of allele loss in VHL tumours. Hum Genet 93: 53–58, 1994 7. Maher ER, Webster AR, Richards FM, Green JS, Crossey PA, Payne SJ, Moore AT. Phenotypic expression in von Hippel-Lindau disease: correlations with germline VHL gene mutations. J Med Genet 33: 328–332, 1996 American Society of Nephrology 8. Ong KR, Woodward ER, Killick P, Lim C, Macdonald F, Maher ER. Genotype-phenotype correlations in von Hippel-Lindau disease. Hum Mutat 28: 143–149, 2007 9. Brauch H, Kishida T, Glavac D, Chen F, Pausch F, Höfler H, Latif F, Lerman MI, Zbar B, Neumann HP. Von Hippel-Lindau (VHL) disease with pheochromocytoma in the Black Forest region of Germany: Evidence for a founder effect. Hum Genet 95: 551–556, 1995 10. Schmid S, Gillessen S, Binet I, Brändle M, Engeler D, Greiner J, Hader C, Heinimann K, Kloos P, Krek W, Krull I, Stoeckli SJ, Sulz MC, van Leyen K, Weber J, Rothermundt C, Hundsberger T. Management of von hippellindau disease: an interdisciplinary review. Oncol Res Treat 37: 761– 771, 2014 11. Tickoo SK, dePeralta-Venturina MN, Harik LR, Worcester HD, Salama ME, Young AN, Moch H, Amin MB. Spectrum of epithelial neoplasms in end-stage renal disease: An experience from 66 tumor-bearing kidneys with emphasis on histologic patterns distinct from those in sporadic adult renal neoplasia. Am J Surg Pathol 30: 141–153, 2006 12. Rao P, Monzon F, Jonasch E, Matin SF, Tamboli P. Clear cell papillary renal cell carcinoma in patients with von Hippel-Lindau syndrome: Clinicopathological features and comparative genomic analysis of 3 cases. Hum Pathol 45: 1966–1972, 2014 13. Walther MM, Choyke PL, Glenn G, Lyne JC, Rayford W, Venzon D, Linehan WM. Renal cancer in families with hereditary renal cancer: Prospective analysis of a tumor size threshold for renal parenchymal sparing surgery. J Urol 161: 1475–1479, 1999 14. Duffey BG, Choyke PL, Glenn G, Grubb RL, Venzon D, Linehan WM, Walther MM. The relationship between renal tumor size and metastases in patients with von Hippel-Lindau disease. J Urol 172: 63–65, 2004 15. Schmidt L, Duh FM, Chen F, Kishida T, Glenn G, Choyke P, Scherer SW, Zhuang Z, Lubensky I, Dean M, Allikmets R, Chidambaram A, Bergerheim UR, Feltis JT, Casadevall C, Zamarron A, Bernues M, Richard S, Lips CJ, Walther MM, Tsui LC, Geil L, Orcutt ML, Stackhouse T, Lipan J, Slife L, Brauch H, Decker J, Niehans G, Hughson MD, Moch H, Storkel S, Lerman MI, Linehan WM, Zbar B. Germline and somatic mutations in the tyrosine kinase domain of the MET proto-oncogene in papillary renal carcinomas. Nat Genet 16: 68–73, 1997 16. Tomlinson IP, Alam NA, Rowan AJ, Barclay E, Jaeger EE, Kelsell D, Leigh I, Gorman P, Lamlum H, Rahman S, Roylance RR, Olpin S, Bevan S, Barker K, Hearle N, Houlston RS, Kiuru M, Lehtonen R, Karhu A, Vilkki S, Laiho P, Eklund C, Vierimaa O, Aittomäki K, Hietala M, Sistonen P, Paetau A, Salovaara R, Herva R, Launonen V, Aaltonen LA; Multiple Leiomyoma Consortium. Germline mutations in FH predispose to dominantly inherited uterine fibroids, skin leiomyomata and papillary renal cell cancer. Nat Genet 30: 406–410, 2002 17. Menko FH, Maher ER, Schmidt LS, Middelton LA, Aittomäki K, Tomlinson I, Richard S, Linehan WM. Hereditary leiomyomatosis and renal cell cancer (HLRCC): Renal cancer risk, surveillance and treatment. Fam Cancer 13: 637–644, 2014 18. Merino MJ, Torres-Cabala C, Pinto P, Linehan WM. The morphologic spectrum of kidney tumors in hereditary leiomyomatosis and renal cell carcinoma (HLRCC) syndrome. Am J Surg Pathol 31: 1578–1585, 2007 19. Bardella C, El-Bahrawy M, Frizzell N, Adam J, Ternette N, Hatipoglu E, Howarth K, O’Flaherty L, Roberts I, Turner G, Taylor J, Giaslakiotis K, Macaulay VM, Harris AL, Chandra A, Lehtonen HJ, Launonen V, Aaltonen LA, Pugh CW, Mihai R, Trudgian D, Kessler B, Baynes JW, Ratcliffe PJ, Tomlinson IP, Pollard PJ. Aberrant succination of proteins in fumarate hydratase-deficient mice and HLRCC patients is a robust biomarker of mutation status. J Pathol 225: 4–11, 2011 20. Chen YB, Brannon AR, Toubaji A, Dudas ME, Won HH, Al-Ahmadie HA, Fine SW, Gopalan A, Frizzell N, Voss MH, Russo P, Berger MF, Tickoo SK, Reuter VE. Hereditary leiomyomatosis and renal cell carcinoma syndrome-associated renal cancer: Recognition of the syndrome by pathologic features and the utility of detecting aberrant succination by immunohistochemistry. Am J Surg Pathol 38: 627–637, 2014 Onco-Nephrology Curriculum 5 21. Nickerson ML, Warren MB, Toro JR, Matrosova V, Glenn G, Turner ML, Duray P, Merino M, Choyke P, Pavlovich CP, Sharma N, Walther M, Munroe D, Hill R, Maher E, Greenberg C, Lerman MI, Linehan WM, Zbar B, Schmidt LS. Mutations in a novel gene lead to kidney tumors, lung wall defects, and benign tumors of the hair follicle in patients with the Birt-Hogg-Dubé syndrome. Cancer Cell 2: 157–164, 2002 22. Pavlovich CP, Grubb RL 3rd, Hurley K, Glenn GM, Toro J, Schmidt LS, Torres-Cabala C, Merino MJ, Zbar B, Choyke P, Walther MM, Linehan WM. Evaluation and management of renal tumors in the Birt-HoggDubé syndrome. J Urol 173: 1482–1486, 2005 23. Benusiglio PR, Giraud S, Deveaux S, Méjean A, Correas JM, Joly D, Timsit MO, Ferlicot S, Verkarre V, Abadie C, Chauveau D, Leroux D, Avril MF, Cordier JF, Richard S; French National Cancer Institute Inherited Predisposition to Kidney Cancer Network. Renal cell tumour characteristics in patients with the Birt-Hogg-Dubé cancer susceptibility syndrome: A retrospective, multicentre study. Orphanet J Rare Dis 9: 163, 2014 24. Houweling AC, Gijezen LM, Jonker MA, van Doorn MB, Oldenburg RA, van Spaendonck-Zwarts KY, Leter EM, van Os TA, van Grieken NC, Jaspars EH, de Jong MM, Bongers EM, Johannesma PC, Postmus PE, van Moorselaar RJ, van Waesberghe JH, Starink TM, van Steensel MA, Gille JJ, Menko FH. Renal cancer and pneumothorax risk in Birt-HoggDubé syndrome; an analysis of 115 FLCN mutation carriers from 35 BHD families. Br J Cancer 105: 1912–1919, 2011 25. Farley MN, Schmidt LS, Mester JL, Peña-Llopis S, Pavia-Jimenez A, Christie A, Vocke CD, Ricketts CJ, Peterson J, Middelton L, Kinch L, Grishin N, Merino MJ, Metwalli AR, Xing C, Xie XJ, Dahia PL, Eng C, Linehan WM, Brugarolas J. A novel germline mutation in BAP1 predisposes to familial clear-cell renal cell carcinoma. Mol Cancer Res 11: 1061–1071, 2013 26. Popova T, Hebert L, Jacquemin V, Gad S, Caux-Moncoutier V, Duboisd’Enghien C, Richaudeau B, Renaudin X, Sellers J, Nicolas A, SastreGarau X, Desjardins L, Gyapay G, Raynal V, Sinilnikova OM, Andrieu N, Manié E, de Pauw A, Gesta P, Bonadona V, Maugard CM, Penet C, Avril MF, Barillot E, Cabaret O, Delattre O, Richard S, Caron O, Benfodda M, Hu HH, Soufir N, Bressac-de Paillerets B, Stoppa-Lyonnet D, Stern MH. Germline BAP1 mutations predispose to renal cell carcinomas. Am J Hum Genet 92: 974–980, 2013 27. Woodward ER, Skytte AB, Cruger DG, Maher ER. Population-based survey of cancer risks in chromosome 3 translocation carriers. Genes Chromosomes Cancer 49: 52–58, 2010 28. Cancer Genome Atlas Research Network. Comprehensive molecular characterization of clear cell renal cell carcinoma. Nature 499: 43–49, 2013 29. Mork M, Hubosky SG, Rouprêt M, Margulis V, Raman J, Lotan Y, O’Brien T, You N, Shariat SF, Matin SF. Lynch syndrome: A primer for urologists and panel recommendations. J Urol 194: 21–29, 2015 30. Sijmons RH, Kiemeney LA, Witjes JA, Vasen HF. Urinary tract cancer and hereditary nonpolyposis colorectal cancer: Risks and screening options. J Urol 160: 466–470, 1998 6 Onco-Nephrology Curriculum 31. Win AK, Lindor NM, Winship I, Tucker KM, Buchanan DD, Young JP, Rosty C, Leggett B, Giles GG, Goldblatt J, Macrae FA, Parry S, Kalady MF, Baron JA, Ahnen DJ, Marchand LL, Gallinger S, Haile RW, Newcomb PA, Hopper JL, Jenkins MA. Risks of colorectal and other cancers after endometrial cancer for women with Lynch syndrome. J Natl Cancer Inst 105: 274–279, 2013 32. Barrow PJ, Ingham S, O’Hara C, Green K, McIntyre I, Lalloo F, Hill J, Evans DG. The spectrum of urological malignancy in Lynch syndrome. Fam Cancer 12: 57–63, 2013 33. Shuch B, Ricketts CJ, Vocke CD, Komiya T, Middelton LA, Kauffman EC, Merino MJ, Metwalli AR, Dennis P, Linehan WM. Germline PTEN mutation Cowden syndrome: An underappreciated form of hereditary kidney cancer. J Urol 190: 1990–1998, 2013 34. Mester JL, Zhou M, Prescott N, Eng C. Papillary renal cell carcinoma is associated with PTEN hamartoma tumor syndrome. Urology 79: 1187. e1–1187.e7, 2012 35. Evenepoel L, Papathomas TG, Krol N, Korpershoek E, de Krijger RR, Persu A, Dinjens WN. Toward an improved definition of the genetic and tumor spectrum associated with SDH germ-line mutations. Genet Med 17: 610–620, 2015 36. Ricketts CJ, Shuch B, Vocke CD, Metwalli AR, Bratslavsky G, Middelton L, Yang Y, Wei MH, Pautler SE, Peterson J, Stolle CA, Zbar B, Merino MJ, Schmidt LS, Pinto PA, Srinivasan R, Pacak K, Linehan WM. Succinate dehydrogenase kidney cancer: An aggressive example of the Warburg effect in cancer. J Urol 188: 2063–2071, 2012 37. Gill AJ, Hes O, Papathomas T, Sedivcová M, Tan PH, Agaimy A, Andresen PA, Kedziora A, Clarkson A, Toon CW, Sioson L, Watson N, Chou A, Paik J, Clifton-Bligh RJ, Robinson BG, Benn DE, Hills K, Maclean F, Niemeijer ND, Vlatkovic L, Hartmann A, Corssmit EP, van Leenders GJ, Przybycin C, McKenney JK, Magi-Galluzzi C, Yilmaz A, Yu D, Nicoll KD, Yong JL, Sibony M, Yakirevich E, Fleming S, Chow CW, Miettinen M, Michal M, Trpkov K. Succinate dehydrogenase (SDH)deficient renal carcinoma: A morphologically distinct entity: a clinicopathologic series of 36 tumors from 27 patients. Am J Surg Pathol 38: 1588–1602, 2014 38. Crino PB, Nathanson KL, Henske EP. The tuberous sclerosis complex. N Engl J Med 355: 1345–1356, 2006 39. Dixon BP, Hulbert JC, Bissler JJ. Tuberous sclerosis complex renal disease. Nephron, Exp Nephrol 118: e15–e20, 2011 40. Guo J, Tretiakova MS, Troxell ML, Osunkoya AO, Fadare O, Sangoi AR, Shen SS, Lopez-Beltran A, Mehra R, Heider A, Higgins JP, Harik LR, Leroy X, Gill AJ, Trpkov K, Campbell SC, Przybycin C, Magi-Galluzzi C, McKenney JK. Tuberous sclerosis-associated renal cell carcinoma: A clinicopathologic study of 57 separate carcinomas in 18 patients. Am J Surg Pathol 38: 1457–1467, 2014 41. Tyburczy ME, Jozwiak S, Malinowska IA, Chekaluk Y, Pugh TJ, Wu CL, Nussbaum RL, Seepo S, Dzik T, Kotulska K, Kwiatkowski DJ. A shower of second hit events as the cause of multifocal renal cell carcinoma in tuberous sclerosis complex. Hum Mol Genet 24: 1836–1842, 2015 American Society of Nephrology REVIEW QUESTIONS 1. What is the exception to the “3-cm rule” of tumor removal in hereditary renal cancer syndromes? a. von Hippel Lindau disease b. Birt-Hogg-Dubé syndrome c. Hereditary leiomyomatosis and renal cell cancer Answer: c is correct. Hereditary leiomyomatosis and renal cell cancer (HLRCC or Reed’s syndrome) is associated with a very aggressive form of renal cancer, and tumors should be removed as soon as they are detected. 2. A patient with renal cancer tells you that a sister had a pheochromocytoma. What further evaluation might that prompt? a. Review of pathology of the renal cancer and potentially additional immunohistochemistry b. Further family history collection and potentially genetics evaluation c. Nothing, probably unrelated d. a and b Answer: d is correct. Renal cancer is associated with hereditary pheochromocytoma and paraganglioma syndromes caused by SDHx mutations. These renal cancers have a characteristic pathology, which is well described. The renal cancers associated with SDHB mutations also American Society of Nephrology have loss of staining on SDHB immunohistochemistry. Collection of additional history also could be useful, in that further history of pheochromocytomas and paragangliomas in the family supports the diagnosis of an SDHx-related tumor. However, it is important to note that genetic testing is recommended for all patients with pheochromocytoma, so the family should be referred to a cancer geneticist independent of family history. 3. One of your patients has a hybrid chromophobe/oncocytic renal tumor, but is aged 65 and has no family history of renal cancer or other cancers. You vaguely remember reading this chapter, and remember that tumor type is a red flag,and deserves further evaluation, but can’t remember for what? a. BAP-1–associated renal cancer b. Birt-Hogg-Dubé syndrome c. Lynch syndrome Answer: b is correct. The three major manifestations of Birt-Hogg-Dubé (BHD) are fibrofolliculomas (facial), lung cysts and pneumothorax, and renal cancer. A hybrid chromophobe/oncocytic renal tumor is considered characteristic of BHD, and the finding on pathology should prompt further evaluation. All three manifestations of BHD are incompletely penetrant, particularly renal cancer. Thus, the lack of family history of renal cancer and older age of diagnosis should not preclude evaluation for BHD. Onco-Nephrology Curriculum 7 Chapter 15: Workup and Management of “Small” Renal Masses Susie L. Hu, MD,* and Anthony Chang, MD† *Division of Kidney Disease and Hypertension, Warren Alpert Medical School of Brown University, Rhode Island Hospital, Providence, Rhode Island; and †Department of Pathology, The University of Chicago Medicine, Chicago, Illinois INTRODUCTION Nephrologists are frequently asked by urology and oncology colleagues to participate in the management of patients diagnosed with a renal mass. This is especially the case when there is associated CKD, hypertension, and/or other medically challenging comorbidities. Renal masses are classified as large and small. Small renal masses, defined as T1a (#4 cm) with no metastases or contralateral kidney involvement, have a 5-year survival rate approaching 100% in most studies. Therefore, as the vast majority of these patients are cured of their kidney cancer, the maintenance of renal function is becoming the major determinant of clinical outcomes. In this era of almost indiscriminate use of diagnostic imaging, .50% of renal masses are incidentally discovered (1,2) and 16%–23% of these are benign (3,4). Given the rise of incidental tumors, nephron-sparing procedures (partial nephrectomy, cryoablation, radiofrequency ablation, or thermal ablation) are increasingly replacing traditional radical nephrectomy (RN), and even diagnostic renal biopsies are a viable option as the concept of tumor seeding along the needle track has been largely unfounded (5). Cancer-specific survival and overall survival between radical nephrectomy and nephron-sparing surgery are comparable (6). For poor or nonsurgical candidates, greater consideration is being given to ablative therapies or even active surveillance, given that small renal masses grow very slowly at an average rate of 1.3 mm/yr (7). This population (with small renal masses) who remain renal cancer free, are dying of other causes, most frequently due to cardiovascular events (8). With improving survival, morbidity related to CKD from nephron mass loss, as well comorbid disease–induced complications, has become more relevant and ultimately impacts survival (with increased risk for cardiovascular American Society of Nephrology death) (9). Consideration of preoperative kidney function, comorbidities (10), nephron-sparing surgical methods, and tumor size (11) should be made when determining the management plan for patients with small renal masses. EPIDEMIOLOGY Individuals with small renal tumors are older (average age of 60 years), predominantly white men (12–14) with notable comorbidity (13,14). In the Medicarelinked US Surveillance Epidemiology and End Results (SEER) database, .10,000 individuals with small tumors #7 cm surveyed had an average tumor size of 4 cm, had a higher median age of 74, were predominantly white, 8.7% were African American, and 65% were male. In this population with high burden of comorbid diseases, almost half had diabetes mellitus (DM) or chronic obstructive pulmonary disease, a third had cerebrovascular disease, and approximately 15% had peripheral vascular disease or preexisting CKD (14). The study population of the sole prospective randomized controlled multicenter trial, European Organization for Research and Treatment of Cancer (EORTC) trial, which included small renal masses (#5cm), had substantial comorbid disease (36%), which was largely cardiovascular disease (8). Overlapping risk factors between renal cell carcinoma and CKD may account for the high prevalence for CKD and cardiovascular disease in this population (Figure 1). Risk factors for renal cell carcinoma in the general population have traditionally Correspondence: Susie L. Hu, Rhode Island Hospital, 593 Eddy St., Providence, Rhode Island 02903. Email: [email protected] Copyright © 2016 by the American Society of Nephrology Onco-Nephrology Curriculum 1 Figure 1. Overlapping risk factors for renal cell carcinoma and CKD in the general population and among those with small renal masses. DM, diabetes mellitus; HLD, hyperlipidemia; HTN, hypertension; Neph, nephrectomy. included tobacco exposure, obesity, DM, and hypertension (HTN) (15). Cystic disease and ESRD patients have a greater predisposition to renal tumors (16). This risk is 100-fold for ESRD patients (16) and a lesser but significant risk has been identified for stage 3 and 4 CKD (17). Therefore, the risk factors for CKD seem to also predispose one to the development of renal cell carcinoma. With extension of survival, quality-of-life issues shaped by postoperative chronic disease burden, primarily CKD, and cardiovascular disease have increasingly become an important factor in the care of these patients. Preoperative CKD and comorbidity Prevalence of CKD seems to vary, considerably ranging from 10% to 52% (18–21) for those with small renal masses, similar to that of all-comers with all sizes of renal masses (11%–32%) depending on the cutoff GFR and age of the general population (22–24). When examining older subgroups, CKD prevalence nearly doubled (19,23), consistent with the finding that increasing age raises CKD risk. Of known CKD risk factors, the most important two, DM and HTN, were also highly prevalent in this population, where 9%–22% had diabetes and 23%–59% were hypertensive (18,20,24–26). The extent of DM and HTN was greater not only among those with a diagnosis of preexisting CKD, but also among those with renal cell carcinoma. Not surprisingly, preexisting CKD patients had more DM (26%) and HTN (60%) in a Korean cohort with small renal masses (n 5 1,928) than those without CKD (DM, 12.7%; HTN, 32%) (19). In a case-matched Taiwanese cohort of 26,460 patients, those diagnosed with renal cell carcinoma had greater burden of DM (19.6%) and HTN (30.6%) compared with case-matched controls (DM, 7.7%; HTN, 14%) without renal cell carcinoma (10). CKD risk factors Predictive factors for new-onset CKD or progression of CKD after therapeutic intervention include older age, male sex (6), tobacco use (27), obesity (27,28), and concomitant DM or 2 Onco-Nephrology Curriculum HTN, which are reflective of the predominant features defining the renal mass cohort, and also include lower baseline estimated GFR and larger tumor size (10,11,20). Hypoalbuminemia (19) and postoperative AKI (29) are other likely determinants of GFR decline (Table 1). With tumor resection, CKD prevalence increased anywhere from 10%–24% to 16%–52% after treatment (18–20). Others reported a mean GFR decrease of 13 mL/min per 1.73 m2, corresponding to a 30% drop in GFR after partial nephrectomy (PN), and renal volume reduction seemed to be a prognostic factor for GFR decline (30). Furthermore, among diabetics, 60% developed CKD compared with only 43% of the entire cohort; the 2-year probability of absence of CKD was poor among patients with diabetes (47%) in contrast to those without DM (76%, P 5 0.006) (20). Incidence of ESRD was 5.6 times greater among renal cell carcinoma patients (4.05%) than for a comparable control group (0.68%) (10). In the US Renal Data System (USRDS), renal cell carcinoma has been reported as a cause for ESRD in 0.5% of the 360,000 patients, with a higher mortality compared with other causes of ESRD (28). MANAGEMENT Preoperative evaluation Historically, small renal mass identification by imaging studies nearly always led to surgical intervention with the possible exception of an angiomyolipoma, which can be suspected when there is a significant component of adipose tissue. However, active surveillance and percutaneous kidney biopsies are viable options that are increasingly utilized, as up to 23% of patients will have benign small renal masses (oncocytoma or angiomyolipoma) and can be spared any additional surgery (3,4). The diagnostic rate of kidney biopsies approaches 80% in experienced centers (31), and the concordance rate approaches 100% compared with the surgical resection specimen (32). Preoperative evaluation of potentially modifiable risk factors including DM, HTN, and CKD may play a role in the preservation of renal function. Optimizing glycemic and BP control, as well as estimation of GFR and prevention of AKI, may minimize risk for deterioration of GFR postoperatively. Prevention of AKI can be achieved through proper medical management to avoid nephrotoxic exposure and renal Table 1. Risk factors for CKD Older age (.65 years) Male sex Tobacco use Comorbid diseases Diabetes mellitus Hypertension Obesity Lower baseline GFR Larger tumor size Surgical procedure: radical nephrectomy Postoperative AKI American Society of Nephrology hypoperfusion (33). Renal nuclear scintigraphy has been used to help determine proportional GFR of each kidney to better assess the potential impact of renal resection (partial or radical nephrectomy). Preoperative proportional GFR assessment used to calculate the expected postoperative GFR tended to underestimate the actual postoperative GFR by 12% in one study (34), presumably due to compensatory hyperfiltration and hypertrophy. Additionally, one study examining pre- and postoperative differential function with nuclear scintigraphy found that postsurgical differential function inversely correlated best with ischemia time and tumor size, which may be more predictive of intraoperative renal damage (35). Evaluation of differential function remains a useful tool in assessing operative risk for CKD, providing we recognize these limitations of total GFR underestimation (due to hyperfiltration by the preserved kidney) and relative GFR overestimation in the surgical kidney. Treatment of small renal masses and outcomes RN has been the mainstay of therapy for generations and remains an essential therapy for those with larger renal masses or with lesions extending beyond the affected kidney. Small renal masses have favorable prognosis and survival, which do not necessitate radical nephrectomy. Partial nephrectomy has equivalent/comparable oncologic and overall survival and greater renal preservation (6,36,37). Innumerable studies have emerged examining these outcomes over the last couple of decades with similar findings and may be best illustrated in a meta-analysis performed by Kim et al. Risk reduction with nephron-sparing surgery assessed from 36 studies (40,000 patients; 31,000 RN and 9,300 PN) was 19% for all-cause mortality, 29% for cancer specific mortality, and 61% for CKD (37). On the contrary, the EORTC study, which was a clinical trial of 541 patients with solitary unilateral small renal masses (#5 cm), found that overall 10-year survival was slightly higher for RN (81.1%) than nephron-sparing surgery (75.7%), with a hazard ratio (HR) of 1.5 (95% CI, 1.03–2.16). The small difference in mortality was no longer present when only patients diagnosed with renal cell carcinoma were considered. Progression of disease and renal cell carcinoma death were no different between the two therapies (8). In the same EORTC study, risk of CKD was retrospectively examined and demonstrated that partial nephrectomy (PN) preserved GFR (baseline creatinine , 1.25 times the upper normal range), particularly early on where the difference of patients reaching GFR ,60 mL/min per 1.73 m2 between RN (85.7%) and PN (64.7%) was 21%. However, over time, this difference of progressive decline to lower GFRs ,30 (RN 10%, PN 6.3%) and ,15 mL/min per 1.73 m2 (RN 1.5%, PN 1.6%) became insignificant. The steeper GFR decline observed initially with RN was not associated with an increase in mortality (36), potentially suggesting that GFR loss from nephrectomy did not confer the same risk of death usually seen with GFR decline from traditional causes of CKD such as DM or HTN (9). The cohort studies, however, generally showed a survival American Society of Nephrology advantage in addition to GFR preservation with nephron-sparing surgery (37), which is clearly limited by its retrospective design. The increased mortality seen with CKD has been attributed to high cardiovascular disease risk typically associated with advanced GFR (9). Among 1,004 case-matched patients with relatively small renal mass size (T1b, 4–7 cm), an association of greater cardiovascular death and GFR decline was observed. GFR loss was measured using the difference between extrapolated values of preoperative and (.3 weeks) postoperative GFRs. The average GFR loss was less for those who had partial nephrectomy (16.6 mL/min) as opposed to radical nephrectomy (23.5 mL/min, P ,0.0001). GFR decline generally occurred within 3 weeks and then stabilized thereafter. Each excess loss of GFR of 7 mL/min per 1.73 m2 resulted in a 17% increase risk of death as well as a 25% greater cardiovascular disease risk. Cancer-specific survival was not different, but overall survival was better for partial nephrectomy (85% versus RN 78%, P 5 0.01) (38). The hard end point of ESRD has also been examined in a Taiwanese incident cohort with 10-year follow-up in a newly diagnosed renal cell carcinoma group (n 5 2940) and a control group (n 5 23,520). Progression to ESRD occurred in 4.05% of the renal cell carcinoma group compared with only 0.68% of the control group (HR, 5.63; 95% CI, 4.37–7.24) with the same risk factors (DM, preexisting CKD) for CKD progression (10). In the USRDS, the higher mortality noted with ESRD from renal cell carcinoma compared with other causes was not observed for those who had undergone nephrectomy, suggesting that even though progression to ESRD from renal cell carcinoma could not be prevented, nephrectomy may still confer a lower risk for mortality (28). Alternative therapy for small renal masses in individuals with high operative risk includes nonsurgical ablative therapies such as radiofrequency ablation and cryoablation, which are currently the two most common approaches used. Older patients were more likely to receive radiofrequency ablation (RFA) with fewer major complications (RFA 3.1 % versus PN 7.2%–7.9%, P ,0.001), but with higher local progression of disease (RFA 4.6% versus PN 1.2%–1.9%, P ,0.001) than seen with partial nephrectomy (39). In one series, however, oncologic outcomes were no different when excluding those with high risk for recurrence (40). Cryoablation was also utilized more frequently for older patients with higher operative risk. In addition to fewer procedural complications, shorter length of hospital stay was noted; however, this was also associated with higher local (relative risk [RR], 9.39; P ,0.0001) and metastatic progression of disease (RR, 4.68; P 5 0.01) after cryoablation compared with partial nephrectomy (both performed laparoscopically) (41). Last, active surveillance with judicious monitoring of tumor size and sometimes with delayed surgical intervention resulted in acceptable outcomes particularly among those age .75, which was no worse than surgical resection for select populations (14). Although oncologic outcomes for ablative therapies may not be as favorable as surgical resection, they provide viable therapeutic options Onco-Nephrology Curriculum 3 with less complications and likely greater renal parenchymal (thus also GFR) preservation for nonoperative candidates. c Older age, male sex, comorbid diseases including diabetes mellitus, hypertension, preexisting CKD, and larger tumor size increase risk for postoperative CKD. c Nephron-sparing surgery (partial nephrectomy) among patients with PATHOLOGIC EVALUATION OF TUMOR NEPHRECTOMY The evaluation of tumor nephrectomy specimens has always centered around the renal mass, but careful assessment of the nonneoplastic kidney parenchyma reveals the presence of common yet undiagnosed nonneoplastic renal diseases. Therefore, the synoptic reports established by the College of American Pathologist required in 2010 that the nonneoplastic parenchyma should be evaluated and reported for every renal malignancy (42). Also, the Accreditation Council for Graduate Medical Education will require that nephropathology be part of the curriculum for all anatomic pathology residents effective July 1, 2015, as the vast majority of pathologists do not receive any exposure to this subspecialty. Several large studies found that diabetic nephropathy and hypertensive nephropathy (or arterionephrosclerosis) can be identified in 8%–20% and 3%–14% of specimens, respectively (43–45), and 60%– 88% of these diagnoses were not identified during the initial evaluation. With nearly 350,000 kidney cancer survivors in the United States, the CKD burden will only increase, especially as more attention is given to the nonneoplastic parenchyma examined by pathologists. In addition, there still remains much room for improvement regarding the coordination of urologists, pathologists, and especially nephrologists in the preoperative and postoperative management of kidney cancer patients. CONCLUSIONS The majority of renal tumors are small renal tumors discovered on routine imaging with excellent oncologic and overall survival. The prolonged survival with earlier discovery has resulted in higher likelihood of nononcologic death, where patients are saddled instead with CKD and associated increased cardiovascular morbidity and mortality. Minimally invasive techniques for diagnosis and nephron-sparing surgery have minimized nephron mass and functional loss. Recognizing and assessing modifiable risk factors for CKD such as HTN, DM, and cardiovascular disease may potentially allow for greater preservation of GFR and reduction of cardiovascular disease– related death. To achieve this goal, communication and coordination of management is essential within the specialty care team, which is comprised of nephrologists, oncologists, urologists, and pathologists. TAKE HOME POINTS c Early diagnosis of small renal tumors is rising due to incidental discovery with favorable prognosis. c New-onset CKD is fairly common after nephrectomy with overlapping risk factors for CKD and renal cell carcinoma. 4 Onco-Nephrology Curriculum small renal masses has equivalent cancer-specific survival, practically similar overall survival, and better GFR preservation compared with radical nephrectomy. REFERENCES 1. Patard JJ, Shvarts O, Lam JS, Pantuck AJ, Kim HL, Ficarra V, Cindolo L, Han KR, De La Taille A, Tostain J, Artibani W, Abbou CC, Lobel B, Chopin DK, Figlin RA, Mulders PF, Belldegrun AS. Safety and efficacy of partial nephrectomy for all T1 tumors based on an international multicenter experience. J Urol 171: 2181–2185, quiz 2435, 2004 2. Chow WH, Devesa SS, Warren JL, Fraumeni JF Jr. Rising incidence of renal cell cancer in the United States. JAMA 281: 1628–1631, 1999 3. McKiernan J, Yossepowitch O, Kattan MW, Simmons R, Motzer RJ, Reuter VE, Russo P. Partial nephrectomy for renal cortical tumors: pathologic findings and impact on outcome. Urology 60: 1003–1009, 2002 4. Kutikov A, Fossett LK, Ramchandani P, Tomaszewski JE, Siegelman ES, Banner MP, Van Arsdalen KN, Wein AJ, Malkowicz SB. Incidence of benign pathologic findings at partial nephrectomy for solitary renal mass presumed to be renal cell carcinoma on preoperative imaging. Urology 68: 737–740, 2006 5. Volpe A, Kachura JR, Geddie WR, Evans AJ, Gharajeh A, Saravanan A, Jewett MA. Techniques, safety and accuracy of sampling of renal tumors by fine needle aspiration and core biopsy. J Urol 178: 379–386, 2007 6. Li L, Lau WL, Rhee CM, Harley K, Kovesdy CP, Sim JJ, Jacobsen S, Chang A, Landman J, Kalantar-Zadeh K. Risk of chronic kidney disease after cancer nephrectomy. Nat Rev Nephrol 10: 135–145, 2014 7. Jewett MAS, Mattar K, Basiuk J, Morash CG, Pautler SE, Siemens DR, Tanguay S, Rendon RA, Gleave ME, Drachenberg DE, Chow R, Chung H, Chin JL, Fleshner NE, Evans AJ, Gallie BL, Haider MA, Kachura JR, Kurban G, Fernandes K, Finelli A. Active surveillance of small renal masses: progression patterns of early stage kidney cancer. Eur Urol 60: 39–44, 2011 8. Van Poppel H, Da Pozzo L, Albrecht W, Matveev V, Bono A, Borkowski A, Colombel M, Klotz L, Skinner E, Keane T, Marreaud S, Collette S, Sylvester R. A prospective, randomised EORTC intergroup phase 3 study comparing the oncologic outcome of elective nephron-sparing surgery and radical nephrectomy for low-stage renal cell carcinoma. Eur Urol 59: 543–552, 2011 9. Go AS, Chertow GM, Fan D, McCulloch CE, Hsu CY. Chronic kidney disease and the risks of death, cardiovascular events, and hospitalization. N Engl J Med 351: 1296–1305, 2004 10. Hung PH, Tsai HB, Hung KY, Muo CH, Chung MC, Chang CH, Chung CJ. Increased risk of end-stage renal disease in patients with renal cell carcinoma: A 12-year nationwide follow-up study. Medicine (Baltimore) 93: e52, 2014 11. Jeon HG, Choo SH, Sung HH, Jeong BC, Seo SI, Jeon SS, Choi HY, Lee HM. Small tumour size is associated with new-onset chronic kidney disease after radical nephrectomy in patients with renal cell carcinoma. Eur J Cancer 50: 64–69, 2014 12. Crépel M, Jeldres C, Sun M, Lughezzani G, Isbarn H, Alasker A, Capitanio U, Shariat SF, Arjane P, Widmer H, Graefen M, Montorsi F, Perrotte P, Karakiewicz PI. A population-based comparison of cancercontrol rates between radical and partial nephrectomy for T1A renal cell carcinoma. Urology 76: 883–888, 2010 13. Lane BR, Campbell SC, Gill IS. 10-year oncologic outcomes after laparoscopic and open partial nephrectomy. J Urol 190: 44–49, 2013 14. Sun M, Becker A, Tian Z, Roghmann F, Abdollah F, Larouche A, Karakiewicz PI, Trinh QD. Management of localized kidney cancer: American Society of Nephrology 15. 16. 17. 18. 19. 20. 21. 22. 23. 24. 25. 26. 27. 28. 29. 30. Calculating cancer-specific mortality and competing risks of death for surgery and nonsurgical management. Eur Urol 65: 235–241, 2014 Chow W-H, Dong LM, Devesa SS. Epidemiology and risk factors for kidney cancer. Nat Rev Urol 7: 245–257, 2010 Denton MD, Magee CC, Ovuworie C, Mauiyyedi S, Pascual M, Colvin RB, Cosimi AB, Tolkoff-Rubin N. Prevalence of renal cell carcinoma in patients with ESRD pre-transplantation: A pathologic analysis. Kidney Int 61: 2201–2209, 2002 Lowrance WT, Ordoñez J, Udaltsova N, Russo P, Go AS. CKD and the risk of incident cancer. J Am Soc Nephrol 25: 2327–2334, 2014 Barlow LJ, Korets R, Laudano M, Benson M, McKiernan J. Predicting renal functional outcomes after surgery for renal cortical tumours: A multifactorial analysis. BJU Int 106: 489–492, 2010 Kim SH, Lee SE, Hong SK, Jeong CW, Park YH, Kim YJ, Kang SH, Hong SH, Choi WS, Byun SS. Incidence and risk factors of chronic kidney disease in Korean patients with t1a renal cell carcinoma before and after radical or partial nephrectomy. Jpn J Clin Oncol 43: 1243–1248, 2013 Jeon HG, Jeong IG, Lee JW, Lee SE, Lee E. Prognostic factors for chronic kidney disease after curative surgery in patients with small renal tumors. Urology 74: 1064–1068, 2009 Huang WC, Levey AS, Serio AM, Snyder M, Vickers AJ, Raj GV, Scardino PT, Russo P. Chronic kidney disease after nephrectomy in patients with renal cortical tumours: A retrospective cohort study. Lancet Oncol 7: 735–740, 2006 Kaushik D, Kim SP, Childs MA, Lohse CM, Costello BA, Cheville JC, Boorjian SA, Leibovich BC, Thompson RH. Overall survival and development of stage IV chronic kidney disease in patients undergoing partial and radical nephrectomy for benign renal tumors. Eur Urol 64: 600–606, 2013 Canter D, Kutikov A, Sirohi M, Street R, Viterbo R, Chen DY, Greenberg RE, Uzzo RG. Prevalence of baseline chronic kidney disease in patients presenting with solid renal tumors. Urology 77: 781–785, 2011 Clark MA, Shikanov S, Raman JD, Smith B, Kaag M, Russo P, Wheat JC, Wolf JS Jr, Matin SF, Huang WC, Shalhav AL, Eggener SE. Chronic kidney disease before and after partial nephrectomy. J Urol 185: 43–48, 2011 Choi YS, Park YH, Kim YJ, Kang SH, Byun SS, Hong SH. Predictive factors for the development of chronic renal insufficiency after renal surgery: A multicenter study. Int Urol Nephrol 46: 681–686, 2014 Takagi T, Kondo T, Iizuka J, Kobayashi H, Hashimoto Y, Nakazawa H, Ito F, Tanabe K. Postoperative renal function after partial nephrectomy for renal cell carcinoma in patients with pre-existing chronic kidney disease: A comparison with radical nephrectomy. Int J Urol 18: 472–476, 2011 Malcolm JB, Bagrodia A, Derweesh IH, Mehrazin R, Diblasio CJ, Wake RW, Wan JY, Patterson AL. Comparison of rates and risk factors for developing chronic renal insufficiency, proteinuria and metabolic acidosis after radical or partial nephrectomy. BJU Int 104: 476–481, 2009 Stiles KP, Moffatt MJ, Agodoa LY, Swanson SJ, Abbott KC. Renal cell carcinoma as a cause of end-stage renal disease in the United States: Patient characteristics and survival. Kidney Int 64: 247–253, 2003 Cho A, Lee JE, Kwon G-Y, Huh W, Lee HM, Kim YG, Kim DJ, Oh HY, Choi HY. Post-operative acute kidney injury in patients with renal cell carcinoma is a potent risk factor for new-onset chronic kidney disease after radical nephrectomy. Nephrol Dial Transplant 26: 3496–3501, 2011 Song C, Bang JK, Park HK, Ahn H. Factors influencing renal function reduction after partial nephrectomy. J Urol 181: 48–53, discussion 53–54, 2009 American Society of Nephrology 31. Volpe A, Finelli A, Gill IS, Jewett MA, Martignoni G, Polascik TJ, Remzi M, Uzzo RG. Rationale for percutaneous biopsy and histologic characterisation of renal tumours. Eur Urol 62: 491–504, 2012 32. Volpe A, Mattar K, Finelli A, Kachura JR, Evans AJ, Geddie WR, Jewett MA. Contemporary results of percutaneous biopsy of 100 small renal masses: A single center experience. J Urol 180: 2333–2337, 2008 33. Thadhani R, Pascual M, Bonventre JV. Acute renal failure. N Engl J Med 334: 1448–1460, 1996 34. Bachrach L, Negron E, Liu JS, Su YK, Paparello JJ, Eggener S, Kundu SD. Preoperative nuclear renal scan underestimates renal function after radical nephrectomy. Urology 84: 1402–1406, 2014 35. Sankin A, Sfakianos JP, Schiff J, Sjoberg D, Coleman JA. Assessing renal function after partial nephrectomy using renal nuclear scintigraphy and estimated glomerular filtration rate. Urology 80: 343–346, 2012 36. Scosyrev E, Messing EM, Sylvester R, Campbell S, Van Poppel H. Renal function after nephron-sparing surgery versus radical nephrectomy: Results from EORTC randomized trial 30904. Eur Urol 65: 372–377, 2014 37. Kim SP, Thompson RH, Boorjian SA, Weight CJ, Han LC, Murad MH, Shippee ND, Erwin PJ, Costello BA, Chow GK, Leibovich BC. Comparative effectiveness for survival and renal function of partial and radical nephrectomy for localized renal tumors: A systematic review and meta-analysis. J Urol 188: 51–57, 2012 38. Weight CJ, Larson BT, Fergany AF, Gao T, Lane BR, Campbell SC, Kaouk JH, Klein EA, Novick AC. Nephrectomy induced chronic renal insufficiency is associated with increased risk of cardiovascular death and death from any cause in patients with localized cT1b renal masses. J Urol 183: 1317–1323, 2010 39. Wang S, Qin C, Peng Z, Cao Q, Li P, Shao P, Ju X, Meng X, Lu Q, Li J, Wang M, Zhang Z, Gu M, Zhang W, Yin C. Radiofrequency ablation versus partial nephrectomy for the treatment of clinical stage 1 renal masses: A systematic review and meta-analysis. Chin Med J (Engl) 127: 2497–2503, 2014 40. Olweny EO, Park SK, Tan YK, Best SL, Trimmer C, Cadeddu JA. Radiofrequency ablation versus partial nephrectomy in patients with solitary clinical T1a renal cell carcinoma: Comparable oncologic outcomes at a minimum of 5 years of follow-up. Eur Urol 61: 1156–1161, 2012 41. Klatte T, Shariat SF, Remzi M. Systematic review and meta-analysis of perioperative and oncologic outcomes of laparoscopic cryoablation versus laparoscopic partial nephrectomy for the treatment of small renal tumors. J Urol 191: 1209–1217, 2014 42. Srigley JR, Amin MB, Delahunt B, Campbell SC, Chang A, Grignon DJ, Humphrey PA, Leibovich BC, Montironi R, Renshaw AA, Reuter VE; Members of the Cancer Committee, College of American Pathologists. Protocol for the examination of specimens from patients with invasive carcinoma of renal tubular origin. Arch Pathol Lab Med 134: e25–e30, 2010 43. Bijol V, Mendez GP, Hurwitz S, Rennke HG, Nosé V. Evaluation of the nonneoplastic pathology in tumor nephrectomy specimens: predicting the risk of progressive renal failure. Am J Surg Pathol 30: 575–584, 2006 44. Salvatore SP, Cha EK, Rosoff JS, Seshan SV. Nonneoplastic renal cortical scarring at tumor nephrectomy predicts decline in kidney function. Arch Pathol Lab Med 137: 531–540, 2013 45. Henriksen KJ, Meehan SM, Chang A. Non-neoplastic renal diseases are often unrecognized in adult tumor nephrectomy specimens: A review of 246 cases. Am J Surg Pathol 31: 1703–1708, 2007 Onco-Nephrology Curriculum 5 REVIEW QUESTIONS 1. What are the common risk factors for both CKD and renal cell carcinoma (RCC)? a. b. c. d. Obesity Diabetes mellitus Hypertension All of the above Answer: d is correct. Diabetes, hypertension, and obesity are independent risk factors for renal cell carcinoma. Diabetes and hypertension can be found in 25% and up to 60% of kidney cancer patients, respectively. These are also the two most common causes of ESRD. Given the link between CKD and RCC, it is not surprising to find these common overlapping risk factors. Obesity is a lesser but also significant risk factor common to both CKD and RCC. 2. Which of the following is the most common pathologic finding in tumor nephrectomy specimens? a. b. c. d. e. Diabetic nephropathy Membranous nephropathy IgA nephropathy Focal segmental glomerulosclerosis Minimal change disease Answer: a is correct. Diabetes is an independent risk factor for RCC and is found in up to 25% of kidney cancer patients. Therefore, diabetic nephropathy is common and can be identified in 8%–20% of tumor nephrectomy specimens (depending on the definition of diabetic nephropathy). IgA 6 Onco-Nephrology Curriculum nephropathy occurs in ,2% of specimens. Amyloidosis occurred in 3% of specimens according to a study in the late 1980s, but is much less common due to the significant stage migration that has occurred toward smaller neoplasms as a result of early detection. Membranous nephropathy and pauci-immune crescentic glomerulonephritis rarely occur in the setting of kidney cancer, with only case reports being available in the literature. 3. Which of the following statements is true regarding outcomes in the management of small renal masses? a. Cancer-specific survival and overall survival is superior with radical nephrectomy, which should be considered primarily in this population b. Partial nephrectomy is associated with GFR preservation and comparable cancer-specific survival and overall survival to that of radical nephrectomy c. Ablative therapies are associated with increased treatmentrelated complication rates but less disease progression d. Active surveillance is not indicated for management of small renal masses Answer: b is correct. Cancer-specific survival and overall survival are equivalent between partial and radical nephrectomy. GFR preservation is greater with partial nephrectomy. For small renal masses, nephron-sparing procedure should be considered first. Ablative therapies have worse oncologic outcomes but lower treatment-related complications. Active surveillance in select populations such as older poor operative candidates have been shown to have comparable outcomes. American Society of Nephrology Chapter 16: Cancer in Solid Organ Transplantation Mona D. Doshi, MD Department of Medicine, Wayne State University, Detroit, Michigan INTRODUCTION Solid organ transplantation provides lifesaving therapy for patients with end-organ disease. The Scientific Registry of Transplant Recipients (SRTR) report announced that 17,654 kidney, 6455 liver, 1946 lung, 2554 heart, and 109 intestinal transplants were performed in 2013, superseding the number of transplants from prior years (1). It also reports continual improvement in death-censored graft survival (2,3). Success of field of transplantation is reflected in the rising number and longevity of the transplant organs. However, the same report also noted an increase in recipient death with graft function, primarily due to increased infections and cancers associated with chronic immunosuppression. Malignancy is the second leading cause of death with graft function (4). The risk of cancer is twoto four-fold higher in transplant recipients than age-, sex-, and race-matched individuals from similar geographic areas (5,6). Not only are cancers common, but they tend to be more aggressive and are associated with increased mortality among transplant recipients than in the general population. The relative risk varies by age. Children have the highest increase (15–30 times), and older individuals (i.e., .65 years of age) experience a two-fold increase. The magnitude of increase in risk for all cancer types is similar across organ recipients; the incidence of specific cancer varies by transplanted organ. Knowledge of cancer types, including the magnitude of increased risk and its clinical course, can help develop prevention and early detection protocols and prompt management (including adjustment of immunosuppression) to minimize cancer related deaths. COMMON CANCER TYPES IN TRANSPLANT RECIPIENTS The incidence of cancer is highest for malignancies related to viral infections, including non-Hodgkin American Society of Nephrology lymphoma (NHL; Epstein-Barr virus [EBV]), Kaposi sarcoma (human herpes virus-8 [HHV8]), liver (hepatitis C virus [HCV] and hepatitis B virus [HBV]), and anogenital cancers (human papilloma virus [HPV]). The increased risk of these cancers is believed to be related to impaired immune control of these oncogenic viruses, which can be present in the recipient prior to transplant or transmitted at time of transplant via the donor organ. This notion is supported by similar amplified risks of these cancers in individuals with compromised immune systems, i.e., HIV/AIDS, and reversal of this heightened risk on withdrawal of immunosuppression, i.e., kidney graft failure and reinitiating of dialysis (7). Independent of immunosuppressive effects, the antirejection medications such as azathioprine and cyclosporine directly enhance the carcinogenic effects of ultraviolet (UV) radiation via inhibiting DNA repair and resulting in apoptosis of keratinocyte (8,9). Certain malignancies with no obvious infectious causes are also reported to be elevated, i.e., colon, bladder, lip, kidney, and thyroid cancer. These cancers have been reported to occur at a greater frequency in patients with kidney disease prior to receiving a kidney transplant but do not occur frequently in people with HIV/AIDS, confirming the lack of a role of the immune system. Instead, they may be occurring due to underlying renal or bladder disease, loss of kidney function, and/ or malignant transformation of acquired cystic kidney disease (common in individuals with renal failure) (6). Traditional risk factors such as smoking and alcohol intake leading to organ failure continue to play a role in development of cancer, i.e., cancer in the native lung for those receiving a solitary lung transplant for chronic obstructive pulmonary disease, liver cancer in alcoholics, and colon cancer in Correspondence: Department of Medicine, Wayne State University, Harper Professional Office Building, 4160 John R, Suite 908, Detroit, Michigan 48201. Copyright © 2016 by the American Society of Nephrology Onco-Nephrology Curriculum 1 patients with ulcerative colitis requiring a liver transplant due to primary sclerosing cholangitis. There are a few case reports of malignancy transmitted via donor organ. To summarize, both immunosuppression and host-related factors play an important role in the increased risk of cancer in transplant recipients. Table 1 lists the names of common malignancies with their incidence rates after transplantation. Of note, the cancer risk of nasopharynx, cervix, prostate, breast, brain, and chronic lymphocytic leukemia are reported to be lower in transplant recipients than in the general population (5). Similar observations have been made in people with HIV infection, together suggesting that the immune system may not be primarily controlling the development and growth of these cancers (10). Alternatively, this observation can also be explained by aggressive screening for cervical, prostate, and breast cancer in the transplant population, leading to prompt treatment of precancerous lesions prior to or after transplant. CLINICAL COURSE OF COMMON CANCERS Non-melanoma skin cancer Non-melanoma skin cancer is the most common malignancy in adult white solid organ transplant recipients. Squamous and basal cell carcinoma account for .90% of all skin cancers. Unlike the general population, squamous cell carcinoma is the most common skin cancer. It occurs 250 times as frequently as that seen in the general population, whereas the risk of basal cell cancer is increased by 10-fold. Thus, the ratio of squamous cell to basal cell cancer in patients without transplant (1:4) is reversed in transplant patients (4:1). A third of patients will have both types of skin cancer. The risk factors for developing skin cancer are as follows: 1) recipient related: history of skin cancer prior to transplant, the presence of premalignant skin lesions (warts or keratosis), exposure to solar UV rays, location of residence (highest incidence in Australia), older age, male sex, and fair skin phenotype; 2) immunosuppression related: duration and type (mainly azathioprine and cyclosporine); and 3) infection related: keratinocytes of transplant recipients are more likely to be infected with HPV than nontransplant people. The Table 1. Common malignancies and incidence rates after transplant in United States Cancer type Incidence/10,000 person-years SIR (95% CI) Skin cancer Kaposi sarcoma PTLD Lung Liver Kidney 23.7 15.5 194.0 173.4 120 97 13.85 (11.92–16.00) 61.46 (50.95–73.49) 7.54 (7.17–7.93) 1.97 (1.86–2.08) 11.56 (10.83–12.33) 4.65 (432–4.99) Data were obtained from reference 5. SIR, standardized incidence ratio (observed/ expected cases). 2 Onco-Nephrology Curriculum appearance and distribution of cancer depends on recipient age. The older recipients are likely to have their first lesion within 3 years of transplant and it will typically develop on their head, whereas in younger folks, it occurs later (typically after 8 years), and lesions are located on dorsum of their hands. Squamous cell cancers in transplant recipients are also more aggressive, especially when poorly differentiated on histology. The treatment of skin cancers depends on the type of lesion and its extent. Superficial lesions can be managed with cryotherapy while deeper lesions require excision with clean margins. Last, changing immunosuppression to a mammalian target of rapamycin inhibitor (mTORi)-based regimen has been shown to decrease the risk of recurrent cancers (11). Kaposi sarcoma Kaposi sarcoma occurs 80–500 times more frequently in transplant recipients than in non-immunosuppressed populations. In addition, it tends to be more aggressive and multicentric with visceral involvement. Most cases occur due to infection with HHV-8, and therefore, are commonly seen in recipients from a high sero-prevalence area, i.e., Mediterranean and African regions. It is also more common in men, with a male to female ratio of 3:1, and often occurs within the first year of transplant. Ninety percent of them present as cutaneous lesions on the legs or mucosal angiomatous lesions. Visceral involvement commonly occurs in heart and liver transplant recipients. The mainstay of treatment is reduction of immunosuppression, but this may lead to graft dysfunction (12). Non-Hodgkin lymphoma NHL, which is more commonly referred to as post-transplant lymphoproliferative disorders (PTLD), occurs seven to eight times more frequently than in the general population (standardized incidence ratio [SIR], 7.54; 95% CI, 7.17–7.93). It occurs more commonly in young (0–34 years) and older ($50 years) male recipients. The incidence is highest in lung and heart recipients and lowest in kidney transplant recipients, possibly due to varying transfer of lymphoid tissue during organ transplant and intensity of immunosuppression. Its occurrence is associated with the use of T cell–depleting agents and a mycophenolic acid–based antirejection regimen. Data on the risk of PTLD with use of tacrolimus are equivocal. Use of cyclosporine and azathioprine is not associated with increased risk of lymphoma. It commonly presents within the first year of transplant or 5 years after transplant. Early-onset lymphoma is related to primary EBV infection, and late-onset lymphoma is independent of infection. Its pathology ranges from benign hyperplasia to lymphoid malignancy. PTLD differs from lymphoma in the general population not only in histopathologic findings, but it is also associated with increased extranodal involvement, predominant occurrence in the transplanted organ, an aggressive clinical course, and poor outcomes. The 5-year survival was 41% and did not vary based on time of presentation. The mortality was higher in heart transplant recipients than in kidney transplant recipients American Society of Nephrology due to inability to withdraw/cease immunosuppression. In addition to conventional therapy, the mainstay of treatment includes reduction in immunosuppression, especially antiproliferative agents, and use of antiviral agents in those with primary EBV infections (13,14). Lung cancer Risk of lung cancer in transplant recipients is moderately increased compared with that in the general population (SIR, 6.13; 95% CI, 5.18–7.21). It is common in older male transplant recipients. It is more common in lung transplant recipients, with the highest risk occurring in the first 6 months of transplant (SIR, 11.17; 95% CI, 7.48–16.04) and falling to a five-fold greater risk thereafter. The elevated early risk of cancer may be due to cancer in the explanted lung (15). A novel study of single-lung versus bilateral-lung transplant recipients matched for underlying disease, smoking history, and age reported a five-fold increase in cancer among those with single-lung transplants, where the primary cancer was noted in the lung of the recipient (16). In addition to smoking and chronic immunosuppression, chronic inflammation and repeated infections may be playing a role in development of lung cancer in native lung (17). Recipients of other organs had smaller elevations in their risk, and its occurrence increased with time. Liver cancer Risk of liver cancer was strongly elevated in liver transplant recipients compared with the general population (SIR, 43.83; 95% CI, 40.90–46.91). Ninety-five percent of liver cancers were diagnosed within the first 6 months after transplant. Like the lung, the increased incidence of cancer within 6 months of transplant may be due to delayed recognition of cancer in the explanted liver. Thereafter, the risk of liver cancer was two-fold higher than the general population. These late-onset liver cancers may be due to recurrent disease related to HCV or HBV. Liver cancer risk is not increased among other organ recipients (5). Kidney cancer The risk of kidney cancer was highest in kidney transplant recipients (SIR, 6.66; 95% CI, 1.57–3.04), but was also elevated in liver and heart recipients. Among all recipients, kidney cancer occurs mainly in older men and had a bimodal pattern of presentation. It occurred within the first 6 months, and a second peak was seen 4–15 years after transplant. Some of the early cases can be explained by malignant transformation of the cysts that develop in patients with ESRD prior to transplant (6). As mentioned previously, the risk of renal cancer is already high among patients with CKD and continues to remain high after transplant (18). for routine age-appropriate screening, as that in general population, is recommended for all (Table 2). Annual instead of biannual pap testing is recommended to detect precancerous lesions that may progress faster to cancer under influence of immunosuppression. There are no data on vaccinating transplant recipients who are HPV naïve. Annual mammograms are also recommended for all women over age 50. The patients should be counseled about higher incidence of false-positive findings (calcification and increased density of breast with chronic steroid use), resulting in increased interventions. In addition, recipients should also be screened for colorectal cancers with yearly fecal occult blood testing and flex sigmoidoscopy or colonoscopy every 5 years. Of note, most of these practices have not been validated in a transplant cohort (19). In the absence of evidence, an individualized approach to screening should be used based on the individual’s cancer risk, existing comorbidities, overall life expectancy, and preference for screening. Skin cancer may be prevented by using sunscreen (SPF 115) and sun hats, avoiding sun peak hours, and covering up the exposed skin with long sleeves. Annual follow-up with an experienced dermatologist for total body skin examination is also advocated for those at high risk. Systemic retinoid should be avoided, and topical retinoid treatments can be tried to treat dysplastic lesions but with caution due to fear of increased risk of rejection. Those with repeated precancerous skin lesions can be counseled to switch to an mTOR inhibitor-based immunosuppressive regimen. Routine screening for renal cancer is not recommended (19). In addition to traditional therapy, reduction in immunosuppression is often recommended. The underlying idea is that this allows immune reconstitution and control of the malignancy by the recipient’s recovering immune system. If Table 2. Common cancers and recommendations for screening Cancer type Breast Colorectal Cervical Prostate Liver Skin SURVEILLANCE AND MANAGEMENT Kidney In view of the higher cancer incidence and poorer prognosis, prevention and screening play an important role. Surveillance American Society of Nephrology Recommendations for screening Annual or biennial mammography for all women older than 50 years; for women between 40 and 49 years, no evidence for or against screening Annual fecal occult blood testing and/or 5-year flexible sigmoidoscopy or colonoscopy for individuals .50 years Annual pap and pelvic examination once sexually active Annual digital rectal examination and PSA in all males after age 50 years a-Fetoprotein and liver ultrasound every 6 months in high-risk individuals, i.e., HBV or HCV infection, but no firm data Monthly self-examination and total body skin examination every 6–12 months by an expert skin physician No firm recommendation, but some have suggested regular ultrasound of native kidneys PSA, prostate-specific antigen. Adapted from reference 24. Onco-Nephrology Curriculum 3 immunosuppression is stopped or lowered, particularly early after transplantation, graft monitoring at short intervals is necessary. Successful reduction or cessation of immunosuppression was reported in transplanted patients who developed NHL and Kaposi sarcoma (20). Use of mTORi has been shown to reduce risk of new squamous cell cancer in patients with a prior history of skin cancer (11,21) and are also very effective in treating Kaposi sarcoma (22). However, tolerability of mTORi is poor and is associated with a 35% discontinuation rate. Although there are strong data favoring the use of an mTORi-based regimen in those with skin cancers and Kaposi sarcoma, there are insufficient data for solid organ cancers (23). In summary, cancer remains a leading cause of morbidity and mortality in transplant recipients. Routine surveillance and early detection with prompt intervention directed at cancer and immunosuppression are recommended to improve the life of the recipient and their transplant organ. TAKE HOME POINTS 8. 9. 10. 11. 12. 13. 14. c Cancer risk is increased by two- to four-fold in transplant recipients and tends to be more aggressive than age-, sex-, and race-matched individuals from the general population. 15. c Skin cancer is the most common cancer, followed by PTLD and cancer of the transplanted organ. c Emphasis should be placed on adherence to recommended cancer 16. surveillance protocols for early detection and prompt management. c Management of cancer developing after transplant includes reduction of immunosuppression and switching to an mTOR inhibitor–based regimen for those with skin cancers. REFERENCES 1. OPTN/SRTR. 2012 annual data report. Introduction. Am J Transplant 14(Suppl 1): 8–10, 2014 2. Matas AJ, Smith JM, Skeans MA, Thompson B, Gustafson SK, Stewart DE, Cherikh WS, Wainright JL, Boyle G, Snyder JJ, Israni AK, Kasiske BL. OPTN/SRTR 2013 annual data report: Kidney. Am J Transplant 15 (Suppl 2): 1–34, 2015 3. Kim WR, Lake JR, Smith JM, Skeans MA, Schladt DP, Edwards EB, Harper AM, Wainright JL, Snyder JJ, Israni AK, Kasiske BL. OPTN/SRTR 2013 Annual Data Report: Liver. Am J Transplant 15(Suppl 2): 1–28, 2015 4. Watt KD, Pedersen RA, Kremers WK, Heimbach JK, Charlton MR. Evolution of causes and risk factors for mortality post-liver transplant: Results of the NIDDK long-term follow-up study. Am J Transplant 10: 1420–1427, 2010 5. Engels EA, Pfeiffer RM, Fraumeni JF Jr, Kasiske BL, Israni AK, Snyder JJ, Wolfe RA, Goodrich NP, Bayakly AR, Clarke CA, Copeland G, Finch JL, Fleissner ML, Goodman MT, Kahn A, Koch L, Lynch CF, Madeleine MM, Pawlish K, Rao C, Williams MA, Castenson D, Curry M, Parsons R, Fant G, Lin M. Spectrum of cancer risk among US solid organ transplant recipients. JAMA 306: 1891–1901, 2011 6. Maisonneuve P, Agodoa L, Gellert R, Stewart JH, Buccianti G, Lowenfels AB, Wolfe RA, Jones E, Disney AP, Briggs D, McCredie M, Boyle P. Cancer in patients on dialysis for end-stage renal disease: An international collaborative study. Lancet 354: 93–99, 1999 7. van Leeuwen MT, Grulich AE, McDonald SP, McCredie MR, Amin J, Stewart JH, Webster AC, Chapman JR, Vajdic CM. Immunosuppression 4 Onco-Nephrology Curriculum 17. 18. 19. 20. 21. 22. 23. 24. and other risk factors for lip cancer after kidney transplantation. Cancer Epidemiol Biomarkers Prev 18: 561–569, 2009 Perrett CM, Walker SL, O’Donovan P, Warwick J, Harwood CA, Karran P, McGregor JM. Azathioprine treatment photosensitizes human skin to ultraviolet A radiation. Br J Dermatol 159: 198–204, 2008 Yarosh DB, Pena AV, Nay SL, Canning MT, Brown DA. Calcineurin inhibitors decrease DNA repair and apoptosis in human keratinocytes following ultraviolet B irradiation. J Invest Dermatol 125: 1020–1025, 2005 Grulich AE, van Leeuwen MT, Falster MO, Vajdic CM. Incidence of cancers in people with HIV/AIDS compared with immunosuppressed transplant recipients: A meta-analysis. Lancet 370: 59–67, 2007 Campbell SB, Walker R, Tai SS, Jiang Q, Russ GR. Randomized controlled trial of sirolimus for renal transplant recipients at high risk for nonmelanoma skin cancer. Am J Transplant 12: 1146–1156, 2012 Penn I. Kaposi’s sarcoma in transplant recipients. Transplantation 64: 669–673, 1997 Parker A, Bowles K, Bradley JA, Emery V, Featherstone C, Gupte G, Marcus R, Parameshwar J, Ramsay A, Newstead C; Haemato-oncology Task Force of the British Committee for Standards in Haematology and British Transplantation Society. Management of post-transplant lymphoproliferative disorder in adult solid organ transplant recipients— BCSH and BTS Guidelines. Br J Haematol 149: 693–705, 2010 Pirsch JD, Stratta RJ, Sollinger HW, Hafez GR, D’Alessandro AM, Kalayoglu M, Belzer FO. Treatment of severe Epstein-Barr virusinduced lymphoproliferative syndrome with ganciclovir: Two cases after solid organ transplantation. Am J Med 86: 241–244, 1989 Ritchie AJ, Mussa S, Sivasothy P, Stewart S. Single-lung transplant complicated by unexpected explant carcinoma: A management dilemma. J Heart Lung Transplant 26: 1206–1208, 2007 Dickson RP, Davis RD, Rea JB, Palmer SM. High frequency of bronchogenic carcinoma after single-lung transplantation. J Heart Lung Transplant 25: 1297–1301, 2006 Engels EA. Inflammation in the development of lung cancer: Epidemiological evidence. Expert Rev Anticancer Ther 8: 605–615, 2008 Vajdic CM, McDonald SP, McCredie MR, van Leeuwen MT, Stewart JH, Law M, Chapman JR, Webster AC, Kaldor JM, Grulich AE. Cancer incidence before and after kidney transplantation. JAMA 296: 2823– 2831, 2006 Kasiske BL, Vazquez MA, Harmon WE, Brown RS, Danovitch GM, Gaston RS, Roth D, Scandling JD, Singer GG; American Society of Transplantation. Recommendations for the outpatient surveillance of renal transplant recipients. J Am Soc Nephrol 11(Suppl 15): S1–S86, 2000 Penn I. Incidence and treatment of neoplasia after transplantation. J Heart Lung Transplant 12: S328–S336, 1993 Euvrard S, Morelon E, Rostaing L, Goffin E, Brocard A, Tromme I, Broeders N, del Marmol V, Chatelet V, Dompmartin A, Kessler M, Serra AL, Hofbauer GF, Pouteil-Noble C, Campistol JM, Kanitakis J, Roux AS, Decullier E, Dantal J; TUMORAPA Study Group. Sirolimus and secondary skin-cancer prevention in kidney transplantation. N Engl J Med 367: 329–339, 2012 Stallone G, Schena A, Infante B, Di Paolo S, Loverre A, Maggio G, Ranieri E, Gesualdo L, Schena FP, Grandaliano G. Sirolimus for Kaposi’s sarcoma in renal-transplant recipients. N Engl J Med 352: 1317–1323, 2005 Salgo R, Gossmann J, Schöfer H, Kachel HG, Kuck J, Geiger H, Kaufmann R, Scheuermann EH. Switch to a sirolimus-based immunosuppression in long-term renal transplant recipients: Reduced rate of (pre-)malignancies and nonmelanoma skin cancer in a prospective, randomized, assessor-blinded, controlled clinical trial. Am J Transplant 10: 1385–1393, 2010 Wong G, Chapman JR, Craig JC. Cancer screening in renal transplant recipients: What is the evidence? Clin J Am Soc Nephrol 3(Suppl 2): S87–S100, 2008 American Society of Nephrology REVIEW QUESTIONS 1. A 50-year-old white woman, status post–kidney transplant 10 years ago, was recently diagnosed with squamous cell skin cancer on her nose. What should you advise her? a. Continue regular follow-up with dermatology b. Discontinue calcineurin inhibitor and switch to mTOR inhibitor–based regimen c. Apply sun screen d. Avoid peak hours of sun exposure e. All of the above Answer: e is correct. The patient should follow-up with dermatology for a complete skin examination as the risk of a second skin cancer is high. Exposure to UV light is one of the main risk factors for development of cancer, and therefore, avoiding sunlight and applying sunscreen may prevent development of new skin cancers. Last, studies have shown that switching to an mTOR inhibitor–based regimen reduces the risk of additional skin cancers. 2. Which of these factors lead to increased risk of cancer in the transplant recipient? a. b. c. d. e. Viral infections Immunosuppression Chronic Infections Smoking All of the above Answer: e is correct. The majority of the cancers in transplant recipients are due to viral infections such as HPV, HCV, HBV, EBV, and HHV-8. However, smoking, chronic infections, and certain immunosuppressants are also reported to increase risk of cancer. American Society of Nephrology 3. A 12-year-old boy underwent kidney transplant 6 months ago. He received thymoglobulin for induction and was maintained on a triple immunosuppressive regimen including mycophenolic acid derivatives. He was EBV negative at the time of transplant. He now presents with low-grade fever and pain over the allograft. The biopsy reveals dense lymphocytic infiltrate with minimal tubulitis. The lymphocytes stained positive for CD3 and CD20. SV-40 stain is negative. What is the most likely diagnosis in this patient? a. b. c. d. Rejection Acute interstitial nephritis Post-transplant lymphoproliferative disorder (PTLD) BK virus nephropathy Answer: c is correct. The boy has developed PTLD as suggested by a mixed lymphocytic population in the biopsy. CD31 indicates the presence of T cells, and CD201 indicates the presence of B cells. Rejection will have only CD31 or T-lymphocytes. Lack of SV-40 staining rules out BK virus nephropathy. The presence of abundant polymorphic lymphocytes with minimal tubulitis should be a clue for PTLD. He had several risk factors for development of PTLD including young age, EBV naïve, use of a T cell–depleting agent for induction, and use of mycophenolic acid derivatives. 4. How will you manage this patient? a. b. c. d. Increase immunosuppression Decrease immunosuppression Discontinue bactrim Start cidofovir Answer: a is correct. Treatment includes reduction of immunosuppression in this patient. Increasing immunosuppression to treat possible rejection may be harmful. Onco-Nephrology Curriculum 5 Chapter 17: Cancer Screening in ESRD Jean L. Holley, MD Department of Medicine, University of Illinois, Urbana-Champaign and Carle Physician Group, Urbana, Illinois INTRODUCTION The American Cancer Society recommends specific age-related screening examinations for colorectal, breast, and cervical cancer and suggests that individuals discuss their risk factors and screening for prostate and lung cancer with their primary care physician (Table 1) (1). Such recommendations are incorporated into guidelines for periodic adult health care for the general population. Cancer screening for any individual is predicated on the risk of developing cancer and the likelihood that the screening test will detect the cancer. An individual’s expected survival is also an integral factor in cancer screening. If expected survival is low, then the cost-effectiveness of routine cancer screening in average-risk individuals argues against screening because the patient will probably die before cancer develops and is detected. In the ESRD population, therefore, when considering routine cancer screening, it is important to ask the following: 1) is there an increased risk of cancer in this patient group?; 2) are screening tests accurate in this population?; and 3) will the patient live long enough for cancer screening to detect a life-threatening disease that can be cured? These issues will be discussed to demonstrate that, because of the high mortality with ESRD, routine cancer screening is not indicated for most patients. CANCER RISK IN ESRD Table 2 shows a summary of the published literature cancer incidence among ESRD patients. The standardized incidence ratio (SIR) is typically used to assess cancer frequency. Viral-mediated cancers like human papilloma virus (HPV)-associated cervical, uterine, and tongue cancer and hepatitis C– and B–associated liver cancer are more common in ESRD patients (2–8). Although there are no clinical data, the SIR for cervical cancer in ESRD patients suggests that young women (and men) with ESRD American Society of Nephrology should receive the HPV vaccine according to the recommendations in the general population (9). Bladder cancer is also more common in ESRD patients, likely in part due to medications associated with the development or treatment of kidney disease (oral cyclophosphamide use, analgesic use leading to chronic tubulointerstitial disease). These cancers, as well as liver cancer, are more common among Asian ESRD patients (6). Because of the development of acquired cystic disease in ESRD, renal cell carcinoma is also more common among dialysis patients, albeit with a relatively low incidence in most studies (5,7) (Table 2). Routine screening for renal cell carcinoma in chronic dialysis patients remains somewhat controversial but most advocate for individual patient-directed screening based on cost-effectiveness (10,11). The relatively low incidence of renal cell carcinoma in the setting of acquired cystic disease and the low expected patient survival with ESRD argues against routine screening. However, for patients on transplant waiting lists, screening may be advisable and required. EFFICACY OF CANCER SCREENING TESTS IN ESRD Cancer screening is primarily based on imaging techniques or laboratory and histopathologic examinations (Table 1). For most of these evaluations, the positive and negative predictive value of the test has not been assessed in ESRD patients. Due to the presence of vascular calcifications, mammography interpretation in women with ESRD may be more difficult (12,13). The higher rates of gastrointestinal bleeding in ESRD may result in higher fecal occult blood tests than the general population (14). This may actually lead to higher rates of Correspondence: Jean L. Holley, Nephrology, S2S2, 611 West Park Street, Urbana, Illinois 61802. Copyright © 2016 by the American Society of Nephrology Onco-Nephrology Curriculum 1 Table 1. American Cancer Society Recommendations for Routine Cancer Screening Cancer Recommended screening Breast Colorectal Cervical Prostate Lung Yearly mammogram beginning at age 40, continuing as long as in good health Clinical breast examination every 3 years from age 20 to 39 and then yearly for age .40 MRI for high-risk women Beginning age 50: Flexible sigmoidoscopy every 5 years or colonoscopy every 10 years or double contrast barium enema every 5 years or CT colonography (virtual colonoscopy) every 5 years, with yearly fecal occult blood test or fecal immunochemical test or stool DNA test done every 3 years Begin screening at age 21: 21–29 years: Pap every 3 years; no HPV unless Pap is abnormal; 30–65 years: Pap 1 HPV every 5 years or Pap alone every 3 years; .65 years: no screening Age 50, discuss pros and cons with MD; age 45 if African American or father or brother with prostate cancer before the age of 65 No recommendation High risk: consider screening age 55–74 in fairly good health with at least 30–pack-years smoking history and either still smoking or quit within the last 15 years Individuals with risk factors for specific cancers may need alternate screening protocols and should discuss with their physicians. Adapted from the American Cancer Society website. colonoscopies in ESRD patients, perhaps resulting in earlier detection of colorectal cancer in ESRD patients compared with the general population (15). There is no information on the reliability of fecal immunochemical or stool DNA testing in ESRD patients. An ongoing study on the performance of fecal occult blood testing in CKD may help to clarify these issues (16). Table 2. Cancer incidence in ESRD: Literature summary Cancer Renal cell Bladder and ureter Tongue Cervical and uterine Liver Thyroid and other Endocrine organs Breast (women) Lung/bronchus Colon/rectum Pancreas Prostate SIR Risk factors in ESRD 3.6–24.1 Acquired cystic disease 1.5–16.4 Analgesic abuse, Balkan nephropathy, oral cyclophosphamide 1.2–1.9 Human papilloma virus 0.9 Human papilloma virus 2.7–4.31 1.4–4.5 Hepatitis B and C 2.2–2.3 2,3,6 6 2,3 2,3,5,6 2,3,6 0.8–1.42 0.5–1.28 1.0–1.27 1.08 0.5–1.08 3,6,8 3,6,8 3,6,8 8 3,6,8 Adapted from reference 25 with additional data and references. 2 References Onco-Nephrology Curriculum 2–8 2,3,5,6,8 Tumor markers are sometimes used as cancer screening tools and may be affected by ESRD. Total prostate specific antigen (PSA) is probably valid in ESRD patients (17–19), but free PSA and free/total PSA ratios are less useful, as free PSA rises with hemoconcentration and high-flux dialysis membranes affect its clearance (18,19). For unclear reasons, prostate cancer is the only tumor diagnosed at a later stage in ESRD patients compared with the general population (15). Prostate cancer has generally not been more common in ESRD patients (2–7). However, a recent study found an SIR of 1.06 for prostate cancer (8), raising the issue of an increasing incidence of this cancer among ESRD patients. Controversy continues about screening for prostate cancer in the general population (Table 1). Most tumor markers are unreliable in ESRD patients; they are generally glycoproteins with high molecular weight that are rarely removed by dialysis and rise with hemoconcentration, yielding false-positive results in ESRD. For example, cancer antigen 125 (CA-125), a tumor marker for ovarian cancer, is produced by mesothelial cells, and patients with any serosal fluid (pleural effusion, ascites) will have elevated levels. This is especially applicable to patients on peritoneal dialysis, making CA-125 less useful in all ESRD patients, particularly those on peritoneal dialysis. b-human chorionic gonadotropin and a-fetoprotein, as well as total PSA, are probably reliable in ESRD patients. The recently published clinical trial on the cost-effectiveness of computed tomography (CT) screening for lung cancer in high-risk individuals reported a 20% reduction in mortality over a 4-year period in patients undergoing three annual CT exams at a cost of $81,000 per quality-adjusted life-year and incremental cost-effectiveness ratios of $52,000 per life-year gained (20). This study led the US Preventive Services Task Force to assign a B rating to the recommendation that annual low-dose CT scanning be performed as a screen for lung cancer in adults 55–80 years of age with a 30–pack-year smoking history. However, remaining questions about the overall efficacy of this screening method prompted the American Cancer Society (Table 1) to avoid endorsing CT scanning as a cancer screen. The Centers for Medicare and Medicaid, despite initial misgivings (21), have now endorsed lung cancer screening with CT scans. Lung cancer has not traditionally been more common in ESRD patients (2–6), and ESRD patients’ reduced survival argues against the adoption of lung cancer screening in this population. Recently, a 1.28 SIR for lung cancer in ESRD patients (8), along with the benefits of CT screening in at-risk individuals (20), suggests additional study may be needed. IS CANCER SCREENING APPROPRIATE IN ESRD GIVEN EXPECTED SURVIVAL? Although survival in ESRD may be improving slightly, it remains poor (22). As noted above, a patient’s expected survival is an important factor to consider when weighing the American Society of Nephrology benefits of cancer screening. Hypothetical modeling done in the 1990s suggested cancer screening in a dialysis patient would on average provide a net gain of 5 days of survival (23). This model was biased toward cancer screening, examining Papanicolaou smears for detecting cervical cancer, mammography as a screen for breast cancer, flexible sigmoidoscopy for colorectal cancer, and digital examination with PSA testing for prostate cancer assuming screening tests were perfectly sensitive and specific and that each detected cancer was instantaneously treated and cured (23). Using this model, the costs per unit of survival benefit provided by cancer screening were 1.6–19.3 times higher among ESRD patients (23). Another study focusing on breast cancer screening in dialysis patients found an absolute reduction in breast cancer mortality of 0.1% with a net gain in life expectancy of 1.3 days (24). Even focusing on the dialysis patient with the best predicted survival (a young black woman without diabetes mellitus) and multiple risk factors for breast cancer, only 250 days of life were estimated to be saved by screening with mammography in another study (25). Such investigations led to the recommendation to perform cancer screening only on dialysis patients assumed to benefit; cancer screening in ESRD should be based on the individual, considering his or her risk factors for cancer, as well as expected survival with ESRD (23–30). Transplant candidacy also needs to be considered when contemplating cancer screening in ESRD patients. The evaluative process for kidney transplantation includes age- and sex- appropriate cancer screening such as mammography, Papanicolaou smears, and PSA testing. Thus, cancer screening for transplant candidates is generally required. However, ESRD patients will need to be assessed on an individual basis, considering cancer risk factors, transplant status, and, importantly, expected survival to proceed with cancer screening in a cost-effective manner. Table 3 suggests an outline for cancer screening in ESRD patients based on these factors. Table 3. Suggested cancer screening in ESRD patients: Individualized, considering expected survival, risk factors, and transplant status Cancer Breast Colorectal Cervical Prostate Renal cell Recommended screening -Yearly mammogram beginning age 40 and on transplant list Clinical breast examination every 3 years for ages 20–39 and yearly for age .40 Beginning age 50: Yearly FIT or FOBT for those on transplant lists and flexible sigmoidoscopy, colonoscopy, double contrast barium enema, or virtual colonoscopy per transplant evaluation protocols Positive FIT or FOBT will require additional evaluation Begin screening at age 21: 21–65, yearly Pap for those on transplant list; consider HPV DNA and HPV vaccine in transplant candidates Age 50, annual PSA and digital rectal examination for men on transplant list Age 45 if African American or father or brother had prostate cancer before the age of 65 Yearly CT or MRI in patients on dialysis .3 years and on transplant list For all the above cancers, consider screening in high-risk patients with long expected survival. FIT, fecal immunochemical test; FOBT, fecal occult blood test. Adapted from references 22–29. TAKE HOME POINTS c Viral-associated cancers like hepatitis B– and C–associated liver cancer and human papilloma virus–associated tongue and cervical cancer are more common in ESRD patients. c Because of acquired cystic disease, renal cell carcinoma is more common in ESRD patients, and exposure to analgesic abuse and oral cyclophosphamide result in an increased incidence of bladder cancer in ESRD patients. c Due to poor expected survival with ESRD, cancer screening is not appropriate for most dialysis patients. Patients with long expected survival, those on transplant waiting lists, and those with increased cancer risk factors are appropriate candidates for cancer screening. SUMMARY AND CONCLUSIONS Although bladder cancer and viral-mediated cancers like HPVassociated cervical cancer and hepatitis C– and B–associated liver cancer are more common in ESRD patients, general routine cancer screening in ESRD patients is not recommended. ESRD patients in whom cancer screening should be considered are those with good expected survival, candidates for kidney transplantation, and certain individuals with a high cancer risk and good expected survival. Although acquired cystic kidney disease is associated with an increased risk of renal cell carcinoma, the same general rules apply; routine screening is not recommended for most patients. The tendency may be to implement routine cancer screening protocols in dialysis units, but individualized patient assessment is required for appropriate cancer screening. The emerging model of personalized cancer screening for the general population is being discussed (31) and seems clearly appropriate for those on dialysis. American Society of Nephrology REFERENCES 1. American Cancer Society: American Cancer Society Guidelines for the Early Detection of Cancer, 2015. Available at: http://www.cancer.org/ healthy/findcancerearly/cancerscreeningguidelines/american-cancersociety-guidelines-for-the-early-detection-of-cancer. Accessed January 21, 2015 2. Buccianti G, Maisonneuve P, Ravasi B, Cresseri D, Locatelli F, Boyle P. Cancer among patients on renal replacement therapy: a populationbased survey in Lombardy, Italy. Int J Cancer 66: 591–593, 1996 3. Maisonneuve P, Agodoa L, Gellert R, Stewart JH, Buccianti G, Lowenfels AB, Wolfe RA, Jones E, Disney APS, Briggs D, McCredie M, Boyle P. Cancer in patients on dialysis for end-stage renal disease: An international collaborative study. Lancet 354: 93–99, 1999 4. Heidland A, Bahner U, Vamvakas S. Incidence and spectrum of dialysisassociated cancer in three continents. Am J Kidney Dis 35: 347–351, discussion 352–353, 2000 5. Chen K-S, Lai M-K, Huang C-C, Chu S-H, Leu M-L. Urologic cancers in uremic patients. Am J Kidney Dis 25: 694–700, 1995 Onco-Nephrology Curriculum 3 6. Lin H-F, Li Y-H, Wang C-H, Chou C-L, Kuo D-J, Fang TC. Increased risk of cancer in chronic dialysis patients: A population-based cohort study in Taiwan. Nephrol Dial Transplant 27: 1585–1590, 2012 7. Farivar-Mohseni H, Perlmutter AE, Wilson S, Shingleton WB, Bigler SA, Fowler JE Jr. Renal cell carcinoma and end stage renal disease. J Urol 175: 2018–2020, discussion 2021, 2006 8. Butler AM, Olshan AF, Kshirsagar AV, Edwards JK, Nielsen ME, Wheeler SB, Brookhart MA. Cancer incidence among US Medicare ESRD patients receiving hemodialysis, 1996-2009. Am J Kidney Dis 65: 763–772, 2015 9. Centers for Disease Control and Prevention: Immunization Schedules, 2015. Available at: http://www.cdc.gov/vaccines/schedules/. Accessed January 21, 2015 10. Sarasin FP, Wong JB, Levey AS, Meyer KB. Screening for acquired cystic kidney disease: A decision analytic perspective. Kidney Int 48: 207–219, 1995 11. Brown EA. Renal tumours in dialysis patients: Who should we screen? Nephron Clin Pract 97: c3–c4, 2004 12. Evans AJ, Cohen ME, Cohen GF. Patterns of breast calcification in patients on renal dialysis. Clin Radiol 45: 343–344, 1992 13. Castellanos M, Varma S, Ahern K, Grosso SJ, Buchbinder S, D’Angelo D, Raia C, Kleiner M, Elsayegh S. Increased breast calcifications in women with ESRD on dialysis: Implications for breast cancer screening. Am J Kidney Dis 48: 301–306, 2006 14. Akmal M, Sawelson S, Karubian F, Gadallah M: The prevalence and significance of occult blood loss in patients with predialysis advanced chronic renal failure (CRF), or receiving dialytic therapy. Clin Nephrol 42: 198–202, 1994 15. Taneja S, Mandayam S, Kayani ZZ, Kuo Y-F, Shahinian VB. Comparison of stage at diagnosis of cancer in patients who are on dialysis versus the general population. Clin J Am Soc Nephrol 2: 1008–1013, 2007 16. Wong G, Howard K, Chapman JR, Tong A, Bourke MJ, Hayen A, Macaskill P, Hope RL, Williams N, Kieu A, Allen R, Chadban S, Pollock C, Webster A, Roger SD, Craig JC. Test performance of faecal occult blood testing for the detection of bowel cancer in people with chronic kidney disease (DETECT) protocol. BMC Public Health 11: 516–522, 2011 17. Morton JJ, Howe SF, Lowell JA, Stratta RJ, Taylor RJ. Influence of endstage renal disease and renal transplantation on serum prostatespecific antigen. Br J Urol 75: 498–501, 1995 18. Djavan B, Shariat S, Ghawidel K, Güven-Marberger K, Remzi M, Kovarik J, Hoerl WH, Marberger M. Impact of chronic dialysis on serum PSA, free PSA, and free/total PSA ratio: Is prostate cancer detection 4 Onco-Nephrology Curriculum 19. 20. 21. 22. 23. 24. 25. 26. 27. 28. 29. 30. 31. compromised in patients receiving long-term dialysis? Urology 53: 1169–1174, 1999 Bruun L, Björk T, Lilja H, Becker C, Gustafsson O, Christensson A. Percent-free prostate specific antigen is elevated in men on haemodialysis or peritoneal dialysis treatment. Nephrol Dial Transplant 18: 598–603, 2003 Black WC, Gareen IF, Soneji SS, Sicks JD, Keeler EB, Aberle DR, Naeim A, Church TR, Silvestri GA, Gorelick J, Gatsonis C; National Lung Screening Trial Research Team. Cost-effectiveness of CT screening in the National Lung Screening Trial. N Engl J Med 371: 1793–1802, 2014 Bindman A. JAMA Forum: Lung cancer screening and evidence-based policy. JAMA 313: 17–18, 2015 US Renal Data System. USRDS 2014 Annual Data Report: Atlas of EndStage Renal Disease in the United States, Bethesda, MD, National Institutes of Health, National Institute of Diabetes and Digestive and Kidney Diseases, 2014 Chertow GM, Paltiel AD, Owen WF Jr, Lazarus JM. Cost-effectiveness of cancer screening in end-stage renal disease. Arch Intern Med 156: 1345–1350, 1996 Wong G, Howard K, Chapman JR, Craig JC. Cost-effectiveness of breast cancer screening in women on dialysis. Am J Kidney Dis 52: 916–929, 2008 LeBrun CJ, Diehl LF, Abbott KC, Welch PG, Yuan CM. Life expectancy benefits of cancer screening in the end-stage renal disease population. Am J Kidney Dis 35: 237–243, 2000 Holley JL. Screening, diagnosis, and treatment of cancer in long-term dialysis patients. Clin J Am Soc Nephrol 2: 604–610, 2007 Kajbaf S, Nichol G, Zimmerman D. Cancer screening and life expectancy of Canadian patients with kidney failure. Nephrol Dial Transplant 17: 1786–1789, 2002 Holley JL. Preventive medical screening is not appropriate for many chronic dialysis patients. Semin Dial 13: 369–371, 2000 Williams AW, Dwyer AC, Eddy AA, Fink JC, Jaber BL, Linas SL, Michael B, O’Hare AM, Schaefer HM, Shaffer RN, Trachtman H, Weiner DE, Falk AR; American Society of Nephrology Quality, and Patient Safety Task Force. Critical and honest conversations: the evidence behind the “Choosing Wisely” campaign recommendations by the American Society of Nephrology. Clin J Am Soc Nephrol 7: 1664–1672, 2012 Walter LC, Lindquist K, O’Hare AM, Johansen KL. Targeting screening mammography according to life expectancy among women undergoing dialysis. Arch Intern Med 166: 1203–1208, 2006 Saini SD, van Hees F, Vijan S. Smarter screening for cancer: Possibilities and challenges of personalization. JAMA 312: 2211–2212, 2014 American Society of Nephrology REVIEW QUESTIONS 1. Is the cancer incidence in ESRD patients higher than in the general population? Answer: Yes, for certain cancers. Virus-associated cancers (liver cancer, cervical cancer, and tongue cancer) and renal cell and bladder cancer (because or risk factors) are more common in ESRD patients. Breast, colorectal, and lung cancer are not more common in ESRD patients. 2. What factors affect the efficacy or cost-effectiveness of cancer screening in general? cancer screening. Screening is predicated on the patient living long enough to develop a specific cancer and the sensitivity and specificity of the screening test to detect that cancer at a stage when cure is possible. 3. Should routine screening protocols be in place in dialysis units? Answer: No, routine cancer screening is not cost-effective for most dialysis patients because their expected survival is short. An individualized approach to cancer screening is most appropriate for ESRD patients, considering the patient’s specific risk factors for cancer development, transplant status, and expected survival. Answer: The cancer risk, the effectiveness of the screening test, and the patient’s expected survival all affect the efficacy of American Society of Nephrology Onco-Nephrology Curriculum 5 Chapter 18: Ethics of RRT, Initiation, and Withdrawal in Cancer Patients Michael J. Germain, MD Department of Medicine, Tufts University, Springfield, Massachusetts Malignancies are common in CKD patients, and the incidence is higher than in the general age-matched population. Because cardiovascular disease and infection are so prevalent in CKD, especially ESRD patients, the mortality rate from cancer in ESRD patients is lower than the age-matched general population due to these competing influences. Thus, the relative risk of mortality from cancer is increased in the younger ESRD population and then declines with age (1,2). Patients with cancer and a need for RRT present very difficult scenarios for making clinical decisions, and an approach grounded in medical ethical principles can be helpful (3–7). Medical ethics reflect the culture and time that we are living in and also include a religious perspective. This chapter will focus on a US perspective that reflects the generally accepted values of our society at the present time. The United States has a wide representation of cultural and religious values, with many patients who are new immigrants from many countries. A discussion of the different medical ethical approaches from these societies is beyond the scope of this discussion, but the clinician should always inquire from the patient and family how they want prognosis, goals of care, and end-of-life issues discussed with them. This discussion will rely heavily on the national clinical practice guideline Shared Decision-Making in the Appropriate Initiation of and Withdrawal from Dialysis, 2nd Ed. (SDMG), in particular the section “Ethical Considerations in Dialysis Decision-Making” (8). Six ethical principles should be strongly considered for patients with cancer when discussing RRT (Table 1). Conflicts between respect for patient autonomy and beneficence/nonmaleficence often can occur with these patients. There are four scenarios where the ethical issues of cancer and RRT intersect: 1) patients with ESRD who develop a terminal malignancy; 2) patients with a terminal malignancy who develop ESRD; 3) patients with a terminal malignancy who develop AKI (AKI can be caused by the treatment of the malignancy, obstruction or invasion of the American Society of Nephrology kidney by the malignancy, or surgical removal of the kidney to remove the malignancy); and 4) a renal transplant patient with a terminal malignancy. In the first scenario, withdrawal of dialysis (9–11) is often the ethical question. In the second and third scenarios, withholding of dialysis or withdrawal may be the ethical issue. An important ethical aspect is the ethical imperative of the clinician to “first, do no harm.” The clinician has the right and duty not to order a treatment that will do more harm than good. Nephrologists often find themselves in the position of being asked to provide dialysis, by a patient, family, or other clinicians, when dialysis may not be in the patient’s best interest. Many clinicians feel they are required to provide dialysis treatment when the patient or health care provider (HCP) requests it. The SDMG (recommendations 5 and 6) clearly state that the clinician has no such obligation. The clinician should document these discussions and make it clear that the patient/HCP has the right to transfer care to another clinician. Clinicians should not fear medical–legal concerns in this scenario; in reality, these rarely, if ever, occur, especially if the SDMG is followed. Instead, shared decision-making is the preferred process where the clinician/care team (SDMG recommendation 1) and the patient/family/HCP make a care plan for the patient. The first step in this process is for the care team to ask, listen, and understand the patient’s understanding of his or her condition and values and goals in life. With the patient’s explicit permission, the care team then explains from their expert perspective the patient’s condition, prognosis, and the risks and benefits of the treatment options. Recent qualitative studies have shown that CKD patients want to know their prognosis. However, our experience is such that patients often do not want a numerical estimate, such Correspondence: Michael J. Germain, Department of Medicine, Tufts University, 100 Wason Avenue, Suite 200, Springfield, Massachusetts 01107. Copyright © 2016 by the American Society of Nephrology Onco-Nephrology Curriculum 1 Table 1. Six medical ethics principles (http://en.wikipedia. org/wiki/Medical_ethics#Values_in_medical_ethics) 1) Respect for autonomy: The patient has the right to refuse or choose their treatment (voluntas aegroti suprema lex). 2) Beneficence: A practitioner should act in the best interest of the patient (salus aegroti suprema lex). 3) Nonmaleficence: “First, do no harm” (primum non nocere). 4) Justice: Concerns the distribution of scarce health resources and the decision of who gets what treatment (fairness and equality). 5) Respect for persons: The patient (and the person treating the patient) has the right to be treated with dignity. 6) Truthfulness and honesty: The concept of informed consent as how many months of life they may have remaining. Rather, they prefer a general statement about overall prognosis (SDMG recommendations 2 and 3). Through the process of consensus building, a shared decision and treatment plan is agreed on (SDMG recommendation 4). In a consensus, each party may not get the plan they originally favored, but they may be convinced by hearing the perspectives put forth by others that a different plan is preferred. Sometimes the party may not like the consensus plan but agrees to accept it. I have seen this in situations where the HCP may want to initiate or continue dialysis in a patient who clearly is getting no benefit (or even suffering harm), but the nephrologist has decided that he or she cannot ethically order the dialysis treatment. After offering to transfer care to another nephrologist, invariably the HCP will accept the clinician’s decision. For this to work, the clinician must show respect for the alternative point of view, listen carefully, and validate the person’s views. If the views are based on religious objections, then it is sensible to involve a clergy person who may help explain the religion’s positions. A time-limited trial of RRT (7) can be undertaken in certain defined situations such as when the benefit to be achieved by dialysis is uncertain or a consensus about the benefit of dialysis cannot be reached (SDMG recommendation 7). When consensus cannot be reached, the SDMG suggests conflict resolution (SDMG recommendation 8; “resolving conflicts about what dialysis decision to make”; Box 1 and Figure 1). The SDMG suggests a practical ethical approach to decision-making. The patient’s case is analyzed from these perspectives (Table 2). Each perspective is viewed through the six ethical principles in Table 1. The SDMG then recommends the following process for ethical decision-making (Table 3). Although the SDMG recommends that patients with a terminal prognosis (,6 months) should not receive dialysis, the guidelines recognize that “palliative dialysis” (12) is an option for those who require more time to finish their life goals. Such goals include activities for significant events like a wedding, birth, or graduation. Palliative dialysis allows the patient to transition to a more comfort-oriented care. The patient may shorten their dialysis treatment time, restrict further hospitalizations or procedures, and, when appropriate, receive hospice services (SDMG recommendation 9). 2 Onco-Nephrology Curriculum Box 1. Suggested steps for implementing recommendation 8 (reproduced with permission) ■ Extended conversation 2 Why does the patient or legal agent desire dialysis when it is not recommended by the renal care team? 2 Why does the patient or legal agent refuse dialysis when it is recommended by the renal care team? 2 Does the patient or legal agent misunderstand the diagnosis, prognosis, and treatment alternatives? 2 Does the nephrologist misunderstand the patient’s or legal agent’s reasons for requesting dialysis? 2 Does the nephrologist understand the psychosocial, cultural, or spiritual concerns and values the patient or legal agent have? 2 Has the nephrologist consulted a psychologist, social worker, or chaplain for assistance in fully understanding the concerns of the patient or legal agent family? ■ Consultation with other physicians 2 Do other physicians agree or disagree with the attending physician’s recommendation to withhold or withdraw dialysis? 2 Is the request for dialysis by the patient or legal agent medically appropriate? ■ Consultation with ethics committee or ethics consultants. 2 Has the patient or legal agent been informed that the purpose of the ethics consult is to clarify issues of disagreement, and ideally, to enable resolution? 2 Has the patient or legal agent met with the ethics committee or ethics consultants to explain their perspective and reasoning behind their request for dialysis? 2 Can the ethics committee identify the reasons why the patient or legal agent is resistant to the physician’s recommendation to forgo dialysis? 2 Can the ethics committee identify the reasons why the health care provider is resistant to the patient’s or legal agent’s desire to begin or continue dialysis? 2 Has the ethics committee explained in understandable terms to the patient or legal agent its conclusions and the reasoning behind them? 2 Can the impasse be resolved with accommodation, negotiation, or mediation? ■ Documentation 2 The physician must document the medical facts and his/her reasons for the recommendation to forgo dialysis and the decision not to agree to the request by the patient or legal agent. 2 The consultants should also document their assessment of the patient’s diagnosis, prognosis, and their recommendations in the chart. ■ An attempt to transfer the patient’s care 2 If reconciliation is not achieved through the above procedure and the physician in good conscience cannot agree to the patient or legal agent’s request, the physician is ethically and legally obligated to attempt to transfer the care of the patient to another physician. 2 Another physician and/or institution may not be found who is willing to accept the patient under the terms of the family’s request. Physicians and institutions that refuse to accept the patient in transfer and their reasons should also be documented in the medical record. 2 Consider consultation with a mediator, extramural ethics committee, or the ESRD Network in the region. American Society of Nephrology Table 2. Perspectives to consider in ethical decisionmaking* 1) Medical indications, the diagnosis, prognosis, and treatment 2) Patient preferences 3) Quality of life 4) Contextual features (social, economic, legal, and administrative) *Adapted from reference 8 with permission from the Renal Physicians Association Table 3. The seven-step process of ethical decision-making in patient care* 1) What are the ethical questions 2) What are the clinically relevant facts 3) What are the values at stake 4) List options (what could you do) 5) What should you do (choice the best option from the ethical point of view balancing all the above factors 6) Justify your choice based on the ethical principles 7) How could this ethical issue have been prevented *Adapted from reference 8 with permission from the Renal Physicians Association. Table 4. Systems approach to American College of Physicians in nephrology practice and the dialysis unit 1) Normalize the conversation: start discussions of EOL issues early in the patient’s interaction with the nephrology team. 2) Involve all members of the care team. In the office, this depends on human resources available. In the dialysis unit, train and utilize the dietician, technicians, social worker, and nurses. 3) Have a champion. Without this, likely there will be little buy-in or progress. Although the nephrologist does not have to be the champion, the nephrologist leader (i.e., medical director in the dialysis unit) needs to show strong support. 4) Teach all staff members simple communication techniques. 5) Integrate ACP into the workflow. 6) Do continuous quality improvement on the process. 7) There are resources available to learn from established successful programs . Figure 1. Systematic approach to resolving conflict between patient and renal care team. Finally, an effective process depends on excellent clinician– patient/family communication (13–15) (SDMG recommendation 10). To have these discussions, appropriate systems must be in place in the nephrology practice and dialysis units to facilitate the process (Table 4,) (16–18). There are excellent resources to help the health care team to accomplish these goals and tasks. Offering meticulous end-of-life care, including hospice, is mandatory for all of our patients with a ,6-month prognosis (17–19). It is important for patients and families to understand that palliative care and hospice do not American Society of Nephrology result in a shortened survival (20). Patients are not harmed, and they appreciate honest communication of bad news (21). It is important to recognize that our patients want to know their prognosis, and there are validated tools available for the clinician to utilize when having this discussion (22,23). In the end, the goal of the communication between the patient (and family or other preferred surrogate decisionmaker) and the kidney care team is shared decision-making. Shared decision-making is the recognized preferred model for medical decision-making because it addresses the ethical need to fully inform patients about the risks and benefits of treatments, as well as the need to ensure that patients’ values and preferences play a prominent role (8). Shared decisionmaking has been referred to as the “pinnacle” of patient-centered care (24). Patient-centered care has been one of the six specific aims for improvement for health care since the Institute Onco-Nephrology Curriculum 3 Table 5. List of resources 1) RPA SDM Toolkit: https://itunes.apple.com/us/app/rpa-sdm-toolkit/ id843971920?mt58 2) ACP DECISIONS: http://www.acpdecisions.org 3) Coalition for Kidney Supportive Care: http://www. kidneysupportivecare.org/Home.aspx 4) Supportive Care for the Renal Patient, edited by Chambers, Brown, Germain, 2nd Ed., London, UK, Oxford Press, 2010 5) RPA SDM Guidelines, 2nd Ed., 2010: http://www.renalmd.org/ catalogue-item.aspx?id5682 6) Five Wishes: https://www.agingwithdignity.org 7) Alberta’s Conversations Matter: http://www.albertahealthservices. ca/9098.asp 8) ASN Geriatric Curriculum: http://asn-online.org/education/ distancelearning/curricula/geriatrics/ 9) KDIGO Renal Supportive Care Initiative: http://kdigo.org/home/ conferences/supportivecare/ 10) Vital Talk communication techniques: http://vitaltalk.org/sites/ default/files/quick-guides/NURSEforVitaltalkV1.0.pdf 11) https://www.prepareforyourcare.org/ 12) http://www.ihi.org/Engage/Initiatives/ConversationProject/ Pages/default.aspx 13) http://www.gundersenhealth.org/respecting-choices 14) Six-month mortality predictor: Under nephrology/HD, http://www. qxmd.com/apps/calculate-by-qxmd 15) Breaking Bad News SPIKE: https://depts.washington.edu/bioethx/ topics/badnws.html of Medicine (IOM) issued its 2001 report, Crossing the Quality Chasm: A New Health System for the 21st Century (25). The IOM noted that the US health care delivery system does not provide consistent, high-quality medical care to all people. The IOM defined patient-centered as “providing care that is respectful of and responsive to individual patient preferences, needs, and values, and ensuring that patient values guide all clinical decisions.” Since the publication of the IOM report, there has been growing national interest in more individualized patient-centered models of care. These models focus on what matters most to individual patients and less on what might matter to providers or health systems (26). A recent qualitative study suggests that patients want to discuss ACP with the nephrologist (27). When effectively done, it can increase use of hospice and provide a “good death” (28). A recent review emphasizes the ethical principles involved in these discussion with the elderly CKD patient (29). This chapter has sought to explain how high-quality, ethical care can be delivered to patients with advanced kidney disease and cancer. TAKE HOME POINTS c The ethics of RRT in cancer balances the principles of respect for patient autonomy with nonmaleficence. c In some cases, palliative dialysis may be an option for these patients. c Good communication skills are the key to shared decision-making and patient-centered care. 4 Onco-Nephrology Curriculum REFERENCES 1. Available at: http://www.usrds.org/2014/view/Default.aspx. Accessed March 24, 2015 2. Oneschuk D, Fainsinger R. Medical and ethical dilemmas when an advanced cancer patient discontinues dialysis. J Palliat Care 18: 123– 126, 2002 3. Del Vecchio L, Locatelli F. Ethical issues in the elderly with renal disease. Clin Geriatr Med 25: 543–553, 2009 4. Davison SN. The ethics of end-of-life care for patients with ESRD. Clin J Am Soc Nephrol 7: 2049–2057, 2012 5. Skold A, Lesandrini J, Gorbatkin S. Ethics and health policy of dialyzing a patient in a persistent vegetative state. Clin J Am Soc Nephrol 9: 366–370, 2014 6. Moss AH. Ethical principles and processes guiding dialysis decisionmaking. Clin J Am Soc Nephrol 6: 2313–2317, 2011 7. Rinehart A. Beyond the futility argument: the fair process approach and time-limited trials for managing dialysis conflict. Clin J Am Soc Nephrol 8: 2000–2006, 2013 8. Renal Physicians Association. Shared Decision Making in the Appropriate Initiation of and Withdrawal from Dialysis, 2nd Ed., Rockville, MD, Renal Physicians Association, 2010 9. Akbar S, Moss AH. The ethics of offering dialysis for AKI to the older patient: Time to re-evaluate? Clin J Am Soc Nephrol 9: 1652–1656, 2014 10. Brown EA. Non-dialysis therapy: A better policy than dialysis followed by withdrawal? Semin Dial 25: 26–27, 2012 11. Murtagh F, Cohen LM, Germain MJ. Dialysis discontinuation: Quo vadis? Adv Chronic Kidney Dis 14: 379–401, 2007 12. Grubbs V, Moss AH, Cohen LM, Fischer MJ, Germain MJ, Jassal SV, Perl J, Weiner DE, Mehrotra R; Dialysis Advisory Group of the American Society of Nephrology. A palliative approach to dialysis care: A patientcentered transition to the end of life. Clin J Am Soc Nephrol 9: 2203– 2209, 2014 13. Schell JO, Cohen RA. A communication framework for dialysis decision-making for frail elderly patients. Clin J Am Soc Nephrol 9: 2014–2021, 2014 14. Schell JO, Green JA, Tulsky JA, Arnold RM. Communication skills training for dialysis decision-making and end-of-life care in nephrology. Clin J Am Soc Nephrol 8: 675–680, 2013 15. Tamura MK, Tan JC, O’Hare AM. Optimizing renal replacement therapy in older adults: A framework for making individualized decisions. Kidney Int 82: 261–269, 2012 16. Da Silva-Gane M, Cohen LM. Planning a supportive care programme and it’s components. In: Supportive Care for the Renal Patient, edited by Chambers J, Brown E, Germain MJ, 2nd Ed., Oxford, UK, Oxford University Press, 2010, 39–48 17. Germain MJ, Kurella Tamura M, Davison SN. Palliative care in CKD: The earlier the better. Am J Kidney Dis 57: 378–380, 2011 18. Wong SP, Kreuter W, O’Hare AM. Treatment intensity at the end of life in older adults receiving long-term dialysis. Arch Intern Med 172: 661– 663, discussion 663–664, 2012 19. Farrington K, Chambers JE. Death and end-of-life care in advanced kidney disease. In: Supportive Care for the Renal Patient, edited by Chambers JE Brown E, Germain MJ, 2nd Ed., Oxford, UK, Oxford University Press, 2010 20. Temel JS, Greer JA, Muzikansky A, Gallagher ER, Admane S, Jackson VA, Dahlin CM, Blinderman CD, Jacobsen J, Pirl WF, Billings JA, Lynch TJ. Early palliative care for patients with metastatic non-small-cell lung cancer. N Engl J Med 363: 733–742, 2010 21. Wachterman MW, Marcantonio ER, Davis RB, Cohen RA, Waikar SS, Phillips RS, McCarthy EP. Relationship between the prognostic expectations of seriously ill patients undergoing hemodialysis and their nephrologists. JAMA Intern Med 173: 1206–1214, 2013 22. Germain MJ. How to integrate predictions in outcomes in planning clinical care. Blood Purif 39: 65–69, 2015 American Society of Nephrology 23. Cohen LM, Ruthazer R, Moss AH, Germain MJ. Predicting six-month mortality for patients who are on maintenance hemodialysis. Clin J Am Soc Nephrol 5: 72–79, 2010 24. Barry MJ, Edgman-Levitan S. Shared decision making: Pinnacle of patient-centered care. N Engl J Med 366: 780–781, 2012 25. Institute of Medicine. Crossing the Quality Chasm: A New Health System for the 21st Century, Washington, DC, National Academies Press, 2001 26. Tinetti ME, Fried T. The end of the disease era. Am J Med 116: 179– 185, 2004 American Society of Nephrology 27. Goff SL, Eneanya ND, Feinberg R, Germain MJ, Marr L, Berzoff J, Cohen LM, Unruh M. Advance care planning: A qualitative study of dialysis patients and families. Clin J Am Soc Nephrol 10: 390–400, 2015 28. Cohen LM, Ruthazer R, Germain MJ. Increasing hospice services for elderly patients maintained with hemodialysis. J Palliat Med 13: 847– 854, 2010 29. Thorsteinsdottir B, Swetz KM, Albright RC. The ethics of chronic dialysis for the older patient: Time to reevaluate the norms. Clin J Am Soc Nephrol 10: 2094–2099, 2015 Onco-Nephrology Curriculum 5 REVIEW QUESTIONS 1. A long-term dialysis patient presents with metastatic sarcoma that is not treatable, and the prognosis is poor. What are the relevant medical ethical principles to consider in this patient? a. b. c. d. e. f. g. Autonomy Nonmaleficence Beneficence Respect for person Truth and honesty All of the above None of the above Answer: f is correct. Patient autonomy, nonmaleficence (avoiding the harms of RRT), beneficence, respect for person, truth, and honesty. 2. The family requests that you withhold the cancer diagnosis/prognosis information from this patient. What is the ethical principle that would guide your decision? a. Nonmaleficence: The information would be harmful to the patient b. Truth and honesty. c. Beneficence Answer: b is correct. In our society, it is not ethical to withhold this information. In some cultures, it is left to the doctor to decide if it would be “harmful” to the patient to give them bad news. If a family asks that you not give bad news to the patient, it is acceptable to ask the patient if they prefer that 6 Onco-Nephrology Curriculum these discussions take place with a family member or HCP instead of with them; this is a common scenario for some cultures in the United States (Native American, some Asian cultures). 3. The patient’s health deteriorates rapidly, and she is in pain whenever she is moved, such as transportation to and from dialysis. She is very lethargic and not communicative. The clinician feels that dialysis is doing more harm than good for the patient. When this is discussed with the family, they insist that dialysis be continued. They believe that in their religion withdrawing dialysis is a sin. Attempts at shared decision making and involving their pastor have not resulted in a resolution of the conflict. The correct approach is to: a. Continue dialysis b. Seek a court order to withdraw dialysis c. Explain to the family that you understand and respect their point of view; explain that you have an ethical duty to do no harm by the treatments that you order for your patients, and at this point, dialysis is doing more harm than good; and you will be discontinuing your order for dialysis and the family can seek another clinician to take over care if they wish Answer: c is correct. After following an shared decision making process and conflict resolution, if there is still no consensus, then the clinician has the right and ethical duty to not order RRT if the principle of nonmaleficence and justice outweighs the principle of autonomy. The patient has the right to refuse a treatment but not to demand a treatment. American Society of Nephrology Chapter 19: Palliative Care in Patients with Kidney Disease and Cancer Alvin H. Moss, MD, FACP, FAAHPM Department of Medicine, Sections of Nephrology and Supportive Care, West Virginia University School of Medicine, Morgantown, West Virginia INTRODUCTION RELEVANCE OF PALLIATIVE CARE Many, if not most, cancer and kidney disease patients have two things in common: they have a shortened life expectancy and a high symptom burden. Both populations benefit from early palliative care interventions. The goal of palliative care is to relieve suffering and to support the best possible quality of life for patients and their families, regardless of their stage of disease or the need for other therapies, in accordance with their values and preferences. By Medicare regulation, hospice care is limited to patients estimated to be in their last 6 months of life if their disease follows the normal course. In patients with kidney disease and cancer (hereafter kidney-cancer patients) who have a higher symptom burden than patients with either disease alone, the need for meticulous pain and symptom management is even more important to maintain quality of life. In addition, there is a unique need for advance care planning for these patients, most of whom have two life-limiting illnesses. As in other populations, for kidney-cancer patients, pain is one of the most common and severe symptoms. Multiple studies in kidney patients show that pain is undertreated. Treatment of pain in patients with stage 4 and 5 CKD and ESRD is more challenging because of the failure of renal excretion of active metabolites from some commonly used opioids, which leads to opioid neurotoxicity. The nephrology community has developed a clinical practice guideline that endorses the process of shared decision-making in reaching decisions about who should undergo dialysis. It recognizes that the burdens of dialysis may substantially outweigh the benefits in some patients and notes that nephrologists may want to recommend forgoing dialysis to kidney-cancer patients who are terminally ill from their cancer. This chapter describes the growing interest in the nephrology and oncology communities in incorporating palliative care into the standard treatment of patients with CKD, ESRD, and cancer. There is an increasing recognition that skills in palliative and end-of-life care are required for physicians, nurses, and others who treat patients who have CKD and ESRD (Table 1). The principal reasons are as follows: first, they have a significantly shortened life expectancy; just over half of dialysis patients (52%) are still alive 3 years after the start of RRT (1). Second, patients with CKD and ESRD have multiple comorbidities and consequently many symptoms such as pain, fatigue, itching, and difficulty with sleep. In one study, the symptoms of CKD and ESRD patients were found to be comparable (mean of 10.7) and severity (2). Similarly, cancer patients have been found to have a high symptom burden compared with age-matched controls, and pain, anxiety and depression, and insomnia were noted as most prevalent in a population-based study of 1,904 cancer survivors (3). An interaction between cancer status and comorbidity was found, resulting in a higher symptom burden for patients with comorbidities such as CKD. Thus, it is reasonable to conclude the CKD or ESRD patients with cancer will have a higher symptom burden than patients with either cancer or kidney disease alone (3). Third, the dialysis population has been growing progressively older. The incidence rates of ESRD are highest in patients 75 years old and older, and they continue to rise in this group (1). Older patients survive the shortest period of time on dialysis, and they withdraw from dialysis significantly more often than younger patients. In consideration of the high symptom burden and the low survival rate for dialysis patients, the American Society of Nephrology Correspondence: Alvin H. Moss, Department of Medicine, Sections of Nephrology and Supportive Care, West Virginia University School of Medicine, 1195 Health Sciences North, Morgantown, West Virginia 26506-9022 Copyright © 2016 by the American Society of Nephrology Onco-Nephrology Curriculum 1 Table 1. Palliative care for CKD/ESRD patients: Need for a systematic approach Table 2. Components of a dialysis facility palliative care program Pain and symptom assessment/management (11,21) Shared decision-making for informed consent Patient-specific estimates of prognosis using the surprise question Timely discussions prompted by prognosis Inclusion of family/legal agent in discussions Completion of advance directives Completion of physician orders for life-sustaining treatment (POLST) paradigm form as appropriate Immediately actionable medical orders Transferrable throughout health care setting Referral to hospice when indicated 1. Palliative care focus a. Educational activities, including dialysis unit in-service trainings b. Quality improvement activities, including morbidity and mortality conferences c. Use of the “Would you be surprised if this patient died within the next year?” question to identify patients appropriate for palliative care d. Collaboration with local hospice programs to coordinate a smooth transition to end-of-life care 2. Pain and symptom assessment and management protocols 3. Systematized advanced care planning 4. Psychosocial and spiritual support to patients and families, including the use of peer counselors 5. Terminal care protocols that include hospice referral 6. Bereavement programs for families that include memorial services Adapted from reference 5. American Society of Nephrology (ASN) and the Renal Physicians Association (RPA) have recommended that dialysis facilities incorporate palliative care into their treatment of patients (4,5). Nephrologists have been encouraged to obtain education and skills in palliative care, and dialysis facilities have been urged to developed protocols, policies, and programs to ensure that palliative care is provided to their patients (Table 2) (5). Also, dialysis units have been urged to develop a working relationship with local hospice programs, so patients with ESRD who stop dialysis or patients undergoing dialysis with a nonrenal terminal diagnosis may be referred for hospice. Similarly, the American Society of Clinical Oncology has issued a provisional clinical opinion that early involvement of palliative care when combined with standard cancer care leads to better patient and caregiver outcomes, including improvement in symptoms, quality of life, and patient satisfaction and reduced caregiver burden (6). SYMPTOM MANAGEMENT Pain As in other patient populations, the burden of symptoms for patients undergoing dialysis is inversely associated with their reported quality of life (7). Pain is one of the most common symptoms reported by patients undergoing dialysis, and several studies have found that approximately 50% of these patients report pain. For most patients undergoing dialysis, the pain is musculoskeletal in origin. Smaller numbers of patients have pain related to the dialysis procedure, peripheral neuropathy, peripheral vascular disease, or carpal tunnel syndrome. Three studies have found that pain is undertreated in 75% of patients undergoing dialysis (8–10). As in cancer patients, use of the World Health Organization (WHO) three-step analgesic ladder has been found to be effective in the treatment of pain in dialysis patients (9). Because the metabolites of some of the opioids on the analgesic ladder are renally excreted and active, these opioids, morphine, codeine, meperidine, and propoxyphene, are not recommended for use in patients with advanced kidney disease (Table 3) (11). 2 Onco-Nephrology Curriculum Adapted from reference 5. Morphine is the best studied of the opioids used for pain management, and its most common metabolites (including morphine-3-glucuronide, morphine-6-glucuronide, and normorphine) are excreted by the kidneys. The clearance of these metabolites is particularly problematic in stage 4 and 5 CKD and ESRD. Morphine-6-glucuronide is an active metabolite with analgesic properties; it crosses the blood–brain barrier and may have prolonged central nervous system effects. A comprehensive review recommended that morphine not be used in patients with kidney disease because it is so difficult to manage the complicated adverse effects of the morphine metabolites (12). Studies of codeine pharmacokinetics suggest that codeine metabolites would accumulate to toxic levels in a majority of patients undergoing hemodialysis. Codeine use is not recommended because serious adverse effects have been reported in patients with CKD (12). Hydromorphone is metabolized in the liver largely to hydromorphone-3-glucuronide. This metabolite accumulates in patients with kidney disease and can cause opioid neurotoxicity. Some studies suggest that hydromorphone is removed with dialysis. It is recommended that hydromorphone be used cautiously, if at all, in patients stopping dialysis (12). Use of oxycodone in patients with kidney disease has not been well studied. The elimination half-life of oxycodone is Table 3. Pain medications for use in advanced kidney failure Recommended Use with caution Do not use Fentanyl Methadone Hydromorphone Acetaminophen Gabapentin Tramadol Hydrocodone/oxycodone Desipramine/nortriptyline Morphine Codeine Meperidine Propoxyphene Adapted from reference 11. Seehttp://www.kidneysupportivecare.org/Files/ PainBrochure9-09.aspx for recommendations for use, dosage reductions, and cautions in advanced kidney failure. American Society of Nephrology lengthened in dialysis patients, and excretion of metabolites is impaired but almost all are inactive. Oxycodone can be used with caution in patients with advanced CKD and ESRD (12). The WHO analgesic ladder recommends the use of fentanyl for severe pain. Fentanyl is metabolized in the liver primarily to norfentanyl. There is no evidence that any fentanyl metabolites are active. Several studies have found that fentanyl can be used safely in patients with CKD and ESRD. It has negligible dialyzability. Fentanyl use is deemed to be one of the safest opioids to use in patients with advanced CKD (12). The WHO analgesic ladder recommends methadone for severe pain. Studies in anuric patients have found that nearly all of methadone and its metabolites doses are excreted in the feces. No dose adjustments are recommended for patients undergoing dialysis. The use of methadone appears safe in patients with advanced CKD and ESRD (12). Opioids are also often used to treat dyspnea at the end of life in patients with CKD, ESRD, and cancer. In the setting of worsening renal function or withdrawal of dialysis, the clinician may be challenged to distinguish uremic encephalopathy from opioid neurotoxicity. Both can cause sedation, hallucinations, and myoclonus. If respiratory depression is also present, it is advisable to stop the opioid until the respiratory depression subsides. If the patient’s respiratory rate is not compromised, the opioid can usually be continued, and a benzodiazepine such as lorazepam is added to control the myoclonus. Occasionally, a lorazepam continuous intravenous infusion at 1 or 2 mg/h is necessary to control the myoclonus. Although nonsteroidal anti-inflammatory drugs are recommended for use in step 1 on the WHO analgesic ladder, the use of these drugs in patients with CKD is contraindicated because of their nephrotoxicity and in dialysis patients because of the increased risk of upper gastrointestinal bleeding. The Mid-Atlantic Renal Coalition and the Coalition for Supportive Care of Kidney Patients assembled a panel of international experts on pain management in CKD and developed an evidence-based algorithm for treating pain in dialysis patients that is accessible online (11). Other symptom management Because of their comorbid illnesses, patients undergoing dialysis are among the most symptomatic of any population with chronic disease. In one study (13) of 162 patients undergoing dialysis from three different dialysis units, the median number of symptoms reported by patients was 9.0. Pain, dyspnea, dry skin, and fatigue were each reported by .50% of the patients. Of the 30 different symptoms reported by the patients, the 6 most bothersome (starting with the most severe first) were as follows: chest pain, bone or joint pain, difficulty becoming sexually aroused, trouble falling asleep, muscle cramps, and itching. Pruritus, or itching, is one of the most frustrating symptoms experienced by CKD and ESRD patients. Secondary hyperparathyroidism, increased calcium–phosphate deposition in the skin, dry skin, inadequate dialysis, anemia, iron deficiency, chronic inflammation, imbalance in endogenous opioids, American Society of Nephrology neuropathic processes, and low-grade hypersensitivity to products used in the dialysis procedure have all been identified as possible contributory factors. In addition to careful management of all these factors, the following interventions have been tried for pruritus with some success: emollient skin creams, phototherapy with ultraviolet B light three times weekly, intravenous lidocaine during dialysis for refractory itching, gabapentin, naltrexone, and thalidomide (14). ADVANCE CARE PLANNING Advance care planning is a process of communication among patients, families, health care providers, and other important individuals about the patient’s preferred decision-maker and appropriate future medical care if and when a patient is unable to make his or her own decisions. Advance care planning has been recommended as a central tenet of CKD, ESRD, and cancer patient care (15). It is especially appropriate because of the lifelimiting nature of these diseases. The “surprise” question— would I be surprised if this patient died in the next year?—has been validated in both the ESRD and cancer patient populations as a reliable trigger to identify patients who are at increased risk of death within 1 year and for whom palliative care consultation including advance care planning is appropriate (16,17). Researchers have developed an evidence-based robust integrated prognostic model with a C-statistic of 0.8 to estimate dialysis patients’ 6- and 12-month survival (18), which is available online at www.touchcalc.com/calculators/sq. The American Society of Nephrology and Renal Physicians Association have recommended that advance care planning for CKD and ESRD patients including a patient-specific estimate of prognosis and shared decision-making occur prior to the initiation of dialysis (5,19). Nephrologists are responsible for advance care planning, although aspects of it can be delegated to other nephrology personnel (15). Advance care planning is important for kidneycancer patients because it can ensure that patients’ wishes for end-of-life care are respected, that unwanted interventions are avoided, and that patients and their families are satisfied with the care provided (15). Although nephrologists are expected to possess primary palliative care skills, they are encouraged to consult palliative care physicians for more complex cases (20). Table 2 presents a comprehensive approach to incorporating palliative care into dialysis facility patient care. CONCLUSIONS There is a growing commitment among the leadership in the nephrology and oncology communities to enhance palliative care for advanced kidney disease and cancer patients. It is highly likely that palliative care for these patients will be significantly improved over the next decade, as nephrologists, oncologists, and palliative care consultants apply the knowledge and skills discussed in this chapter. Onco-Nephrology Curriculum 3 TAKE HOME POINTS c Early palliative care intervention is becoming the standard of care for kidney-cancer patients. It improves patients’ quality of life and respects their treatment wishes. c CKD, ESRD, and cancer patients have shortened life expectancy. The 5- year survival rate for incident dialysis patients is 40%, which is 30% less than that of incident cancer patients (66%). c Because they do not have active metabolites excreted by the kidneys, fentanyl and methadone are the safest drugs to use for severe nociceptive pain in patients with advanced kidney disease. c Palliative care consultation can help with complex pain and symptom management and advance care planning, including shared decisionmaking about the goals of care. Collaboration with hospices can help dialysis units implement a palliative care program and appropriately refer patients for hospice care at the end of life. REFERENCES 1. United States Renal Data System. 2013 USRDS Annual Data Report: Atlas of Chronic Kidney Disease and End-Stage Renal Disease in the United States, Bethesda, MD, National Institutes of Health, National Institutes of Diabetes and Digestive and Kidney Diseases, 2013 2. Abdel-Kader K, Unruh ML, Weisbord SD. Symptom burden, depression, and quality of life in chronic and end-stage kidney disease. Clin J Am Soc Nephrol 4: 1057–1064, 2009 3. Mao JJ, Armstrong K, Bowman MA, Xie SX, Kadakia R, Farrar JT. Symptom burden among cancer survivors: impact of age and comorbidity. J Am Board Fam Med 20: 434–443, 2007 4. Renal Physicians Association and American Society of Nephrology. Shared Decision-Making in the Appropriate Initiation of and Withdrawal from Dialysis, Washington, DC, Renal Physicians Association, 2000 5. Renal Physicians Association. Shared Decision-Making in the Appropriate Initiation of and Withdrawal from Dialysis, 2nd ed., Rockville, MD: Renal Physicians Association, 2010 6. Smith TJ, Temin S, Alesi ER, Abernethy AP, Balboni TA, Basch EM, Ferrell BR, Loscalzo M, Meier DE, Paice JA, Peppercorn JM, Somerfield M, Stovall E, Von Roenn JH. American Society of Clinical Oncology provisional clinical opinion: The integration of palliative care into standard oncology care. J Clin Oncol 30: 880–887, 2012 7. Kimmel PL, Emont SL, Newmann JM, Danko H, Moss AH. ESRD patient quality of life: Symptoms, spiritual beliefs, psychosocial factors, and ethnicity. Am J Kidney Dis 42: 713–721, 2003 4 Onco-Nephrology Curriculum 8. Davison SN. Pain in hemodialysis patients: Prevalence, cause, severity, and management. Am J Kidney Dis 42: 1239–1247, 2003 9. Barakzoy AS, Moss AH. Efficacy of the world health organization analgesic ladder to treat pain in end-stage renal disease. J Am Soc Nephrol 17: 3198–3203, 2006 10. Bailie GR, Mason NA, Bragg-Gresham JL, Gillespie BW, Young EW. Analgesic prescription patterns among hemodialysis patients in the DOPPS: Potential for underprescription. Kidney Int 65: 2419–2425, 2004 11. Coalition M-AR. Clinical algorithm and preferred medications to treat pain in dialysis patients. Available at: http://www.kidneysupportivecare.org/ Files/PainBrochure9-09.aspx. Accessed March 25, 2015 12. Dean M. Opioids in renal failure and dialysis patients. J Pain Symptom Manage 28: 497–504, 2004 13. Weisbord SD, Fried LF, Arnold RM, Fine MJ, Levenson DJ, Peterson RA, Switzer GE. Prevalence, severity, and importance of physical and emotional symptoms in chronic hemodialysis patients. J Am Soc Nephrol 16: 2487–2494, 2005 14. Murtagh F, Weisbord S. Symptoms in renal disease: Their epidemiology, assessment, and management. In: Supportive Care for the Renal Patient, edited by Chambers EJ, Germain M, Brown E, 2nd Ed., Oxford, UK, Oxford University Press, 2010, pp 103–138 15. Holley JL, Davison SN. Advance care planning for patients with advanced CKD: A need to move forward. Clin J Am Soc Nephrol 10: 344– 346, 2015 16. Moss AH, Ganjoo J, Sharma S, Gansor J, Senft S, Weaner B, Dalton C, MacKay K, Pellegrino B, Anantharaman P, Schmidt R. Utility of the “surprise” question to identify dialysis patients with high mortality. Clin J Am Soc Nephrol 3: 1379–1384, 2008 17. Moss AH, Lunney JR, Culp S, Auber M, Kurian S, Rogers J, Dower J, Abraham J. Prognostic significance of the “surprise” question in cancer patients. J Palliat Med 13: 837–840, 2010 18. Cohen LM, Ruthazer R, Moss AH, Germain MJ. Predicting six-month mortality for patients who are on maintenance hemodialysis. Clin J Am Soc Nephrol 5: 72–79, 2010 19. Williams AW, Dwyer AC, Eddy AA, Fink JC, Jaber BL, Linas SL, Michael B, O’Hare AM, Schaefer HM, Shaffer RN, Trachtman H, Weiner DE, Falk AR; American Society of Nephrology Quality, and Patient Safety Task Force. Critical and honest conversations: The evidence behind the “Choosing Wisely” campaign recommendations by the American Society of Nephrology. Clin J Am Soc Nephrol 7: 1664– 1672, 2012 20. Quill TE, Abernethy AP. Generalist plus specialist palliative care: Creating a more sustainable model. N Engl J Med 368: 1173–1175, 2013 21. Davison SN, Jhangri GS, Johnson JA. Cross-sectional validity of a modified Edmonton symptom assessment system in dialysis patients: A simple assessment of symptom burden. Kidney Int 69: 1621–1625, 2006 American Society of Nephrology REVIEW QUESTIONS 1. Which one of the following is an advantage of the physician orders for life-sustaining treatment (POLST) form compared with an advance directive for a patient with stage 5 kidney disease, terminal cancer, and loss of decisionmaking capacity? a. It is legal in all fifty states b. It is an immediately actionable medical order c. Checklist format prevents contradictory orders from being issued d. It is appropriate for patients in all stages of CKD Answer: b is correct. The POLST form or variant is endorsed in 22 states at present and being developed in another 23. Contradictory orders could be written between Sections A and B such that the patient is to receive CPR in Section A and comfort measures in Section B of the form. The form is only appropriate for patients who are seriously ill and for whom the physician would not be surprised if the patient died in the next year. Reference Citko J, Moss AH, Carley M, Tolle SW. The National POLST Paradigm Initiative, 2nd ed. FAST FACTS AND CONCEPTS. Available at: http://www.mypcnow.org/#!blank/k2dh9. Accessed January 28, 2016. See www.polst.org for more information. 2. Which one of the following medications would be the preferred, recommended medication for a patient with stage 5 CKD and lung cancer with painful metastases to the bone who describes his pain as aching and 10/10? a. b. c. d. Morphine Acetaminophen Codeine Fentanyl Answer: d is correct. Fentanyl is appropriate for severe nociceptive pain in patients with advanced kidney failure. American Society of Nephrology Fentanyl does not have active metabolites excreted by the kidneys. Morphine and codeine are contraindicated in advanced kidney failure because of the toxic accumulation of active metabolites in kidney failure. Acetaminophen is not appropriate for severe pain. Reference Dean M. Opioids in renal failure and dialysis patients. J Pain Symptom Manage 28: 497–504, 2004 3. Which one of the following statements best summarizes the role of shared decision-making for patients with advanced kidney disease and cancer approaching the need for dialysis? a. Shared decision-making is an outmoded concept from the 1980s b. Shared decision-making fits well with a disease-oriented approach to CKD patient treatment c. Shared decision-making for CKD patients defaults to dialysis modality choices d. Shared decision-making is the recognized preferred model for medical decision-making Answer: d is correct. Shared decision-making was introduced in the 1980s as a process to promote informed consent and decisions that adequately take account of patients’ preferences. It fits well with an individualized, patient-centered approach to decision-making and not a disease-oriented approach. Shared decision-making for CKD patients encompasses decisions about whether to start or stop dialysis and not just dialysis modality. References Barry MJ, Edgman-Levitan S. Shared decision making: Pinnacle of patient-centered care. N Engl J Med 366: 780–781, 2012 Renal Physicians Association. Shared Decision-Making in the Appropriate Initiation of and Withdrawal from Dialysis, 2nd Ed., Rockville, MD, Renal Physicians Association, 2010 Onco-Nephrology Curriculum 5